You are on page 1of 212

Topic:

Subtopic:
Provision:
Case Name:

Definitions and Attributes of Corporation


Concession Theory and Corporate Franchises
Section 2, Corporation Code
Tayag v. Benguet Consolidated, Inc.

FACTS:
Idonah Slade Perkins died in New York City on March 27, 1960. Prospero Sanidad instituted ancillary administration
proceedings in the Court of First Instance of Manila. Lazaro A. Marquez was appointed ancillary administrator. He was
later on substituted by the Renato D. Tayag.
A dispute arose between the domiciliary administrator in New York and the ancillary administrator in the Philippines as
to which of them was entitled to the possession of the stock certificates owned by Perkins in Benguet Consolidated,
Inc.
CFI of Manila ordered the domiciliary administrator, County Trust Company, to "produce and deposit" them with the
ancillary administrator or with the Clerk of Court. The domiciliary administrator did not comply with the order.
Thereafter, the ancillary administrator petitioned the court to "issue an order declaring the certificate or certificates of
stocks covering the 33,002 shares issued in the name of Idonah Slade Perkins by Benguet Consolidated, Inc., be
declared or considered as lost granted
Benguet Consolidated, Inc. appealed opposing the CFI order alleging that the said stock certificates are in existence,
they are today in the possession of the domiciliary administrator, the County Trust Company, in New York, U.S.A.
ISSUE:
WON the Benguet Consolidated Incs appeal/ opposition to the CFI order was proper
HELD:
No. SC affirmed CFI.
A corporation is defined as an artificial being created by operation of law. It owes its life to the state, its birth being
purely dependent on its will. A corporation as known to Philippine jurisprudence is a creature without any existence
until it has received the imprimatur of the state according to law. It is logically inconceivable therefore that it will have
rights and privileges of a higher priority than that of its creator. It is not immune from judicial control where a duty
under the law as ascertained in an appropriate legal proceeding is cast upon it. Therefore, Benguet Consolidated Inc, a
corporation created under the Philippine laws may not adamantly oppose the CFI order.

1
3B-Corporation Law

Topic:
Subtopic:
Provision:
Case Name:

Definitions and Attributes of Corporation


Concession Theory and Corporate Franchises
Section 2, Corporation Code
JRS Business Corp v Imperial Insurance Inc.

FACTS:
Imperial Insurance Inc., filed against JRS Business Corp, an establishment duly franchised by the Congress of the
Philippines to conduct a messenger and delivery express service, a complaint for sum of money. The parties entered
into a Compromise Agreement where defendants promised to pay their obligation in the amount of P 61,172.32 within
60 days and should they fail to pay, Imperial Insurance shall be entitled to move for the execution of the decision.
JRS failed to pay its judgment debt. Imperial Insurance Inc. then filed a motion for the issuance of a Writ of Execution.
A Writ of Execution was issued and Notices of Sale were sent out for the auction of the personal properties of J.R.S.
Business Corporation.
Notice of Sale of the "whole capital stocks of the defendants JRS Business Corporation, the business name, right of
operation, the whole assets, furnitures and equipments, the total liabilities, and Net Worth, books of accounts, etc., of
the petitioner corporation was, handed down.
JRS filed an "Urgent Petition for Postponement of Auction Sale and for Release of Levy on the Business Name and Right
to Operate of Defendant JRS Business Corporation" stating that the judgment was for money only. Thus, Imperial
Insurance may not use the business name of JRS Business Corp and its right to operate under the franchise is not
transferable and could not be subject to levy and sale. CFI of Manila denied the petition for postponement.
Auction sale was conducted and all the properties of JRS Business Corporation, the business name, right of operation,
the whole assets, furnitures and equipments, the total liabilities and net worth, books of accounts and etc. were
bought by respondent Imperial Insurance, Inc., for P10,000.00, which was the highest bid offered. After the sale,
respondent Insurance Company took possession of the proper ties and started running the affairs and operating the
business of the JRS Business Corporation.
ISSUE:
Won the business name or trade name, franchise (right to operate) and capital stocks of the petitioner could be the
subject of levy, execution and sale.
SC RULING:
No. The right to operate a messenger and express delivery service, by virtue of a legislative enactment, is admittedly a
secondary franchise (R.A. No. 3260, entitled "An Act granting the JRS Business Corporation a franchise to conduct a
messenger and express service)" and, as such, under our corporation law, is subject to levy and sale on execution
together and including all the property necessary for the enjoyment thereof. The law, however, indicates the procedure
under which the same (secondary franchise and the properties necessary for its enjoyment) may be sold under
execution. Said franchise can be sold under execution, when such sale is especially decreed and ordered in the
judgment and it becomes effective only when the sale is confirmed by the Court after due notice (Sec. 56, Corp. Law).
The compromise agreement and the judgment based thereon, do not contain any special decree or order making the
franchise answerable for the judgment debt. The same thing may be stated with respect to petitioner's trade name or
business name and its capital stock. Incidentally, the trade name or business name corresponds to the initials of the
President of the petitioner corporation and there can be no serious dispute regarding the fact that a trade name or
business name and capital stock are necessarily included in the enjoyment of the franchise. Like that of a franchise,
the law mandates, that property necessary for the enjoyment of said franchise, can only be sold to satisfy a judgment
debt if the decision especially so provides. As we have stated heretofore, no such directive appears in the decision.
Moreover, a trade name or business name cannot be sold separately from the franchise, and the capital stock of the
petitioner corporation or any other corporation, for the matter, represents the interest and is the property of
stockholders in the corporation, who can only be deprived thereof in the manner provided by law

2
3B-Corporation Law

3
3B-Corporation Law

Topic:
Subtopic:
Provision:
Case Name:

Definitions and Attributes of Corporation


Doctrine of Separate Personality
Section 2, Corporation Code
Philippine National Bank v Merelo Aznar

FACTS:
In 1958, RISCO ceased operation due to business reverses. In plaintiffs desire to rehabilitate RISCO, they contributed a
total amount of P212,720.00 which was used in the purchase of the three parcels of land (2 in Talisay and 1 in Cebu).
After the purchase of the above lots, titles were issued in the name of RISCO. The amount contributed by plaintiffs
constituted as liens and encumbrances on the properties as annotated in the titles of said lots. Such annotation was
made pursuant to the Minutes of the Special Meeting of the Board Of Directors of RISCO on March 14, 1961, and a part
of it says: And that the respective contributions above-mentioned (Aznar et al.) shall constitute as theirlien or interest
on the property described above, if and when said property are titled in the nameof RISCO, subject to registration as
their adverse claim in pursuance of the Provision of LandRegistration Act, until such time their respective contributions
are refunded to themcompletely
Thereafter, various subsequent annotations were made on the same titles, including the Notice of Attachment and Writ
of Execution both dated August 3, 1962 in favor of herein defendant PNB. Certificate of Sale was issued in favor of
Philippine National Bank, being the lone and highest bidder of the three (3) parcels of land. A new certificate of title
was issued in the name of PNB.
This prompted plaintiffs-appellees to file the instant complaint seeking the quieting of their supposed title to the
subject properties, declaratory relief, cancellation of TCT and reconveyance with temporary restraining order and
preliminary injunction. Plaintiffs alleged that the subsequent annotations on the titles are subject to the prior
annotation of their liens and encumbrances. Plaintiffs further contended that the subsequent writs and processes
annotated on the titles are all null and void for want of valid service upon RISCO and on them, as stockholders. They
argued that the Final Deed of Sale and TCT No. 119848 are null and void as these were issued only after 28 years and
that any right which PNB may have over the properties had long become stale.
Defendant PNB on the other hand countered that plaintiffs have no right of action for quieting of title since the order of
the court directing the issuance of titles to PNB had already become final and executory. Defendant further asserted
that plaintiffs, as mere stockholders of RISCO do not have any legal or equitable right over the properties of the
corporation. PNB posited that even if plaintiffs monetary lien had not expired, their only recourse was to require the
reimbursement or refund of their contribution.51awphi1
TC ruled against PNB on the basis that there was an express trust created over the subject properties whereby RISCO
was the trustee and the stockholders, Aznar, et al., were the beneficiaries.
CA- set aside the judgment of the trial court. Although the Court of Appeals agreed with the trial court that a judgment
on the pleadings was proper, the appellate court opined that the monetary contributions made by Aznar, et al., to
RISCO can only be characterized as a loan secured by a lien on the subject lots, rather than an express trust. Thus, it
directed PNB to pay Aznar, et al., the amount of their contributions plus legal interest from the time of acquisition of
the property until finality of judgment.lawphil
Both parties filed an MR - denied. Hence, each party filed with this Court their respective petitions for review on
certiorari under Rule 45 of the Rules of Court.
ISSUE:
Whether or not there was a trust contract between RISCO and Aznar, et al
RULING:
No. Trust relations between parties may either be express or implied. An express trust is created by the intention of
the trustor or of the parties. An implied trust comes into being by operation of law. 21 Express trusts, sometimes
referred to as direct trusts, are intentionally created by the direct and positive acts of the settlor or the trustor - by
some writing, deed, or will or oral declaration. It is created not necessarily by some written words, but by the direct
and positive acts of the parties. In other words, the creation of an express trust must be manifested with reasonable
certainty and cannot be inferred from loose and vague declarations or from ambiguous circumstances susceptible of
other interpretations.23
No such reasonable certitude in the creation of an express trust obtains in the case at bar. In fact, a careful scrutiny of
the plain and ordinary meaning of the terms used in the Minutes does not offer any indication that the parties thereto
intended that Aznar, et al., become beneficiaries under an express trust and that RISCO serve as trustor.

4
3B-Corporation Law

Indeed, we find that Aznar, et al., have no right to ask for the quieting of title of the properties at issue because they
have no legal and/or equitable rights over the properties that are derived from the previous registered owner which is
RISCO, the pertinent provision of the law is Section 2 of the Corporation Code (Batas Pambansa Blg. 68), which states
that "[a] corporation is an artificial being created by operation of law, having the right of succession and the powers,
attributes and properties expressly authorized by law or incident to its existence."
As a consequence thereof, a corporation has a personality separate and distinct from those of its stockholders and
other corporations to which it may be connected. 24 Thus, we had previously ruled in Magsaysay-Labrador v. Court of
Appeals25 that the interest of the stockholders over the properties of the corporation is merely inchoate and therefore
does not entitle them to intervene in litigation involving corporate property.
Here, the interest, if it exists at all, of petitioners-movants is indirect, contingent, remote, conjectural, consequential
and collateral. At the very least, their interest is purely inchoate, or in sheer expectancy of a right in the management
of the corporation and to share in the profits thereof and in the properties and assets thereof on dissolution, after
payment of the corporate debts and obligations.
While a share of stock represents a proportionate or aliquot interest in the property of the corporation, it does not vest
the owner thereof with any legal right or title to any of the property, his interest in the corporate property being
equitable or beneficial in nature. Shareholders are in no legal sense the owners of corporate property, which is owned
by the corporation as a distinct legal person.
In the case at bar, there is no allegation, much less any proof, that the corporate existence of RISCO has ceased and
the corporate property has been liquidated and distributed to the stockholders. The records only indicate that, as per
Securities and Exchange Commission (SEC) Certification 27 dated June 18, 1997, the SEC merely suspended RISCOs
Certificate of Registration beginning on September 5, 1988 due to its non-submission of SEC required reports and its
failure to operate for a continuous period of at least five years.
Verily, Aznar, et al., who are stockholders of RISCO, cannot claim ownership over the properties at issue in this case on
the strength of the Minutes which, at most, is merely evidence of a loan agreement between them and the company.
There is no indication or even a suggestion that the ownership of said properties were transferred to them which would
require no less that the said properties be registered under their names. For this reason, the complaint should be
dismissed since Aznar, et al., have no cause to seek a quieting of title over the subject properties.

5
3B-Corporation Law

Topic:
Subtopic:
Provision:
Case Name:

Definitions and Attributes of Corporation


Doctrine of Limited Liability
Section 2, Corporation Code
Donina Halley v Printwell

DOCTRINE OF LIMITED LIABILITY


DONNINA HALLEY vs PRINTWELL, INC.
FACTS:

Halley was an incorporator of Business Media Philippines, Inc., with authorized capital stock of 3M, whereby at
par value of 10/share, Halley has 35,000 shares of which 87,500 was paid.

From 1988-1989, BMPI commissioned Printwell to print its magazines.

Apparently, BMPI only paid for 25,000, out of the 300,000 it owes Printwell due to the printing service.

In January 1990, Printwell thus sued BMPI for collection of sum of money.

Printwell amended the complaint to implead as defendants all the original stockholders and incorporators to
recover on their unpaid subscriptions, Halley for 350,000-87,500=262,500.

Defendants aver that they had all paid their subscriptions in full, and that BMPIs personality is separate, and
that the stockholders resolved to dissolve BMPI in February 1990.

RTC granted the petition, holding that the defendants used BMPIs corporate personality to perpetuate an
injustice, and that the ORs for the paid subscription is irregular.

CA Affirmed the RTC.


ISSUE:
Whether the Trust Fund doctrine is applicable. (i.e. creditors can go after unpaid subscriptions for fulfillment of claims)
HELD:
Yes.
Creditors may go after the unpaid subscriptions.
Trust Fund Doctrine - It is established doctrine that subscriptions to the capital of a corporation constitute a fund to
which creditors have a right to look for satisfaction of their claims and that the assignee in insolvency can maintain an
action upon any unpaid stock subscription in order to realize assets for the payment of its debts. It is the rule that the
property of a corporation is a trust fund for the payment of creditors, but such property can be called a trust fund only
by way of analogy or metaphor. As between the corporation itself and its creditors it is a simple debtor, and as
between its creditors and stockholders its assets are in equity a fund for the payment of its debts.
We clarify that the trust fund doctrine is not limited to reaching the stockholders unpaid subscriptions. The scope of
the doctrine when the corporation is insolvent encompasses not only the capital stock, but also other property and
assets generally regarded in equity as a trust fund for the payment of corporate debts. All assets and property
belonging to the corporation held in trust for the benefit of creditors that were distributed or in the possession of the
stockholders, regardless of full payment of their subscriptions, may be reached by the creditor in satisfaction of its
claim.

6
3B-Corporation Law

Topic:
Subtopic:
Provision:
Case Name:

Definitions and Attributes of Corporation


Doctrine of Piercing the Veil of Corporate Fiction
Concept Builders Inc. v NLRC

DOCTRINE OF PIERCING THE CORPORATE VEIL


CONCEPT BUILDERS v NLRC
FACTS:

Petitioner was the employer of private petitioners who filed with LA a case of illegal dismissal. Decision was
made in favor of the private respondents and order petitioner to pay for their back wages and other financial
claims.

To satisfy the liability of petitioner, a writ of execution was issued in order to levy personal properties of the
petitioner. When the sheriff tried to serve the writ, he was informed that the premises is now occupied by
another company, HPPI and that Concept Builders had already suspended its operation.

A certain Dennis Cuyegkeng filed a third-party claim with the Labor Arbiter alleging that the properties sought
to be levied upon by the sheriff were owned by Hydro (Phils.), Inc. (HPPI) of which he is the Vice-President.
Private respondents filed a "Motion for Issuance of a Break-Open Order," alleging that HPPI and petitioner
corporation were owned by the same incorporator/stockholders.

LA dismissed the motion. NLRC reversed the decision. Hence the appeal.
ISSUE:
Whether the Piercing of the Corporate Veil Doctrine can be applied in the absence of proof that HPPI was created to
evade its liabilities to the private respondent
HELD:
Yes.
It cannot be pierced in this case.
This is true likewise when the corporation is merely an adjunct, a business conduit or an alter ego of another
corporation.
The test in determining the applicability of the doctrine of piercing the veil of corporate fiction is as follows:
1.
2.
3.

Control, not mere majority or complete stock control, but complete domination, not only of finances but of policy
and business practice in respect to the transaction attacked so that the corporate entity as to this transaction had
at the time no separate mind, will or existence of its own;
Such control must have been used by the defendant to commit fraud or wrong, to perpetuate the violation of a
statutory or other positive legal duty or dishonest and unjust act in contravention of plaintiff's legal rights; and
The aforesaid control and breach of duty must proximately cause the injury or unjust loss complained of.

The absence of any one of these elements prevents "piercing the corporate veil." In applying the "instrumentality" or
"alter ego" doctrine, the courts are concerned with reality and not form, with how the corporation operated and the
individual defendant's relationship to that operation. Clearly, petitioner ceased its business operations in order to
evade the payment to private respondents of back wages and to bar their reinstatement to their former positions. HPPI
is obviously a business conduit of petitioner corporation and its emergence was skillfully orchestrated to avoid the
financial liability that already attached to petitioner corporation.

7
3B-Corporation Law

Topic:
Subtopic:
Provision:
Case Name:

Definitions and Attributes of Corporation


Doctrine of Piercing the Veil of Corporate Fiction
Donina Halley v Printwell, Inc.

DOCTRINE OF PIERCING THE CORPORATE VEIL


DONNINA HALLEY vs PRINTWELL, INC.
FACTS:

Halley was an incorporator of Business Media Philippines, Inc., with authorized capital stock of 3M, whereby at
par value of 10/share, Halley has 35,000 shares of which 87,500 was paid.

From 1988-1989, BMPI commissioned Printwell to print its magazines.

Apparently, BMPI only paid for 25,000, out of the 300,000 it owes Printwell due to the printing service.

In January 1990, Printwell thus sued BMPI for collection of sum of money.

Printwell amended the complaint to implead as defendants all the original stockholders and incorporators to
recover on their unpaid subscriptions, Halley for 350,000-87,500=262,500.

Defendants aver that they had all paid their subscriptions in full, and that BMPIs personality is separate, and
that the stockholders resolved to dissolve BMPI in February 1990.

RTC granted the petition, holding that the defendants used BMPIs corporate personality to perpetuate an
injustice, and that the ORs for the paid subscription is irregular.

CA affirmed the RTC.


ISSUE:
Whether the veil of corporate fiction can be pierced.
HELD:
Yes.
It can be pierced when the corporation is used as a clock for illegality.
While a corporation has a personality separate and distinct from those of its stockholders, directors, or officers, such
separate and distinct personality is merely a fiction created by law for the sake of convenience and to promote the
ends of justice. The corporate personality may be disregarded, and the individuals composing the corporation will be
treated as individuals, if the corporate entity is being used as a cloak or cover for fraud or illegality; as a justification
for a wrong; as an alter ego, an adjunct, or a business conduit for the sole benefit of the stockholders. As a general
rule, a corporation is looked upon as a legal entity, unless and until sufficient reason to the contrary appears. Thus, the
courts always presume good faith, and for that reason accord prime importance to the separate personality of the
corporation, disregarding the corporate personality only after the wrongdoing is first clearly and convincingly
established.
Although nowhere in Printwells amended complaint or in the testimonies Printwell offered can it be read or inferred
from that the petitioner was instrumental in persuading BMPI to renege on its obligation to pay; or that she induced
Printwell to extend the credit accommodation by misrepresenting the solvency of BMPI to Printwell, her personal
liability, together with that of her co-defendants, remained because the CA found her and the other defendant
stockholders to be in charge of the operations of BMPI at the time the unpaid obligation was transacted and incurred.

8
3B-Corporation Law

Topic:
Subtopic:
Provision:
Case Name:

Definitions and Attributes of Corporation


Doctrine of Piercing the Veil of Corporate Fiction
Siain Enterprises, Inc. v Cupertino Realty Corp

SIAIN ENTERPRISES v. CUPERTINO


G.R. No. 170782 | June 22, 2009
FACTS:

This is a is a petition for review on certiorari under Rule 45 of the Rules of Court assailing the decision of the
Court of Appeals which affirmed the decision of the Regional Trial Court

Siain Enterprises, Inc. obtained a loan of P37,000,000.00 from respondent Cupertino Realty Corporation
(Cupertino) covered by a promissory note signed by both petitioners and Cupertinos respective presidents,
Cua Le Leng and Wilfredo Lua.

The parties then executed an amendment to promissory note which provided for a seventeen percent (17%)
interest per annum on the P37,000,000.00 loan.

The amendment to promissory note was likewise signed by Cua Le Leng and Wilfredo Lua on behalf of
petitioner and Cupertino, respectively

Another promissory note was signed by Cua Le Leng in favor of Cupertino for P160,000,000.00. Cua Le Leng
signed the second promissory note as maker, on behalf of petitioner, and as co-maker, liable to Cupertino in
her personal capacity.

However, on March 11, 1996, through counsel, wrote Cupertino and demanded the release of the
P160,000,000.00 loan. In complete refutation, Cupertino, likewise through counsel, responded and denied that
it had yet to release the P160,000,000.00 loan. Cupertino maintained that petitioner had long obtained the
proceeds of the aforesaid loan. With this, Cupertino instituted extrajudicial foreclosure proceedings over the
properties subject of the amended real estate mortgage.

RTC rendered a decision dismissing petitioners complaint and ordering it to pay Cupertino P100,000.00 each
for actual and exemplary damages, and P500,000.00 as attorneys fees. The RTC recalled and set aside its
previous order declaring the notarial foreclosure of the mortgaged properties as null and void.

On appeal, the CA, as previously adverted to, affirmed the RTCs ruling.

In this regard, the lower courts applied the doctrine of piercing the veil of corporate fiction to preclude
petitioner from disavowing receipt of the P160,000,000.00 and paying its obligation under the amended real
estate mortgage.
ISSUE:

Petitioner asseverates that the lower courts erroneously applied the doctrine of piercing the veil of corporate
fiction when both gave credence to Cupertinos evidence showing that petitioners affiliates were the previous
recipients of part of the P160,000,000.00 indebtedness of petitioner to Cupertino. Is this valid?
HELD:

Yes.

As a general rule, a corporation will be deemed a separate legal entity until sufficient reason to the contrary
appears. But the rule is not absolute. A corporations separate and distinct legal personality may be
disregarded and the veil of corporate fiction pierced when the notion of legal entity is used to defeat public
convenience, justify wrong, protect fraud, or defend crime. In this case, Cupertino presented overwhelming
evidence that petitioner and its affiliate corporations had received the proceeds of the P160,000,000.00 loan
increase which was then made the consideration for the Amended Real Estate Mortgage.

The facts established in the case at bar has convinced the Court of the propriety to apply the principle by
virtue of which, the juridical personalities of the various corporations involved are disregarded and the ensuing
liability of the corporation to attach directly to its responsible officers and stockholders These are as follows:
1.That the checks, debit memos and the pledges of the jewelries, condominium units and trucks were
constituted not exclusively in the name of [petitioner] but also either in the name of Yuyek Manufacturing
Corporation, Siain Transport, Inc., Cua Leleng and Alberto Lim is of no moment. 2. Siain and Yuyek have [a]
common set of [incorporators], stockholders and board of directors; 3. They have the same internal
bookkeeper and accountant in the person of Rosemarie Ragodon; 4. They have the same office address at 306
Jose Rizal St., Mandaluyong City; 5. They have the same majority stockholder and president in the person of
Cua Le Leng; and 6. In relation to Siain Transport, Cua Le Leng had the unlimited authority by and on herself,
without authority from the Board of Directors, to use the funds of Siain Trucking to pay the obligation incurred
by the [petitioner] corporation. 7. As such, [petitioner] corporation is now estopped from denying the above
apparent authorities of Cua Le Leng who holds herself to the public as possessing the power to do those acts,
against any person who dealt in good faith as in the case of Cupertino.

9
3B-Corporation Law

10
3B-Corporation Law

Topic:
Subtopic:
Provision:
Case Name:

Definitions and Attributes of Corporation


Doctrine of Piercing the Veil of Corporate Fiction
Kukan International Corporation v Hon. Amor Reyes

FACTS:

Private respondent Romeo M. Morales doing business under the name RM Morales Trophies and Plaques was
awarded a P5 million contract for the supply and installation of signages in a building constructed in Makati
sometime in March 1998.

The contract price was later reduced to P3,388,502 because some items were deleted from the contract.
Morales complied with his contractual obligations but he was paid only the amount of P1,976,371.07 leaving a
balance of P1,412,130.93.

He filed a case against Kukan, Inc., for sum of money with the RTC of Manila docketed as Civil Case No. 9993173. Kukan Inc., stopped participating in the proceedings in November 2000, hence, it was declared in
default and Morales presented his evidence ex-parte against petitioner.

On November 28, 2002, the RTC rendered a decision in favor of Morales and against Kunkan, Inc. ordering the
latter to pay the sum of P1,201,724.00 with legal interest of 12% per annum until fully paid; P50,000.00 as
moral damages,P20,000.00 as attorney's fees and P7,960.06 as litigation expenses.

The counterclaimfiled by Kunkan, Inc. was dismissed. The decision became final and executory During the
execution, the sheriff levied the personal properties found at the office of Kukan, Inc..

Claiming it owned the properties levied, Kukan International Corporation (KIC) fied an Affidavit of Third Party
Claim. Morales filed an Omnibus Motion praying to apply the principle of piercing the veil of corporate entity.
He alleged that Kankun, Inc. and KIC are one and the same corporation His Motion was denied. On Motion of
Morales the presiding Judge of Branch 17 of RTC Manila inhibited himself from hearing the case. It was raffled
to Branch 21 which granted the Motion filed by Morales on March 12, 2007 and decreed that Kukan, Inc. and
Kukan International Inc., as one and the same corporation; that the levy made on the properties of KIC is valid;
and ordering Kunkan International Corp. and Michael Chan as jointly and severally liable to pay the award
pursuant to the Decision dated November 28, 2002.

KIC filed a Motion for Reconsideration which was denied.KIC brought the case to the Court of Appeals which
rendered the Decision n January 23, 2008 denying KIC's petition.

The CA also denied its Motion for Reconsideration in the Resolution dated June 7, 2007. Hence, this case.
ISSUE/S:

One of the issues raised is whether or not the trial court and the appellate court correctly applied the principle
of piercing the veil of corporate entity.
HELD:

The Supreme Court ruled that the doctrine of piercing the veil of corporate entity finds no application in this
case. According to the Supreme Court, the principle of piercing the veil of corporate entity and the resulting
treatment of two related corporation as one and the same juridical person applies only to established liability
and not to confer jurisdiction. In this case, the Supreme Court ruled that KIC was not made a party defendant
in Civil Case No. 99-93173.

It entered a special but not a voluntary appearance in the trial court to assert that it was a separate entity and
has a separate legal personality from Kunkan, Inc. KIC was not impleaded nor served with summons.

Hence, it could only assert its claim through the affidavits, comments and motions filed by special apperance
before the RTC that it is a separate juridical entity. The Supreme stated that the doctrine of piercing the veil of
corporate entity comes to play during the trial of the case after the court has already acquired jurisdiction over
the corporation. To justify the piercing of the veil of corporate fiction, it must be shown by clear and convincing
proof that the separate and distinct personality of the corporation was purposely employed to evade a
legitimate and binding comittment and perpetuate a fraud or like wrongdoings.

In those instances when the Court pierced the veil of corporate fiction of two corporations, there was a
confluence of the following factors: 1. A first corporation is dissolved; 2. The assets of the first corporation is
transferred to a second corporation to avoid a financial liability of the first corporation; and 3. Both
corporations are owned and controlled by the same persons such that the second corporation should be
considered as a continuation and successor of the first corporation.

In this case, the second and third factors are conspicuously absent. There is, therefore, no compelling
justification for disregarding the fiction of corporate entity separating Kukan, Inc. from KIC. In applying the
principle, both the RTC and the CA miserably failed to identify the presence of the abovementioned factors.
The High Court stated that neither should the level of paid-up capital of Kukan, Inc. upon its incorporation be
viewed as a badge of fraud, for it is in compliance with Sec. 13 of the Corporation Code, which only requires a

11
3B-Corporation Law

minimum paid-up capital of PhP 5,000. The suggestion that KIC is but a continuation and successor of Kukan,
Inc., owned and controlled as they are by the same stockholders, stands without factual basis.
The fact that Michael Chan, a.k.a. Chan Kai Kit, owns 40% of the outstanding capital stock of both corporations
standing alone, is insufficient to establish identity. There must be at least a substantial identity of stockholders
for both corporations in order to consider this factor to be constitutive of corporate identity. Petition granted

12
3B-Corporation Law

Topic:
Subtopic:
Provision:
Case Name:

Definitions and Attributes of Corporation


Doctrine of Piercing the Veil of Corporate Fiction
Koppel Phil Inc. v Yatco

Facts:

Koppel Philippines Inc. (KPI) has a capital stock divided into thousand (1,000) shares of P100 each.

The Koppel Industrial Car and Equipment Company (KICEC) owns 995 shares of the total capital stock. KICEC is
organized under US laws and not licensed to do business in the Philippines. The remaining five (5) shares only
were and are owned one each by officers of the KPI.

They have the following business process: (1) "When a local buyer was interested in the purchase of railway
materials, machinery, and supplies, it asked for price quotations from KPI"; (2) "KPI then cabled for the quotation
desired from Koppel Industrial Car and Equipment Company"; (3) "KPI, however, quoted to the purchaser a selling
price above the figures quoted by Koppel Industrial Car and Equipment Company"; (4) "On the basis of these
quotations, orders were placed by the local purchasers KPI paid under protest the P64,122.51 demanded by the
CIR.
Total profit
Php 3,772,403,82
KPI Share
Php 132,201.30
KPI paid commercial brokers tax (4% of KPI Share)
Php 5,288.05
CIR demanded (1% of Total Profit) + 25% surcharge for late Php 64,122.51
payment Paid tax

It appears that KICEC is the only foreign principal of KPI.

The KPI corporation bore alone incidental expenses - as, for instance, cable expenses-not only those of its own
cables but also those of its "principal" .

The KPI's "share in the profits" realized from the transactions in which it intervened was left virtually in the hands
of KICEC

Where drafts were not paid by the purchasers, the local banks were instructed not to protest them but to refer
them to KPI which was fully empowered by KICEC to instruct the banks with regards to disposition of the drafts and
documents

Where the goods were European origin, consular invoices, bill of lading, and, in general, the documents necessary
for clearance were sent directly to KPI

If the KPI had in stock the merchandise desired by local buyers, it immediately filled the orders of such local
buyers and made delivery in the Philippines without the necessity of cabling its principal in America either
for price quotations or confirmation or rejection of that agreed upon between it and the buyer

Whenever the deliveries made by KICEC were incomplete or insufficient to fill the local buyer's orders, KPI used to
make good the deficiencies by deliveries from its own local stock, but in such cases it charged its principal only
the actual cost of the merchandise thus delivered by it from its stock and in such transactionsKPI did
not realize any profit CFI:

KPI is a mere dummy or branch ("hechura") of KICEC. did not deny legal personality to Koppel (Philippines), Inc. for
any and all purposes, but in effect its conclusion was that, in the transactions involved herein, the public interest
and convenience would be defeated and what would amount to a tax evasion perpetrated, unless
resort is had to the doctrine of "disregard of the corporate fiction."
Issues/Ruling:

WON KPI is a domestic corporation distinct and separate from, and not a mere branch of KICEC
KPI:

Its corporate existence as cannot be collaterally attacked and that the Government is estopped from so doing.

Koppel (Philippines), Inc. was a mere branch or agency or dummy ("hechura") of Koppel Industrial Car and
Equipment Co. The lower court did not hold that the corporate personality of KPI would also be disregarded in
other cases or for other purposes. It would have had no power to so hold. The courts' action in this regard must
be confined to the transactions involved in the case at bar "for the purpose of adjudging the rights and
liabilities of the parties in the case. They have no jurisdiction to do more peaches United States vs. Milwaukee
Refrigeration Transit
General Rule: a corporation will be looked upon as a legal entity as a general rule, and until sufficient reason to
the contrary appears
Exception: Wthe notion of legal entity is used to defeat public convenience, justify wrong, protect fraud, or
defend crime, the law will regard the corporation as an association of persons.

SC:

13
3B-Corporation Law

Manifestly, the principle is the same whether the "person" be natural or artificial. A very numerous and
growing class of cases wherein the corporate entity is disregarded is that (it is so organized and controlled, and
its affairs are so conducted, as to make it merely an instrumentality, agency, conduit or adjunct of another
corporation)." Where it appears that two business enterprises are owned, conducted and controlled by the
same parties, both law and equity will, when necessary to protect the rights of third persons, disregard the
legal fiction that two corporations are distinct entities, and treat them as identical.
The fact that KPI is a mere branch is conclusively borne out by the fact, among others, that the amount of the
so-called "share in the profits" of KPIwas ultimately left to the sole, unbridled control of KICEC . If
KPI was intended to function as a bona fide separate corporation, we cannot conceive how this arrangement
could have been adopted.
No group of businessmen could be expected to organize a mercantile corporation if the amount of that profit
were to be subjected to such a unilateral control of another corporation, unless indeed the former has
previously been designed by the incorporators to serve as a mere subsidiary, branch or agency of the latter.
KPI charged the parent corporation no more than actual cost - without profit whatsoever - for merchandise
allegedly of its own to complete deficiencies of shipments made by said parent corporation.

14
3B-Corporation Law

Topic:
Subtopic:
Provision:
Case Name:

Definitions and Attributes of Corporation


Doctrine of Piercing the Veil of Corporate Fiction
Heirs of Fe Tan Uy v International Exchange Bank

Facts:

June to September 1997, International Exchange Bank granted loan to Hammer Garments Incorporation
covered by promissory notes for a total of P24,938,898.08.
These were made pursuant to the Letter-Agreement,4 dated March 23, 1996, between iBank and Hammer,
represented by its President and General Manager, Manuel Chua (Chua) a.k.a. Manuel Chua Uy Po Tiong,
granting Hammer a P 25 Million-Peso Omnibus Line.5 The loans were secured by a P 9 Million-Peso Real Estate
Mortgage executed on July 1, 1997 by Goldkey Development Corporation (Goldkey) over several of its
properties and a P 25 Million-Peso Surety Agreement7 signed by Chua and his wife, Fe Tan Uy (Uy), on April 15,
1996.
Hammer defaulted, prompting iBank to foreclose on properties leaving an unpaid balance of Php 13Million
iBank filed a complaint for collection for the deficiency against Chua, Uy, and Goldkey
Goldkey denies liability citing it only acted as 3rd party guarantor
Chua and Hammer did not file response
iBank wants to pierce to veil of corporate personality and was granted by RTC due to:
o 1. both were family corporations of Chua and Uy, with Chua as the President and Chief Operating
Officer;
o (2) both corporations shared the same office and transacted business from the same place,
o (3) the assets of Hammer and Goldkey were co-mingled; and
o (4) when Chua absconded, both Hammer and Goldkey ceased to operate. As such, the piercing of the
veil of corporate fiction was warranted. Uy, as an officer and stockholder of Hammer and Goldkey, was
found liable to iBank together with Chua, Hammer and Goldkey for the deficiency of P13,420,177.62.

Issue:
Whether or not there is guilt by association in those cases where the veil of corporate fiction may be pierced
Whether or not the "alter ego" theory in disregarding the corporate personality of a corporation is applicable to
Goldkey

Ruling:
Uy is not liable; The piercing of the veil of corporate fiction is not justified
The heirs of Uy argue that the latter could not be held liable for being merely an officer of Hammer and
Goldkey because it was not shown that she had committed any actionable wrong22 or that she had
participated in the transaction between Hammer and iBank. They further claim that she had cut all ties with
Hammer and her husband long before the execution of the loan.
The Court finds in favor of Uy.
Basic is the rule in corporation law that a corporation is a juridical entity which is vested with a legal
personality separate and distinct from those acting for and in its behalf and, in general, from the people
comprising it. Following this principle, obligations incurred by the corporation, acting through its directors,
officers and employees, are its sole liabilities. A director, officer or employee of a corporation is generally not
held personally liable for obligations incurred by the corporation.24 Nevertheless, this legal fiction may be
disregarded if it is used as a means to perpetrate fraud or an illegal act, or as a vehicle for the evasion of an
existing obligation, the circumvention of statutes, or to confuse legitimate issues.25 This is consistent with the
provisions of the Corporation Code of the Philippines

15
3B-Corporation Law

Topic:
Subtopic:
Provision:
Case Name:

Definitions and Attributes of Corporation


Doctrine of Piercing the Veil of Corporate Fiction
WPM International Trading Inc. v Labayen

Facts:
- WPM, a domestic corporation engaged in the restaurant business with Manlapaz as its President, entered into a
management agreement with Labayen, owner of HBO Systems consultants, which authorizes the latter to operate,
manage and rehabilitate Quickbite which is a restaurant owned and controlled by WPM.
- Labayen then engaged the services of CLN Engineering Services for the renovation of Quickbite, Divisoria branch, at
the cost of P432,876.02. However out of the P432,876.02 renovation cost, only P320,000 was paid to CLN leaving a
balance of P112,876.02.
- CLN then filed a case for the collection of sum of money and damages against Labayen and Manlapaz, with the latter
being later on excluded by CLN through an amendment of the complaint.
- Labayen was declared in default for her failure to file a responsive pleading, the RTC later found Labayen liable to pay
CLN the amount of P112,876.02 with 12% interest per annum and 20% of the amount recoverable as attorneys fees.
- Labayen then instituted a complaint for damages against WPM and Manlapaz claiming that she only introduced
Manlapaz to CLNs general manager and they were the ones who agreed on the terms and conditions of the renovation
agreement.
- Among Manlapazs defense, he claims that since WPM has a separate and distinct personality, Manlapaz cannot be
made liable for Labayens claims.
Issue: Whether or not Manlapaz is liable in his own personal capacity
Held: No, Manlapaz is not liable in his own personal capacity
- The rule is settled that a corporation has a personality separate and distinct from the persons acting for and in its
behalf and, in general, from the people comprising it. The doctrine of piercing the corporate veil applies in three (3)
basic instances, namely:
a) when the separate and distinct corporate personality defeats public convenience, as when the corporate
fiction is used as a vehicle for the evasion of an existing obligation
b) in fraud cases, or when the corporate entity is used to justify a wrong, protect a fraud, or defend a crime or
c) is used in alter ego cases, i.e., where a corporation is essentially a farce, since it is a mere alter
ego or business conduit of a person, or where the corporation is so organized and controlled and
its affairs so conducted as to make it merely an instrumentality, agency, conduit or adjunct of
another corporation.
- Piercing the corporate veil based on the alter ego theory requires the concurrence of three elements, namely:
(1) Control, not mere majority or complete stock control, but complete domination, not only of finances but of
policy and business practice in respect to the transaction attacked so that the corporate entity as to this
transaction had at the time no separate mind, will or existence of its own.
(2) Such control must have been used by the defendant to commit fraud or wrong, to perpetuate the violation
of a statutory or other positive legal duty, or dishonest and unjust act in contravention of plaintiffs legal right
and
(3) The aforesaid control and breach of duty must have proximately caused the injury or unjust loss
complained of. The absence of any of these elements prevents piercing the corporate veil.
- In the present case, the attendant circumstances do not establish that WPM is a mere alter ego of Manlapaz. Aside
from the fact that Manlapaz was the principal stockholder of WPM, records do not show that WPM was organized and
controlled, and its affairs conducted in a manner that made it merely an instrumentality, agency, conduit or adjunct of
Manlapaz.
- Likewise, the records of the case do not support the lower courts finding that Manlapaz had control or domination
over WPM or its finances. That Manlapaz concurrently held the positions of president, chairman and treasurer, or that
the Manlapazs residence is the registered principal office of WPM, are insufficient considerations to prove that he had
exercised absolute control over WPM.

16
3B-Corporation Law

- In this connection, the Supreme Court stress that the control necessary to invoke the instrumentality or alter ego rule
is not majority or even complete stock control but such domination of finances, policies and practices that the
controlled corporation has, so to speak, no separate mind, will or existence of its own, and is but a conduit for its
principal. The control must be shown to have been exercised at the time the acts complained of took place. Moreover,
the control and breach of duty must proximately cause the injury or unjust loss for which the complaint is made.
- Finally, the Supreme Court emphasized that the piercing of the veil of corporate fiction is frowned upon and thus,
must be done with caution. It can only be done if it has been clearly established that the separate and distinct
personality of the corporation is used to justify a wrong, protect fraud, or perpetrate a deception. The court must be
certain that the corporate fiction was misused to such an extent that injustice, fraud, or crime was committed against
another, in disregard of its rights it cannot be presumed.

17
3B-Corporation Law

Topic:
Definitions and Attributes of Corporation
Subtopic:
Doctrine of Piercing the Veil of Corporate Fiction
Provision:
Case Name:
Lanuza Jr. v BF Corporation
Name: Glenn Nio M. Sartillo
Topic: Doctrine of Piercing the Veil of Corporate Fiction
Name of Case: Lanuza, Jr. vs. BF Corporation (GR No. 174938, October 1, 2014)
Corporation Code Provision: Section 2
FACTS:
BF Corporation and Shangri-La entered into a contract, wherein BF Corporation will construct the Shangri-La mall and
parking along EDSA. It provided for arbitration in the event of a dispute, and that such award by the recognized arbiter
named therein will be binding to the parties.
Shangri-La at first was paying the fees diligently, but eventually started to renege on the obligation. BF Corporation
continued to build the mall and parking building until it was completed. Shangri-La proceeded to take control of the
mall and building, while BF Corporation alleged that it remains unpaid to this day.
BF Corp filed a suit against Shangri-La and its directors. Shangri-La and the directors filed a motion to suspend
proceedings, alleging that the arbitration was not done yet. RTC denied. CA reversed, and ordered the submission of
the dispute to arbitration. SC affirmed.
BF Corp and Shangri-La initiated arbitration proceedings. RTC issued the order directing the parties to proceed in
accordance with RA 876. Shangri-La and BF Corp filed motions for clarification, to clarify the parties referred to in the
order, and whether Shangri-Las directors should be included in the arbitration.
ISSUE: Whether or not the directors can be compelled to submit to the arbitration clause in the contract, despite
being non-parties to the contract between BF Corp and Shangri-La
HELD:
Yes.
A consequence of a corporations separate personality is that consent by a corporation through its
representatives is not consent of the representative, personally. Its obligations, incurred through official acts of its
representatives, are its own. Hence, a corporations representatives are generally not bound by the terms of the
contract executed by the corporation.
The rule is settled that a corporation has a personality separate and distinct from the persons acting for and in its
behalf and, in general, from the people comprising it. The doctrine of piercing the corporate veil applies only in three
(3) basic instances, namely: when the separate and distinct corporate personality defeats public convenience; in fraud
cases, and when used in alter ego cases.
When there are allegations of bad faith or malice against corporate directors or representatives, it becomes the duty of
courts or tribunals to determine if these persons and the corporation should be treated as one. Without a trial, courts
and tribunals have no basis for determining whether the veil of corporate fiction should be pierced.
Hence, when the directors, as in this case, are impleaded in a case against a corporation, alleging malice or bad faith
on their part in directing the affairs of the corporation, complainants are effectively alleging that the directors and the
corporation are not acting as separate entities. In that case, complainants have no choice but to institute only one
proceeding against the parties. Under the Rules of Court, filing of multiple suits for a single cause of action is
prohibited.
In cases alleging solidary liability with the corporation or praying for the piercing of the corporate veil, parties who are
normally treated as distinct individuals should be made to participate in the arbitration proceedings in order to
determine if such distinction should indeed be disregarded and, if so, to determine the extent of their liabilities.

18
3B-Corporation Law

Topic:
Subtopic:
Provision:
Case Name:

Definitions and Attributes of Corporation


Rules of Attribution
Lanuza Jr. v BF Corporation

19
3B-Corporation Law

Topic:
Subtopic:
Provision:
Case Name:

Definitions and Attributes of Corporation


Rules of Attribution
Ellice Agro-Industrial Corp v Rodel Young

Rules of Attribution
Ellice Agro-Industrial Corp Vs. Rodel T. Young (G.R. No. 174077 November 21, 2012)

Private respondent and Petitioner through the petitioners alleged corporate secretary and attorney-in-fact
Domingo, entered to a contract to sell the respondents parcel of land located in Quezon.
Pursuant to the contract to sell, respondent through Domingo made a partial payment for the acquisition of the
subject property.
Despite such payment, Petitioner failed to deliver the owners duplicate certificate of title of the subject
property and the corresponding deed of sale.
Respondent filed a complaint for specific performance against Domingo and the Petitioner with the RTC
While the case was pending Respondent Wee sent a letter to petitioners chairman and president, seeking a
conference for the execution of an absolute deed of sale pursuant to the contract to sell entered between the
parties.
In response, the petitioners counsel sent a letter to Respondent Wee, informing him that Domingo lack the
authority to represent the petitioner.
Due to Petitioners failure to appear on the Pre-trial, the respondent was allowed to present their evidence ex
parte.
RTC rendered a decision ordering petitioner to deliver the owners duplicate copy and execute a deed of sale in
favor of the respondent. The decision became final and executor.
Petitioner represented by Gala filed a petition for relief from judgment premised on the alleged fraud
committed by Domingo in concealing the existence of both the contract to sell and the civil case from the
petitioner.
RTC denied the petition for relief from judgment for being filed out of time
A Petition for Annulment of judgment was initiated before the CA by the petitioner on the ground of lack of
jurisdiction and intrinsic fraud committed by Domingo, asserting that Domingo was not its President, Manager,
Secretary, Cashier, Agent or Director
CA dismissed the petition for annulment of judgment

Issue: W/N the action of Doming who represented as the Petitioners Corporate Secretary on the Civil Case for Specific
Performance is attributable to the Petitioner?
Ruling: No the action of Domingo cannot be imputed/attributed to the Petitioner

Domingo was not the corporate secretary of the petitioner as she claimed to be.
Domingos lack of authority to properly represent Petitioner resulted to a no valid service of summons binding
the petitioner.
Domingo was not an officer of the Petitioner, much less duly authorized by any board resolution or secretarys
certificate from petitioner to file an answer with counterclaim in behalf of the petitioner.
In view of this petitioner was not validly served with summons and did not voluntarily appear in the civil case.
The RTC did not validly acquire jurisdiction over the person of the Petitioner, Thus the decision by the RTC was
null and void.

20
3B-Corporation Law

Topic:
Subtopic:
Provision:
Case Name:

Definitions and Attributes of Corporation


Rules of Attribution
Trinidad J. Francisco v GSIS

Rules of Attribution
Trinidad Francisco VS. GSIS (G.R. No. L-18287, March 30, 1963)

Trinidad Francisco obtained a P400k loan from the Government Service Insurance System (GSIS). She secured
the loan with a parcel of land.

Due to some difficulties, Trinidad incurred huge arrears. This prompted GSIS to foreclose the property.

Trinidads father, Atty. Vicente Francisco, wrote a letter to GSIS offering that he pay 30,000 Pesos off the loan
and allow GSIS to administer the mortgaged property instead of foreclosing it and GSIS shall receive rents from
the tenants of the land until the arrears are paid and the account is made current or up to date

GSIS, through its general manager Rodolfo Andal, responded with a letter which states that the GSIS Board had
accepted Vicentes offer.

GSIS did not take over the property but the Franciscos still collected rents and turned them over to GSIS.

GSIS demanded Francisco to pay off the loan.

Vicente Francisco assailed with GSIS the agreement in a form of compromise binding upon GSIS.

GSIS averred that the letter sent to Vicente Francisco in response to his offer was poorly worded and was not
the actual intention of GSIS.

ISSUE: W/N GSIS is bound by the acts of its officers acting in their apparent authority?
HELD: Yes, GSIS is bounded by the compromised entered by its officer.

A third party transacting with a corporation cannot be expected to know what occurs within a corporation, its
meetings, without any external manifestations from the corporation.

In this case response by GSIS to Vicente Sotto by way of a telegram, is within the apparent authority of Andal.

Vicente Francisco cannot therefore be faulted for relying on the telegramthat GSIS accepted his offer.

GSIS cannot now ask Francisco to suddenly pay off the debt. If a corporation knowingly permits one of its
officers, or any other agent, to do acts within the scope of an apparent authorityand thus holds him out to the
public as possessing power to do those acts, the corporation will, as against anyone who has in good faith
dealt with the corporation through such agent, be estopped from denying his authority; and where it is said if
the corporation permits this means the same as if the thing is permitted by the directing power of the
corporation.

GSIS cannot also deny that it has knowledge of the acceptance. A corporation cannot see, or know, anything
except through its officers.

Knowledge of facts acquired or possessed by an officer or agent of a corporation in the course of his
employment, and in relation to matters within the scope of his authority, is notice to the corporation, whether
he communicates such knowledge or not.

21
3B-Corporation Law

Andal is presumed to have knowledge of the acceptance because it was his office which sent it to Vicente.
Knowledge of Andal, an officer of GSIS, is deemed knowledge of GSIS.

Even if the compromise agreement is void, GSISs silence and acceptance of the subsequent remittances of
the Franciscos ratified the compromise agreement.

Topic:
Subtopic:
Provision:
Case Name:

Definitions and Attributes of Corporation


Nationality, Citizenship and Foreign Equity (Place of Incorporation Test, Control
Test, and Grandfather Rule)
Filipinas Compania De Seguros v Christern, Huenefeld and Co

FACTS:

Christern Huenefeld Corporation bought a fire insurance policy from Filipinas Compania de Seguros to cover
merchandise contained in a building. During the Japanese military occupation, this same merchandise and the building
were burned, so Huenefeld filed a claim under the policy.
However, Filipinas Compania de Seguros refused to pay alleging that the policy had ceased to be in force when the
United States declared war against Germany. Filipinas Compania contended that although organized and created under
Philippine laws, Huenefeld is a German subject, and hence, a public enemy, since majority of its stockholders are
Germans. On the other hand, Filipinas Compania is under American jurisdiction.
The Director of Bureau of Financing, Philippine Executive Commission ordered Filipinas Compania to pay, so Filipinas
Compania did pay. The case at bar is about the recovery of that sum paid.
ISSUES:
1.
2.

Whether
Whether
the

or
fire

not
Christern
insurance
policy

Huenefeld
is
is
enforceable

a
against

German
an
enemy

subject.
state.

HELD:
1. There is no question that majority of the stockholders of the respondent corporation were German subjects. This
being so, we have to rule that said respondent became an enemy corporation upon the outbreak of the war between
the United States and Germany.
2. The Philippine Insurance Law (Act No. 2427, as amended,) in section 8, provides that "anyone except a public
enemy may be insured." It stands to reason that an insurance policy ceases to be allowable as soon as an insured
becomes a public enemy.
The respondent having become an enemy corporation on December 10, 1941, the insurance policy issued in its favor
on October 1, 1941, by the petitioner (a Philippine corporation) had ceased to be valid and enforcible, and since the
insured goods were burned after December 10, 1941, and during the war, the respondent was not entitled to any
indemnity under said policy from the petitioner. However, elementary rules of justice (in the absence of specific
provision in the Insurance Law) require that the premium paid by the respondent for the period covered by its policy
from December 11, 1941, should be returned by the petitioner.

22
3B-Corporation Law

23
3B-Corporation Law

Topic:
Subtopic:
Provision:
Case Name:

Definitions and Attributes of Corporation


Nationality, Citizenship and Foreign Equity (Place of Incorporation Test, Control
Test, and Grandfather Rule)
Heirs of Wilson Gamboa v Teves

FACTS:
This is a petition to nullify the sale of shares of stock of Philippine Telecommunications Investment Corporation (PTIC)
by the government of the Republic of the Philippines, acting through the Inter-Agency Privatization Council (IPC), to
Metro Pacific Assets Holdings, Inc. (MPAH), an affiliate of First Pacific Company Limited (First Pacific), a Hong Kongbased investment management and holding company and a shareholder of the Philippine Long Distance Telephone
Company (PLDT).
The petitioner questioned the sale on the ground that it also involved an indirect sale of 12 million shares (or about 6.3
percent of the outstanding common shares) of PLDT owned by PTIC to First Pacific. With the this sale, First Pacifics
common shareholdings in PLDT increased from 30.7 percent to 37 percent, thereby increasing the total common
shareholdings of foreigners in PLDT to about 81.47%. This, according to the petitioner, violates Section 11, Article XII of
the 1987 Philippine Constitution which limits foreign ownership of the capital of a public utility to not more than 40%,
thus:
Section 11. No franchise, certificate, or any other form of authorization for the operation of a public utility shall be
granted except to citizens of the Philippines or to corporations or associations organized under the laws of the
Philippines, at least sixty per centum of whose capital is owned by such citizens; x x x
ISSUE:
Whether or not PLDT is a Filipino corporation compliant with the requirements of the Consitution based on its
composition.
RULING:
NO. Section 11, Article XII (National Economy and Patrimony) of the 1987 Constitution mandates the Filipinization of
public utilities, to wit: Section 11. No franchise, certificate, or any other form of authorization for the operation of a
public utility shall be granted except to citizens of the Philippines or to corporations or associations organized under
the laws of the Philippines, at least sixty per centum of whose capital is owned by such citizens; nor shall such
franchise, certificate, or authorization be exclusive in character or for a longer period than fifty years. Neither shall any
such franchise or right be granted except under the condition that it shall be subject to amendment, alteration, or
repeal by the Congress when the common good so requires. The State shall encourage equity participation in public
utilities by the general public. The participation of foreign investors in the governing body of any public utility
enterprise shall be limited to their proportionate share in its capital, and all the executive and managing officers of
such corporation or association must be citizens of the Philippines.
It was proven that: (1) foreigners own 64.27% of the common shares of PLDT, which class of shares exercises
the sole right to vote in the election of directors, and thus exercise control over PLDT; (2) Filipinos own only 35.73% of
PLDTs common shares, constituting a minority of the voting stock, and thus do not exercise control over PLDT; (3)
preferred shares, 99.44% owned by Filipinos, have no voting rights; (4) preferred shares earn only 1/70 of the
dividends that common shares earn; (5) preferred shares have twice the par value of common shares; and (6)
preferred shares constitute 77.85% of the authorized capital stock of PLDT and common shares only 22.15%. This kind
of ownership and control of a public utility is a mockery of the Constitution.
It must be stressed, and respondents do not dispute, that foreigners hold a majority of the common shares of PLDT. In
fact, based on PLDTs 2010 General Information Sheet (GIS), which is a document required to be submitted annually to
the Securities and Exchange Commission, foreigners hold 120,046,690 common shares of PLDT whereas Filipinos hold
only 66,750,622 common shares. In other words, foreigners hold 64.27% of the total number of PLDTs common
shares, while Filipinos hold only 35.73%. Since holding a majority of the common shares equates to control, it is clear
that foreigners exercise control over PLDT. Such amount of control unmistakably exceeds the allowable 40 percent limit
on foreign ownership of public utilities expressly mandated in Section 11, Article XII of the Constitution.
As shown in PLDTs 2010 GIS, as submitted to the SEC, the par value of PLDT common shares is P5.00 per share,
whereas the par value of preferred shares is P10.00 per share. In other words, preferred shares have twice the par
value of common shares but cannot elect directors and have only 1/70 of the dividends of common shares. Moreover,
99.44% of the preferred shares are owned by Filipinos while foreigners own only a minuscule 0.56% of the preferred

24
3B-Corporation Law

shares. Worse, preferred shares constitute 77.85% of the authorized capital stock of PLDT while common shares
constitute only 22.15%. This undeniably shows that beneficial interest in PLDT is not with the non-voting preferred
shares but with the common shares, blatantly violating the constitutional requirement of 60 percent Filipino control
and Filipino beneficial ownership in a public utility.
The legal and beneficial ownership of 60 percent of the outstanding capital stock must rest in the hands of Filipinos in
accordance with the constitutional mandate. Full beneficial ownership of 60 percent of the outstanding capital stock,
coupled with 60 percent of the voting rights, is constitutionally required for the States grant of authority to operate a
public utility. The undisputed fact that the PLDT preferred shares, 99.44% owned by Filipinos, are non-voting and earn
only 1/70 of the dividends that PLDT common shares earn, grossly violates the constitutional requirement of 60
percent Filipino control and Filipino beneficial ownership of a public utility.
In short, Filipinos hold less than 60 percent of the voting stock, and earn less than 60 percent of the dividends, of
PLDT. This directly contravenes the express command in Section 11, Article XII of the Constitution that [n]o franchise,
certificate, or any other form of authorization for the operation of a public utility shall be granted except to x x x
corporations x x x organized under the laws of the Philippines, at least sixty per centum of whose capital is owned by
such citizens x x x.

25
3B-Corporation Law

Topic:
Subtopic:

Definitions and Attributes of Corporation


Nationality, Citizenship and Foreign Equity (Place of Incorporation Test, Control
Test, and Grandfather Rule)

Provision:
Case Name:
Heirs of Wilson Gamboa v Teves (Ruling only, 2012)
Name: Glenn Nio M. Sartillo
Topic: Nationality, Citizenship and Foreign Equity
Name of Case: Heirs of Wilson Gamboa vs. Teves, et al. (GR No. 176579, 9 October 2012)
Corporation Code Provision: Sections 6 and 17 (4)
FACTS:
This resolves the motions for reconsideration of the 28 June 2011 Decision in the case Heirs of Wilson Gamboa vs.
Teves, et al., dated 28 June 2011.
ISSUE:
WHETHER OR NOT BOTH THE VOTING CONTROL TEST AND THE BENEFICIAL OWNERSHIP TEST MUST BE APPLIED TO
DETERMINE WHETHER A CORPORATION IS A "PHILIPPINE NATIONAL."
RULING:
Yes.
The Constitution expressly declares as State policy the development of an economy "effectively controlled" by
Filipinos. Consistent with such State policy, the Constitution explicitly reserves the ownership and operation of public
utilities to Philippine nationals, who are defined in the Foreign Investments Act of 1991 as Filipino citizens, or
corporations or associations at least 60 percent of whose capital with voting rights belongs to Filipinos. The FIAs
implementing rules explain that "[f]or stocks to be deemed owned and held by Philippine citizens or Philippine
nationals, mere legal title is not enough to meet the required Filipino equity. Full beneficial ownership of the
stocks, coupled with appropriate voting rights is essential." In effect, the FIA clarifies, reiterates and confirms
the interpretation that the term "capital" in Section 11, Article XII of the 1987 Constitution refers to shares with
voting rights, as well as with full beneficial ownership. This is precisely because the right to vote in the election
of directors, coupled with full beneficial ownership of stocks, translates to effective control of a corporation.
Any other construction of the term "capital" in Section 11, Article XII of the Constitution contravenes the letter and
intent of the Constitution. Any other meaning of the term "capital" openly invites alien domination of economic
activities reserved exclusively to Philippine nationals. Therefore, respondents interpretation will ultimately result in
handing over effective control of our national economy to foreigners in patent violation of the Constitution, making
Filipinos second-class citizens in their own country.

26
3B-Corporation Law

Topic:
Subtopic:
Provision:
Case Name:

Definitions and Attributes of Corporation


Tort Liability and Doctrine of Corporate Responsibility
Professional Health Services Inc. v CA

Professional health Services, Inc. vs. CA


Facts:
PSI, together with Dr. Miguel Ampil (Dr. Ampil) and Dr. Juan Fuentes (Dr. Fuentes), was impleaded by Enrique Agana
and Natividad Agana (later substituted by her heirs), in a complaintfor damages filed in the Regional Trial Court (RTC)
of Quezon City, Branch 96, for the injuries suffered by Natividad when Dr. Ampil and Dr. Fuentes neglected to remove
from her body two gauzeswhich were used in the surgery they performed on her on April 11, 1984 at the Medical City
General Hospital. PSI was impleaded as owner, operator and manager of the hospital.
RTC held PSI solidarily liable with Dr. Ampil and Dr. Fuentes for damages. Thus, the appeal
Issue: Whether or not PSI can be held solidarily liable with Drs. Ampil and Dr. Fuentes
Held: Petition dismissed. PSI is solidarily liable.
Ratio:
PSI as owner, operator and manager of Medical City Hospital, "did not perform the necessary supervision nor exercise
diligent efforts in the supervision of Drs. Ampil and Fuentes and its nursing staff, resident doctors, and medical interns
who assisted Drs. Ampil and Fuentes in the performance of their duties as surgeons.
Recent years have seen the doctrine of corporate negligence as the judicial answer to the problem of allocating
hospitals liability for the negligent acts of health practitioners, absent facts to support the application of responde at
superior or apparent authority. Its formulation proceeds from the judiciarys acknowledgment that in these modern
times, the duty of providing quality medical service is no longer the sole prerogative and responsibility of the
physician. The modern hospitals have changed structure. Hospitals now tend to organize a highly professional medical
staff whose competence and performance need to be monitored by the hospitals commensurate with their inherent
responsibility to provide quality medical care.
In the present case, it was duly established that PSI operates the Medical City Hospital for the purpose and under the
concept of providing comprehensive medical services to the public. Accordingly, it has the duty to exercise reasonable
care to protect from harm all patients admitted into its facility for medical treatment. Unfortunately, PSI failed to
perform such duty.
PSIs liability is traceable to its failure to conduct an investigation of the matter reported in the nota bene of the count
nurse. Such failure established PSIs part in the dark conspiracy of silence and concealment about the gauzes. Ethical
considerations, if not also legal, dictated the holding of an immediate inquiry into the events, if not for the benefit of
the patient to whom the duty is primarily owed, then in the interest of arriving at the truth. The Court cannot accept
that the medical and the healing professions, through their members like defendant surgeons, and their institutions
like PSIs hospital facility, can callously turn their backs on and disregard even a mere probability of mistake or
negligence by refusing or failing to investigate a report of such seriousness as the one in Natividads case.

Topic:
Subtopic:
Provision:
Case Name:
UP vs. Dizon

Definitions and Attributes of Corporation


Right to Moral Damages
University of the Philippines v Dizon

Facts:

27
3B-Corporation Law

UP entered into a General Construction Agreement with respondent Stern Builders Corporation for the construction of
the extension building and the renovation of the College of Arts and Sciences Building the campus of UP in Los Banos.
In the course of the implementation of the contract, Stern Builders submitted three progress billings corresponding to
the work accomplished, but the UP paid only two of the billings. The third billing was not paid due to its disallowance
by the COA. Despite the lifting of the disallowance, the UP failed to pay the billing, prompting Stern Builders to sue the
UP and officials to collect the unpaid billing and to recover various damages.
Meanwhile, the sheriff served notices of garnishment on the UPs depository banks, namely: LDP and DBP. The UP
assailed said garnishment of funds. Stern Builders and dela Cruz, meanwhile, again sought the release of garnished
funds.
Issue: Whether or not Stern is entitled to moral damages.
Held: Petition granted.
Ratio:
UP correctly submits here that the garnishments of its funds to satisfy the judgment awards of actual and moral
damages was not validly made if there was no special appropriation by Congress to cover the liability.
The CA and RTC thereby unjustifiably ignored the legal restriction imposed on the trust funds of the Government and
its agencies and instrumentalities to be used exclusively to fulfill the purposes for which the trusts were created or for
which the funds were received except upon express authorization by Congress or by the head of a government agency
in control of funds, and subject to pertinent budgetary laws, rules and regulations.

28
3B-Corporation Law

Topic:
Subtopic:
Provision:
Case Name:

Definitions and Attributes of Corporation


Right to Moral Damages
ABS-CBN Broadcasting Corp v CA

ABS-CBN Broadcasting Corp v. CA


FACTS:

Viva, through Del Rosario, offered ABS-CBN through its vice-president Charo Santos-Concio, a list of 3 film
packages or 36 titles from which ABS-CBN may exercise its right of first refusal

Mrs. Concio informed Vic through a letter that they can only purchase 10 titles to be schedules on nonprimetime slots because they were very adult themes which the ruling of the MTRCB advises to be aired at
9:00 p.m

February 27, 1992: Del Rosario approached ABS-CBN's Ms. Concio with a list consisting of 52 original movie
titles as well as 104 re-runs proposing to sell to ABS-CBN airing rights for P60M (P30M cash and P30M worth of
television spots)

April 2, 1992: Del Rosario and ABS-CBN general manager, Eugenio Lopez III met wherein Del Rosario allegedly
agreed to grant rights for 14 films for P30M

April 06, 1992: Del Rosario and Mr. Graciano Gozon of RBS Senior vice-president for Finance discussed the
terms and conditions of Viva's offer to sell the 104 films, after the rejection of the same package by ABS-CBN

April 07, 1992: Ms. Concio sent the proposal draft of 53 films for P35M which Viva's Board rejected since they
will not accept anything less than P60M

April 29, 1992: Viva granted RBS exclusive grants for P60M

RTC: Issued TRO against RBS in showing 14 films as filed by ABS-CBN.


o

RBS also set up a cross-claim against VIVA

RTC: ordered ABS-CBN to pay RBS P107,727 premium paid by RBS to the surety which issued their bond to lift
the injunction, P191,843.00 for the amount of print advertisement for "Maging Sino Ka Man" in various
newspapers, P1M attorney's fees, P5M moral damages, P5M exemplary damages and costs. Cross-claim to
VIVA was dismissed.

ABS-CBN appealed. VIVA and Del Rosario also appealed seeking moral and exemplary damages and additional
attorney's fees.

CA: reduced the awards of moral damages to P2M, exemplary damages to P2M and attorney's fees to
P500,000. Denied VIVA and Del Rosario's appeal because it was RBS and not VIVA which was actually
prejudiced when the complaint was filed by ABS-CBN

ISSUE:
1. W/N RBS is entitled to damages. -YES
2. W/N VIVA is entitled to damages. - NO
HELD: REVERSED except as to unappealed award of attorney's fees in favor of VIVA Productions, Inc.
1. YES.

One is entitled to compensation for actual damages only for such pecuniary loss suffered by him as he has
duly proved. The indemnification shall comprehend not only the value of the loss suffered, but also that of the

29
3B-Corporation Law

profits that the obligee failed to obtain. In contracts and quasi-contracts the damages which may be awarded
are dependent on whether the obligor acted with good faith or otherwise, It case of good faith, the damages
recoverable are those which are the natural and probable consequences of the breach of the obligation and
which the parties have foreseen or could have reasonably foreseen at the time of the constitution of the
obligation. If the obligor acted with fraud, bad faith, malice, or wanton attitude, he shall be responsible for all
damages which may be reasonably attributed to the non-performance of the obligation. In crimes and quasidelicts, the defendant shall be liable for all damages which are the natural and probable consequences of the
act or omission complained of, whether or not such damages has been foreseen or could have reasonably
been foreseen by the defendant. Actual damages may likewise be recovered for loss or impairment of earning
capacity in cases of temporary or permanent personal injury, or for injury to the plaintiff's business standing or
commercial credit.

The claim of RBS for actual damages did not arise from contract, quasi-contract, delict, or quasi-delict. It arose
from the fact of filing of the complaint despite ABS-CBN's alleged knowledge of lack of cause of action.
Needless to state the award of actual damages cannot be comprehended under the above law on actual
damages. RBS could only probably take refuge under Articles 19, 20, and 21 of the Civil Code.

In this case, ABS-CBN had not yet filed the required bond; as a matter of fact, it asked for reduction of the bond
and even went to the Court of Appeals to challenge the order on the matter, Clearly then, it was not necessary
for RBS to file a counterbond. Hence, ABS-CBN cannot be held responsible for the premium RBS paid for the
counterbond

Neither could ABS-CBN be liable for the print advertisements for "Maging Sino Ka Man" for lack of sufficient
legal basis.

Article 2217 thereof defines what are included in moral damages, while Article 2219 enumerates the cases
where they may be recovered, Article 2220 provides that moral damages may be recovered in breaches of
contract where the defendant acted fraudulently or in bad faith. RBS's claim for moral damages could possibly
fall only under item (10) of Article 2219
o

(10) Acts and actions referred to in Articles 21, 26, 27, 28, 29, 30, 32, 34, and 35.

The award of moral damages cannot be granted in favor of a corporation because, being an artificial person
and having existence only in legal contemplation, it has no feelings, no emotions, no senses, It cannot,
therefore, experience physical suffering and mental anguish, which call be experienced only by one having a
nervous system. A corporation may recover moral damages if it "has a good reputation that is debased,
resulting in social humiliation" is an obiter dictum. On this score alone the award for damages must be set
aside, since RBS is a corporation.

exemplary damages are imposed by way of example or correction for the public good, in addition to moral,
temperate, liquidated or compensatory damages. They are recoverable in criminal cases as part of the civil
liability when the crime was committed with one or more aggravating circumstances in quasi-contracts, if the
defendant acted with gross negligence and in contracts and quasi-contracts, if the defendant acted in a
wanton, fraudulent, reckless, oppressive, or malevolent manner

It may be reiterated that the claim of RBS against ABS-CBN is not based on contract, quasi-contract, delict, or
quasi-delict, Hence, the claims for moral and exemplary damages can only be based on Articles 19, 20, and 21
of the Civil Code.

There is no adequate proof that ABS-CBN was inspired by malice or bad faith. If damages result from a
person's exercise of a right, it is damnum absque injuria.

30
3B-Corporation Law

Topic:
Subtopic:
Provision:
Case Name:

Definitions and Attributes of Corporation


Constitutional Rights
Bataan Shipyard and Engineering Co v PCGG

Facts:
President Corazon Aquino promulgated Executive Orders No. 1 and 2 addressed to the PCGG to sequester among
others, the properties and assets of petitioner BASECO. The PCGG took over BASECO, it terminated existing contracts
and ordered the officers to produce books and documents of the corporation. BASECO on the other hand alleged that
the sequestration without resorting to judicial action, might be made within the context of Executive Orders Nos. 1 and
2 before March 25, 1986 when the Freedom Constitution was promulgated however considering that the new
Constitution provides that "No person shall be deprived of life, liberty and property without due process of law."
(Const., Art. I V, Sec. 1)." Hence, it assailed the Constitutionality of the alleged Executive Orders, the Sequestration
and Takeover Order because First, no notice and hearing was accorded before its properties and business were taken
over; Second, the PCGG is not a court, but a purely investigative agency and therefore not competent to act as
prosecutor and judge in the same cause; Third, there is nothing in the issuances which envisions any proceeding,
process or remedy by which petitioner may expeditiously challenge the validity of the takeover after the same has
been effected. It also asserted that by ordering the turnover of records and books, their right against selfincrimination was violated.
Issue: WON the acts of the PCGG infringe the Constitutional rights of BASECO.
Held:
It is elementary that the right against self-incrimination has no application to juridical persons. While an
individual may lawfully refuse to answer incriminating questions unless protected by an immunity statute, it does not
follow that a corporation, vested with special privileges and franchises, may refuse to show its hand when charged
with an abuse of such privileges. Corporations are not entitled to all of the constitutional protections which private
individuals have. It is also settled that an officer of the company cannot refuse to produce its records in its possession
upon the plea that they will either incriminate him or may incriminate it." The corporation is a creature of the state.
There is a reserve right in the legislature to investigate its contracts and find out whether it has exceeded its powers. It
would be a strange anomaly to hold that a state, having chartered a corporation to make use of certain franchises,
could not, in the exercise of sovereignty, inquire how these franchises had been employed, and whether they had been
abused, and demand the production of the corporate books and papers for that purpose. Hence, PCGG did not infringe
any constitutional right of BASECO.

31
3B-Corporation Law

Topic:
Subtopic:
Provision:
Case Name:
Facts:

Definitions and Attributes of Corporation


Constitutional Rights
Bache and Co v Ruiz

Respondent Misael P. Vera, Commissioner of Internal Revenue, requested respondent Judge Vivencio M. Ruiz for the
issuance of a search warrant against bache & Co. et al. alleging that the corporation violated several provisions of the
Internal Revenue Code. The warrant was granted by the Judge even though he did not personally take the depositions
of the respondent-complainant. BIR agents served the search warrant at the offices of petitioner corporation.
Petitioners lawyers protested but the agents nevertheless proceeded with their search which yielded six boxes of
documents. The BIR then made tax assessments based on the said documents. Bache & Co filed a petition to nullify
the said warrant on the ground that their Constitutional right against unreasonable searches was violated.
Respondents on the other hand assert that a Corporation is not entitled to protection against unreasonable search and
seizures. Again, we find no merit in the contention.
Issue: WON the Constitutional Right of Bache & Co. was violated.
Held:
In Linn v. United States, 163 C.C.A. 470, 251 Fed. 476, 480, it was thought that a different rule applied to a corporation,
the ground that it was not privileged from producing its books and papers. But the rights of a corporation against
unlawful search and seizure are to be protected even if the same result might have been achieved in a lawful way. The
Supreme Court has impliedly recognized the he right of a corporation to object against unreasonable searches and
seizures in the case of Stonehill, Et. Al. v. Diokno, Et Al. In the case at bar, the corporation to whom the seized
documents belong, and whose rights have thereby been impaired, is itself a petitioner. On that score, petitioner
corporation here stands on a different footing from the corporations in Stonehill where only the officers of the various
corporations in whose offices documents, papers and effects were searched and seized. Hence, yes the Corporations
Constitutional Right against Unreasonable Searches was violated.

32
3B-Corporation Law

Topic:
Subtopic:
Provision:
Case Name:
FACTS:

Definitions and Attributes of Corporation


Constitutional Rights
Stonehill v Diokno

In violation of Central Bank Laws, Tariff and Customs Laws, Internal Revenue (Code) and the Revised Penal
Code, 42 warrants were issued against petitioners or the corporation where they are officers to search the persons
above-named and/or the premises of their offices, warehouses and/or residences, and to seize and take possession
of their books of accounts, financial records, vouchers, correspondence, receipts, ledgers, journals, portfolios,
credit journals, typewriters, and other documents and/or papers showing all business transactions including
disbursements receipts, balance sheets and profit and loss statements and Bobbins (cigarette wrappers) which are
the subject of the offense.

Petitioners filed with the Supreme Court this original action for certiorari, prohibition, mandamus and
injunction, and prayed that, pending final disposition of the present case, a writ of preliminary injunction be issued
alleging the search warrants to be void since (1) they do not describe with particularity the documents, books and
things to be seized; (2) cash money, not mentioned in the warrants, were actually seized; (3) the warrants were
issued to fish evidence against the aforementioned petitioners in deportation cases filed against them; (4) the
searches and seizures were made in an illegal manner; and (5) the documents, papers and cash money seized

were not delivered to the courts that issued the warrants, to be disposed of in accordance with law
ISSUE: W/N the seizure is valid
HELD: YES. warrants for the search of 3 residences null and void; searches and seizures made are illegal; that the writ
of preliminary injunction issued

the documents, papers, and things seized under the alleged authority of the warrants in question may be split
into two (2) major groups, namely:

(a) those found and seized in the offices of the aforementioned corporations, and
have no cause of action to assail the legality of the contested warrants and of the seizures
made in pursuance thereof, for the simple reason that said corporations have their respective personalities,
separate and distinct from the personality of herein petitioners, regardless of the amount of shares of stock or of
the interest of each of them in said corporations, and whatever the offices they hold therein may be.

question of the lawfulness of a seizure can be raised only by one whose rights have been
invaded. Certainly, such a seizure, if unlawful, could not affect the constitutional rights of defendants whose
property had not been seized or the privacy of whose homes had not been disturbed

(b) those found and seized in the residences of petitioners herein.


2 points must be stressed in connection with this constitutional mandate, namely:
(1) that no warrant shall issue but upon probable cause, to be determined by the judge in the manner
set forth in said provision; and - not met

(2) that the warrant shall particularly describe the things to be seized. - not met

without reference to any determinate provision of said laws

the warrants authorized the search for and seizure of records pertaining to all business
transactions of petitioners herein, regardless of whether the transactions were legal or illegal.

33
3B-Corporation Law

To uphold the validity of the warrants in question would be to wipe out completely one of the most
fundamental rights guaranteed in our Constitution, for it would place the sanctity of the domicile and the privacy
of communication and correspondence at the mercy of the whims caprice or passion of peace officers.

Corporations are protected by Constitutional Rights as well such as the due process clause which was blatantly
disregarded in this case although there was no representative in this case of the corporation to assail such
violation.

34
3B-Corporation Law

Topic:
Definitions and Attributes of Corporation
Subtopic:
Criminal Liability
Provision:
Case Name:
Ching v Secretary of Justice
Criminal Liability
25) Ching v. Sec. of Justice
Facts:
Alfredo Ching was the Senior Vice President of Philippine Blooming Mills Inc. PBMi, through Ching, applied with
Rizal Commercial Banking Corporation, RCBC for issuance of commercial letters of credit. RCBC approved the
application. Goods were purchased and delivered in trust to PBMI. Ching signed 13 trust receipts as surety. Under the
receipts, Ching agreed to hold the goods in trust for the banks. In case the goods remained unsold within the specified
period, the goods were to be returned to the bank without need of any demand.
When the trust receipts matured, Ching failed to return the goods to RCBC despite demands. RCBC filed a
criminal complaint for Estafa against Ching. City Prosecutor found probable cause estafa. 13 Information of Estafa were
filed against Ching in RTC of Manila.
RTC ruled that there was no estafa. City prosecutor also ruled there was no probable cause. Secretary of Justice
reversed the ruling and assailed that Ching was responsible for the offense. Ching executed the trust receipts. The
execution of said receipts is enough to indict petitioner as official responsible for violation of PD 115. Ching bound
himself as corporate official of PBMI and also as its surety.
CA dismissed Chings petition on procedural grounds. CA stated that Ching failed to establish that Secretary of
Justice commited grave abuse of discretion in finding probable cause against petitioner for violation of estafa under
Article 315.
Issue:
Whether or not Ching is guilty of Estafa under PD No. 115?
Held:
The Court ruled that Ching is guilty of estafa. PD No. 115 is malum prohibitum but is classified as estafa under RPC
315. It may be committed by a corporation or other juridical entity or by natural persons.
If the crime is committed by a corporation or other juridical entity, the directors, officers, employees or other officers
thereof responsible for the offense shall be charged and penalized for the crime, precisely because of the nature of the
crime and the penalty therefor. A Corporation cannot be arrested and imprisoned hence cannot be penalized for a
crime punishable by imprisonment. However, a corporation may be charged and prosecuted for a crime if the
imposable penalty is fine. Even if the statute prescribes both fine and imprisonment as a penalty, a corporation may
be prosecuted and if found guilty may be fined.
In this case, Ching signed the trust receipt in question. He is the actual, present and efficient actor. He cannot hide
behind the cloak of the separate corporate personality of PBMI.

Topic:
Subtopic:
Provision:
Case Name:

Definitions and Attributes of Corporation


Criminal Liability
Arnel U. Ty v NBI Supervising Agent Marvin E. Jemil

35
3B-Corporation Law

FACTS:
Petitioners are stockholders of Omni Gas Corporation (Omni) as per Omnis General Information Sheet (GIS)
dated March 6, 2004 submitted to the Securities and Exchange Commission (SEC). Omni is in the business of trading
and refilling of Liquefied Petroleum Gas (LPG) cylinders and holds Pasig City Mayors Permit No. RET-04-001256 dated
February 3, 2004.
The case all started when Joaquin Guevara Adarlo & Caoile Law Offices (JGAC Law Offices) sent a letter dated
March 22, 2004 to the NBI requesting, on behalf of their clients Shellane Dealers Association, Inc., Petron Gasul Dealers
Association, Inc., and Totalgaz Dealers Association, Inc., for the surveillance, investigation, and apprehension of
persons or establishments in Pasig City that are engaged in alleged illegal trading of petroleum products and
underfilling of branded LPG cylinders in violation ofBatas Pambansa Blg. (BP) 33, as amended by Presidential Decree
No. (PD) 1865.
Earlier, the JGAC Law Offices was furnished by several petroleum producers/brand owners their respective
certifications on the dealers/plants authorized to refill their respective branded LPG cylinders, to wit: (1) On October 3,
2003, Pilipinas Shell Petroleum Corporation (Pilipinas Shell) issued a certification of the list of entities duly authorized
to refill Shellane LPG cylinders; (2) on December 4, 2003, Petron Corporation (Petron) issued a certification of their
dealers in Luzon, Visayas, and Mindanao authorized to refill Petron Gasul LPG cylinders; and (3) on January 5, 2004,
Total (Philippines) Corporation (Total) issued two certifications of the refilling stations and plants authorized to refill
their Totalgaz and Superkalan Gaz LPG cylinders.
Agents De Jemil and Kawada attested to conducting surveillance of Omni in the months of March and April
2004 and doing a test-buy on April 15, 2004. They brought eight branded LPG cylinders of Shellane, Petron
Gasul, Totalgaz, and Superkalan Gaz to Omni for refilling. The branded LPG cylinders were refilled, for which the
National Bureau of Investigation (NBI) agents paid PhP 1,582 as evidenced by Sales Invoice No. 90040 issued by Omni
on April 15, 2004. The refilled LPG cylinders were without LPG valve seals and one of the cylinders was actually
underfilled, as found by LPG Inspector Noel N. Navio of the Liquefied Petroleum Gas Industry Association (LPGIA) who
inspected the eight branded LPG cylinders on April 23, 2004 which were properly marked by the NBI after the test-buy.
The NBIs test-buy yielded positive results for violations of BP 33, Section 2(a) in relation to Secs. 3(c) and
4, i.e., refilling branded LPG cylinders without authority; and Sec. 2(c) in relation to Sec. 4, i.e., underdelivery or
underfilling of LPG cylinders. Thus, on April 28, 2004, Agent De Jemil filed an Application for Search Warrant (With
Request for Temporary Custody of the Seized Items) before the Regional Trial Court (RTC) in Pasig City, attaching,
among others, his affidavit and the affidavit of Edgardo C. Kawada, an NBI confidential agent.
On the same day of the filing of the application for search warrants on April 28, 2004, the RTC, Branch 167
in Pasig City issued Search Warrants No. 2624 and 2625. The NBI served the warrants the next day or on April 29,
2004 resulting in the seizure of several items from Omnis premises duly itemized in the NBIs Receipt/Inventory of
Property/Item Seized. On May 25, 2004, Agent De Jemil filed his Consolidated Return of Search Warrants with Ex-Parte
Motion to Retain Custody of the Seized Items before the RTC Pasig City.
Subsequently, Agent De Jemil filed before the Department of Justice (DOJ) his Complaint-Affidavits against
petitioners for: (1) Violation of Section 2(a), in relation to Sections 3(c) and 4, of B.P. Blg. 33, as amended by P.D.
1865; and (2) Violation of Section 2(c), in relation to Section 4, of B.P. Blg. 33, as amended by P.D. 1865, docketed as
I.S. Nos. 2004-616 and 2004-618, respectively.
During the preliminary investigation, petitioners submitted their Joint Counter-Affidavit, which was replied to by
Agent De Jemil with a corresponding rejoinder from petitioners.

ISSUE: WHETHER OR NOT PETITIONERS CAN BE HELD LIABLE UNDER BATAS PAMBANSA BLG. 33, AS AMENDED, FOR
BEING MERE DIRECTORS, NOT ACTUALLY IN CHARGE OF THE MANAGEMENT OF THE BUSINESS AFFAIRS OF THE
CORPORATION.

HELD:
YES.

36
3B-Corporation Law

Sec. 4 of BP 33, as amended, provides for the penalties and persons who are criminally liable, thus:

Sec. 4.
Penalties. Any person who commits any act herein prohibited shall, upon conviction, be
punished with a fine of not less than twenty thousand pesos (P20,000) but not more than fifty
thousand pesos (P50,000), or imprisonment of at least two (2) years but not more than five (5) years,
or both, in the discretion of the court. In cases of second and subsequent conviction under this Act, the
penalty shall be both fine and imprisonment as provided herein. Furthermore, the petroleum and/or
petroleum products, subject matter of the illegal trading, adulteration, shortselling, hoarding,
overpricing or misuse, shall be forfeited in favor of the Government: Provided, That if the petroleum
and/or petroleum products have already been delivered and paid for, the offended party shall be
indemnified twice the amount paid, and if the seller who has not yet delivered has been fully paid, the
price received shall be returned to the buyer with an additional amount equivalent to such price; and
in addition, if the offender is an oil company, marketer, distributor, refiller, dealer, sub-dealer and other
retail outlets, or hauler, the cancellation of his license.
Trials of cases arising from this Act shall be terminated within thirty (30) days after arraignment.
When the offender is a corporation, partnership, or other juridical person, the president,
the general manager, managing partner, or such other officer charged with the management
of the business affairs thereof, or employee responsible for the violation shall be criminally
liable; in case the offender is an alien, he shall be subject to deportation after serving the sentence.
If the offender is a government official or employee, he shall be perpetually disqualified from
office. (Emphasis supplied.)

Relying on the third paragraph of the above statutory proviso, petitioners argue that they cannot be held liable
for any perceived violations of BP 33, as amended, since they are mere directors of Omni who are not in charge of the
management of its business affairs. Reasoning that criminal liability is personal, liability attaches to a person from his
personal act or omission but not from the criminal act or negligence of another. Since Sec. 4 of BP 33, as amended,
clearly provides and enumerates who are criminally liable, which do not include members of the board of directors of a
corporation, petitioners, as mere members of the board of directors who are not in charge of Omnis business affairs,
maintain that they cannot be held liable for any perceived violations of BP 33, as amended. To bolster their position,
they attest to being full-time employees of various firms as shown by the Certificates of Employment they submitted
tending to show that they are neither involved in the day-to-day business of Omni nor managing it. Consequently, they
posit that even if BP 33, as amended, had been violated by Omni they cannot be held criminally liable thereof not
being in any way connected with the commission of the alleged violations, and, consequently, the criminal complaints
filed against them based solely on their being members of the board of directors as per the GIS submitted by Omni to
SEC are grossly discriminatory.

On this point, we agree with petitioners except as to petitioner Arnel U. Ty who is indisputably the President of
Omni.

It may be noted that Sec. 4 above enumerates the persons who may be held liable for violations of the law,
viz: (1) the president, (2) general manager, (3) managing partner, (4) such other officer charged with the management
of the business affairs of the corporation or juridical entity, or (5) the employee responsible for such violation. A
common thread of the first four enumerated officers is the fact that they manage the business affairs of the

37
3B-Corporation Law

corporation or juridical entity. In short, they are operating officers of a business concern, while the last in the list is selfexplanatory.

It is undisputed that petitioners are members of the board of directors of Omni at the time pertinent. There can
be no quibble that the enumeration of persons who may be held liable for corporate violators of BP 33, as amended,
excludes the members of the board of directors. This stands to reason for the board of directors of a corporation is
generally a policy making body. Even if the corporate powers of a corporation are reposed in the board of directors
under the first paragraph of Sec. 23 of the Corporation Code, it is of common knowledge and practice that the board of
directors is not directly engaged or charged with the running of the recurring business affairs of the
corporation.Depending on the powers granted to them by the Articles of Incorporation, the members of the board
generally do not concern themselves with the day-to-day affairs of the corporation, except those corporate officers
who are charged with running the business of the corporation and are concomitantly members of the board, like the
President. Section 25 of the Corporation Code requires the president of a corporation to be also a member of the board
of directors.

Thus, the application of the legal maxim expressio unius est exclusio alterius, which means the mention of one
thing implies the exclusion of another thing not mentioned.If a statute enumerates the thing upon which it is to
operate, everything else must necessarily and by implication be excluded from its operation and effect. The fourth
officer in the enumerated list is the catch-all such other officer charged with the management of the business affairs of
the corporation or juridical entity which is a factual issue which must be alleged and supported by evidence.
A scrutiny of the GIS reveals that among the petitioners who are members of the board of directors are the
following who are likewise elected as corporate officers of Omni: (1) Petitioner Arnel U. Ty (Arnel) as President; (2)
petitioner Mari Antonette Ty as Treasurer; and (3) petitioner Jason Ong as Corporate Secretary. Sec. 4 of BP 33, as
amended, clearly indicated firstly the president of a corporation or juridical entity to be criminally liable for violations
of BP 33, as amended.
Evidently, petitioner Arnel, as President, who manages the business affairs of Omni, can be held liable for
probable violations by Omni of BP 33, as amended. The fact that petitioner Arnel is ostensibly the operations manager
of Multi-Gas Corporation, a family owned business, does not deter him from managing Omni as well. It is well-settled
that where the language of the law is clear and unequivocal, it must be taken to mean exactly what it says. As to the
other petitioners, unless otherwise shown that they are situated under the catch-all such other officer charged with the
management of the business affairs, they may not be held liable under BP 33, as amended, for probable
violations.Consequently, with the exception of petitioner Arnel, the charges against other petitioners must perforce be
dismissed or dropped.

38
3B-Corporation Law

Topic:
Subtopic:
Provision:
Case Name:

Definitions and Attributes of Corporation


Classes of Corporation
Boy Scouts of the Philippines v COA

Facts:
COA issued Resolution No. 99-011 on Aug. 19, 1999 with the subject Defining the Commissions policy with
respect to the audit of the Boy Scouts of the Philippines. This resolution stated that the BSP was created as a public
corporation under Commonwealth Act No. 111, as amended by PD No. 460 and RA 7278; and in the case of BSP vs
NLRC the Supreme Court held that the BSP, under its charter, was a government controlled corporation within the
meaning of Art. IX (B) (2) (1) of the Constitution and that the BSP is appropriately regarded as a government
instrumentality under the 1987 Administrative Code.
BSP sought reconsideration from the COA resolution in a letter signed by the BSP president Jejomar C. Binay. It
was stated that BSP is not subject to COAs jurisdiction and the latest amendment to its charter (RA 7278) converted it
to a private corporation. It does not receive special funds from the government nor was there any appropriation of
special funds for it. Funds of BSP came from donations or membership fees. It cannot also be considered an agency
as it is only an attached-agency to DECS.
Issue:
WON BSP is considered a public corporation subject to COAs jurisdiction?
Held:
Yes.
BSPs charter (Commonwealth Act No. 111) entitled An Act to create a Public Corporation to be known as The Boy
Scouts of the Philippines and to define its powers and purposes created the BSP as a public corporation.
The amendment of RA 7278 only reduced the government representation in its governing body. And being an
attached-agency does not make it a private corporation.

39
3B-Corporation Law

Topic:
Subtopic:
Provision:
Case Name:

Definitions and Attributes of Corporation


Classes of Corporation
Republic of the Philippines v City of Paranaque

CLASSES OF CORPORATIONS (Sec. 3 & 4, Corp. Code; Sec. 2 par. 13, Administrative Code of 1987; Sec. 16, Art. XII,
1987 Constitution)
Republic of the Philippines vs City of Paranaque
FACTS:

This is a petition for review on certiorari assailing the Order of the Regional Trial Court, Branch 195, Paranaque
City (RTC), which ruled that petitioner Philippine Reclamation Authority (PRA) is a government-owned and
controlled corporation (GOCC), a taxable entity, and, therefore, not exempt from payment of real property
taxes.

The Public Estates Authority (PEA) is a government corporation created by virtue of P.D. No. 1084 to provide a
coordinated, economical and efficient reclamation of lands. EO 525 issued by Marcos designated PEA to be the
primary agency for coordinating all reclamation projects for the National Government. EO 380 issued by Arroyo
transformed PEA to PRA.

By virtue of its mandate, PRA reclaimed several portions of the foreshore and offshore areas of Manila Bay,
including those located in Paraaque City. Paraaque City Treasurer issued Warrants of Levy on PRAs
reclaimed properties based on the assessment for delinquent real property for tax years 2001 and 2002.
PRA asserted that:

It is not a GOCC under the Administrative Code, nor is it a GOCC under Section 16, Article XII of the 1987
Constitution because it is not required to meet the test of economic viability.

It is a government instrumentality vested with corporate powers and performing an essential public service.
Although it has a capital stock divided into shares, it may not be classified as a stock corporation because it
lacks the second requisite of a stock corporation: to distribute dividends and allotment of surplus and profits to
its stockholders.

It may not be classified as a non-stock corporation because it has no members and it is not organized for
charitable, religious, educational, professional, cultural, recreational, fraternal, literary, scientific, social, civil
service, or similar purposes, like trade, industry, agriculture and like chambers as provided in Section 88 of the
Corporation Code.

It was not created to compete in the market place as there was no competing reclamation company operated
by the private sector. Also, while PRA is vested with corporate powers under P.D. No. 1084, such circumstance
does not make it a corporation but merely an incorporated instrumentality and that the mere fact that an
incorporated instrumentality of the National Government holds title to real property does not make said
instrumentality a GOCC.
City of Paraaque (respondent) argued that:

PRA since its creation consistently represented itself to be a GOCC. PRAs very own charter (P.D. No. 1084)
declared it to be a GOCC and that it has entered into several thousands of contracts where it represented itself
to be a GOCC. In fact, PRA admitted in its original and amended petitions and pre-trial brief filed with the RTC
of Paraaque City that it was a GOCC.

It argues that PRA is a stock corporation with an authorized capital stock divided into 3 million no par value
shares, out of which 2 million shares have been subscribed and fully paid up. Section 193 of the LGC of 1991
has withdrawn tax exemption privileges granted to or presently enjoyed by all persons, whether natural or
juridical, including GOCCs.
ISSUE:
Whether or not petitioner is an incorporated instrumentality of the national government and is, therefore, exempt from
payment of real property tax under sections 234(a) and 133(o) of Republic Act 7160 or the Local Government Code vis-vis Manila International Airport Authority v. Court of Appeals.
HELD:
Yes it is a Government Instrumentality. However, it is not a GOCC. When the law vests in a government instrumentality
corporate powers, the instrumentality does not necessarily become a corporation. Unless the government
instrumentality is organized as a stock or non-stock corporation, it remains a government instrumentality exercising
not only governmental but also corporate powers.
Introductory Provisions of the Administrative Code of 1987 defines a GOCC as any agency organized as a stock or nonstock corporation, vested with functions relating to public needs whether governmental or proprietary in nature, and

40
3B-Corporation Law

owned by the Government directly or through its instrumentalities either wholly, or, where applicable as in the case of
stock corporations, to the extent of at least fifty-one (51) percent of its capital stock: x x x.
From the above definitions, it is clear that a GOCC must be "organized as a stock or non-stock corporation" while an
instrumentality is vested by law with corporate powers. Likewise, when the law makes a government instrumentality
operationally autonomous, the instrumentality remains part of the National Government machinery although not
integrated with the department framework.
The fundamental provision above authorizes Congress to create GOCCs through special charters on two conditions: 1)
the GOCC must be established for the common good; and 2) the GOCC must meet the test of economic viability. In this
case, PRA may have passed the first condition of common good but failed the second one - economic viability.
Undoubtedly, the purpose behind the creation of PRA was not for economic or commercial activities. Neither was it
created to compete in the market place considering that there were no other competing reclamation companies being
operated by the private sector.
Many government instrumentalities are vested with corporate powers but they do not become stock or non-stock
corporations, which is a necessary condition before an agency or instrumentality is deemed a GOCC. These
instrumentalities perform essential public services for the common good, services that every modern State must
provide its citizens. These instrumentalities need not be economically viable since the government may even subsidize
their entire operations. These instrumentalities are not the "government-owned or controlled corporations" referred to
in Section 16, Article XII of the 1987 Constitution.
The facts, the evidence on record and jurisprudence on the issue support the position that PRA was not organized
either as a stock or a non-stock corporation. Neither was it created by Congress to operate commercially and compete
in the private market. Instead, PRA is a government instrumentality vested with corporate powers and performing an
essential public service pursuant to Section 2(10) of the Introductory Provisions of the Administrative Code. Being an
incorporated government instrumentality, it is exempt from payment of real property tax.

41
3B-Corporation Law

Topic:
Subtopic:
Provision:
Case Name:

Definitions and Attributes of Corporation


Classes of Corporation
Collector of Internal Revenue v Club Filipino

CLASSES OF CORPORATIONS (Sec. 3 & 4, Corp. Code)


Collector of Internal Revenue vs Club Filipino
FACTS:

The Club Filipino, is a civic corporation organized under the laws of the Philippines with an original authorized
capital stock of P22,000, which was subsequently increased to P200,000 to operate and maintain a golf course,
tennis, gymnasiums, bowling alleys, billiard tables and pools, and all sorts of games not prohibited by general
laws and general ordinances, and develop and nurture sports of any kind and any denomination for recreation
and healthy training of its members and shareholders" (sec. 2, Escritura de Incorporacion (Deed of
Incorporation) del Club Filipino, Inc.). There is no provision either in the articles or in the by-laws relative to
dividends and their distribution, although it is covenanted that upon its dissolution, the Club's remaining
assets, after paying debts, shall be donated to a charitable Phil. Institution in Cebu (Art. 27, Estatutos del
(Statutes of the) Club).

The Club owns and operates a club house, a bowling alley, a golf course (on a lot leased from the government),
and a bar-restaurant where it sells wines and liquors, soft drinks, meals and short orders to its members and
their guests. The bar-restaurant was a necessary incident to the operation of the club and its golf-course. The
club is operated mainly with funds derived from membership fees and dues. Whatever profits it had, were used
to defray its overhead expenses and to improve its golf-course. In 1951, as a result of a capital surplus, arising
from the re-valuation of its real properties, the value or price of which increased, the Club declared stock
dividends; but no actual cash dividends were distributed to the stockholders.

In 1952, a BIR agent discovered that the Club has never paid percentage tax on the gross receipts of its bar
and restaurant, although it secured licenses. In a letter, the Collector assessed against and demanded from the
Club P12,068.84 as fixed and percentage taxes, surcharge and compromise penalty. Also, the Collector denied
the Clubs request to cancel the assessment.

On appeal, the CTA reversed the Collector and ruled that the Club is not liable for the assessed tax liabilities of
P12,068.84 allegedly due from it as a keeper of bar and restaurant as it is a non-stock corporation. Hence, the
Collector filed the instant petition for review.
ISSUE:
WON the Club is a stock corporation
HELD:
NO. It is a non-stock corporation.
The facts that the capital stock of the Club is divided into shares, does not detract from the finding of the trial court
that it is not engaged in the business of operator of bar and restaurant. What is determinative of whether or not
the Club is engaged in such business is its object or purpose, as stated in its articles and by-laws. The
actual purpose is not controlled by the corporate form or by the commercial aspect of the business prosecuted, but
may be shown by extrinsic evidence, including the by-laws and the method of operation. From the extrinsic evidence
adduced, the CTA concluded that the Club is not engaged in the business as a barkeeper and restaurateur.
For a stock corporation to exist, two requisites must be complied with:
1. a capital stock divided into shares and
2. an authority to distribute to the holders of such shares, dividends or allotments of the surplus profits on the
basis of the shares held (sec. 3, Act No. 1459).
Nowhere in its articles of incorporation or by-laws could be found an authority for the distribution of its dividends or
surplus profits. Strictly speaking, it cannot, therefore, be considered a stock corporation, within the contemplation of
the corpo law.

42
3B-Corporation Law

Topic:
Definitions and Attributes of Corporation
Subtopic:
Classes of Corporation
Provision:
Case Name:
Dante Liban v Richard Gordon
Name: Glenn Nio M. Sartillo
Topic: Nationality, Citizenship and Foreign Equity
Name of Case: Dante Liban, et al. vs. Richard Gordon (GR No L-39841 June 20, 1988)
Corporation Code Provision: Sections 3 and 4
FACTS:
Richard J. Gordon, while being an incumbent member of the Senate of the Philippines, was elected Chairman of the
Philippine National Red Cross (PNRC) during the February 23, 2006 meeting of the PNRC Board of Governors.
Petitioners, who are officers of the Board of Directors of the Quezon City Red Cross Chapter, alleged that by accepting
the chairmanship of the PNRC Board of Governors, Gordon has ceased to be a member of the Senate following Section
13, Article VI of the Constitution, which provides, among others, that no Senator may hold any other office or
employment in the Government, or any subdivision, agency, or instrumentality thereof, including government-owned
or controlled corporations or their subsidiaries, during his term without forfeiting his seat.
ISSUE:
WHETHER OR NOT PNRC IS A GOVERNMENT- OWNED OR CONTROLLED CORPORATION FOR PURPOSES OF THE
PROHIBITION IN SEC. 13, ARTICLE VI OF THE CONSTITUTION
RULING:
No. PNRC is a Private Organization Performing Public Functions. RA 95, otherwise known as the PNRC Charter, provides
that PNRC is a non-profit, donor-funded, voluntary, humanitarian organization, whose mission is to bring timely,
effective, and compassionate humanitarian assistance for the most vulnerable without consideration of nationality,
race, religion, gender, social status, or political affiliation. The PNRC is a member National Society of the International
Red Cross and Red Crescent Movement. In order to be recognized as a National Society, the PNRC has to be
autonomous. To ensure and maintain its autonomy, neutrality, and independence, the PNRC cannot be owned or
controlled by the government.
Indeed, the Philippine government does not own the PNRC. The PNRC does not have government assets and does not
receive any appropriation from the Philippine Congress. The PNRC is financed primarily by contributions from private
individuals and private entities obtained through solicitation campaigns organized by its Board of Governors.
The government does not control the PNRC. The PNRC Chairman is not appointed by the President or by any
subordinate government official. Only six of the 30 members of the PNRC Board of Governors are appointed by the
President of the Philippines. Thus, twenty-four members, or four-fifths (4/5), of the PNRC Board of Governors are not
appointed by the President.

43
3B-Corporation Law

Topic:
Subtopic:
Provision:
Case Name:

Definitions and Attributes of Corporation


Nature and Classes of Shares
Philippine National Bank v Merelo B. Aznar

Facts:

This case is consolidated with G.R. 172021, Merelo and Matias Aznar v. PNB

1958: Rural Insurance and Surety Company, Inc. (RISCO) ceased operation due to business reverses

In plaintiffs (Anzar et al.) desire to rehabilitate RISCO, they contributed a total amount of P212,720.00
-

This was used to purchase 3 parcels of land in Cebu

2 in the Minicipality of Talisay and 1 in the District of Lahug, Cebu City

Marami yung nag-contribute for the P212k, lahat sila kasama ni Aznar as defendants

After the purchase of the lots, titles were issued in the name of RISCO

The amount contributed by plaintiffs constituted as liens and encumbrances on the properties as annotated in
the titles of said lots

Such annotation was made pursuant to the Minutes of the Special Meeting of the Board of

Directors of RISCO on March 14, 1961, and a part of it says:


-

And that the respective contributions above-mentioned (Aznar et al.) shall constitute as their lien or
interest on the property described above, if and when said property are titled in the name of RISCO,
subject to registration as their adverse claim in pursuance of the Provision of Land Registration Act,
until such time their respective contributions are refunded to them completely

Thereafter, various subsequent annotations were made on the same titles, including the Notice of Attachment
and Writ of Execution both dated August 3,1962 in favour of Philippine National Bank

(PNB)

As a result, a Certificate of Sale was issued in favor of PNB, being the lone and highest bidder of the 3

parcels of land

This prompted Aznar et al. to file the instant case seeking the quieting of their supposed title to the

subject properties

Trial court ruled against PNB on the basis that there was an express trust created over the subject properties
whereby RISCO was the trustee and the stockholders, Aznar, et al., were the beneficiaries

Court of Appeals opined that the monetary contributions made by Aznar, et al. to RISCO can only be

Characterized as a load secured by a lien on the subjected lots, rather than an expressed trust

Issue:
Whether or not there was a trust contract between RISCO and Aznar, et al.

Held:
NO. At the outset, the Court agrees with the Court of Appeals that the agreement contained in the Minutes of the
Special Meeting of the RISCO Board of Directors held on March 14, 1961 was a loan by the therein named stockholders
to RISCO. Careful perusal of the Minutes relied upon by plaintiffs-appellees in their claim, showed that their
contributions shall constitute as lien or interest on the property. The term lien as used in the Minutes is defined as "a
discharge on property usually for the payment of some debt or obligation. Hence, from the use of the word "lien" in

44
3B-Corporation Law

the Minutes, We find that the money contributed by plaintiffs-appellees was in the nature of a loan, secured by their
liens and interests duly annotated on the titles. The annotation of their lien serves only as collateral and does not in
any way vest ownership of property to plaintiffs.

We are not persuaded by the contention of Aznar, et al., that the language of the subject Minutes created an express
trust.

Trust is the right to the beneficial enjoyment of property, the legal title to which is vested in another. It is a fiduciary
relationship that obliges the trustee to deal with the property for the benefit of the beneficiary. Express trusts are
intentionally created by the direct and positive acts of the settlor or the trustor - by some writing, deed, or will or oral
declaration. It is created not necessarily by some written words, but by the direct and positive acts of the parties. The
creation of an express trust must be manifested with reasonable certainty and cannot be inferred from loose and
vague declarations or from ambiguous circumstances susceptible of other interpretations.

At most, what Aznar, et al., had was merely a right to be repaid the amount loaned to RISCO. Unfortunately, the right
to seek repayment or reimbursement of their contributions used to purchase the subject properties is already barred
by prescription

10 Years because it was based on a written contract (the minutes by the Board of Directors) in 1961 and the quieting
of the title suit was brought only in 1998

45
3B-Corporation Law

Topic:
Subtopic:
Provision:
Case Name:

Definitions and Attributes of Corporation


Nature and Classes of Shares
Heirs of Wilson Gamboa v Teves

FACTS:
This is a petition to nullify the sale of shares of stock of Philippine Telecommunications Investment Corporation (PTIC)
by the government of the Republic of the Philippines, acting through the Inter-Agency Privatization Council (IPC), to
Metro Pacific Assets Holdings, Inc. (MPAH), an affiliate of First Pacific Company Limited (First Pacific), a Hong Kongbased investment management and holding company and a shareholder of the Philippine Long Distance Telephone
Company (PLDT).
The petitioner questioned the sale on the ground that it also involved an indirect sale of 12 million shares (or about 6.3
percent of the outstanding common shares) of PLDT owned by PTIC to First Pacific. With the this sale, First Pacifics
common shareholdings in PLDT increased from 30.7 percent to 37 percent, thereby increasing the total common
shareholdings of foreigners in PLDT to about 81.47%. This, according to the petitioner, violates Section 11, Article XII of
the 1987 Philippine Constitution which limits foreign ownership of the capital of a public utility to not more than 40%,
thus:
Section 11. No franchise, certificate, or any other form of authorization for the operation of a public utility shall be
granted except to citizens of the Philippines or to corporations or associations organized under the laws of the
Philippines, at least sixty per centum of whose capital is owned by such citizens; x x x
ISSUE:
Whether or not PLDT is a Filipino corporation compliant with the requirements of the Consitution based on its
composition.
RULING:
NO. Section 11, Article XII (National Economy and Patrimony) of the 1987 Constitution mandates the Filipinization of
public utilities, to wit: Section 11. No franchise, certificate, or any other form of authorization for the operation of a
public utility shall be granted except to citizens of the Philippines or to corporations or associations organized under
the laws of the Philippines, at least sixty per centum of whose capital is owned by such citizens; nor shall such
franchise, certificate, or authorization be exclusive in character or for a longer period than fifty years. Neither shall any
such franchise or right be granted except under the condition that it shall be subject to amendment, alteration, or
repeal by the Congress when the common good so requires. The State shall encourage equity participation in public
utilities by the general public. The participation of foreign investors in the governing body of any public utility
enterprise shall be limited to their proportionate share in its capital, and all the executive and managing officers of
such corporation or association must be citizens of the Philippines.
It was proven that: (1) foreigners own 64.27% of the common shares of PLDT, which class of shares exercises
the sole right to vote in the election of directors, and thus exercise control over PLDT; (2) Filipinos own only 35.73% of
PLDTs common shares, constituting a minority of the voting stock, and thus do not exercise control over PLDT; (3)
preferred shares, 99.44% owned by Filipinos, have no voting rights; (4) preferred shares earn only 1/70 of the
dividends that common shares earn; (5) preferred shares have twice the par value of common shares; and (6)
preferred shares constitute 77.85% of the authorized capital stock of PLDT and common shares only 22.15%. This kind
of ownership and control of a public utility is a mockery of the Constitution.
It must be stressed, and respondents do not dispute, that foreigners hold a majority of the common shares of PLDT. In
fact, based on PLDTs 2010 General Information Sheet (GIS), which is a document required to be submitted annually to
the Securities and Exchange Commission, foreigners hold 120,046,690 common shares of PLDT whereas Filipinos hold
only 66,750,622 common shares. In other words, foreigners hold 64.27% of the total number of PLDTs common
shares, while Filipinos hold only 35.73%. Since holding a majority of the common shares equates to control, it is clear
that foreigners exercise control over PLDT. Such amount of control unmistakably exceeds the allowable 40 percent limit
on foreign ownership of public utilities expressly mandated in Section 11, Article XII of the Constitution.
As shown in PLDTs 2010 GIS, as submitted to the SEC, the par value of PLDT common shares is P5.00 per share,
whereas the par value of preferred shares is P10.00 per share. In other words, preferred shares have twice the par
value of common shares but cannot elect directors and have only 1/70 of the dividends of common shares. Moreover,
99.44% of the preferred shares are owned by Filipinos while foreigners own only a minuscule 0.56% of the preferred
shares. Worse, preferred shares constitute 77.85% of the authorized capital stock of PLDT while common shares

46
3B-Corporation Law

constitute only 22.15%. This undeniably shows that beneficial interest in PLDT is not with the non-voting preferred
shares but with the common shares, blatantly violating the constitutional requirement of 60 percent Filipino control
and Filipino beneficial ownership in a public utility.
The legal and beneficial ownership of 60 percent of the outstanding capital stock must rest in the hands of Filipinos in
accordance with the constitutional mandate. Full beneficial ownership of 60 percent of the outstanding capital stock,
coupled with 60 percent of the voting rights, is constitutionally required for the States grant of authority to operate a
public utility. The undisputed fact that the PLDT preferred shares, 99.44% owned by Filipinos, are non-voting and earn
only 1/70 of the dividends that PLDT common shares earn, grossly violates the constitutional requirement of 60
percent Filipino control and Filipino beneficial ownership of a public utility.
In short, Filipinos hold less than 60 percent of the voting stock, and earn less than 60 percent of the dividends, of
PLDT. This directly contravenes the express command in Section 11, Article XII of the Constitution that [n]o franchise,
certificate, or any other form of authorization for the operation of a public utility shall be granted except to x x x
corporations x x x organized under the laws of the Philippines, at least sixty per centum of whose capital is owned by
such citizens x x x.

47
3B-Corporation Law

Topic:
Definitions and Attributes of Corporation
Subtopic:
Nature and Classes of Shares
Provision:
Case Name:
Republic Planters Bank v Agana
Republic Planters Bank vs. Agana
[GR 51765, 3 March 1997]
First Division, Hermosisima Jr. (J): 3 concur, 1 concurs in result
Facts: On 18 September 1961, the Robes-Francisco Realty & Development Corporation (RFRDC) secured a loan from
the Republic Planters Bank in the amount of P120,000.00. As part of the proceeds of the loan, preferred shares of
stocks were issued to RFRDC through its officers then, Adalia F. Robes and one Carlos F. Robes. In other words, instead
of giving the legal tender totaling to the full amount of the loan, which is P120,000.00, the Bank lent such amount
partially in the form of money and partially in the form of stock certificates numbered 3204 and 3205, each for 400
shares with a par value of P10.00 per share, or for P4,000.00 each, for a total of P8,000.00. Said stock certificates were
in the name of Adalia F. Robes and Carlos F. Robes, who subsequently, however, endorsed his shares in favor of Adalia
F. Robes. Said certificates of stock bear the following terms and conditions: "The Preferred Stock shall have the
following rights, preferences, qualifications and limitations, to wit: 1. Of the right to receive a quarterly dividend of 1%,
cumulative and participating. xxx 2. That such preferred shares may be redeemed, by the system of drawing lots, at
any time after 2 years from the date of issue at the option of the Corporation." On 31 January 1979, RFRDC and Robes
proceeded against the Bank and filed a complaint anchored on their alleged rights to collect dividends under the
preferred shares in question and to have the bank redeem the same under the terms and conditions of the stock
certificates. The bank filed a Motion to Dismiss 3 private respondents' Complaint on the following grounds: (1) that the
trial court had no jurisdiction over the subject-matter of the action; (2) that the action was unenforceable under
substantive law; and (3) that the action was barred by the statute of limitations and/or laches. The bank's Motion to
Dismiss was denied by the trial court in an order dated 16 March 1979. The bank then filed its Answer on 2 May 1979.
Thereafter, the trial court gave the parties 10 days from 30 July 1979 to submit their respective memoranda after the
submission of which the case would be deemed submitted for resolution. On 7 September 1979, the trial court
rendered the decision in favor of RFRDC and Robes; ordering the bank to pay RFRDC and Robes the face value of the
stock certificates as redemption price, plus 1% quarterly interest thereon until full payment. The bank filed the petition
for certiorari with the Supreme Court, essentially on pure questions of law.
Issue [1]: Whether the bank can be compelled to redeem the preferred shares issued to RFRDC and Robes.
Held [1]: While the stock certificate does allow redemption, the option to do so was clearly vested in the bank. The
redemption therefore is clearly the type known as "optional". Thus, except as otherwise provided in the stock
certificate, the redemption rests entirely with the corporation and the stockholder is without right to either compel or
refuse the redemption of its stock. Furthermore, the terms and conditions set forth therein use the word "may". It is a
settled doctrine in statutory construction that the word "may" denotes discretion, and cannot be construed as having a
mandatory effect. The redemption of said shares cannot be allowed. The Central Bank made a finding that the Bank
has been suffering from chronic reserve deficiency, and that such finding resulted in a directive, issued on 31 January
1973 by then Gov. G. S. Licaros of the Central Bank, to the President and Acting Chairman of the Board of the bank
prohibiting the latter from redeeming any preferred share, on the ground that said redemption would reduce the assets
of the Bank to the prejudice of its depositors and creditors. Redemption of preferred shares was prohibited for a just
and valid reason. The directive issued by the Central Bank Governor was obviously meant to preserve the status quo,
and to prevent the financial ruin of a banking institution that would have resulted in adverse repercussions, not only to
its depositors and creditors, but also to the banking industry as a whole. The directive, in limiting the exercise of a
right granted by law to a corporate entity, may thus be considered as an exercise of police power.
Issue [2]: Whether RFRDC and Robes are entitled to the payment of certain rate of interest on the stocks as a matter
of right without necessity of a prior declaration of dividend.
Held [2]: Both Section 16 of the Corporation Law and Section 43 of the present Corporation Code prohibit the
issuance of any stock dividend without the approval of stockholders, representing not less than two-thirds (2/3) of the
outstanding capital stock at a regular or special meeting duly called for the purpose. These provisions underscore the
fact that payment of dividends to a stockholder is not a matter of right but a matter of consensus. Furthermore,
"interest bearing stocks", on which the corporation agrees absolutely to pay interest before dividends are paid to
common stockholders, is legal only when construed as requiring payment of interest as dividends from net earnings or
surplus only. In compelling the bank to redeem the shares and to pay the corresponding dividends, the Trial committed

48
3B-Corporation Law

grave abuse of discretion amounting to lack or excess of jurisdiction in ignoring both the terms and conditions
specified in the stock certificate, as well as the clear mandate of the law.

49
3B-Corporation Law

Topic:
Incorporation and Organization
Subtopic:
Corporate Contract Law
Provision:
Sections 10 18, Corporation Code
Case Name:
Lanuza v Court of Appeals
Facts:
- In 1952, the Philippine Merchant Marine School, Inc. (PMMSI) was incorporated, with 700 founders shares and 76
common shares as its initial capital stock subscription reflected in the articles of incorporation
-Onrubia et. al, who were in control of PMMSI registered the companys stock and transfer book for the first time in
1978, recording 33 common shares as the only issued and outstanding shares of PMMSI.
- In 1979, a special stockholders meeting was called and held on the basis of what was considered as a quorum of
twenty-seven (27) common shares, representing more than 2/3 of the common shares issued and outstanding.
- In 1982, Juan Acayan, one of the heirs of the incorporators filed a petition for the registration of their property rights
was filed before the SEC over 120 founders shares and 12 common shares owned by their father
-SEC Hearing Officer: heirs of Acayan were entitled to the claimed shares and called for a special stockholders
meeting to elect a new set of officers.
-SEC en banc: affirmed the decision
-Acayans shares were recorded in the stock and transfer book.
-On May 6, 1992, a special stockholders meeting was held to elect a new set of directors
-Onrubia et al filed a petition with SEC questioning the validity of said meeting. They alleged that the quorum for the
said meeting should not be based on the 165 issued and outstanding shares as per the stock and transfer book, but on
the initial subscribed capital stock of 776 shares, as reflected in the 1952 Articles of Incorporation.
-Petition was dismissed
-SC: shares of the deceased incorporators should be duly represented by their respective administrators or heirs
concerned. Called for a stockholders meeting on the basis of the stockholdings reflected in the articles of incorporation
for the purpose of electing a new set of officers for the corporation
-CA: 1) the quorum should be based on the outstanding capital stock as found in the articles of incorporation; 2) To
require a separate judicial declaration to recognize the shares of the original incorporators would entail unnecessary
delay and expense. Incorporators have already proved their stockholdings through the provisions of the articles of
incorporation.
-Lanuza et al contention:
1. 1992 stockholders meeting was valid and legal
2. Reliance on the 1952 articles of incorporation for determining the quorum negates the existence and validity of
the stock and transfer book Onrubia et al prepared
3. Onrubia et al must show and prove entitlement to the founders and common shares in a separate and
independent action/proceeding in order to avail of the benefits secured by the heirs of Acayan
-Onrubia et als contention, based on the Memorandum: petition should be dismissed on the ground of res judicata
-Lanuza et als contention: instant petition is separate and distinct from G.R. No. 131315, there being no identity of
parties, and more importantly, the parties in the two petitions have their own distinct rights and interests in relation to
the subject matter in litigation
-Onrubia et als manifestation and motion: moved for the dismissal of the case
Issue: What should be the basis of quorum for a stockholders meetingthe outstanding capital stock as indicated in
the articles of incorporation or that contained in the companys stock and transfer book?
Held: WHEREFORE, the petition is DENIED and the assailed Decision is AFFIRMED. Costs against petitioners
Articles of Incorporation
- Defines the charter of the corporation and the contractual relationships between the State and the corporation, the
stockholders and the State, and between the corporation and its stockholders.
- Contents are binding, not only on the corporation, but also on its shareholders.
Stock and transfer book
- Book which records the names and addresses of all stockholders arranged alphabetically, the installments paid and
unpaid on all stock for which subscription has been made, and the date of payment thereof; a statement of every
alienation, sale or transfer of stock made, the date thereof and by and to whom made; and such other entries as may
be prescribed by law
-necessary as a measure of precaution, expediency and convenience since it provides the only certain and accurate
method of establishing the various corporate acts and transactions and of showing the ownership of stock and like
matters
-Not public record, and thus is not exclusive evidence of the matters and things which ordinarily are or should be
written therein

50
3B-Corporation Law

In the case at bar, the articles of incorporation indicate that at the time of incorporation, the incorporators
were bona fide stockholders of 700 founders shares and 76 common shares. Hence, at that time, the corporation had
776 issued and outstanding shares.
According to Sec. 52 of the Corp Code, a quorum shall consist of the stockholders representing a majority of the
outstanding capital stock. As such, quorum is based on the totality of the shares which have been subscribed and
issued, whether it be founders shares or common shares
To base the computation of quorum solely on the obviously deficient, if not inaccurate stock and transfer book, and
completely disregarding the issued and outstanding shares as indicated in the articles of incorporation would work
injustice to the owners and/or successors in interest of the said shares.
The stock and transfer book of PMMSI cannot be used as the sole basis for determining the quorum as it does not
reflect the totality of shares which have been subscribed, more so when the articles of incorporation show a
significantly larger amount of shares issued and outstanding as compared to that listed in the stock and transfer book.
One who is actually a stockholder cannot be denied his right to vote by the corporation merely because the
corporate officers failed to keep its records accurately. A corporations records are not the only evidence of the
ownership of stock in a corporation.
It is no less than the articles of incorporation that declare the incorporators to have in their name the founders and
several common shares. Thus, to disregard the contents of the articles of incorporation would be to pretend that the
basic document which legally triggered the creation of the corporation does not exist and accordingly to allow great
injustice to be caused to the incorporators and their heirs.

51
3B-Corporation Law

Topic:
Subtopic:
Provision:
Case Name:

Incorporation and Organization


Corporate Contract Law
Sections 10 18, Corporation Code
Excellent Quality Apparrel Inc. v Win Multi Rich Builders Inc.

Facts:

On March 26, 1996 petitioner Excellent Quality Apparel, Inc., represented by Max Ying Vice-President for
Productions and Alfiero R. Orden entered into a contract with Multi Rich Builders represented by Wilson Chua,
its President and General Manager, for the construction of a garment factory.
The said project was completed on Nov. 27, 1996
Respondent Win Multi-Rich Builders, Inc. (Win) was incorporated with the SEC on Feb. 20, 1997 with Wilson
Chua as its President and General Manager. On Jan. 26, 2004 Win filed a complaint for a collection of sum of
money against petitioner. It prayed for the issuance of writ of attachment claiming that Mr. Ying was about to
abscond and that petitioner was about to close.
Petitioner denied the allegations, it claimed that it was neither about to close and owing anything to
respondent.
On Feb. 10, 2004 Win moved that its name in the case be changed from Win Multi-Rich Builders, Inc. to
Multi-Rich Builders, Inc. Hence, petitioner became aware of the variance in the name of the plaintiff. It then
filed a motion to dismiss on the ground that Win Multi-Rich (plaintiff) was not the contractor and neither a
party to the contract, thus it cannot institute the case. Petitioner presented a Certificate of Non-Registration of
Corporation/Partnership from the SEC which certified that the latter did not have any records of a Multi Rich
Builders, Inc. It was admitted by Win that Multi Rich Builders is a sole proprietorship and was issued a
business permit by Office of the Mayor of Manila.
RTC dismissed the motion and proceeded with the trial.
Petitioner filed a Review on Certiorari under Rule 65 questioning the jurisdiction of the RTC. CA dismissed the
petition.

Issue:
Does Win have a legal personality to institute the present case?
Held:
No.
The contract executed on May 26, 1996 was between Excellent Quality Apparel, Inc. and Multi-Rich Builders.
Win Multi-Rich Builders, Inc. was only registered on Feb. 20, 1997. It was not even a party to the contract, further it
was not shown that it acquired assets and liabilities of Multi-Rich Builders that can at least prove that it was a
predecessor-in-business and hold that the latter has a standing to institute the collection suit.
Changing the name of the case will not also hold since Win Multi-Rich Builders, Inc. is a corporation while Multi-Rich
Builders is a sole proprietorship that has no separate juridical personality from its owner (in this case Chua). These two
names in effect has a separate and distinct personality from each other and not just a mere alter-ego of each.

52
3B-Corporation Law

Topic:
Subtopic:
Provision:
Case Name:

Incorporation and Organization


Purpose Clause
Sections 10 18, Corporation Code
Gala v Ellice Agro-Industrial Corp

Facts:
On March 28, 1979, the spouses Manuel and Alicia Gala, their children Guia Domingo, Ofelia Gala, Raul Gala,
and Rita Benson, and their encargados Virgilio Galeon and Julian Jader formed and organized the Ellice AgroIndustrialCorporation.
As payment for their subscriptions, the Gala spouses transferred several parcels of land located in the
provinces of Quezon and Laguna to Ellice.
In 1982, Manuel Gala, Alicia Gala and Ofelia Gala subscribed to an additional 3,299 shares, 10,652.5 shares and 286.5
shares, respectively.
On June 28, 1982, Manuel Gala and Alicia Gala acquired an additional 550 shares and 281 shares, respectively.
Subsequently, on September 16, 1982, Guia Domingo, Ofelia Gala, Raul Gala, Virgilio Galeon and Julian Jader
incorporated the Margo Management and Development Corporation (Margo).
On November 10, 1982, Manuel Gala sold 13,314 of his shares in Ellice to Margo.
Alicia Gala transferred 1,000 of her shares in Ellice to a certain Victor de Villa on March 2, 1983. That same day, de
Villa transferred said shares to Margo. A few months later, on August 28, 1983, Alicia Gala transferred 854.3 of her
shares to Ofelia Gala, 500 to Guia Domingo and 500 to Raul Gala.
Years later, on February 8, 1988, Manuel Gala transferred all of his remaining holdings in Ellice, amounting to
2,164 shares, to Raul Gala.
On July 20, 1988, Alicia Gala transferred 10,000 of her shares to Margo.
On June 23, 1990, a special stockholders meeting of Margo was held, where a new board of directors was
elected. That same day, the newly-elected board elected a new set of officers. Raul Gala was elected as chairman,
president and general manager. During the meeting, the board approved several actions, including the
commencement of proceedings to annul certain dispositions of Margos property made by Alicia Gala. The board also
resolved to change the name of the corporation to MRG Management and Development Corporation.
Similarly, a special stockholders meeting of Ellice was held on August 24, 1990 to elect a new board of
directors. In the ensuing organizational meeting later that day, a new set of corporate officers was elected. Likewise,
Raul Gala was elected as chairman, president and general manager.
On March 27, 1990, respondents filed against petitioners with the Securities and Exchange Commission (SEC)
a petition for the appointment of a management committee or receiver, accounting and restitution by the directors
and officers, and the dissolution of Ellice Agro-Industrial Corporation for alleged mismanagement, diversion of funds,
financial losses and the dissipation of assets, docketed as SEC Case No. 3747. The petition was amended to delete the
prayer for the appointment of a management committee or receiver and for the dissolution of Ellice. Additionally,
respondents prayed that they be allowed to inspect the corporate books and documents of Ellice.
In turn, petitioners initiated a complaint against the respondents on June 26, 1991, docketed as SEC Case No.
4027, praying for, among others, the nullification of the elections of directors and officers of both Margo Management
and Development Corporation and Ellice Industrial Corporation; the nullification of all board resolutions issued by
Margo from June 23, 1990 up to the present and all board resolutions issued by Ellice from August 24, 1990 up to the
present; and the return of all titles to real property in the name of Margo and Ellice, as well as all corporate papers and
records of both Margo and Ellice which are in the possession and control of the respondents.
The two cases were consolidated in an Order dated November 23, 1993.
Issues:
Whether or not the lower court erred in not declaring as illegal and contrary to public policy the purposes and manner
in which respondent corporations were organizedwhich were, to
(1) prevent the gala estate from being brought under the coverage of the comprehensive agrarian reform program
(CARP); and
(2) purportedly for estate planning.
Held:
No.

53
3B-Corporation Law

If a corporations purpose, as stated in the Articles of Incorporation, is lawful, then the SEC has no authority to inquire
whether the corporation has purposes other than those stated, and mandamus will lie to compel it to issue the
certificate of incorporation. The best proof of the purpose of a corporation is its articles of incorporation and by-laws.
The articles of incorporation must state the primary and secondary purposes of the corporation, while the by-laws
outline the administrative organization of the corporation, which, in turn, is supposed to insure or facilitate the
accomplishment of said purpose. In the case at bar, a perusal of the Articles of Incorporation of Ellice and Margo shows
no sign of the allegedly illegal purposes that petitioners are complaining of. It is well to note that, if a corporations
purpose, as stated in the Articles of Incorporation, is lawful, then the SEC has no authority to inquire whether the
corporation has purposes other than those stated, and mandamus will lie to compel it to issue the certificate of
incorporation.

54
3B-Corporation Law

Topic:
Incorporation and Organization
Subtopic:
Nominees and Anti-Dummy Law
Provision:
Sections 10 18, Corporation Code, Anti-Dummy Law
Case Name:
Marsman and Company Inc, v. First Coconut Central Co
Name: Glenn Nio M. Sartillo
Topic: Nationality, Citizenship and Foreign Equity
Name of Case: Marsman & Company, Inc. vs. First Coconut Central Co. (GR No L-39841 June 20, 1988)
Corporation Code Provision: Sections 3 and 4
FACTS:
First Coconut Central Co., Inc. (First Coconut) purchased on installment one diesel generating unit worth P21,000 from
Madrid Trading. As down payment, First Coconut paid the P4,000. The balance of P17,000 will be paid in three (3) equal
monthly installments. As security for the satisfaction of the said obligation, a chattel mortgage over the same diesel
generating unit was constituted. Madrid Trading assigned all its rights to Marsman & Company, Inc. (Marsman). First
Coconut paid Marsman P2,000, leaving a balance of P15,000. Later, First Coconut wrote Marsman appealing that they
be given thirty (30) days to settle the obligation, and enclosing in said letter a check for P1,000. After repeated failure
by First Coconut to meet its obligation, Marsman brought an action to recover the balance.
The Court of First Instance ruled in favor of Marsman. On appeal, CA ruled that the sale violated RA 1180 or the Retail
Trade Nationalization Law.
ISSUE:
WHETHER OR NOT THE SALE OF A DIESEL GENERATING SET TO FIRST COCONUT WAS NULL AND VOID FOR HAVING
BEEN MADE IN VIOLATION OF REPUBLIC ACT NO. 1180
RULING:
As per Section 4 of Republic Act No. 1180, for a sale to be considered, the following elements should concur: 1) The
seller should be habitually engaged in selling; 2) The sale must be direct to the general public; and 3) The object of the
sale is limited to merchandise, commodities or goods for consumption.
In this case, the first two elements are present. It is the presence of the third element that must be determined. The
last element refers to the subject of the retailer's activities or what he is selling, i.e., consumption goods or consumer
goods. Consumer goods may be defined as "goods which are used or bought for use primarily for personal, family or
household purposes. Such goods are not intended for resale or further use in the production of other products."
Producer goods have been defined as "goods (as tools and raw material) that are factors in the production of other
goods and that satisfy wants only indirectly- called also auxiliary goods, instrumental goods, intermediate goods."
The article in controversy is a piece of industrial machinerya diesel generating unit, used in the coconut central and
as such may be classified as "production or producer goods." Since the diesel generating unit is not a consumer item,
it necessarily does not come within the ambit of retail business as defined by Republic Act No. 1180.
Hence, Marsman & Company, Inc. may engage in the business of selling producer goods. It necessarily follows that
petitioner cannot be guilty of violating the Anti- Dummy Law or of using a dummy since it is not prohibited by the
Retail Trade Nationalization Law from selling the diesel generating unit to herein respondent.

55
3B-Corporation Law

Topic:
Incorporation and Organization
Subtopic:
Commencement of Corporate Existence, Corporate Team
Provision:
Section 19, Corporation Code
Case Name:
Cagayan Fishing Development Co. v Sandiko
Name: Glenn Nio M. Sartillo
Topic: Commencement of Corporate Existence, Corporate Term
Name of Case: Cagayan Fishing Development Co. Inc. vs. Sandiko (GR No. L-43350, December 23, 1937)
Corporation Code Provision: Section 19
FACTS:
Manuel Tabora is the registered owner of four parcels of land. To guarantee the payment of two loans, he executed in
favor of PNB two mortgages over the four parcels of land. Later, a third mortgage on the same lands was executed in
favor of Severina Buzon.
Thereafter, on May, 1930, Tabora executed a public document, by virtue of which the four parcels of land was sold to
the plaintiff company (Cagayan), said to be under process of incorporation. Cagayan filed its article incorporation with
the Bureau of Commerce and Industry only on October, 1930.
A year later, the board of directors of Cagayan adopted a resolution authorizing its president to sell the four parcels of
lands in question to Teodoro Sandiko. Through a deed of sale, Cagayan sold, ceded and transferred to Sandiko all its
right, titles, and interest in and to the four parcels of land. A promissory note was drawn by the Sandiko in favor of
Cagayan, payable after one year from the date thereof.
Sandiko failed to pay. Cagayan brought this action in the Court of First Instance, which absolved Sandiko.
ISSUE: Whether or not Cagayan has juridical capacity to enter into contract with Tabora
HELD:
No.
The transfer made by Tabora to Cagayan was affected on May 31, 1930 and the actual incorporation of said company
was affected later on October 22, 1930. In other words, the transfer was made almost five months before the
incorporation of the company.
Unquestionably, a duly organized corporation has the power to purchase and hold such real property as the purposes
for which such corporation was formed may permit and for this purpose may enter into such contracts as may be
necessary. But before a corporation may be said to be lawfully organized, many things have to be done. Among other
things, the law requires the filing of articles of incorporation.
In this case, Cagayan was not yet incorporated when it entered into a contract of sale. Not being in legal existence
then, it did not possess juridical capacity to enter into the contract.
Boiled down to its naked reality, the contract was entered into not between Tabora and a non-existent corporation, but
between the Manuel Tabora as owner of the four parcels of lands on the one hand and the same Manuel Tabora, his
wife and others, as mere promoters of a corporations on the other hand. These promoters could not have acted as
agent for a projected corporation since that which has no legal existence could have no agent. A corporation, until
organized, has no life and therefore no faculties.

56
3B-Corporation Law

Topic:
Incorporation and Organization
Subtopic:
Commencement of Corporate Existence, Corporate Team
Provision:
Section 19, Corporation Code
Case Name:
Marc II Marketing v Alfredo Joson
Name: Glenn Nio M. Sartillo
Topic: Commencement of Corporate Existence, Corporate Term
Name of Case: Marc II Marketing, Inc. vs. Alfredo Joson (GR No. 171993, December 12, 2011)
Corporation Code Provision: Section 19
FACTS:
Marc II Marketing, Inc. (Marc II) is primarily engaged in buying, marketing, selling and distributing in retail or wholesale
for export or import household appliances and products and other items. Petitioner Lucila V. Joson is the President and
majority stockholder of the corporation. Before Marc II Marketing, Inc. was officially incorporated, Alfredo M. Joson has
already been engaged by Lucila, in her capacity as President, to work as General Manager of the corporation, which
was formalized through the execution of a Management Contract dated in 1994 under Marc Marketing, Inc., as Marc II
was yet to be incorporated. For occupying the said position, respondent was among the corporations corporate
officers by the express provision of Section 1, Article IV of its by-laws.
Alfredo was appointed General Manager. However, Marc II decided to stop and cease its operation due to poor sales
collection. Alfredo was informed of the termination of his services as General Manager. He filed action for
reinstatement and money claim against petitioners.
ISSUE:
Whether or not Marc IIs Board of Directors could create a position for corporate officers through an enabling clause
found in its corporate by-laws
HELD:
No.
The Court held that in the context of PD 902-A, corporate officers are those officers of a corporation who are given that
character either by the Corporation Code or by the corporations by-laws.
Section 25 of the Corporation Code specifically enumerated these corporate officers: president, secretary, treasurer
and such other officers as may be provided for in the by-laws. A careful examination of Marc II Marketing Inc.s by-laws
explicitly revealed that its corporate officers are composed only of chairman, president, one/more vice president,
treasurer and secretary. The position of general manager was not among those enumerated.
Meanwhile, paragraph 2, Section 1 of Article IV of the corporations by-laws empowered its Board of Directors to
appoint such officers as it may determine necessary or proper, making this an enabling provision for approving a
resolution to make the position of general manager a corporate officer. All of these acts were done without first
amending its by-laws so as to include the General Manager in its roster of corporate officers. Though the Board of
Directors may create appointive positions other than the positions of corporate officers, the persons occupying such
positions cannot be viewed as corporate officers under Section 25 of the Corporation Code. The said provision of the
Corporation Code safeguards the constitutionally enshrined right of every employee to security of tenure and prevents
the creation of a corporate officer position by a simple inclusion in the corporate by-laws of an enabling clause
empowering the Board of Directors.

57
3B-Corporation Law

Topic:
Subtopic:
Provision:
Case Name:

Incorporation and Organization


Commencement of Corporate Existence, Corporate Team
Section 19, Corporation Code
Majority Stockholders of Ruby Industrial Corporation v Miguel Lim

FACTS:
Ruby Industrial Corporation (RUBY) is a domestic corporation engaged in glass manufacturing. Reeling from severe
liquidity problems beginning in 1980, RUBY filed on December 13, 1983 a petition for suspension of payments with the
Securities and Exchange Commission (SEC) docketed as SEC Case No. 2556.On December 20, 1983, the SEC issued an
order declaring RUBY under suspension of payments and enjoining the disposition of its properties pending hearing of
the petition, except insofar as necessary in its ordinary operations, and making payments outside of the necessary or
legitimate
expenses
of
its
business.
On August 10, 1984, the SEC Hearing Panel created the management committee (MANCOM) for RUBY, composed of
representatives from Allied Leasing and Finance Corporation (ALFC), Philippine Bank of Communications (PBCOM),
China Banking Corporation (China Bank), Pilipinas Shell Petroleum Corporation (Pilipinas Shell), and RUBY represented
by Mr. Yu Kim Giang.The MANCOM was tasked to perform the following functions: (1) undertake the management of
RUBY; (2) take custody and control over all existing assets and liabilities of RUBY; (3) evaluate RUBYs existing assets
and liabilities, earnings and operations; (4) determine the best way to salvage and protect the interest of its investors
and creditors; and (5) study, review and evaluate the proposed rehabilitation plan for RUBY.
Subsequently, two (2) rehabilitation plans were submitted to the SEC: the BENHAR/RUBY Rehabilitation Plan of the
majority stockholders led by Yu Kim Giang, and the Alternative Plan of the minority stockholders represented by Miguel
Lim
(Lim).
Both

plans

were

endorsed

by

the

SEC

to

the

MANCOM

for

evaluation.

OnApril 26, 1991, over ninety percent (90%) of RUBYs creditors objected to the Revised BENHAR/RUBY Plan and the
creation of a new management committee.Instead, they endorsed the minority stockholders Alternative Plan. At the
hearing of the petition for the creation of a new management committee, three (3) members of the original
management committee (Lim, ALFC and Pilipinas Shell) opposed the Revised BENHAR/RUBY Plan on grounds that:(1) it
would legitimize the entry of BENHAR, a total stranger, to RUBY as BENHAR would become the biggest creditor of
RUBY;(2) it would put RUBYs assets beyond the reach of the unsecured creditors and the minority stockholders; and (3)
it
was
not
approved
by
RUBYs
stockholders
in
a
meeting
called
for
the
purpose.
Notwithstanding the objections of 90% of RUBYs creditors and three members of the MANCOM, the SEC Hearing Panel
approved on September 18, 1991the Revised BENHAR/RUBY Plan and dissolved the existing management committee.
It also created a new management committee and appointed BENHAR as one of its members. In addition to the powers
originally conferred to the management committee under Presidential Decree (P.D.) No. 902-A, the new management
committee was tasked to oversee the implementation by the Board of Directors of the revised rehabilitation plan for
RUBY.
ISSUE:
Whether the minoritys pre-emptive rights were violated
RULING:
Yes. Pre-emptive right under Sec. 39 of the Corporation Code refers to the right of a stockholder of a stock corporation
to subscribe to all issues or disposition of shares of any class, in proportion to their respective shareholdings. The right
may be restricted or denied under the articles of incorporation, and subject to certain exceptions and limitations. The
stockholder must be given a reasonable time within which to exercise their preemptive rights. Upon the expiration of
said period, any stockholder who has not exercised such right will be deemed to have waived it.
The validity of issuance of additional shares may be questioned if done in breach of trust by the controlling
stockholders. Thus, even if the pre-emptive right does not exist, either because the issue comes within the exceptions
in Section 39 or because it is denied or limited in the articles of incorporation, an issue of shares may still be
objectionable if the directors acted in breach of trust and their primary purpose is to perpetuate or shift control of the
corporation, or to "freeze out" the minority interest. In this case, the following relevant observations should have
signaled greater circumspection on the part of the SEC -- upon the third and last remand to it pursuant to our January
20, 1998 decision -- to demand transparency and accountability from the majority stockholders, in view of the illegal
assignments and objectionable features of the Revised BENHAR/RUBY Plan, as found by the CA and as affirmed by this
Court:

58
3B-Corporation Law

There can be no gainsaying the well-established rule in corporate practice and procedure that the will of the majority
shall govern in all matters within the limits of the act of incorporation and lawfully enacted by-laws not proscribed by
law.It is, however, equally true that other stockholders are afforded the right to intervene especially during critical
periods in the life of a corporation like reorganization, or in this case, suspension of payments, more so,when the
majority seek to impose their will and through fraudulent means, attempt to siphon off Rubys valuable assets to the
great prejudice of Ruby itself, as well as the minority stockholders and the unsecured creditors.
Certainly, the minority stockholders and the unsecured creditors are given some measure of protection by the law from
the abuses and impositions of the majority, more so in this case, considering the give-away signs of private
respondents perfidy strewn all over the factual landscape. Indeed, equity cannot deprive the minority of a remedy
against the abuses of the majority, and the present action has been instituted precisely for the purpose of protecting
the true and legitimate interests of Ruby against the Majority Stockholders. On this score, the Supreme Court, has
ruled
that:
"Generally speaking, the voice of the majority of the stockholders is the law of the corporation, but there are
exceptions to this rule. There must necessarily be a limit upon the power of the majority. Without such a limit the will
of the majority will be absolute and irresistible and might easily degenerate into absolute tyranny. x x x" (Additional
emphasis
supplied.)
Lamentably, the SEC refused to heed the plea of the minority stockholders and MANCOM for the SEC to order RUBY to
commence liquidation proceedings, which is allowed under Sec. 4-9 of the Rules on Corporate Recovery. Under the
circumstances, liquidation was the only hope of the minority stockholders for effecting an orderly and equitable
settlement of RUBYs obligations, and compelling the majority stockholders to account for all funds, properties and
documents in their possession, and make full disclosure on the nullified credit assignments. Oblivious to these pending
incidents so crucial to the protection of the interest of the majority of creditors and minority shareholders, the SEC
simply stated that in the interim, RUBYs corporate term was validly extended, as if such extension would provide the
solution
to
RUBYs
myriad
problems.
Extension of corporate term requires the vote of 2/3 of the outstanding capital stock in a stockholders meeting called
for the purpose. The actual percentage of shareholdings in RUBY as of September 3, 1996 -- when the majority
stockholders allegedly ratified the board resolution approving the extension of RUBY's corporate life to another 25
years was seriously disputed by the minority stockholders, and we find the evidence of compliance with the notice and
quorum requirements submitted by the majority stockholders insufficient and doubtful. Consequently, the SEC had no
basis for its ruling denying the motion of the minority stockholders to declare as without force and effect the extension
of RUBY's corporate existence

59
3B-Corporation Law

Topic:
Incorporation and Organization
Subtopic:
Defectively Formed Corporations: De Facto and Corporation by Estoppel
Provision:
Section 20 & 21, Corporation Code
Case Name:
Pioneer Insurance and Surety Corp v Court of Appeals
Topic: Defectively Formed Corporations: De Facto and Corporation by Estoppel
Case: PIONEER INSURANCE & SURETY CORPORATION vs CA
Name: Carmela Abergos
FACTS:
In 1965, Jacob S. Lim (petitioner in G.R. No. 84157) was engaged in the airline business as owner-operator of Southern
Air Lines (SAL) a single proprietorship. On May 17, 1965, at Tokyo, Japan, Japan Domestic Airlines (JDA) and Lim
entered into and executed a sales contract for the sale and purchase of two (2) DC-3A Type aircrafts and one (1) set of
necessary spare parts for the total agreed price of US $109,000.00 to be paid in installments. One DC-3 Aircraft with
Registry No. PIC-718, arrived in Manila on June 7,1965 while the other aircraft, arrived in Manila on July 18,1965.
On May 22, 1965, Pioneer Insurance and Surety Corporation, as surety executed and issued its Surety Bond No. 6639
in favor of JDA, in behalf of its principal, Lim, for the balance price of the aircrafts and spare parts. On June 10, 1965,
Lim doing business under the name and style of SAL executed in favor of Pioneer as deed of chattel mortgage as
security for the latter's suretyship in favor of the former. It was stipulated therein that Lim transfer and convey to the
surety the two aircrafts. The deed was duly registered with the Office of the Register of Deeds of the City of Manila and
with the Civil Aeronautics Administration pursuant to the Chattel Mortgage Law and the Civil Aeronautics Law (Republic
Act No. 776), respectively.
Lim defaulted on his subsequent installment payments prompting JDA to request payments from the surety. Pioneer
paid a total sum of P298,626.12.
Pioneer then filed a petition for the extrajudicial foreclosure of the said chattel mortgage before the Sheriff of Davao
City. The Cervanteses and Maglana, however, filed a third party claim alleging that they are co-owners of the aircrafts,
On July 19, 1966, Pioneer filed an action for judicial foreclosure with an application for a writ of preliminary attachment
against Lim and respondents, the Cervanteses, Bormaheco and Maglana.
In their Answers, Maglana, Bormaheco and the Cervanteses filed cross-claims against Lim alleging that they were not
privies to the contracts signed by Lim and, by way of counterclaim, sought for damages for being exposed to litigation
and for recovery of the sums of money they advanced to Lim for the purchase of the aircrafts in question.
After trial on the merits, a decision was rendered holding Lim liable to pay Pioneer but dismissed Pioneer's complaint
against all other defendants.
ISSUES:
What legal rules govern the relationship among co-investors whose agreement was to do business through the
corporate vehicle but who failed to incorporate the entity in which they had chosen to invest? How are the losses to be
treated in situations where their contributions to the intended 'corporation' were invested not through the corporate
form?
RULING:
No de facto partnership was created among the parties which would entitle the petitioner to a reimbursement of the
supposed losses of the proposed corporation. The record shows that the petitioner was acting on his own and not in
behalf of his other would-be incorporators in transacting the sale of the airplanes and spare parts.
Ordinarily, when co-investors agreed to do business through a corporation but failed to incorporate, a de facto
partnership would have been formed, and as such, all must share in the losses and/or gains of the venture in
proportion to their contribution. But in this case, it was shown that Lim did not have the intent to form a corporation
with Maglana et al. This can be inferred from acts of unilaterally taking out a surety from Pioneer Insurance and not
using the funds he got from Maglana et al. The record shows that Lim was acting on his own and not in behalf of his
other would-be incorporators in transacting the sale of the airplanes and spare parts.

60
3B-Corporation Law

Topic:
Incorporation and Organization
Subtopic:
Commencement of Corporate Existence, Corporate Team
Provision:
Section 20 & 21, Corporation Code
Case Name:
Hall v Piccio
Topic: Defectively Formed Corporations: De Facto and Corporation by Estoppel
Case: Hall vs. Piccio
Name: Carmela Abergos
FACTS:
On 28 May 1947, C. Arnold Hall and Bradley P. Hall, and Fred Brown, Emma Brown, Hipolita D. Chapman and Ceferino
S. Abella, signed and acknowledged in Leyte, the article of incorporation of the Far Eastern Lumber and Commercial
Co., Inc., organized to engage in a general lumber business to carry on as general contractors, operators and
managers, etc. Attached to the article was an affidavit of the treasurer stating that 23,428 shares of stock had been
subscribed and fully paid with certain properties transferred to the corporation described in a list appended thereto.
Immediately after the execution of said articles of incorporation, the corporation proceeded to do business with the
adoption of by-laws and the election of its officers. On 2 December 1947, the said articles of incorporation were filed in
the office of the Securities and Exchange Commissioner, for the issuance of the corresponding certificate of
incorporation. On 22 March 1948, pending action on the articles of incorporation by the aforesaid governmental office,
Fred Brown, Emma Brown, Hipolita D. Chapman and Ceferino S. Abella filed before the Court of First Instance of Leyte
the civil case, alleging among other things that the Far Eastern Lumber and Commercial Co. was an unregistered
partnership; that they wished to have it dissolved because of bitter dissension among the members, mismanagement
and fraud by the managers and heavy financial losses. C. Arnold Hall and Bradley P. Hall, filed a motion to dismiss,
contesting the court's jurisdiction and the sufficiently of the cause of action. After hearing the parties, the Hon.
Edmund S. Piccio ordered the dissolution of the company; and at the request of Brown, et. al., appointed Pedro A.
Capuciong as the receiver of the properties thereof, upon the Commercial Law - Corporation Law, 2005 filing of a
P20,000 bond. Hall and Hall offered to file a counter-bond for the discharge of the receiver, but Judge Piccio refused to
accept the offer and to discharge the receiver. Whereupon, Hall and Hall instituted the present special civil action with
the Supreme Court.
ISSUE:
Whether Brown, et. al. may file an action to cause the dissolution of the Far Eastern Lumber and Commercial Co.,
without State intervention.
HELD:
The Securities and Exchange Commission has not issued the corresponding certificate of incorporation. The personality
of a corporation begins to exist only from the moment such certificate is issued not before. Not having obtained the
certificate of incorporation, the Far Eastern Lumber and Commercial Co. even its stockholders may not probably
claim "in good faith" to be a corporation. Under the statue it is to be noted that it is the issuance of a certificate of
incorporation by the Director of the Bureau of Commerce and Industry which calls a corporation into being. The
immunity if collateral attack is granted to corporations "claiming in good faith to be a corporation under this act." Such
a claim is compatible with the existence of errors and irregularities; but not with a total or substantial disregard of the
law. Unless there has been an evident attempt to comply with the law the claim to be a corporation "under this act"
could not be made "in good faith." This is not a suit in which the corporation is a party. This is a litigation between
stockholders of the alleged corporation, for the purpose of obtaining its dissolution. Even the existence of a de jure
corporation may be terminated in a private suit for its dissolution between stockholders, without the intervention of
the state.

61
3B-Corporation Law

Topic:
Incorporation and Organization
Subtopic:
Commencement of Corporate Existence, Corporate Team
Provision:
Section 20 & 21, Corporation Code
Case Name:
Lim v Philippine Fishing Gear Industries
DEFECTIVELY FORMED CORPORATIONS
LIM v PHILIPPINE FISHING GEAR INDUSTRIES
FACTS:

Lim Tong Lim requested Peter Yao and Antonio Chuato engage in commercial fishing with him.

The three agreed to purchase two fishing boats but since they do not have the money they borrowed from one
Jesus Lim the brother of Lim Tong Lim.

Subsequently, they again borrowed money for the purchase of fishing nets and other fishing equipment.

Yao and Chua represented themselves as acting in behalf of Ocean Quest Fishing Corporation (OQFC) and
they contracted with Philippine Fishing Gear Industries (PFGI) for the purchase of fishing nets amounting to
more than P500k.

However, they were unable to pay PFGI and hence were sued in their own names as Ocean Quest Fishing
Corporation is a non-existent corporation.

Chua admitted his liability while Lim Tong Lim refused such liability alleging that Chua and Yao acted without
his knowledge and consent in representing themselves as a corporation.
ISSUE:
Whether there exists a corporation by estoppel
HELD:
Yes.
It is apparent from the factual milieu that the three decided to engage in a fishing business. Moreover, their
Compromise Agreement had revealed their intention to pay the loan with the proceeds of the sale and to divide
equally among them the excess or loss. The boats and equipment used for their business entails their common fund.
The contribution to such fund need not be cash or fixed assets; it could be an intangible like credit or industry. That the
parties agreed that any loss or profit from the sale and operation of the boats would be divided equally among them
also shows that they had indeed formed a partnership. The principle of corporation by estoppel cannot apply in the
case as Lim Tong Lim also benefited from the use of the nets in the boat, which was an asset of the partnership.
Under the law on estoppel, those acting in behalf of a corporation and those benefited by it, knowing it to be without
valid existence are held liable as general partners. Hence, the question as to whether such was legally formed for
unknown reasons is immaterial to the case.

62
3B-Corporation Law

Topic:
Incorporation and Organization
Subtopic:
Commencement of Corporate Existence, Corporate Team
Provision:
Section 20 & 21, Corporation Code
Case Name:
International Express Travel and Tours v CA
DEFECTIVELY FORMED CORPORATIONS
INTERNATIONAL EXPRESS TRAVEL & TOURS v COURT OF APPEALS
FACTS:

International Express Travel and Tour Services, Inc. (IETTSI), through its managing director, wrote a letter to the
Philippine Football Federation (Federation), through its president, Henri Kahn, wherein the former offered its
services as a travel agency to the latter. The offer was accepted.

IETTSI secured the airline tickets for the trips of the athletes and officials of the Federation to the South East
Asian Games in Kuala Lumpur as well as various other trips to the People's Republic of China and Brisbane.

After the Federation made two partial payments, IETTSI wrote the Federation, through Kahn a demand letter
requesting for the balance.

The Federation, through made partial payments through the following:


o Project Gintong Alay, and
o Kahn, through his personal check partial payment.

Thereafter, no further payments were made despite repeated demands, which then prompted IETTSI to file a
civil case before the RTC of Manila.

IETTSI sued Henri Kahn in his personal capacity and as President of the Federation and impleaded the
Federation as an alternative defendant. IETTSI sought to hold Kahn liable for the unpaid balance for the tickets
purchased by the Federation on the ground that Kahn allegedly guaranteed the said obligation.

Kahn filed his answer with counterclaim, while the Federation failed to file its answer and was declared in
default by the trial court.

RTC ruled in favor of IETTSI and declared Henri Kahn personally liable for the unpaid obligation of the
Federation. The complaint of IETTSI against the Philippine Football Federation and the counterclaims of Kahn
were dismissed.

Only Kahn elevated the decision to the CA. The appellate court rendered a decision reversing the trial court.

IETTSI filed a motion for reconsideration and as an alternative prayer pleaded that the Federation be held liable
for the unpaid obligation. The same was denied by the appellate court, hence this petition.
ISSUE:
Whether the appellate court properly applied the doctrine of corporation by estoppel.
HELD:
Yes.
The Court cannot subscribe to the position taken by the appellate court that even assuming that the Federation was
defectively incorporated, IETTSI cannot deny the corporate existence of the Federation because it had contracted and
dealt with the Federation in such a manner as to recognize and in effect admit its existence. The doctrine of
corporation by estoppel is mistakenly applied by the appellate court to IETTSI. The application of the doctrine applies
to a third party only when he tries to escape liabilities on a contract from which he has benefited on the irrelevant
ground of defective incorporation.
In the instant case, IETTSI is not trying to escape liability from the contract but rather is the one claiming from the
contract.

63
3B-Corporation Law

Topic:
Incorporation and Organization
Subtopic:
Non-Use of Charter and Inoperation
Provision:
Section 22, Corporation Code
Case Name:
Chung Ka Bio v Intermediate Appellate Court
NON-USE OF CHARTER AND INOPERATION
CHUNG KA BIO v IAC
FACTS:

Philippine Blooming Mills Company, Inc. was incorporated for a term of 25 years.

The members of its board of directors executed a deed of assignment of all of the accounts receivables,
properties, obligations and liabilities of the old PBM in favor of Chung Siong Pek in his capacity as treasurer of
the new PBM, then in the process of reincorporation.

The new PBM was issued a certificate of incorporation by the Securities and Exchange Commission.

Chung Ka Bio and the other petitioners herein, all stockholders of the old PBM, filed with the SEC a petition for
liquidation of both the old PBM and the new PBM.

The allegation was that the former had become legally non-existent for failure to extend its corporate life and
that the latter had likewise beenipso facto dissolved for non-use of the charter and continuous failure to
operate within 2 years from incorporation.
ISSUE:
Whether the new corporation has not substantially complied with the two-year requirement of Section 22 of the new
Corporation Code on non-user because its stockholders never adopted a set of by-laws
HELD:
No.
Non-filing of the by-laws will not result in automatic dissolution of the corporation. Under Section 6(i) of PD 902-A, the
SEC is empowered to suspend or revoked, after proper notice and hearing, the franchise or certificate of registration
of a corporation on the ground inter alia of failure to file by-laws within the required period. It is clear from this
provision that there must first of all be a hearing to determine the existence of the ground, and secondly, assuming
such finding, the penalty is not necessarily revocation but may be only suspension of the charter. In fact, under the
rules and regulations of the SEC, failure to file the by-laws on time may be penalized merely with the imposition of an
administrative fine without affecting the corporate existence of the erring firm.

64
3B-Corporation Law

Topic:
By-Laws
Subtopic:
Binding Effect
Provision:
Sections 46-48, Corporation Code
Case Name:
PMI Colleges v NLRC
FACTS:

PMI, an educational institution offering courses on basic seamans training and other marine-related courses,
hired private respondent as contractual instructor, pursuant to this engagement, private respondent then
organized classes in marine engineering.

Initially, private respondent and other instructors were compensated for services rendered during the first
three periods of the abovementioned contract. However, for reasons unknown to private respondent, he
stopped receiving payment for the succeeding rendition of services.

Then he filed a complaint 4 before the National Capital Region Arbitration Branch on September 14, 1993
seeking payment for salaries earned including the shipyard and plant visits in Cavite Naval Base.

PMI manifested that Mr. Tomas G. Cloma, Jr., a member of the petitioners Board of Trustees wrote a letter 5 to
the Chairman of the Board on May 23, 1994, clarifying the case of private respondent and stating therein, inter
alia, that under petitioners by-laws only the Chairman is authorized to sign any contract and that private
respondent, in any event, failed to submit documents on the alleged shipyard and plant visits in Cavite Naval
Base.
ISSUE:

Whether the contract would be invalid just because the signatory thereon was not the Chairman of the Board
which allegedly violated petitioners by-laws?

HELD:

No.
Since by-laws operate merely as internal rules among the stockholders, they cannot affect or prejudice third
persons who deal with the corporation, unless they have knowledge of the same. 11 No proof appears on
record that private respondent ever knew anything about the provisions of said by-laws. In fact, petitioner itself
merely asserts the same without even bothering to attach a copy or excerpt thereof to show that there is such
a provision.

65
3B-Corporation Law

Topic:
Subtopic:
Provision:
Case Name:

By-Laws
Binding Effect
Sections 46-48, Corporation Code
Government of Philippine Islands v El Hogar Filipino

The Government of the Philippine Islands vs. El Hogar Filipino G.R. No. L-26649 July 13, 1917
FACTS:

The Philippine Commission enacted Act No. 1459, also known as the Corporation Law, on March 1, 1906. El
Hogar Filipino, organized in 1911 under the laws of the Philippine Islands, was the first corporation organized
under Sec. 171-190 Act No. 1459, devoted to the subject of building and loan associations, their organization
and administration.

In the said law, the capital of the corporation was not permitted to exceed P3M, but Act No. 2092 amended the
statute, permitting capitalization to the amount of ten millions. El Hogar took advantage of the amendment of
Act No. 1459 and amended its AOI as a result thereof, stating that the amount of capital must not exceed what
has been stated in Act No. 2092. This resulted to El Hogar having 5,826 shareholders, 125,750 shares with
paid-up value of P8.7M.

The corporation paid P7.16M to its withdrawing stockholders. The Government of the Philippine Islands filed an
action against El Hogar due to the alleged illegal holding title to real property for a period exceeding five (5)
years after the same was bought in a foreclosure sale. Sec. 13(5) of the Corporation Law states that
corporations must dispose of real estate obtained within 5 years from receiving the title. The Philippine
Government also prays that El Hogar be excluded from all corporate rights and privileges and effecting a final
dissolution of said corporation.

It appears from the records that El Hogar was the holder of a recorded mortgage on the San Clemente land as
security for a P24K loan to El Hogar. However, shareholders and borrowers defaulted in payment so El Hogar
foreclosed the mortgage and purchased the land during the auction sale. A deed of conveyance in favor of El
Hogar was executed and sent to the Register of Deeds of Tralac with a request that the certificate of title be
cancelled and a new one be issued in favor of El Hogar from the Register of Deeds of Tarlac. However, no reply
was received. El Hogar filed a complaint with the Chief of the General Land Registration Office.

The certificate of title to the San Clemente land was received by El Hogar and a board resolution authorizing
Benzon to find a buyer was issued. Alcantara, the buyer of the land, was given extension of time to make
payment but defaulted so the contract treated rescinded. Efforts were made to find another buyer. Respondent
acquired title in December 1920 until the property was finally sold to Felipa Alberto in July 1926. The interval
exceeded 5 years but the period did not commence to run until May 7, 1921 when the register of deeds
delivered the new certificate of title.

It has been held that a purchaser of land registered under the Torrens system cannot acquire the status of an
innocent purchaser for value unless the vendor is able to place the owners duplicate in his hands showing the
title to be in the vendor. During the period before May 1921, El Hogar was not in a position to pass an
indefeasible title to any purchaser.

Therefore, El Hogar cannot be held accountable for this delay which was not due to its fault. Likewise, the
period from March 25, 1926 to April 20, 1926 must not be part of the five-year period because this was the
period where respondent was under the obligation to sell the property to Alcantara prior to the contracts
rescission due to Alcantaras non-payment.

Another circumstance causing the delay is the fact that El Hogar purchased the property in the full amount of
the loan made by the former owner which is nearly P24K when it was subsequently found that the property
was not salable and later sold for P6K notwithstanding El Hogars efforts to find a purchaser upon better terms.
ISSUE:

Whether the acts of respondent corporation merit its dissolution or deprivation of its corporate franchise and to
exclude it from all corporate rights and privileges
HELD:

SUSTAINED only as to administering of real property not owned by it and when permitted by contract. Causes
of action:

1) Alleged illegal holding of real property for a period exceeding five years from receipt of title-Cause of delay
is not respondents fault

2) That respondent is owning and holding a business lot with the structure thereon in excess of its reasonable
requirements and in contravention of Sec. 13(5) of Corpo. Law- WITHOUT MERIT Every corporation has the
power to purchase, hold and lease such real property as the transaction would of the lawful business may
reasonably and necessarily require.

3) That respondent is engaged in activities foreign to the purposes for which the corporation was created and
not reasonably necessary to its legitimate ends-VALID The administration of property, payment of real estate

66
3B-Corporation Law

taxes, causing necessary repairs, managing real properties of nonborrowing shareholders is more befitting to
the business of a real estate agent or a trust company than a building and loan association.
4) That the by-laws of the association stating that, the board of directors by the vote of an absolute majority
of its members is empowered to cancel shares and to return the balance to the owner by reason of their
conduct or any other motive or liquidation is in direct conflict with Sec. 187 of the Corporation Law which
provides that the board of directors shall not have the power to force the surrender and withdrawal of
unmatured stock except in case of liquidation or forfeiture of stock for delinquency-WITHOUT MERIT There is no
provision of law making it a misdemeanor to incorporate an invalid provision in the by-laws of a corporation;
and if there were such, the hazards incident to corporate effort would be largely increased.
5) Art. 61 of El Hogars by-laws which states that attendance in person or by proxy by shareholders owning
one-half plus one of the shareholders shall be necessary to constitute a quorum for the election of directors is
contrary to Sec. 31 of the Corpo Law which provides that owners of the majority of the subscribed capital stock
entitled to vote must be present either in person or by proxy at all elections of directors- WITHOUT MERIT No
fault can be imputed to the corporation on account of the failure of the shareholders to attend the annual
meetings and their non-attendance in meetings is doubtless to be interpreted in part as expressing their
satisfaction of the way in which things have been conducted. Mere failure of a corporation to elect officers
does not terminate the terms of existing officers nor dissolve the corporation. The general rule is to allow the
officer to holdover until his successor is duly qualified.
6) That the directors of El Hogar, instead of receiving nominal pay or serving without pay, have been receiving
large compensation, varying in amount from time to time, out of respondents profits- WITHOUT MERIT With
the growth of the corporation, the amount paid as compensation to the directors has increased beyond what
would probably be necessary is a matter that cannot be corrected in this action. Nor can it properly be made a
basis for depriving respondent of its franchise or enjoining it from compliance with the provisions of its own bylaws. If a mistake has been made, the remedy is to lie rather in publicity and competition.
7) That the promoter and organizer of El Hogar was Mr. Antonio Melian and that in the early stages of the
organization of the association, the board of directors authorized the association to make a contract with him
and that the royalty given to him as founder is unconscionable, excessive and out of proportion to the
services rendered-NOT SUSTAINED The mere fact that compensation is in excess of what may be considered
appropriate is not a proper consideration for the court to resolve. That El Hogar is in contact with its promoter
did not affect the associations legal character. The court is of the opinion that the traditional respect for the
sanctity of the contract obligation should prevail over the radical and innovating tendencies.
8) That Art. 70 of El Hogars by-laws, requiring persons elected as board of directors to be holders of shares of
the paid up value of P5,000 which shall be held as security, is objectionable since a poor member or wage
earner cannot serve as a director irrespective of other qualifications- NOT SUSTAINED Corpo. Law expressly
gives the power to the corporation to provide in its by-laws for the qualification of its directors and the
requirement of security from them for the proper discharge of the duties of their pffice in the manner
prescribed in Art. 70 is highly prudent and in conformity with good practice.
9) That respondent abused its franchise in issuing special shares alleged to be illegal and inconsistent with
the plan and purposes of building and loan associations- WITHOUT MERIT The said special shares are generally
known as advance payment shares which were evidently created for the purpose of meeting the condition
caused by the prepayment of dues that is permitted. Sec. 178 of Corpo Law allows payment of dues or interest
to be paid in advance but the corporation shall not allow interest on advance payment grater than 6% per
annum nor for a period longer than one year. The amount is satisfied by applying a portion of the shareholders
participation in the annual earnings.The mission of special shares does not involve any violation of the
principle that the shares must be sold at par.
10) That in making purchases at foreclosure sales constituting as security for 54 of the loans, El Hogar bids the
full amount after deducting the withdrawal value, alleged to be pusuing a policy of depreciating at the rate of
10 percent per annum, the value of the real properties it acquired and that this rate is excessiveUNSUSTAINABLE The board of directors possess discretion in this matter. There is no provision of law
prohibiting the association from writing off a reasonable amount for depreciation on its assets for the purpose
of determining its real profits. Art. 74 of its by-laws expressly authorizes the board of directors to determine
each year the amount to be written down upon the expenses for the installation and the property of the
corporation. The court cannot control the discretion of the board of directors about an administrative matter as
to which they have no legitimate power of action.
11) That respondent maintains excessive reserve funds-UNFOUNDED The function of this fund is to insure
stockholders against losses. When the reserves become excessive, the remedy is in the hands of the
Legislature. No prudent person would be inclined to take a policy in a company which had so improvidently
conducted its affairs that it only retained a fund barely sufficient to pay its present liabilities and therefore was
in a condition where any change by the reduction of interest upon or depreciation in the value of securities or
increase of mortality would render it insolvent and subject to be placed in the hands of a receiver.
12) That the board of directors has settled upon the unlawful policy of paying a straight annual dividend of 10
percent per centum regardless of losses suffered and profits made by the corporation, in contravention with
the requirements of Sec. 188 of the Corpo law- UNFOUNDED As provided in the previous cause of action, the

67
3B-Corporation Law

profits and losses shall be determined by the board of directors and this means that they shall exercise the
usual discretion of good businessmen in allocating a portion of the annual profits to purposes needful of the
welfare of the association. The law contemplates distribution of earnings and losses after legitimate obligations
have been met.
13) That El Hogar has made loans to the knowledge of its officers which were intended to be used by the
borrowers for other purposes than the building of homes and no attempt has been made to control the
borrowers with respect to the use made of the borrowed funds- UNFOUNDED There is no statute expressly
declaring that loans may be made by these associations SOLELY for the purpose of building homes. The
building of himes in Sec. 171 of Corpo Law is only one among several ends which building and loan
associations are designed to promote and Sec. 181 authorizes the board of directors of the association to fix
the premium to be charged.
14) That the loans made by defendant for purposes other than building or acquiring homes have been
extended in extremely large amounts and to wealthy persons and large companies- WITHOUT MERIT The
question of whether the making of large loans constitutes a misuser of the franchise as would justify the court
in depriving the association of its corporate life is a matter confided to the discretion of the board of directors.
The law states no limit as to the size of the loans to be made by the association. Resort should be had to the
legislature because it is not a matter amenable to judicial control
15) That when the franchise expires, supposing the corporation is not reorganized, upon final liquidation of the
corporation, a reserve fund may exist which is out of all proportion to the requirements that may fall upon it in
the liquidation of the company-NO MERIT This matter may be left to the discretion of the board of directors or
to legislative action if it should be deemed expedient to require the gradual suppression of reserve funds as
the time for dissolution approaches. It is no matter for judicial interference and much less could the resumption
of the franchise be justified on this ground.
16) That various outstanding loans have been made by the respondent to corporations and partnerships and
such entities subscribed to respondents shares for the sole purpose of obtaining such loans-NO MERIT Sec.
173 of Corpo Law declares that any person may become a stockholder in building and loan associations. The
phrase ANY PERSON does not prevent a finding that the phrase may not be taken in its proper and broad sense
of either a natural or artificial person.
17) That in disposing real estate purchased by it, some of the properties were sold on credit and the persons
and entities to which it was sold are not members nor shareholders nor were they made members or
shareholders, contrary to the provision of Corpo Law requiring requiring loans to be stockholders only- NOT
SUSTAINED The law does not prescribe that the property must be sold for cash or that the purchaser shall be a
shareholder in the corporation. Such sales can be made upon the terms and conditions approved by the
parties. Respondent is enjoined in the future from administering real property not owned by itself, except as
may be permitted to it by contract when a borrowing shareholder defaults in his obligation. In all other
respects, the complaint is DISMISSED.

68
3B-Corporation Law

Topic:
Subtopic:
Provision:
Case Name:

By-Laws
Effect of Non-Submission of By-Laws
Sections 48, Corporation Code
Sawadjaan v Court of Appeals

FACTS:

Sappari K. Sawadjaan was among the first employees of the Philippine Amanah Bank (PAB) when it was
created by virtue of Presidential Decree No. 264 on 02 August 1973.

He started as a security guard and was eventually promoted to a loans analyst. In February 1988, while
designated as appraiser/investigator, Sawadjaan was assigned to inspect the properties offered as collaterals
by Compressed Air Machineries and Equipment Corporation (CAMEC) for a credit line of Five Million Pesos
(P5,000,000.00).

On the basis of his Inspection and Report, the PAB granted the loan application. When the loan matured on 17
May 1989, CAMEC requested an extension of 180 days, but was granted only 120 days to repay the loan.
Sawadjaan was then promoted to Loans Analyst I on 01 July 1989. In January 1990, Congress passed Republic
Act 6848 creating the AIIBP and repealing P.D. No. 264 (which created the PAB).

All assets, liabilities and capital accounts of the PAB were transferred to the AIIBP, and the existing personnel
of the PAB were to continue to discharge their functions unless discharged. In the reorganization, Sawadjaan
was among the personnel retained by the AIIBP. Upon failure of CAMEC to pay, the bank, now AIIBP, discovered
the ff: the TCT offered by CAMEC was spurious, the property described therein was inexistent, and that the
property covered by another TCT had a prior existing mortgage in favor of one Divina Pablico. On 08 June
1993, the Board of Directors of the AIIBP created an Investigating Committee to look into the CAMEC
transaction, which had cost the bank P6,000,000.00 in losses.

The Board eventually held Sawadjaan liable for the administrative offense of conduct prejudicial to the best
interest of the service and imposed on him the penalty from (originally) dismissal to suspension for 6 months.

Sawadjaan appealed the decision but the court affirmed the banks finding. Sawadjaan filed a motion for new
trial claiming that he had recently discovered that at the time his employment was terminated, the AIIBP had
not yet adopted its corporate by-laws.
ISSUE:

W/N the alleged Islamic Bank has no valid by-laws and has lost its juridical personality as a corporation on 16
April 1990 W/N Islamic Bank and its alleged Board of Directors have no jurisdiction to act in the manner they
did in the absence of a valid by-laws;
RULING:

AIIBP has a juridical personality to act as corporation! The AIIBP was created by Rep. Act No. 6848. It has a
main office where it conducts business, has shareholders, corporate officers, a board of directors, assets, and
personnel.

It is, in fact, here represented by the Office of the Government Corporate Counsel, "the principal law office of
government-owned corporations, one of which is respondent bank." At the very least, by its failure to submit
its by-laws on time, the AIIBP may be considered a de facto corporation whose right to exercise corporate
powers may not be inquired into collaterally in any private suit to which such corporations may be a party.

Moreover, a corporation which has failed to file its by-laws within the prescribed period does not ipso facto lose
its powers as such. The SEC Rules on Suspension/Revocation of the Certificate of Registration of Corporations,
details the procedures and remedies that may be availed of before an order of revocation can be issued. There
is no showing that such a procedure has been initiated in this case.

In any case, petitioners argument is irrelevant because this case is not a corporate controversy, but a labor
dispute; and it is an employers basic right to freely select or discharge its employees, if only as a measure of
self-protection against acts inimical to its interest. Regardless of whether AIIBP is a corporation, a partnership,
a sole proprietorship, or a sari-sari store, it is an undisputed fact that AIIBP is the petitioners employer.

AIIBP chose to retain his services during its reorganization, controlled the means and methods by which his
work was to be performed, paid his wages, and, eventually, terminated his services. SECTION 21 Sec. 21.
Corporation by Estoppel. All persons who assume to act as a corporation knowing it to be without authority to
do so shall be liable as general partners for all debts, liabilities and damages incurred or arising as a result
thereof: Provided, however, that when any such ostensible corporation is sued on any transaction entered by it
as a corporation or on any tort committed by it as such, it shall not be allowed to use as a defense its lack of
corporate personality. One who assumes an obligation to an ostensible corporation as such, cannot resist
performance thereof on the ground that there was in fact no corporation.

69
3B-Corporation Law

Topic:
Board of Directors/Trustees/Officers
Subtopic:
Powers of the Board
Provision:
Section 23, Corporation Code
Case Name:
Robern Development Corp. v Peoples Landless Association
Powers Of The Board
Robern Development Corporation VS. Peoples Landless Association (G.R. No. 173622, March 11, 2013)

Al-Amanah owned a 2000-square meter from which thru its officer-in-charge Febe O. Dalig, asked some of the
members of People's Landless Association to desist from building their houses on the lot and to vacate the,
unless they are interested to buy the same.
The informal settlers thus expressed their interest to buy the lot at P100.00 per square meter, which AlAmanah turned down for being far below its asking price.
Consequently, Al-Amanah reiterated its demand to the informal settlers to vacate the lot.
Through a letter the informal settlers together with other members comprising PELA offered to purchase the
lot for P300,000.00, half of which shall be paid as down payment and the remaining half to be paid within one
year. In the lower portion of the said letter, Al- Amanah made the following annotation:
Note: Subject offer has been acknowledged/received but processing to take effect upon putting up of
the partial amt. of P150,000.00 on or before April 15, 1993.

PELA had deposited P150,000.00 as evidenced by four bank receipts.


For the first three receipts, the bank labelled the payments as "Partial deposit on sale of TCT No. 138914",
while it noted the 4th receipt as "Partial/Full payment on deposit on sale of A/asset TCT No. 138914."
PELA members remained in the property and introduced further improvements.
Al-Amanah, thru Davao Branch Manager Abraham D. Ututalum-Al Haj, wrote then PELA President Bonifacio
Cuizon, Sr. informing him of the Head Offices disapproval of PELAs offer to buy the said 2,000-square meter
lot.
Al-Amanah sent similarly worded letters to 19 PELA members demanding that they vacate the lot.
In a letter of PELA, through Atty. Pedro S. Castillo, replied that it had already reached an agreement with AlAmanah regarding the sale of the subject lot based on their offered price. Dear Mr.
Acting on Roberns undated written offer, Al-Amanah issued a Recommendation Sheet addressed to its Board
Operations Committee, indicating therein that Robern is interested to buy the lot for P400,000.00; that it has
already deposited 20% of the offered purchase price; that it is buying the lot on "as is" basis; and, that it is
willing to shoulder the relocation of all informal settlers therein.
The Head Office informed the Davao Branch Manager that the Board Operations Committee had accepted
Roberns offer, from which Robern was informed of the acceptance.
Al-Amanah stressed that it is Roberns responsibility to eject the occupants in the subject lot, if any, as well as
the payment of the remaining amount within 15 days; otherwise, the P80,000.00 deposit shall be forfeited.
In a letter Robern expressed to Al-Amanah its uncertainty on the status of the subject lot because PELA made a
representation with their office bringing with them copies of official receipts issued by Al-Amanah.
Robern also requested for a definite statement from the bank on whether the official receipts being brandished
by PELA are genuine or not.
To convince Robern that it has no existing contract with PELA, Al-Amanah furnished it with copies of the Head
Offices rejection letter of PELAs bid, the demand letters to vacate, and the proof of consignment of PELAs
P150,000.00 deposit to the RTC of Davao City that PELA refused to withdraw.
Al-Amanah requested for assistance for the removal of the houses not only from the Office of the City Engineer
but also from Mayor Rodrigo Duterte.
Robern paid the balance of the purchase price. The Deed of Sale over the realty was executed and Title was
issued in Roberns name the following day. A week later, PELA consigned P150,000.00 in the RTC of Davao
City.
PELA wrote to Al-Amanah asking the latter to withdraw the amount consigned. Three months later, as its
members were already facing eviction and possible demolition of their houses, and in order to protect their
rights as vendees, PELA filed a suit for Annulment and Cancellation of Void Deed of Sale against Al-Amanah, its
Director Engr. Farouk Carpizo ,OIC Dalig, Robern, and Roberns President and General Manager, petitioner
Rodolfo Bernardo before the RTC of Davao City. It insisted that as early as March 1993 it has a perfected
contract of sale with Al-Amanah. However, in an apparent act of bad faith and in cahoots with Robern, AlAmanah proceeded with the sale of the lot despite the prior sale to PELA.

70
3B-Corporation Law

The trial court granted PELAs prayer for a TRO, which as eventually Affirmed by CA
Al-Amanah and Engr. Carpizo claimed that the bank has every right to sell its lot to any interested buyer with
the best offer and thus they chose Robern. They clarified that the P150,000.00 PELA handed to them is not
part of the payment but merely a deposit in connection with its offer. They asserted that PELA was properly
apprised that its offer to buy was subject to the approval of Al- Amanahs Head Office.
It was stressed that Al- Amanah never entered into a sale with PELA for there was no perfected agreement as
to the price since the Head Office rejected.
RTC dismissed PELAs Complaint. It ruled that being a corporation, only Al- Amanahs board of directors can
bind the bank with third persons involving the sale of its property. Thus, the purported offer made by AlAmanahs OIC, who was never conferred authority by the board of directors to sell the lot, cannot bind the
bank. In contrast, Unlike when it accepted Roberns offered price, it was duly approved by the board of
directors, giving birth to a perfected contract of sale between Al-Amanah and Robern.
CA Reversed RTCs Decision

ISSUE: W/N there was a perfected contract of sale between PELA and Al-Amanah?
Ruling: No, there is no perfected contract of sale between PELA and Al-Amanah

A contract of sale is perfected at the moment there is a meeting of minds upon the thing which is the object of
the contract and upon the price.
For a contract of sale to be valid, all of the following essential elements must concur: "a) consent or meeting
of the minds; b) determinate subject matter; and c) price certain in money or its equivalent."
It is undisputed, and PELA even acknowledge that OIC Dalig made it clear that the acceptance of the offer
notwithstanding the deposit is subject to the approval of the Head Office. Recognizing the corporate nature of
the bank and that the power to sell its real properties is lodged in the higher authorities. No false
representation on her part.
The transaction between Al-Ahmanah and PELA remained in the negotiation stage. The offer never
materialized into a perfected sale.
There was no approval from the Board Operations Committee that they accepted the sale of the lot to PELA
but unlike in Roberns case an approval was obtained through the Board Operations Committee for the sale of
the said lot.

71
3B-Corporation Law

Topic:
Board of Directors/Trustees/Officers
Subtopic:
Powers of the Board
Provision:
Section 23, Corporation Code
Case Name:
Heirs of Fausto Ignacio v Home Bankers Savings and Trust Company
Powers Of The Board
Heirs Of Fausto C. Ignacio Vs. Home Bankers Savings and Trust Company (G.R. No. 177783, January 23,
2013)

Fausto C. Ignacio mortgaged the properties to Home Bankers Savings and Trust Company as security for a loan
extended by the Bank.
Ignacio defaulted in the payment of the loan, the property was foreclosed and subsequently sold to the Bank in
a public auction
Ignacio offered to repurchase the property. Universal Properties Inc., the banks collecting agent sent Ignacio a
letter which contained the terms of the repurchase.
Ignacio annotated in the letter new terms and conditions. He claimed that these were verbal agreements
between himself and the Banks collection agent, UPI.
No repurchase agreement was finalized between Ignacio and the Bank. Thereafter the Bank sold the property
to third parties.
Ignacio then filed an action for specific performance against the Bank for the reconveyance of the properties
after payment of the balance of the purchase price. He argued that there was implied acceptance of the
counter-offer of the sale through the receipt of the terms by representatives of UPI.
The Bank denied that it gave its consent to the counter-offer of Ignacio. It countered that it did not approve the
unilateral amendments placed by Ignacio.
RTC: Render decision in favor Ignacio.
CA Reversed the RTCs decision.

Issue: W/N a contract for the repurchase of the foreclosed properties was perfected between petitioner and
respondent bank.
Ruling: No. a contract of repurchase was not perfected

Bank as a corporation can only exercise its powers and transact business through its board of directors or
officers and agents authorized by a board resolution or its by-laws.
A person representing the corporation in negotiations must be authorized by the corporation to accept the
counter-offer to a sale. Since the Bank did not accede to the counter proposal of Ignacio, there was no valid
acceptance of the offer.
Section 23 of the Corporation Code expressly provides that the corporate powers of all corporations shall be
exercised by the board of directors. Just as a natural person may authorize another to do certain acts in his
behalf, so may the board of directors of a corporation validly delegate some of its functions to individual
officers or agents appointed by it.
Contracts or acts of a corporation must be made either by the board of directors or by a corporate agent duly
authorized by the board. Absent such valid delegation/authorization, the rule is that the declarations of an
individual director relating to the affairs of the corporation, but not in the course of, or connected with, the
performance of authorized duties of such director, are held not binding on the corporation.
A corporation can only execute its powers and transact its business through its Board of Directors and through
its officers and agents when authorized by a board resolution or its by-laws.
An agent cannot bind a corporation in any contract without delegation of powers from the board. Mere
communication of modified terms to a bank agent who gave his assent has no effect on the corporation.
In the absence of conformity or acceptance by properly authorized bank officers of petitioners counterproposal, no perfected repurchase contract was born out of the talks or negotiations between petitioner and
Mr. Lazaro and Mr. Fajardo.

72
3B-Corporation Law

Topic:
Board of Directors/Trustees/Officers
Subtopic:
Business Judgment Rule
Provision:
Section 23, Corporation Code
Case Name:
Gamboa v Victoriano
RICARDO L. GAMBOA, LYDIA R. GAMBOA, HONORIO DE 1A RAMA, EDUARDO DE LA RAMA, and the HEIRS
OF MERCEDES DE LA RAMA-BORROMEO
vs.
HON. OSCAR R. VICTORIANO as Presiding Judge of the Court of First Instance of Negros Occidental,
Branch II, BENJAMIN LOPUE, SR., BENJAMIN LOPUE, JR., LEONITO LOPUE, and LUISA U. DACLES
G.R. No. L-40620. May 5, 1979
FACTS:
The herein petitioners were sued by herein defendants to nullify the issuance of 823 shares of stock of the Inocentes
de la Rama, Inc. in favor of the petitioners.
On April 4, 1972, the respondents, are the owners of 1,328 shares of stock of the Inocentes de la Rama, Inc., a
domestic corporation, with an authorized capital stock of 3,000 shares, with a par value of P100.00 per share, 2,177 of
which were subscribed and issued, thus leaving 823 shares unissued. Then President and Vice-President of the
corporation, respectively, the defendants Mercedes R. Borromeo, Honorio de la Rama, and Ricardo Gamboa, remaining
members of the board of directors of the corporation, in order to forestall the takeover by the plaintiffs of the aforenamed corporation, surreptitiously met and elected Ricardo L. Gamboa and Honorio de la Rama as president and vicepresident of the corporation, respectively, and passed a resolution authorizing the sale of the 823 unissued shares of
the corporation to the defendants, at par value, after which the petitioners were elected to the board of directors of
the corporation.
The respondents claimed that the sale of the unissued 823 shares of stock of the corporation was in violation of the
plaintiffs' and pre-emptive rights and made without the approval of the board of directors representing 2/3 of the
outstanding capital stock, and is in disregard of the strictest relation of trust existing between the defendants, as
stockholders. The respondents prayed that a writ of preliminary injunction be issued restraining the defendants from
committing, or continuing the performance of an act tending to prejudice, diminish or otherwise injure the plaintiffs'
rights in the corporate properties and funds of the corporation, and from disposing, transferring, selling, or otherwise
impairing the value of the 823 shares of stock illegally issued. The respondent court granted the prayer.
ISSUES:
Whether or not the issuance of a writ of preliminary attachment was proper.
RULING:
NO. The claim of the petitioners, in their Addendum to the motion for reconsideration of the order denying the motion
to dismiss the complaint, questioning the trial court's jurisdiction on matters affecting the management of the
corporation, is without merit. The well-known rule is that courts cannot undertake to control the discretion of the board
of directors about administrative matters as to which they have legitimate power of, action and contracts intra
vires entered into by the board of directors are binding upon the corporation and courts will not interfere unless such
contracts are so unconscionable and oppressive as to amount to a wanton destruction of the rights of the minority. In
the instant case, the plaintiffs aver that the defendants have concluded a transaction among themselves as will result
to serious injury to the interests of the plaintiffs, so that the trial court has jurisdiction over the case.
The petitioners further contend that the proper remedy of the plaintiffs would be to institute a derivative suit against
the petitioners in the name of the corporation in order to secure a binding relief after exhausting all the possible
remedies available within the corporation.
An individual stockholder is permitted to institute a derivative suit on behalf of the corporation wherein he holds stock
in order to protect or vindicate corporate rights, whenever the officials of the corporation refuse to sue, or are the ones
to be sued or hold the control of the corporation. In such actions, the suing stockholder is regarded as a nominal party,
with the corporation as the real party in interest. In the case at bar, however, the plaintiffs are alleging and vindicating
their own individual interests or prejudice, and not that of the corporation. At any rate, it is yet too early in the
proceedings since the issues have not been joined. Besides, misjoinder of parties is not a ground to dismiss an action.

73
3B-Corporation Law

Topic:
Subtopic:
Provision:
Case Name:

Board of Directors/Trustees/Officers
Business Judgment Rule
Section 23, Corporation Code
Philippine Association of Stock Transfer and Registry Agencies, Inc. v Court of
Appeals
Business Judgment Rule
Philippine Association Of Stock Transfer And Registry Agencies VS. CA (G.R. No. 137321, October 15, 2007)

Petitioner Philippine Association of Stock Transfer and Registry Agencies, Inc. is an association of stock transfer
agents principally engaged in the registration of stock transfers in the stock-and-transfer book of corporations.
Petitioners Board of Directors unanimously approved a resolution allowing its members to increase the
transfer processing fee they charge their clients from P45 per certificate to P75 per certificate and eventually
to P100 per certificate.
The resolution also authorized the imposition of a processing fee for the cancellation of stock certificates at
P20 per certificate.
According to petitioner, the rates had to be increased since it had been over five years since the old rates were
fixed and an increase of its fees was needed to sustain the financial viability of the association and upgrade
facilities and services.
Philippine Association of Securities Brokers and Dealers, Inc. registered its objection to the measure advanced
by petitioner and requested the SEC to defer its implementation.
The SEC advised petitioner to hold in abeyance the implementation of the increases until the matter was
cleared with all the parties concerned.
Petitioner nonetheless proceeded with the implementation of the increased fees.

Issue: W/N the SEC has the power to regulate fees?


Ruling: Yes the SEC has the power to regulate fees.

The regulatory and supervisory powers of the Commission under Section 40 of the then Revised Securities Act,
were broad enough to include the power to regulate petitioners fees.
Section 47 gave the Commission the power to enjoin motu proprio any act or practice of petitioner which could
cause grave or irreparable injury or prejudice to the investing public.
The intentional omission in the law of any qualification as to what acts or practices are subject to the control
and supervision of the SEC under Section 47 confirms the broad extent of the SECs regulatory powers over the
operations of securities-related organizations like petitioner.

74
3B-Corporation Law

Topic:
Board of Directors/Trustees/Officers
Subtopic:
Business Judgment Rule
Provision:
Section 23, Corporation Code
Case Name:
Montelibano v Bacolod-Murcia Milling
Business Judgment Rule
Montelibano VS Bacolod-Murcia Milling

Petitioners-appellants, Alfredo Montelibano, Alejandro Montelibano, and the Limited co-partnership Gonzaga
and Company, had been and are sugar planters
They adhered to the defendants sugar central mill under identical milling contracts. The said contracts were
stipulated to be in force for 30 years and provided that the resulting product should be divided in the ratio of
45% for the mill and 55%for the planters.
Petitioner proposed to execute an amended milling contract, increasing the planters share to 60% of the
manufactured sugar and resulting molasses, besides other concessions, but extending the operation of the
milling contract from the original 30 years to 45 years.
The Board of Directors of the appellee Bacolod-Murcia Milling Co., Inc., adopted a resolution granting further
concessions to the planters over and above those contained in the printed Amended Milling Contract.
The appellants initiated the present action, contending that three Negros sugar centrals with a total annual
production exceeding one-third of the production of all the sugar central mills in the province, had already
granted increased participation (of 62.5%) to their planters, and that under the resolution the appellee had
become obligated to grant similar concessions to the plaintiffs.
The appellee Bacolod-Murcia Milling Co., inc., resisted thec laim, and defended by urging that the stipulations
contained in the resolution were made without consideration; that the resolution in question was, therefore,
null and void abinitio, being in effect a donation that was ultra vires and beyond the powers of the corporate
directors to adopt.

Issue: W/N the board resolution is an ultra vires act and in effect a donation from the board of directors?
Ruling: No, the resolution was not an ultra vires act

There can be no doubt that the directors of the appellee company had authority to modify the proposed terms
of the Amended Milling Contract for the purpose of making its terms more acceptable to the other contracting
parties.
As the resolution in question was passed in good faith by the board of directors, it is valid and binding, and
whether or not it will cause losses or decrease the profits of the central, th ecourt has no authority to review
them.
Whether the business of a corporation should be operated at a loss during depression, or close down at a
smaller loss, is a purely business and economic problem to be determined by the directors of the corporation
and not by the court.
The appellee Bacolod-Murcia Milling Company is, under the terms of its Resolution and is duty bound to grant
similar increases to plaintiffs-appellants herein.

75
3B-Corporation Law

Topic:
Board of Directors/Trustees/Officers
Subtopic:
Business Judgment Rule
Provision:
Section 23, Corporation Code
Case Name:
Filipinas Port Services, Inc. v Go
Business Judgment Rule
Filipinas Port Services VS. GO (G.R. No. 161886 March 16, 2007)

FilPort is a domestic corporation engaged in stevedoring services


Eliodoro C. Cruz (Cruz) was president of Filipinas Port Services, Inc. (Filport) since 1968. He lost his bid for reelection in 1991. A year thereafter, Cruz wrote a letter to the corporations Board of Directors questioning the
creation of six (6) positions and the election of certain members of the board thereto.
Cruz was unhappy with the Boards action on the matter, for a year later he filed a petition with the SEC,
joined by Mindanao Terminal and Brokerage Services, Inc as co-petitioner, in a derivative suit supposedly in
representation of Filport and its stockholders.
Cruz contented that the creation of an executive committee is not provided for in the by-laws and the increase
in the emoluments of several members of the board is greatly disproportionate to the volume and character of
work of said directors.
He also questioned the re-creation of the positions of Assistant Vice President for corporate planning,
operations, finance and administration and additional positions where those holding said offices are not doing
any work but earning compensation.
These acts of mismanagement according to Cruz are detrimental to the corporation and its stockholders and
so the board must account for the amounts incurred in creating these positions and made to pay damages.
This intra-corporate case was transferred from the SEC to the Manila Regional Trial Court(RTC) and eventually
landing in the Davao RTC.
RTC found that Filports Board of Directors had the power to create positions not provided for in the by-laws
and the increases in salaries are reasonable, nevertheless it ordered the directors holding the positions of
Assistant Vice President for Corporate Planning, Special Assistant to the President and Special Assistant to the
Board Chairman to refund to the corporation the salaries they have received as such officers considering that
Filipinas Port Services is not a big corporation requiring multiple executive positions and that said positions
were just created for accommodation.
CA Reverse the decision of the RTC

ISSUE: W/N the creation of an executive committee and other offices in the corporation with corresponding
remunerations are within the powers of the Board of Directors.

HELD: Yes it is within the powers of the Board of Directors

The governing body of a corporation is its board of directors.


Section 23 of the Corporation C ode explicitly provides that unless otherwise provided therein, the corporate
powers of all corporations formed under the Code shall be exercised, all business conducted and all property
of the corporation shall be controlled and held by a board of directors. Thus, with the exception only of some
powers expressly granted by law to stockholders (or members, in case of non-stock corporations), the board of
directors (or trustees, in case of non-stock corporations) has the sole authority to determine policies, enter
into contracts, and conduct the ordinary business of the corporation within the scope of its charter, i.e., its
articles of incorporation, by-laws and relevant provisions of law.
The authority of the board of directors is restricted to the management of the regular business affairs of the
corporation, unless more extensive power is expressly conferred.
The concentration in the board of the powers of control of corporate business and of appointment of corporate
officers and managers is necessary for efficiency in any large organization. Stockholders are too numerous,
scattered and unfamiliar with the business of a corporation to conduct its business directly.
In the present case, the boards creation of the positions of Assistant Vice Presidents for Corporate Planning,
Operations, Finance and Administration, and those of the Special Assistants to the President and the Board
Chairman, was in accordance with the regular business operations of Filport as it is authorized to do so by the
corporations by-laws, pursuant to the Corporation Code.
The election of officers of a corporation is provided for under Section 25 of the Code

76
3B-Corporation Law

Sec. 25. Corporate officers, quorum. Immediately after their election, the directors of a corporation must
formally organize by the election of a president, who shall be a director, a treasurer who may or may not be a
director, a secretary who shall be a resident and citizen of the Philippines, and such other officers as may be
provided for in the by-laws.

The amended Bylaws of Filport provides the following:


Officers of the corporation, as provided for by the by-laws, shall be elected by the board of directors at their
first meeting after the election of Directors.
The officers of the corporation shall be a Chairman of the Board, President, a Vice-President, a Secretary, a
Treasurer, a General Manager and such other officers as the Board of Directors may from time to time provide,
and these officers shall be elected to hold office until their successors are elected and qualified.

The fixing of the corresponding remuneration for the positions in question is provided for in the same by-laws
of the corporation,
Under Section 35 of the Corporation Code, the creation of an executive committee must be provided for in the
bylaws of the corporation.
Notwithstanding the silence of Filports bylaws on the matter, we cannot rule that the creation of the executive
committee by the board of directors is illegal or unlawful. One reason is the absence of a showing as to the
true nature and functions of said executive committee considering that the executive committee, referred to
in Section 35 of the Corporation Code which is as powerful as the board of directors and in effect acting for the
board itself, should be distinguished from other committees which are within the competency of the board to
create at anytime and whose actions require ratification and confirmation by the board.
Another reason as held by the courts that the Board of Directors has the power to create positions not
provided for in Filports bylaws since the board is the corporations governing body, clearly upholding the
power of its board to exercise its prerogatives in managing the business affairs of the corporation.
It was pointed out that as testified to and admitted by petitioner Cruz himself, it was during his incumbency as
Filport president that the executive committee in question was created, and that he was even the one who
moved for the creation of the positions of the AVPs for Operations, Finance and Administration. By his
acquiescence and/or ratification of the creation of the aforesaid offices, Cruz is virtually precluded from suing
to declare such acts of the board as invalid or illegal. And it makes no difference that he sues in behalf of
himself and of the other stockholders.

77
3B-Corporation Law

Topic:
Subtopic:
Provision:
Case Name:

Board of Directors/Trustees/Officers
Business Judgment and the Minority Stockholders
Section 23, Corporation Code
Ching v Subic Bay Golf and Country Club, Inc.

NESTOR CHING and ANDREW WELLINGTON, Petitioners,


vs.
SUBIC BAY GOLF AND COUNTRY CLUB, INC., HU HO HSIU LIEN alias SUSAN HU, HU TSUNG CHIEH alias
JACK HU, HU TSUNG HUI, HU TSUNG TZU and REYNALD R. SUAREZ, Respondents.
G.R. No. 174353, September 10, 2014
FACTS:
On February 26, 2003, Nestor Ching and Andrew Wellington filed a Complaint with the RTC of Olongapo City on behalf
of the members of Subic Bay Golf and Country Club, Inc. (SBGCCI) against the said country club and its Board of
Directors and officers under the provisions of Presidential Decree No. 902-A in relation to Section 5.2 of the Securities
Regulation Code. The Subic Bay Golfers and Shareholders Incorporated (SBGSI), a corporation composed of
shareholders of the defendant corporation, was also named as plaintiff. The officers impleaded as defendants were the
following: (1) its President, Hu Ho Hsiu Lien alias Susan Hu; (2) its treasurer, Hu Tsung Chieh alias Jack Hu; (3)
corporate secretary Reynald Suarez; and (4) directors Hu Tsung Hui and Hu Tsung Tzu. The complaint alleged that the
defendant corporation sold shares to plaintiffs at US$22,000.00 per share, presenting to them the Articles of
Incorporation which contained the following provision:
No profit shall inure to the exclusive benefit of any of its shareholders, hence, no dividends shall be declared in their
favor. Shareholders shall be entitled only to a pro-rata share of the assets of the Club at the time of its dissolution or
liquidation.
However, an amendment to the Articles of Incorporation was approved by the Securities and Exchange Commission
(SEC), wherein the above provision was changed as follows:
No profit shall inure to the exclusive benefit of any of its shareholders, hence, no dividends shall be declared in their
favor. In accordance with the Lease and Development Agreement by and between Subic Bay Metropolitan Authority
and The Universal International Group of Taiwan, where the golf course and clubhouse component thereof was
assigned to the Club, the shareholders shall not have proprietary rights or interests over the properties of the Club.
Petitioners claimed in the Complaint that defendant corporation did not disclose to them the above amendment which
allegedly makes the shares non-proprietary, as it takes away the right of the shareholders to participate in the pro-rata
distribution of the assets of the corporation after its dissolution. According to petitioners, this is in fraud of the
stockholders who only discovered the amendment when they filed a case for injunction to restrain the corporation from
suspending their rights to use all the facilities of the club. Furthermore, petitioners alleged that the Board of Directors
and officers of the corporation did not call any stockholders meeting from the time of the incorporation, in violation of
Section 50 of the Corporation Code and the By-Laws of the corporation. Neither did the defendant directors and
officers furnish the stockholders with the financial statements of the corporation nor the financial report of the
operation of the corporation in violation of Section 75 of the Corporation Code. Petitioners also claim that on August
15, 1997, SBGCCI presented to the SEC an amendment to the By-Laws of the corporation suspending the voting rights
of the shareholders except for the five founders shares. Said amendment was allegedly passed without any
stockholders meeting or notices to the stockholders in violation of Section 48 of the Corporation Code. They
furthermore enumerated several instances of fraud in the management of the corporation allegedly committed by the
Board of Directors and officers of the corporation.
Alleging that the stockholders suffered damages as a result of the fraudulent mismanagement of the corporation,
petitioners prayed that upon the filing of this case a temporary restraining order be issued enjoining the defendants
from acting as Officers and Board of Directors of the Corporation. After hearing, a writ of preliminary injunction be
issued enjoining defendants to act as Board of Directors and Officers of the Corporation.
The RTC held that petitioners failed to exhaust their remedies within the respondent corporation itself. The RTC further
observed that petitioners Ching and Wellington were not authorized by their co-petitioner Subic Bay Golfers and
Shareholders Inc. to file the Complaint, and therefore had no personality to file the same on behalf of the said
shareholders corporation. According to the RTC, the shareholdings of petitioners comprised of two shares out of the
409 alleged outstanding shares or 0.24% is an indication that the action is a nuisance or harassment suit which may
be dismissed either motu proprio or upon motion in accordance with Section 1(b) of the Interim Rules of Procedure for
Intra-Corporate Controversies. Petitioners Ching and Wellington elevated the case to the Court of Appeals, the Court of
Appeals rendered the assailed Decision affirming that of the RTC. Hence, petitioners resort to the present Petition.

78
3B-Corporation Law

ISSUE:
Whether or not the Complaint is a nuisance or harassment suit.
HELD:
Upon a careful examination of the Complaint, the Court found that the complaint should not have been dismissed on
the ground that it is a nuisance or harassment suit. Although the shareholdings of petitioners are indeed only two out
of the 409 alleged outstanding shares or 0.24%, the Court has held that it is enough that a member or a minority of
stockholders file a derivative suit for and in behalf of a corporation. The reliefs sought in the Complaint, namely that of
enjoining defendants from acting as officers and Board of Directors of the corporation, the appointment of a receiver,
and the prayer for damages in the amount of the decrease in the value of the shares of stock, clearly show that the
Complaint was filed to curb the alleged mismanagement of SBGCCI. The causes of action pleaded by petitioners do not
accrue to a single shareholder or a class of shareholders but to the corporation itself.
However, as minority stockholders, petitioners do not have any statutory right to override the business judgments of
SBGCCIs officers and Board of Directors on the ground of the latters alleged lack of qualification to manage a golf
course. Contrary to the arguments of petitioners, Presidential Decree No. 902-A, which is entitled REORGANIZATION OF
THE SECURITIES AND EXCHANGE COMMISSION WITH ADDITIONAL POWERS AND PLACING THE SAID AGENCY UNDER
THE ADMINISTRATIVE SUPERVISION OF THE OFFICE OF THE PRESIDENT, does not grant minority stockholders a cause
of action against waste and diversion by the Board of Directors, but merely identifies the jurisdiction of the SEC over
actions already authorized by law or jurisprudence. It is settled that a stockholders right to institute a derivative suit is
not based on any express provision of the Corporation Code, or even the Securities Regulation Code, but is impliedly
recognized when the said laws make corporate directors or officers liable for damages suffered by the corporation and
its stockholders for violation of their fiduciary duties.

79
3B-Corporation Law

Topic:
Subtopic:
Provision:
Case Name:

Board of Directors/Trustees/Officers
Term of Directors: Hold-over Directors
Section 23, 24, 26, 27 Corporation Code
Valle Verde Country Club v Africa
VALLE VERDE COUNTRY CLUB, INC., et al.
vs.
VICTOR AFRICA
G.R. No. 151969, September 4, 2009

FACTS:
During the Annual Stockholders Meeting of petitioner Valle Verde Country Club, Inc. (VVCC), the following were elected
as members of the VVCC Board of Directors: Ernesto Villaluna, Jaime C. Dinglasan, Eduardo Makalintal, Francisco
Ortigas III, Victor Salta, Amado M. Santiago, Jr., Fortunato Dee, Augusto Sunico, and Ray Gamboa. In the years 1997,
1998, 1999, 2000, and 2001, however, the requisite quorum for the holding of the stockholders meeting could not be
obtained. Consequently, the above-named directors continued to serve in the VVCC Board in a hold-over capacity. Two
of the said members resigned (Makalintal and Dinglasan). After the resignation of Dinglasan, Eric Roxas was elected.
Makalintal was replaced by Jose Ramirez.
Respondent Africa, a member of VVCC, questioned the election of Roxas and Ramirez as members of the VVCC Board
with the Securities and Exchange Commission (SEC) and the Regional Trial Court. Africa alleged that the election of
Roxas was contrary to Section 29, in relation to Section 23, of the Corporation Code of the Philippines. The respective
trial courts ruled in favor of Africa.
ISSUE:
Whether or not the elections were valid.
RULING:
YES.
Section 23 of the Corporation Code declares that "the board of directors shall hold office for one (1) year until their
successors are elected and qualified," we construe the provision to mean that the term of the members of the board of
directors shall be only for one year; their term expires one year after election to the office. The holdover period that
time from the lapse of one year from a members election to the Board and until his successors election and
qualification is not part of the directors original term of office, nor is it a new term; the holdover period, however,
constitutes part of his tenure. Corollary, when an incumbent member of the board of directors continues to serve in a
holdover capacity, it implies that the office has a fixed term, which has expired, and the incumbent is holding the
succeeding term.
After the lapse of one year from his election as member of the VVCC Board in 1996, Makalintals term of office is
deemed to have already expired. That he continued to serve in the VVCC Board in a holdover capacity cannot be
considered as extending his term. This holdover period, however, is not to be considered as part of his term, which, as
declared, had already expired.
With the expiration of Makalintals term of office, a vacancy resulted which, by the terms of Section 29 of the
Corporation Code, must be filled by the stockholders of VVCC in a regular or special meeting called for the purpose. As
correctly pointed out by the RTC, when remaining members of the VVCC Board elected Ramirez to replace Makalintal,
there was no more unexpired term to speak of, as Makalintals one-year term had already expired. Pursuant to law, the
authority to fill in the vacancy caused by Makalintals leaving lies with the VVCCs stockholders, not the remaining
members of its board of directors.

80
3B-Corporation Law

Topic:
Subtopic:
Provision:
Case Name:

Board of Directors/Trustees/Officers
Term of Directors: Hold-over Directors
Section 23, 24, 26, 27 Corporation Code
Barayuga v Adventist University of the Philippines
BARAYUGA VS. ADVENTIST UNIVERSITY OF THE PHILIPPINES
G.R. NO. 168008

Facts:
AUP is a non-stock and non-profit domestic educational institution incorporated under Philippine laws was directly
under the North Philippine Union Mission (NPUM) of the Southern Asia Pacific Division of the Seventh Day Adventists.
During the 3rd Quinquennial Session of the General Conference of Seventh Day Adventists held f, the NPUM Executive
Committee elected the members of the Board of Trustees of AUP, including the Chairman and the
Secretary. Respondent Nestor D. Dayson was elected Chairman while the petitioner was chosen Secretary.
Following the conclusion of the 3rd Quinquennial Session, the Board of Trustees appointed the petitioner President of
AUP. During his tenure ( November 11 to November 13, 2002) a group from the NPUM conducted an external
performance audit. The audit revealed the petitioners autocratic management style, like making major decisions
without the approval or recommendation of the proper committees, including the Finance Committee; and that he had
himself done the canvassing and purchasing of materials and made withdrawals and reimbursements for expenses
without valid supporting receipts and without the approval of the Finance Committee. The audit concluded that he had
committed serious violations of fundamental rules and procedure in the disbursement and use of funds. The NPUM
Upon receipt of the CGAS report that confirmed the initial findings of the auditors informed the petitioner of the
findings and required him to explain.
In the January 27, 2003 special meeting, the members voted to remove him as President because of his serious
violations of fundamental rules and procedures in the disbursement and use of funds as revealed by the special audit.
The petitioner brought his suit for injunction and damages in the RTC, with prayer for the issuance of a temporary
restraining order against the Board of Trustees. He alleged that:
1. He was relieved as President without valid grounds despite his five-year term by the Board of Trustees;
2. that the Board of Trustees had thereby acted in bad faith; and
3. That his being denied ample and reasonable time to present his evidence deprived him of his right to due
process.
The respondents denied the allegations of the petitioner, and claimed that petitioner had been validly removed for
cause and was given the opportunity to be heard in his defense.
Trial Court: granted the TRO
Court of Appeals: reversed the RTC decision
Issue :
Whether or not petitioner has a vested right in office
Held:
In AUPs case, its amended By-Laws provided the term of the members of the Board of Trustees, and the period within
which to elect the officers, thusly:
Board of Trustees
Section 1. At the first meeting of the members of the corporation, and thereafter every two
years, a Board of Trustees shall be elected. It shall be composed of fifteen members in good and
regular standing in the Seventh-day Adventist denomination, each of whom shall hold his office
for a term of two years, or until his successor has been elected and qualified. If a trustee
ceases at any time to be a member in good and regular standing in the Seventh-day Adventist
denomination, he shall thereby cease to be a trustee.
Officers
Section 1. Election of officers. At their organization meeting, the members of the Board of
Trustees shall elect from among themselves a Chairman, a Vice-Chairman, a President, a Secretary,

81
3B-Corporation Law

a Business Manager, and a Treasurer. The same persons may hold and perform the duties of more than
one office, provided they are not incompatible with each other.
In light of foregoing, the members of the Board of Trustees were to serve a term of office of only two years; and the
officers, who included the President, were to be elected from among the members of the Board of Trustees during their
organizational meeting, which was held during the election of the Board of Trustees every two years. Naturally, the
officers, including the President, were to exercise the powers vested by Section 2 of the amended By-Laws for a term
of only two years, not five years.
Ineluctably, the petitioner, having assumed as President of AUP on January 23, 2001, could serve for only two years, or
until January 22, 2003. By the time of his removal for cause as President on January 27, 2003, he was already
occupying the office in a hold-over capacity, and could be removed at any time, without cause, upon the election or
appointment of his successor. His insistence on holding on to the office was untenable, therefore, and with more
reason when one considers that his removal was due to the loss of confidence on the part of the Board of Trustees.

82
3B-Corporation Law

Topic:
Subtopic:
Provision:
Case Name:

Board of Directors/Trustees/Officers
Term of Directors: Removal and Replacement of Directors or Trustees
Sections 28 and 29, Corporation Code
Tan v Sycip

Tan v. Sycip
Grace Christian High School (GCHS) is a nonstock, non-profit educational corporation w/ 15 regular members, who also
constitute the board of trustees. During the annual members meeting only 11 living member-trustees, as 4 had
already died. 7 attended the meeting through their respective proxies. The meeting was convened and chaired by
Atty. Sabino Padilla Jr. over the objection of Atty. Antonio C. Pacis, who argued that there was no quorum. In the
meeting, Petitioners Ernesto Tanchi, Edwin Ngo, Virginia Khoo, and Judith Tan were voted to replace the 4 deceased
member-trustees. SEC: meeting void due to lack of quorum (NOT living but based on AIC)
ISSUE: Whether or not dead members should still be counted in the quorum
HELD: NO. remaining members of the board of trustees of GCHS may convene and fill up the vacancies in the board
Ratio:
Except as provided, the vote necessary to approve a particular corporate act as provided in this Code shall be deemed
to refer only to stocks with voting rights:
o

1. Amendment of the articles of incorporation;

2. Adoption and amendment of by-laws;

3. Sale, lease, exchange, mortgage, pledge or other disposition of all or substantially all of the
corporation property;

4. Incurring, creating or increasing bonded indebtedness;

5. Increase or decrease of capital stock;

6. Merger or consolidation of the corporation with another corporation or other corporations;

7. Investment of corporate funds in another corporation or business in accordance with this Code; and

8. Dissolution of the corporation.

Quorum in a members meeting is to be reckoned as the actual number of members of the corporation. Section 91 of
the Corporation Code: termination extinguishes all the rights of a member of the corporation, unless otherwise
provided in the articles of incorporation or the bylaws. Whether or not "dead members" are entitled to exercise their
voting rights (through their executor or administrator), depends on those articles of incorporation or bylaws

By-Laws of GCHS: membership in the corporation shall be terminated by the death of the
member

With 11 remaining members, the quorum = 6.

SECTION 29. Vacancies in the office of director or trustee. -- Any vacancy occurring in the board of directors or trustees
other than by removal by the stockholders or members or by expiration of term, may be filled by the vote of at least a
majority of the remaining directors or trustees, if still constituting a quorum; otherwise, said vacancies must be filled
by the stockholders in a regular or special meeting called for that purpose. A director or trustee so elected to fill a
vacancy shall be elected only for the unexpired term of his predecessor in office.

83
3B-Corporation Law

The filling of vacancies in the board by the remaining directors or trustees constituting a quorum is merely permissive,
not mandatory either by the remaining directors constituting a quorum, or by the stockholders or members in a regular
or special meeting called for the purpose
By-Laws of GCHS prescribed the specific mode of filling up existing vacancies in its board of directors; that is, by a
majority vote of the remaining members of the board

84
3B-Corporation Law

Topic:
Board of Directors/Trustees/Officers
Subtopic:
Duties of Directors: Obedience, Diligence, and Loyalty
Provision:
Sections 31, 32, 33, and 34, Corporation Code
Case Name:
Sanchez v Republic
Sanchez vs. Republic
Facts:
Petitioners Khan and Sanchez are key officers of University of Life Complex (ULFI). The complex was guilt using public
fund and the management was given to ULFI on the condition that the latter would remit the revenue to the
government through DECS, net of all expenses. DECS demanded the payment of its arrears but petitioners failed to
remit the same. This promoted DECS to file a collection suit against petitioners. Petitioners counter that they are mere
employees and thus cannot be held personally liable.
Issue: Whether or not petitioner Sanchez, a director and chief executive officer of ULFI, can be held liable in damages
under Section 31 of the Corporation Code for gross neglect or bad faith in directing the corporation's affairs;
andcralawlibrary
Held: They can be held personally liable.
Ratio:
Petitioner Sanchez claims that there is no ground for the courts below to pierce the veil of corporate identity and hold
him and Kahn, who were mere corporate officers, personally liable for ULFI's obligations to the DECS. But this is not a
case of piercing the veil of corporate fiction. The DECS brought its action against Sanchez and Kahn under Section 31
of the Corporation Code, which should not be confused with actions intended to pierce the corporate fiction.
Section 31 of the Corporation Code makes directors-officers of corporations jointly and severally liable even to third
parties for their gross negligence or bad faith in directing the affairs of their corporations. Thus:
Sec. 31. Liability of directors, trustees or officers. - Directors or trustees who willfully and knowingly vote for or assent
to patently unlawful acts of the corporation or who are guilty of gross negligence or bad faith in directing the
affairs of the corporation or acquire any personal or pecuniary interest in conflict with their duty as such directors
or trustees shall be liable jointly and severally for all damages resulting therefrom suffered by the corporation, its
stockholders or members and other persons. (Emphasis supplied)cralawlibrary
The DECS does not have to invoke the doctrine of piercing the veil of corporate fiction. Section 31 above expressly lays
down petitioner Sanchez and Kahn's liability for damages arising from their gross negligence or bad faith in directing
corporate affairs. The doctrine mentioned, on the other hand, is an equitable remedy resorted to only when the
corporate fiction is used, among others, to defeat public convenience, justify wrong, protect fraud or defend a crime

85
3B-Corporation Law

Topic:
Subtopic:
Provision:
Case Name:

Board of Directors/Trustees/Officers
Corporate Officers
Section 25, Corporation Code
Marc II Marketing, Inc. v Alfredo Joson

Mar II Marketing, Inc v. Alfredo Joson


Facts: Respondent Joson was appointed as General Manager of Marc II Marketing. Thru their agreement, respondent
Joson was granted a 30% share in the profit of the company. However, because of poor sales, respondent company
ceased operation and served termination notice to respondent. Respondent filed a case against petitioner for illegal
dismissal. Petitioner counters that there is no employer-employee relationship because as a General Manager, he is a
corporate officer thus Labor Arbiter has no jurisdiction on the case.
Issue: Whether the position of General Manager is part of corporate officer divesting the LA from acquiring jurisdiction
of the case.
Held: Petition dismissed. He is not a corporate officer.
Ratio:
A careful perusal of petitioner corporations by-laws, particularly paragraph 1, Section 1, Article IV, would explicitly
reveal that its corporate officers are composed only of: (1) Chairman; (2) President; (3) one or more Vice-President; (4)
Treasurer; and (5) Secretary. The position of General Manager was not among those enumerated.
Paragraph 2, Section 1, Article IV of petitioner corporations by-laws, empowered its Board of Directors to appoint such
other officers as it may determine necessary or proper. It is by virtue of this enabling provision that petitioner
corporations Board of Directors allegedly approved a resolution to make the position of General Manager a corporate
office, and, thereafter, appointed respondent thereto making him one of its corporate officers. All of these acts were
done without first amending its by-laws so as to include the General Manager in its roster of corporate officers.
With the given circumstances and in conformity with Matling Industrial and Commercial Corporation v. Coros, this
Court rules that respondent was not a corporate officer of petitioner corporation because his position as General
Manager was not specifically mentioned in the roster of corporate officers in its corporate by-laws. The enabling clause
in petitioner corporations by-laws empowering its Board of Directors to create additional officers, i.e., General
Manager, and the alleged subsequent passage of a board resolution to that effect cannot make such position a
corporate office. Matling clearly enunciated that the board of directors has no power to create other corporate offices
without first amending the corporate by-laws so as to include therein the newly created corporate office. Though the
board of directors may create appointive positions other than the positions of corporate officers, the persons
occupying such positions cannot be viewed as corporate officers under Section 25 of the Corporation Code. In view
thereof, this Court holds that unless and until petitioner corporations by-laws is amended for the inclusion of General
Manager in the list of its corporate officers, such position cannot be considered as a corporate office within the realm
of Section 25 of the Corporation Code.

86
3B-Corporation Law

Topic:
Subtopic:
Provision:
Case Name:

Board of Directors/Trustees/Officers
Corporate Officers
Section 25, Corporation Code
Violeta Tudtud Banate v Phil. Countryside Rural Bank

Facts:
Spouses Maglasang obtained a loan from PCRB and mortgaged the house and lot owned by their daughter and son-inlaw, the spouses Cortel. They also obtained 2 other loans secured by their other properties. Before the maturity of
their said loan, Sps. Maglasang alleged that they requested PCRB to permit them to sell their subject properties if they
will pay the full amount of the loans. Allegedly, PCRB, acting through its Branch Manager, Pancrasio Mondigo, verbally
agreed to their request hence they sold their property to Violeta Banate and used the proceeds to pay the loans. PCRB
gave the owners duplicate certificate of title and Banate secured a title in her name, however the mortgage lien was
carried over hence they requested PCRB to release the morthgaged which the bank refused. They then filed an action
for specific performance. The lower court ruled in favor of Banate but CA reversed the decision on the ground that It
ruled that Mondigo cannot orally amend the mortgage contract hence it was not binding with PCRB.
Issue: WON the verbal agreement with the banks corporate officer binds PCRB.
Held:
Section 23 of the Corporation Code expressly provides that the corporate powers of all corporations shall be
exercised by the board of directors. The power and the responsibility to decide whether the corporation should enter
into a contract that will bind the corporation are lodged in the board, subject to the articles of incorporation, bylaws, or
relevant provisions of law. In the absence of authority from the board of directors, no person, not even its officers, can
validly bind a corporation. The authority of a corporate officer or agent in dealing with third persons may be actual or
apparent. The doctrine of apparent authority, on the other hand, with special reference to banks, had long been
recognized in this jurisdiction. The existence of apparent authority may be ascertained through: 1) the general manner
in which the corporation holds out an officer or agent as having the power to act, or in other words, the apparent
authority to act in general, with which it clothes him; or 2) the acquiescence in his acts of a particular nature, with
actual or constructive knowledge thereof, within or beyond the scope of his ordinary powers.
In this case, Modrigo is not clothed with apparent authority to bind PCRB and the latter did not ratified the alleged acts
of Modrigo. Although a branch manager, is the general agent and is in general charge of the corporation, with
apparent authority commensurate with the ordinary business entrusted him and the usual course and conduct thereof
yet the power to modify or nullify corporate contracts remains generally in the board of directors.

87
3B-Corporation Law

Topic:
Subtopic:
Provision:
Case Name:

Board of Directors/Trustees/Officers
Corporate Officers
Section 25, Corporation Code
Cosare v Broadcom Asia, Inc.

Facts:
Cosare claimed he was employed as a salesman by Arevalo and incorporator of Broadcom Asian Inc. He became the
AVP of Sales under Abiog, his superior. Cosare sent letters to Arevalo about the anomalies allegedly committed by
Abiog against the company. Arevalo failed to act on Cosares accusations and instead was asked to tender his
resignation charging him of serious misconduct and willful breach of trust. He was also given a show-cause memo. He
was not allowed to go inside the premises of the corporation and was made to wait in the receiving area. After
sometime and he never got any feedback from the corporation, he left the premises and asked the assistance of the
Barangay Officers. He then filed for illegal dismissal with the DOLE. The LA ruled in favor of Bancom Inc.
NLRC
reversed the decision stating e fact that Cosare was suspended from using the assets of Broadcom hence Cosare did
not abandon his work. Bancom then on appeal raised the issue that the issue is an intra-corporate controversy which
was within the jurisdiction of the RTC, instead of the LA since it involved a complaint against a corporation filed by a
stockholder, who, at the same time, was a corporate officer.
Issue: WON the contention of Bancom Asia Inc. is correct.
Held:
The mere fact that Cosare was a stockholder and an officer of Broadcom at the time the subject controversy developed
failed to necessarily make the case an intra-corporate dispute. The definition of corporate officers for the purpose of
identifying an intra-corporate controversy provides that Corporate officers are those officers of the corporation who
are given that character by the Corporation Code or by the corporations by-laws. There are three specific officers
whom a corporation must have under Section 25 of the Corporation Code. These are the president, secretary and the
treasurer. The number of officers is not limited to these three. A corporation may have such other officers as may be
provided for by its by-laws like, but not limited to, the vice-president, cashier, auditor or general manager. The number
of corporate officers is thus limited by law and by the corporations by-laws. As may be deduced from the foregoing,
there are two circumstances which must concur in order for an individual to be considered a corporate officer, as
against an ordinary employee or officer, namely: (1) the creation of the position is under the corporations charter or
by-laws; and (2) the election of the officer is by the directors or stockholders. It is only when the officer claiming to
have been illegally dismissed is classified as such corporate officer that the issue is deemed an intra-corporate dispute
which falls within the jurisdiction of the trial courts. Cosare in this case is not a corporate officer since records and the
GIS does not show he was a corporate officer.

88
3B-Corporation Law

Topic:
Subtopic:
Provision:
Case Name:

Board of Directors/Trustees/Officers
Authority of Officers: Implied Authority
Section 25, Corporation Code
Board of Liquidators v Heris of Maximo Kalaw

Facts:
Maximo M. Kalaw was the General Manager and board chairman of NACOCO. Together with other Board Members, they
approved 60 contracts with various corporations to deliver coconut products even though they were well aware that
there was a decrease in copra productivity due to several typhoons that ravaged the country. As a consequence,
NACOCO failed to deliover the said copra products and was sued by the other parties of the contracts. NACOCO
through the Board of Liquidators then sued Kalaw and the others for the damages incurred by the Corporation due to
the acts of Kalaw et al. They alleged that Kalaw et. Al. approved the said contracts without authority.
Issue: WON Kalaw and the other Board of Directors are personally liable.
Held:
Previous contracts signed by Kalaw reaped NACOCO tremendous profits. These previous contract it should be stressed,
were signed by Kalaw without prior authority from the board. Said contracts were known all along to the board
members. Nothing was said by them. The aforesaid contracts stand to prove one thing: Obviously, NACOCO board met
the difficulties attendant to forward sales by leaving the adoption of means to end, to the sound discretion of
NACOCO's general manager Maximo M. Kalaw. Our law pronounces that "[r]atification cleanses the contract from
all its defects from the moment it was constituted." By corporate confirmation, the contracts executed by Kalaw are
thus purged of whatever vice or defect they may have. Kalaw had authority to execute the contracts without need of
prior approval. Everybody, including Kalaw himself, thought so, and for a long time. Doubts were first thrown on the
way only when the contracts turned out to be unprofitable for NACOCO. Hence, Kalaw and the other Board Mmebers
are not liable as corporate officers considering he acted with implied authority.

89
3B-Corporation Law

Topic:
Subtopic:
Provision:
Case Name:
Facts:

Board of Directors/Trustees/Officers
Agency by Estoppel and Doctrine of Apparent Authority
Section 25, Corporation Code
Advance Paper Corp. v Arma Traders Corp

Advance Paper is a domestic corporation engaged in the business of producing, printing, manufacturing,
distributing and selling of various paper products. Petitioner George Haw is the President. While respondent Arma
Traders is also a domestic corporation engaged in the wholesale and distribution of school and office supplies, and
novelty products. Respondent Antonio Tan was formerly the President while respondent Uy Seng Kee Willy is the
Treasurer of Arma Traders. They represented Arma Traders when dealing with its supplier, Advance Paper, for about 14
years.

Arma Traders purchased on credit notebooks and other paper products amounting to P7,533,001.49 from
Advance Paper. Upon the representation of Tan and Uy, Arma Traders also obtained three loans from Advance Paper in
the amounts of P3,380,171.82, P1,000,000.00, and P3,408,623.94 or a total of P7,788,796.76.
As payment, Arma Traders issued 82 postdated checks payable to cash or to Advance Paper. Tan and Uy were
Arma Traders authorized bank signatories who signed and issued these checks which had the aggregate amount of
P15,130,636.87.
Advance Paper presented the checks to the drawee bank but were dishonored either for insufficiency of
funds or account closed. Despite repeated demands, however, Arma Traders failed to settle its account with
Advance Paper. The petitioners filed a complaint for collection of sum of money with application for preliminary
attachment against Arma Traders, Tan, Uy, Ting, Gui, and Ng.
Claims of the respondents:
As to the loan transactions, the respondents countered that these were the personal obligations of Tan and Uy
to Advance Paper. These loans were never intended to benefit the respondents.
The respondents also claimed that the loan transactions were ultra vires because the board of directors of
Arma Traders did not issue a board resolution authorizing Tan and Uy to obtain the loans from Advance Paper. They
claimed that the borrowing of money must be done only with the prior approval of the board of directors because
without the approval, the corporate officers are acting in excess of their authority or ultra vires. When the acts of the
corporate officers are ultra vires, the corporation is not liable for whatever acts that these officers committed in excess
of their authority. Further, the respondents claimed that Advance Paper failed to verify Tan and Uys authority to
transact business with them. Hence, Advance Paper should suffer the consequences.
RTC ordered Arma Traders to pay Advance Paper the sum of P15,321,798. The RTC held that the respondents
failed to present hard, admissible and credible evidence to prove that the sale invoices were forged or fictitious, and
that the loan transactions were personal obligations of Tan and Uy. Nonetheless, the RTC dismissed the complaint
against Tan, Uy, Ting, Gui and Ng due to the lack of evidence showing that they bound themselves, either jointly or
solidarily, with Arma Traders for the payment of its account.
CA held that the petitioners failed to prove by preponderance of evidence the existence of the purchases on
credit and loans
Arma Traders was not liable for the loan in the absence of a board resolution authorizing Tan and Uy to obtain
the loan from Advance Paper. The CA acknowledged that Tan and Uy were Arma Traders authorized bank signatories.
However, the CA explained that this is not sufficient because the authority to sign the checks is different from the
required authority to contract a loan.
Petitioner Arma Traders led the petitioners to believe that Tan and Uy had the authority to obtain loans since
the respondents left the active and sole management of the company to Tan and Uy since 1984. In fact, Ng testified
that Arma Traders stockholders and board of directors never conducted a meeting from 1984 to 1995. Citing Lipat v.
Pacific Banking Corporation, the petitioners said that if a corporation knowingly permits one of its officers or any other
agent to act within the scope of an apparent authority, it holds him out to the public as possessing the power to do
those acts; thus, the corporation will, as against anyone who has in good faith dealt with it through such agent, be
estopped from denying the agents authority.

90
3B-Corporation Law

Respondents argue that while as a general rule, a corporation is estopped from denying the authority of its
agents which it allowed to deal with the general public; this is only true if the person dealing with the agent dealt in
good faith. In the present case, the respondents claim that the petitioners are in bad faith because the petitioners
connived with Tan and Uy to make Arma Traders liable for the non-existent deliveries of notebooks and other paper
products. As to the loans, the respondents aver that these were Tan and Uys personal obligations with Advance Paper.
ISSUE:
Whether Arma Traders is liable to pay the loans applying the doctrine of apparent authority.
Held:
Yes.
HELD:
The doctrine of apparent authority provides that a corporation will be estopped from denying the agents
authority if it knowingly permits one of its officers or any other agent to act within the scope of an apparent authority,
and it holds him out to the public as possessing the power to do those acts.
In Inter-Asia Investment Industries v. Court of Appeals,78 we explained:
Apparent authority is derived not merely from practice. Its existence may be ascertained through (1) the
general manner in which the corporation holds out an officer or agent as having the power to act or, in other words the
apparent authority to act in general, with which it clothes him; or (2) the acquiescence in his acts of a particular
nature, with actual or constructive knowledge thereof, within or beyond the scope of his ordinary powers. It requires
presentation of evidence of similar act(s) executed either in its favor or in favor of other parties. It is not the quantity
of similar acts which establishes apparent authority, but the vesting of a corporate officer with the power to bind the
corporation. Also, in Peoples Aircargo and Warehousing Co., Inc. v. Court of Appeals, we ruled that the doctrine of
apparent authority is applied when the petitioner, through its president Antonio Punsalan Jr., entered into the First
Contract without first securing board approval. Despite such lack of board approval, petitioner did not object to or
repudiate said contract, thus "clothing" its president with the power to bind the corporation.
In the present petition, we do not agree with the CAs findings that Arma Traders is not liable to pay the loans
due to the lack of board resolution authorizing Tan and Uy to obtain the loans. To begin with, Arma Traders Articles of
Incorporation provides that the corporation may borrow or raise money to meet the financial requirements of its
business by the issuance of bonds, promissory notes and other evidence of indebtedness. Likewise, it states that Tan
and Uy are not just ordinary corporate officers and authorized bank signatories because they are also Arma Traders
incorporators along with respondents Ng and Ting, and Pedro Chao. Furthermore, the respondents, through Ng who is
Arma Traders corporate secretary, incorporator, stockholder and director, testified that the sole management of Arma
Traders was left to Tan and Uy and that he and the other officers never dealt with the business and management of
Arma Traders for 14 years. He also confirmed that since 1984 up to the filing of the complaint against Arma Traders, its
stockholders and board of directors never had its meeting.
Thus, Arma Traders bestowed upon Tan and Uy broad powers by allowing them to transact with third persons
without the necessary written authority from its non-performing board of directors. Arma Traders failed to take
precautions to prevent its own corporate officers from abusing their powers. Because of its own laxity in its business
dealings, Arma Traders is now estopped from denying Tan and Uys authority to obtain loan from Advance Paper.

91
3B-Corporation Law

Topic:
Subtopic:
Provision:
Case Name:

Board of Directors/Trustees/Officers
Liabilities of Directors, Trustees, and Officers
Section 31, Corporation Code
Prisma Construction and Development Corporation v Menchavez

FACTS:
Pantaleon, President and Chairman of the Board of PRISMA, obtained a P1M loan from the respondent, with a
monthly interest of P40,000 payable for 6 months or P1,240,000 under the following schedule of payments:
January 8, 1994 . P40,000.00
February 8, 1994 ... P40,000.00
March 8, 1994 ... P40,000.00
April 8, 1994 . P40,000.00
May 8, 1994 .. P40,000.00
June 8, 1994 P1,040,000.00
Total

P1,240,000.00

Pantaleon signed the promissory note in his personal capacity and as duly authorized by the Board of Directors of
PRISMA. Petitioners failed to completely pay the loan within the stipulated period and paid such amounts to
respondent:
September 8, 1994 P320,000.00
October 8, 1995.P600,000.00
November 8, 1995.....P158,772.00
January 4, 1997 P30,000.00

January 4, 1997: Petitioners had already paid a total of P1,108,772, but still has an outstanding balance of
P1,364,151, applied a 4% monthly interest.
Respondent filed a complaint for sum of money with the RTC to enforce the unpaid balance, plus 4% monthly
interest, P30,000 in attorneys fees, P1,000 per court appearance and costs of suit
RTC: petitioners liable for payment of P3,526,117.00 to Menchavez
CA: affirmed RTC Decision, but modified the interest rate from 4% per month to 12% per annum, computed
from the filing of the complaint to full payment.
Petitioners admitted the loan of P1,240,000.00, but denied the 4% monthly interest for not being provided in
the promissory note. Pantaleon also denied that he made himself personally liable and made representations that the
loan would be repaid 6 months. CA mistakenly relied on their board resolution to conclude that the parties agreed to a
4% monthly interest because the board resolution was not an evidence of a loan or forbearance of money, but merely
an authorization for Pantaleon to perform certain acts, including the power to enter into a contract of loan
ISSUE: Whether the parties agreed to the 4% monthly interest on the loan. If so, does the rate of interest apply to the
6-month payment period only or until full payment of the loan?
HELD:
Petition is meritorious. CA Decision SET ASIDE. petitioners loan of P1,000,000.00 shall bear interest of
P40,000.00 per month for six (6) months from December 8, 1993 as indicated in the promissory note. Any portion of
this loan, unpaid as of the end of the six-month payment period, shall thereafter bear interest at 12% per annum. Case
is REMANDED to RTC for proper computation.
Doctrine of Estoppel not applicable

92
3B-Corporation Law

We cannot apply the doctrine of estoppel in the present case since the facts and circumstances, as established
by the record, negate its application. Under the promissory note, what the petitioners agreed to was the payment of a
specific sum of P40,000.00 per month for six months not a 4% rate of interest per month for six (6) months on a
loan whose principal is P1,000,000.00, for the total amount of P1,240,000.00. The board resolution simply authorizes
Pantaleon to contract for a loan with a monthly interest of not more than 4%. This resolution merely embodies the
extent of Pantaleons authority to contract and does not create any right or obligation except as between Pantaleon
and the board. Again, no cause exists to place the petitioners in estoppel.
Piercing the corporate veil unfounded
The Court finds it unfounded and unwarranted for the lower courts to pierce the corporate veil of PRISMA.
The doctrine of piercing the corporate veil applies only in 3 basic instances: a) when the separate and distinct
corporate personality defeats public convenience, as when the corporate fiction is used as a vehicle for the evasion of
an existing obligation; b) in fraud cases, or when the corporate entity is used to justify a wrong, protect a fraud, or
defend a crime; or c) is used in alter ego cases, i.e., where a corporation is essentially a farce, since it is a mere alter
ego or business conduit of a person, or where the corporation is so organized and controlled and its affairs so
conducted as to make it merely an instrumentality, agency, conduit or adjunct of another corporation.
In the absence of malice, bad faith, or a specific provision of law making a corporate officer liable, such corporate
officer cannot be made personally liable for corporate liabilities.
In the present case, there is no competent and convincing evidence of any wrongful, fraudulent or unlawful act
on the part of PRISMA to justify piercing its corporate veil. While Pantaleon denied personal liability in his Answer, he
made himself accountable in the promissory note in his personal capacity and as authorized by the Board Resolution
of PRISMA. With this statement of personal liability and in the absence of any representation on the part of PRISMA that
the obligation is all its own because of its separate corporate identity, we see no occasion to consider piercing the
corporate veil as material to the case.

93
3B-Corporation Law

Topic:
Subtopic:
Provision:
Case Name:

Board of Directors/Trustees/Officers
Liabilities of Directors, Trustees, and Officers
Section 31, Corporation Code
Magaling et al v Ong

Facts:
Respondent Ong instituted with the RTC Lipa a complaint of Collection of Sum of Money in the amount of P
389,000.00 against the herein petitioners.
The complaint alleged among others that spouses Magaling is the controlling stockholders owners of Thermo
Loans & Credit Corporation, Mr. Reynaldo Magaling being the President of the corporation; that Reynaldo Magaling
induced and obtained aloan from Mr. Ong the amount of P350.000.00 with interest of 2&1/2 a month; that Reynaldo
Magaling later on issued seven (7) post dated checks where only two checks were cleared by the bank and the rest
were dishonored; that despite demands, spouses Magaling and/or Thermo & Credit Loans failed, refused and neglected
to pay; that Ong's prayer for Preliminary Attachment was granted by the Court attaching the two parcel of land owned
by the corporation. In their defense, spouses Magaling argued that Ong at his own risk invested the money with
Thermo Loans Corporation; that the promissory note was issued by the corporation and they are not signatories in the
checks issued by the corporation.
In the first decision of RTC, it ruled in favor of Ong and against the Thermo Loans Corporation. However, in the
second decision, the Court absolved spouses from any obligation or liability. Thus, Ong appealed the case to the Court
of appeal wherein the appellate Court reversed the RTC decision and declared spouses Magaling jointly and severally
liable to Peter Ong for the corporation obligation of Thermo Loans. The Court of Appeals pierced the veil of corporate
fiction and holds the spouses Magaling liable with Thermo Loans for the corporate obligations since Reynaldo Magaling
was grossly negligent in managing the affairs of the corporation hence, this petition.
Issue: Whether or not Reynaldo Magaling personally liable for corporate liability of Thermo loans corporation?
Held:
Petition is not meritorious. It is basic that the corporation is a juridical entity with legal personality separate
and distinct from those acting for and in its behalf and, in general, from the people comprising it. The general rule is
that obligations incurred by the corporation, acting through its directors, officers and employees, are its sole liabilities
and vice versa. However, there are times when solidary liabilities may be incurred and the veil of corporate fiction may
be pierced. One of the circumstances warranting the disregard of separate personality is that when directors and
trustees or, in appropriate case, the officers of a corporation acted in bad faith or with gross negligence in directing the
corporate affairs. Considering that Reynaldo Magaling was grossly negligent in directing the affairs of Thermo Loans
without due regard to the plight of its investors and thus should be held jointly and severally liable for the corporate
obligation of Thermo Loans to Peter Ong.

94
3B-Corporation Law

Topic:
Board of Directors/Trustees/Officers
Subtopic:
Liabilities of Directors, Trustees, and Officers
Provision:
Section 31, Corporation Code
Case Name:
Heirs of Fe Tan Uy v International Exchange Bank
LIABILITIES OF DIRECTORS, TRUSTEES AND OFFICERS (Sec. 31, Corp. Code)
Heirs of Fe Tan Uy vs International Exchange Bank
FACTS:

Respondent International Exchange Bank (iBank), granted loans to Hammer Garments Corporation (Hammer).
The loans were signed by Hammer President Chua (Uys husband) and was secured by a 3 rd party REM by
Goldkey Devt and a Peso Surety Agreement signed by Chua and Uy.

Hammer defaulted. iBank foreclosed some the 3rd party mortgage but was however deficient. iBank now brings
suit against Hammer, Chua, Uy, and Goldkey.

Hammer and Chua failed to file an Answer. Uy denied signing any surety agreement. Goldkey averred that its
liability is only limited to the 3 rd party REM, and that it cannot be held liable for Hammers loan since it has a
separate personality apart from Hammer.

RTC ruled among others that while Uys signature was forged on the surety agreement, she nonetheless should
be held liable for the outstanding obligation of Hammer because she was an officer and stockholder of that
corporation.

CA affirmed the RTC without explaining its ruling on the liability of Uy.

The heirs of Uy argue that the latter could not be held liable for being merely an officer of Hammer because it
was not shown that she had committed any actionable wrong or that she had participated in the transaction
between Hammer and iBank. They further claim that she had cut all ties with Hammer and her husband long
before the execution of the loan.
ISSUE:
WON Uy should be held liable for the outstanding obligation.
HELD:
NO. Piercing of the veil for Uy is not justified.
Before a director or officer of a corporation can be held personally liable for corporate obligations, however, the
following requisites must concur: (1) the complainant must allege in the complaint that the director or officer assented
to patently unlawful acts of the corporation, or that the officer was guilty of gross negligence or bad faith; and (2) the
complainant must clearly and convincingly prove such unlawful acts, negligence or bad faith.
In this case, petitioners are correct to argue that it was not alleged, much less proven, that Uy committed an act as an
officer of Hammer that would permit the piercing of the corporate veil. A reading of the complaint reveals that with
regard to Uy, iBank did not demand that she be held liable for the obligations of Hammer because she was a corporate
officer who committed bad faith or gross negligence in the performance of her duties such that the lifting of the
corporate mask would be merited. What the complaint simply stated is that she, together with her errant husband
Chua, acted as surety of Hammer, as evidenced by her signature on the Surety Agreement which was later found by
the RTC to have been forged.
Considering that the only basis for holding Uy liable for the payment of the loan was proven to be a falsified document,
there was no sufficient justification for the RTC to have ruled that Uy should be held jointly and severally liable to iBank
for the unpaid loan of Hammer. Neither did the CA explain its affirmation of the RTCs ruling against Uy. The Court
cannot give credence to the simplistic declaration of the RTC that liability would attach directly to Uy for the sole
reason that she was an officer and stockholder of Hammer.
At most, Uy could have been charged with negligence in the performance of her duties as treasurer of Hammer by
allowing the company to contract a loan despite its precarious financial position. Furthermore, if it was true, as
petitioners claim, that she no longer performed the functions of a treasurer, then she should have formally resigned as
treasurer to isolate herself from any liability that could result from her being an officer of the corporation. Nonetheless,
these shortcomings of Uy are not sufficient to justify the piercing of the corporate veil which requires that the
negligence of the officer must be so gross that it could amount to bad faith and must be established by clear and
convincing evidence. Gross negligence is one that is characterized by the lack of the slightest care, acting or failing to
act in a situation where there is a duty to act, wilfully and intentionally with a conscious indifference to the
consequences insofar as other persons may be affected.

95
3B-Corporation Law

There is no showing that Uy committed gross negligence. And in the absence of any of the aforementioned requisites
for making a corporate officer, director or stockholder personally liable for the obligations of a corporation, Uy, as a
treasurer and stockholder of Hammer, cannot be made to answer for the unpaid debts of the corporation.

96
3B-Corporation Law

Topic:
Subtopic:
Provision:
Case Name:

Board of Directors/Trustees/Officers
Liabilities of Directors, Trustees, and Officers
Section 31, Corporation Code
Domingo v Domingo

FACTS:

Vicente Domingo granted to Gregorio Domingo, a real estate broker, the exclusive agency to sell his property
in a document. Said lot has an area of 88,477 sq. m. According to the document, said lot must be sold for P2
per sq. m. Gregorio is entitled to 5% commission on the total price if the property is sold by Vicente or by
anyone else during the 30-day duration of the agency or by Vicente within 3 months from the termination of
the agency to a purchaser to whom it was submitted by Gregorio during the effectivity of the agency with
notice to Vicente.

Subsequently, Gregorio authorized Teofilo Purisima to look for a buyer without notifying Vicente. Gregorio
promised Teofilo of the 5% commission. Teofilo introduced Oscar de Leon to Gregorio as a prospective buyer.

Oscar submitted a written offer which was very much lower than the P2 per sq. m. price. Vicente directed
Gregorio to tell Oscar to raise his offer. After several conferences between Gregorio and Oscar, Oscar raised his
offer to P1.20 per sq. m. or P109,000 in total. Vicente agreed to said offer.

Upon Vicentes demand, Oscar issued a P1,000 check to him as earnest money. Vicente, then, advanced P300
to Gregorio. Subsequently, Vicente asked for an additional P1,000 as earnest money, which Oscar promised to
deliver to Vicente. The written agreement, Exhibit C, between the parties was amended.

Oscar will vacate on or about September 15, 1956 his house and lot at Denver St., QC, which is part of the
purchase price. Later on, it was again amended to state that Oscar will vacate his house and lot on Dec. 1,
1956 because his wife was pregnant at that time. Oscar gave Gregorio P1,000 as a gift or propina for
succeeding in persuading Vicente to sell his lot at P1.20 per sq. m. Gregorio did not disclose said gift or propina
to Vicente. Moreover, Oscar did not pay Vicente the additional P1,000 Vicente asked from him as earnest
money.

The deed of sale was not executed since Oscar gave up on the negotiation when he did not receive his money
from his brother in the US, which he communicated to Gregorio. Gregorio did not see Oscar for several weeks
thus sensing that something fishy might be going on. So, he went to Vicentes house where he read a portion
of the agreement to the effect that Vicente was still willing to pay him 5% commission, P5,450.

Thereafter, Gregorio went to the Register of Deeds of QC, where he discovered that a Deed of sale was
executed by Amparo de Leon, Oscars wife, over their house and lot in favor of Vicente. After discovering that
Vicente sold his lot to Oscars wife, Gregorio demanded in writing the payment of his commission.

Gregorio also conferred with Oscar. Oscar told him that Vicente went to him and asked him to eliminate
Gregorio in the transaction and that he would sell his property to him for P104,000.

In his reply, Vicente stated that Gregorio is not entitled to the 5% commission because he sold the property not
to Gregorio's buyer, Oscar de Leon, but to another buyer, Amparo Diaz, wife of Oscar de Leon.

RTC held that Vicente should pay the commission to Gregorio.

CA upheld.
ISSUE:
WON Gregorios act of accepting the gift or propina from Oscar constitutes a fraud which would cause the forfeiture of
his 5% commission.
HELD:
YES.
The pertinent New Civil Code provisions state that:
Art. 1891. Every agent is bound to render an account of his transactions and to deliver to the principal
whatever he may have received by virtue of the agency, even though it may not be owing to the
principal.
Every stipulation exempting the agent from the obligation to render an account shall be void.
xxx xxx xxx
Art. 1909. The agent is responsible not only for fraud but also for negligence, which shall be judged with
more less rigor by the courts, according to whether the agency was or was not for a compensation.
The aforecited provisions demand the utmost good faith, fidelity, honesty, candor and fairness on the part of the
agent, the real estate broker in this case, to his principal, the vendor. The law imposes upon the agent the absolute
obligation to make a full disclosure or complete account to his principal of all his transactions and other material facts
relevant to the agency, so much so that the law as amended does not countenance any stipulation exempting the
agent from such an obligation and considers such an exemption as void. The duty of an agent is likened to that of a
trustee. This is not a technical or arbitrary rule but a rule founded on the highest and truest principle of morality as
well as of the strictest justice.

97
3B-Corporation Law

Hence, an agent who takes a secret profit in the nature of a bonus, gratuity or personal benefit from the vendee,
without revealing the same to his principal, the vendor, is guilty of a breach of his loyalty to the principal and forfeits
his right to collect the commission from his principal, even if the principal does not suffer any injury by reason of such
breach of fidelity, or that he obtained better results or that the agency is a gratuitous one, or that usage or custom
allows it; because the rule is to prevent the possibility of any wrong, not to remedy or repair an actual damage.
The fact that the buyer appearing in the deed of sale is Amparo Diaz, the wife of Oscar de Leon, does not materially
alter the situation; because the transaction, to be valid, must necessarily be with the consent of the husband Oscar de
Leon, who is the administrator of their conjugal assets including their house and lot at No. 40 Denver Street, Cubao,
Quezon City, which were given as part of and constituted the down payment on, the purchase price of herein
petitioner-appellant's lot No. 883 of Piedad Estate. Hence, both in law and in fact, it was still Oscar de Leon who was
the buyer.
As a necessary consequence of such breach of trust, defendant-appellee Gregorio Domingo must forfeit his right to the
commission and must return the part of the commission he received from his principal.

98
3B-Corporation Law

Topic:
Board of Directors/Trustees/Officers
Subtopic:
Compensation of Directors
Provision:
Section 30, Corporation Code
Case Name:
Singsong v COA
COMPENSATION OF DIRECTORS (Sec. 30, Corp. Code)
Singson vs COA
FACTS:

Petitioners are the members of the Board of Philippine International Convention Center, Inc. (PICCI).

By virtue of the PICCI By-Laws, petitioners were authorized to receive P1,000.00 per diem each for every
meeting attended. An amended resolution further granted the Members of the additional monthly RATA, in the
amount of P1,500.00, to each of the petitioners. However, payment for such grants were denied.

The disallowance was questioned but it was upheld by herein respondent.

Hence this petition.


ISSUE:
WON the grant of the compensation as well as the monthly RATA are valid.
HELD:
NO.
Section 30 of the Corporation Code, which authorizes the stockholders to grant compensation to its directors, states: In
the absence of any provision in the by-laws fixing their compensation, the directors shall not receive any
compensation, as such directors, except for reasonable per diems; Provided, however, that any such compensation
(other than per diems) may be granted to directors by the vote of the stockholders representing at least a majority of
the outstanding capital stock at a regular or special stockholders meeting. In no case shall the total yearly
compensation of directors, as such directors, exceed ten (10%) percent of the net income before income tax of the
corporation during the preceding year.
From this, it is clear that the directors of a corporation shall not receive any compensation for being members of the
board of directors, except for reasonable per diems. The two instances where the directors are to be entitled to
compensation shall be when it is fixed by the corporations by-laws or when the stockholders, representing at least a
majority of the outstanding capital stock, vote to grant the same at a regular or special stockholders meeting, subject
to the qualification that, in any of the two situations, the total yearly compensation of directors, as such directors, shall
in no case exceed ten (10%) percent of the net income before income tax of the corporation during the preceding year.
In this regard, the Court upholds the findings of respondent that petitioners right to compensation as members of the
PICCI Board of Directors is limited only to per diem of P1,000.00 for every meeting attended, by virtue of the PICCI ByLaws.

99
3B-Corporation Law

Topic:
Subtopic:
Provision:
Case Name:
Facts:

Powers of Corporations
General Powers
Section 36, Corporation Code
Lily Sy v Hon. Sec. of Justice

Lily Sy, Merry Sy, Jennifer Sy, and Glenn Sy are all incorporators and members of family corporation Fortune
Wealth Mansion Corporation

Lily Sy alleges robbery against the said co-incorporators. Merry, Jennifer, and Glenn authorized their people to
open the safe and take out items allegedly of personal property of Lily Sy.

The defendants allege said property are owned by the corporation

Issue:
-

WoN there is robbery considering that the items allegedly stolen are properties of the corporation and the
alleged robbers are incorporators

Held:
-

In this case, it was shown that respondents believed in good faith that they and the corporation own not only
the subject unit but also the properties found inside. If at all, they took them openly and avowedly under that
claim of ownership.56 This is bolstered by the fact that at the time of the alleged incident, petitioner had
been staying in another unit because the electric service in the 10th floor was disconnected. We quote with
approval the CA conclusion in their Amended Decision, thus:

Indeed, on second look, We note that what is involved here is a dispute between and among members of a
family corporation, the Fortune Wealth Mansion Corporation. Petitioner Lily Sy and respondents Merry,
Jennifer, and Glenn, all surnamed Sy, are the owners-incorporators of said corporation, which owns and
manages the Fortune Wealth Mansion where petitioner allegedly resided and where the crime of robbery was
allegedly committed. As part-owners of the entire building and of the articles allegedly stolen from the 10th
floor of said building the very same properties that are involved between the same parties in a pending
estate proceeding, the respondents cannot, as co-owners, be therefore charged with robbery. The fact of coownership negates any intention to gain, as they cannot steal properties which they claim to own.

Hence, even if we are to assume that private respondents took the said personal properties from the 10th
floor of the Fortune Wealth Mansion, they cannot be charged with robbery because again, the taking was
made under a claim of ownership x x x57

Respondents should not be held liable for the alleged unlawful act absent a felonious intent.1wphi1 "Actus
non facit reum, nisi mens sit rea. A crime is not committed if the mind of the person performing the act
complained of is 58 innocent.

The Court adheres to the view that a preliminary investigation serves not only the purposes of the State, but
more importantly, it is a significant part of freedom and fair play which every individual is entitled to. It is
thus the duty of the prosecutor or the judge, as the case may be, to relieve the accused of going through a
trial once it is determined that there is no sufficient evidence to sustain a finding of probable cause to form a
sufficient belief that the accused has committed a crime. In this case, absent sufficient evidence to establish
probable cause for the prosecution of respondents for the crime of robbery, the filing of information against
respondents constitute grave abuse of discretion.

Topic:
Subtopic:
Provision:

Powers of Corporations
General Powers
Section 36, Corporation Code

100
3B-Corporation Law

Case Name:
Facts:

Violeta Tudtud Banate v Phil Countryside Rural Bank

Spouses Maglasang obtained a loan from PCRB and mortgaged the house and lot owned by their daughter and son-inlaw, the spouses Cortel. They also obtained 2 other loans secured by their other properties. Before the maturity of
their said loan, Sps. Maglasang alleged that they requested PCRB to permit them to sell their subject properties if they
will pay the full amount of the loans. Allegedly, PCRB, acting through its Branch Manager, Pancrasio Mondigo, verbally
agreed to their request hence they sold their property to Violeta Banate and used the proceeds to pay the loans. PCRB
gave the owners duplicate certificate of title and Banate secured a title in her name, however the mortgage lien was
carried over hence they requested PCRB to release the morthgaged which the bank refused. They then filed an action
for specific performance. The lower court ruled in favor of Banate but CA reversed the decision on the ground that It
ruled that Mondigo cannot orally amend the mortgage contract hence it was not binding with PCRB.
Issues:
Whether or not the Branch Manager was validly
Whether or not the doctrine of apparent authority applies

clothed

in

authority

to

novate

the

contract

or

Held:
No.
Section 23 of the Corporation Code16 expressly provides that the corporate powers of all corporations shall be
exercised by the board of directors. The power and the responsibility to decide whether the corporation should enter
into a contract that will bind the corporation are lodged in the board, subject to the articles of incorporation, bylaws, or
relevant provisions of law. In the absence of authority from the board of directors, no person, not even its officers, can
validly bind a corporation.
However, just as a natural person may authorize another to do certain acts for and on his behalf, the board of directors
may validly delegate some of its functions and powers to its officers, committees or agents. The authority of these
individuals to bind the corporation is generally derived from law, corporate bylaws or authorization from the board,
either expressly or impliedly by habit, custom or acquiescence in the general course of business.17
The authority of a corporate officer or agent in dealing with third persons may be actual or apparent. Actual authority
is either express or implied. The extent of an agents express authority is to be measured by the power delegated to
him by the corporation, while the extent of his implied authority is measured by his prior acts which have been ratified
or approved, or their benefits accepted by his principal.18 The doctrine of "apparent authority," on the other hand,
with special reference to banks, had long been recognized in this jurisdiction. The existence of apparent authority may
be ascertained through:
1) the general manner in which the corporation holds out an officer or agent as having the power to act,
or in other words, the apparent authority to act in general, with which it clothes him; or
2) the acquiescence in his acts of a particular nature, with actual or constructive knowledge thereof,
within or beyond the scope of his ordinary powers.
Accordingly, the authority to act for and to bind a corporation may be presumed from acts of recognition in other
instances when the power was exercised without any objection from its board or shareholders.19
Notably, the petitioners action for specific performance is premised on the supposed actual or apparent authority of
the branch manager, Mondigo, to release the subject properties from the mortgage, although the other obligations
remain unpaid. In light of our discussion above, proof of the branch managers authority becomes indispensable to
support the petitioners contention. The petitioners make no claim that Mondigo had actual authority from PCRB,
whether express or implied. Rather, adopting the trial courts observation, the petitioners posited that PCRB should be
held liable for Mondigos commitment, on the basis of the latters apparent authority.
We disagree with this position.

101
3B-Corporation Law

Under the doctrine of apparent authority, acts and contracts of the agent, as are within the apparent scope of the
authority conferred on him, although no actual authority to do such acts or to make such contracts has been conferred,
bind the principal.20 The principals liability, however, is limited only to third persons who have been led reasonably to
believe by the conduct of the principal that such actual authority exists, although none was given. In other words,
apparent authority is determined only by the acts of the principal and not by the acts of the agent.21 There can be no
apparent authority of an agent without acts or conduct on the part of the principal; such acts or conduct must have
been known and relied upon in good faith as a result of the exercise of reasonable prudence by a third party as
claimant, and such acts or conduct must have produced a change of position to the third partys detriment.22
In the present case, the decision of the trial court was utterly silent on the manner by which PCRB, as supposed
principal, has "clothed" or "held out" its branch manager as having the power to enter into an agreement, as claimed
by petitioners. No proof of the course of business, usages and practices of the bank about, or knowledge that the
board had or is presumed to have of, its responsible officers acts regarding bank branch affairs, was ever adduced to
establish the branch managers apparent authority to verbally alter the terms of mortgage contracts.23 Neither was
there any allegation, much less proof, that PCRB ratified Mondigos act or is estopped to make a contrary claim.24
Further, we would be unduly stretching the doctrine of apparent authority were we to consider the power to undo or
nullify solemn agreements validly entered into as within the doctrines ambit. Although a branch manager, within his
field and as to third persons, is the general agent and is in general charge of the corporation, with apparent authority
commensurate with the ordinary business entrusted him and the usual course and conduct thereof,25 yet the power to
modify or nullify corporate contracts remains generally in the board of directors.26 Being a mere branch manager
alone is insufficient to support the conclusion that Mondigo has been clothed with "apparent authority" to verbally alter
terms of written contracts, especially when viewed against the telling circumstances of this case: the unequivocal
provision in the mortgage contract; PCRBs vigorous denial that any agreement to release the mortgage was ever
entered into by it; and, the fact that the purported agreement was not even reduced into writing considering its legal
effects on the parties interests. To put it simply, the burden of proving the authority of Mondigo to alter or novate the
mortgage contract has not been established.27
It is a settled rule that persons dealing with an agent are bound at their peril, if they would hold the principal liable, to
ascertain not only the fact of agency but also the nature and extent of the agents authority, and in case either is
controverted, the burden of proof is upon them to establish it.28 As parties to the mortgage contract, the petitioners
are expected to abide by its terms. The subsequent purported agreement is of no moment, and cannot prejudice
PCRB, as it is beyond Mondigos actual or apparent authority, as above discussed.
Rescission has no legal basis; there can be no restitution of the amount paid

102
3B-Corporation Law

Topic:
Subtopic:
Provision:
Case Name:
Facts:

Powers of Corporations
General Powers
Section 36, Corporation Code
Antonio P. Salenga v Court of Appeals

1998: President/Chief Executive Officer (CEO) Rufo Colayco issued an Order informing petitioner that, pursuant to the
decision of the board of directors of respondent CDC, the position of head executive assistant the position held by
petitioner was declared redundant. Petitioner received a copy of the Order on the same day and immediately went to
see Colayco. The latter informed him that the Order had been issued as part of the reorganization scheme approved by
the board of directors. Thus, petitioners employment was to be terminated thirty (30) days from notice of the Order.
1999: Petitioner filed a case for illegal dismissal. LA ruled in favor of Petitioner.
2000: CD was already under a different head who instructed not to file an appeal. However, an appeal was still filed by
Colayco. The OGCC reiterated its allegation that petitioner was a corporate officer, and that the termination of his
employment was an intra-corporate matter. The Memorandum of Appeal was verified and certified by Hilana TimbolRoman, the executive vice president of respondent CDC. The Memorandum was accompanied by a UCPB General
Insurance Co., Inc. supersedeas bond covering the amount due to petitioner as adjudged by LA Darlucio. TimbolRoman and OGCC lawyer Roy Christian Mallari also executed on 17 March 2000 a Joint Affidavit of Declaration wherein
they swore that they were the respective authorized representative and counsel of respondent corporation. However,
the Memorandum of Appeal and the Joint Affidavit of Declaration were not accompanied by a board resolution from
respondents board of directors authorizing either Timbol-Roman or Atty. Mallari, or both, to pursue the case or to file
the appeal on behalf of respondent.
Petitioner assails in the SC the said appeal stating that the lack of an accompanying Board Resolution from the CDC
board makes the appeal useless.
Issue:
Whether or not the lack of the board resolution removes the representation rights of the lawyer to appeal the case in
the CA
Held:
Yes. Absent the board resolution, the appeal is useless. The lack of a board resolution is a fatal mistake.
It is clear from the NLRC Rules of Procedure that appeals must be verified and certified against forum-shopping by the
parties-in-interest themselves. In the case at bar, the parties-in-interest are petitioner Salenga, as the employee, and
respondent Clark Development Corporation as the employer.
A corporation can only exercise its powers and transact its business through its board of directors and through its
officers and agents when authorized by a board resolution or its bylaws. The power of a corporation to sue and be sued
is exercised by the board of directors. The physical acts of the corporation, like the signing of documents, can be
performed only by natural persons duly authorized for the purpose by corporate bylaws or by a specific act of the
board. The purpose of verification is to secure an assurance that the allegations in the pleading are true and correct
and have been filed in good faith.[41]
Thus, we agree with petitioner that, absent the requisite board resolution, neither Timbol-Roman nor Atty. Mallari, who
signed the Memorandum of Appeal and Joint Affidavit of Declaration allegedly on behalf of respondent corporation,
may be considered as the appellant and employer referred to by Rule VI, Sections 4 to 6 of the NLRC Rules of
Procedure

103
3B-Corporation Law

Topic:
Subtopic:
Provision:
Case Name:

Powers of Corporations
General Powers
Section 36, Corporation Code
Advance Paper Corp v Arma Traders Corp

Facts:
Advance Paper is a domestic corporation engaged in the business of producing, printing, manufacturing,
distributing and selling of various paper products. Petitioner George Haw is the President. While respondent Arma
Traders is also a domestic corporation engaged in the wholesale and distribution of school and office supplies, and
novelty products. Respondent Antonio Tan was formerly the President while respondent Uy Seng Kee Willy is the
Treasurer of Arma Traders. They represented Arma Traders when dealing with its supplier, Advance Paper, for about 14
years.
Arma Traders purchased on credit notebooks and other paper products amounting to P7,533,001.49 from
Advance Paper. Upon the representation of Tan and Uy, Arma Traders also obtained three loans from Advance Paper in
the amounts of P3,380,171.82, P1,000,000.00, and P3,408,623.94 or a total of P7,788,796.76.
As payment, Arma Traders issued 82 postdated checks payable to cash or to Advance Paper. Tan and Uy were
Arma Traders authorized bank signatories who signed and issued these checks which had the aggregate amount of
P15,130,636.87.
Advance Paper presented the checks to the drawee bank but were dishonored either for insufficiency of
funds or account closed. Despite repeated demands, however, Arma Traders failed to settle its account with
Advance Paper. The petitioners filed a complaint for collection of sum of money with application for preliminary
attachment against Arma Traders, Tan, Uy, Ting, Gui, and Ng.
Claims of the respondents
As to the loan transactions, the respondents countered that these were the personal obligations of Tan and Uy to
Advance Paper. These loans were never intended to benefit the respondents.
The respondents also claimed that the loan transactions were ultra vires because the board of directors of Arma
Traders did not issue a board resolution authorizing Tan and Uy to obtain the loans from Advance Paper. They claimed
that the borrowing of money must be done only with the prior approval of the board of directors because without the
approval, the corporate officers are acting in excess of their authority or ultra vires. When the acts of the corporate
officers are ultra vires, the corporation is not liable for whatever acts that these officers committed in excess of their
authority. Further, the respondents claimed that Advance Paper failed to verify Tan and Uys authority to transact
business with them. Hence, Advance Paper should suffer the consequences.
RTC ordered Arma Traders to pay Advance Paper the sum of P15,321,798. The RTC held that the respondents failed to
present hard, admissible and credible evidence to prove that the sale invoices were forged or fictitious, and that the
loan transactions were personal obligations of Tan and Uy. Nonetheless, the RTC dismissed the complaint against Tan,
Uy, Ting, Gui and Ng due to the lack of evidence showing that they bound themselves, either jointly or solidarily, with
Arma Traders for the payment of its account.
CA held that the petitioners failed to prove by preponderance of evidence the existence of the purchases on credit and
loans
Arma Traders was not liable for the loan in the absence of a board resolution authorizing Tan and Uy to obtain the loan
from Advance Paper. The CA acknowledged that Tan and Uy were Arma Traders authorized bank signatories. However,
the CA explained that this is not sufficient because the authority to sign the checks is different from the required
authority to contract a loan.
Petitioner: Arma Traders led the petitioners to believe that Tan and Uy had the authority to obtain loans since the
respondents left the active and sole management of the company to Tan and Uy since 1984. In fact, Ng testified that
Arma Traders stockholders and board of directors never conducted a meeting from 1984 to 1995. Citing Lipat v. Pacific
Banking Corporation, the petitioners said that if a corporation knowingly permits one of its officers or any other agent
to act within the scope of an apparent authority, it holds him out to the public as possessing the power to do those
acts; thus, the corporation will, as against anyone who has in good faith dealt with it through such agent, be estopped
from denying the agents authority.
Respondents: argue that while as a general rule, a corporation is estopped from denying the authority of its agents
which it allowed to deal with the general public; this is only true if the person dealing with the agent dealt in good
faith. In the present case, the respondents claim that the petitioners are in bad faith because the petitioners connived
with Tan and Uy to make Arma Traders liable for the non-existent deliveries of notebooks and other paper products. As
to the loans, the respondents aver that these were Tan and Uys personal obligations with Advance Paper.
ISSUE:
Whether Arma Traders is liable to pay the loans applying the doctrine of apparent authority.

104
3B-Corporation Law

Held:
Yes.
RATIO:
The doctrine of apparent authority provides that a corporation will be estopped from denying the agents
authority if it knowingly permits one of its officers or any other agent to act within the scope of an apparent authority,
and it holds him out to the public as possessing the power to do those acts.
In Inter-Asia Investment Industries v. Court of Appeals,78 we explained:
Apparent authority is derived not merely from practice. Its existence may be ascertained through (1) the general
manner in which the corporation holds out an officer or agent as having the power to act or, in other words the
apparent authority to act in general, with which it clothes him; or (2) the acquiescence in his acts of a particular
nature, with actual or constructive knowledge thereof, within or beyond the scope of his ordinary powers. It requires
presentation of evidence of similar act(s) executed either in its favor or in favor of other parties. It is not the quantity
of similar acts which establishes apparent authority, but the vesting of a corporate officer with the power to bind the
corporation. Also, in Peoples Aircargo and Warehousing Co., Inc. v. Court of Appeals, we ruled that the doctrine of
apparent authority is applied when the petitioner, through its president Antonio Punsalan Jr., entered into the First
Contract without first securing board approval. Despite such lack of board approval, petitioner did not object to or
repudiate said contract, thus "clothing" its president with the power to bind the corporation.
In the present petition, we do not agree with the CAs findings that Arma Traders is not liable to pay the loans
due to the lack of board resolution authorizing Tan and Uy to obtain the loans. To begin with, Arma Traders Articles of
Incorporation provides that the corporation may borrow or raise money to meet the financial requirements of its
business by the issuance of bonds, promissory notes and other evidence of indebtedness. Likewise, it states that Tan
and Uy are not just ordinary corporate officers and authorized bank signatories because they are also Arma Traders
incorporators along with respondents Ng and Ting, and Pedro Chao. Furthermore, the respondents, through Ng who is
Arma Traders corporate secretary, incorporator, stockholder and director, testified that the sole management of Arma
Traders was left to Tan and Uy and that he and the other officers never dealt with the business and management of
Arma Traders for 14 years. He also confirmed that since 1984 up to the filing of the complaint against Arma Traders, its
stockholders and board of directors never had its meeting.
Thus, Arma Traders bestowed upon Tan and Uy broad powers by allowing them to transact with third persons without
the necessary written authority from its non-performing board of directors. Arma Traders failed to take precautions to
prevent its own corporate officers from abusing their powers. Because of its own laxity in its business dealings, Arma
Traders is now estopped from denying Tan and Uys authority to obtain loan from Advance Paper.

105
3B-Corporation Law

Topic:
Subtopic:
Provision:
Case Name:

Powers of Corporations
General Powers
Section 36, Corporation Code
Riosa v Tabaco La Suerte Corporation

Facts:
Riosa alleged that he was the owner and in actual possession of a 52-square meter commercial lot situated in
Barangay Quinale, Tabaco City, Albay; that he declared the property in his name and had been religiously paying the
realty tax on the said property; that thereafter, his daughter renovated the commercial building on the lot and
introduced improvements. Subsequently, he obtained loans from Sia Ko Pio in the total amount ofP50,000.00. Riosa
claimed that by means of fraud, misrepresentation and deceit employed by Sia Ko Pio, he was made to sign the
document which he thought was a receipt and undertaking to pay the loan, only to find out later that it was a
document of sale. Riosa averred that he did not appear before the notary public to acknowledge the sale. He thus
prayed for the nullification of the deed of sale and certificate of title in the name of La Suerte and the reconveyance of
the subject property to him.
In its Answer, La Suerte averred that it was the actual and lawful owner of the commercial property, after
purchasing it from Riosa on December 7, 1990; that it allowed Riosa to remain in possession of the property to avoid
the ire of his father from whom he had acquired property inter vivos, subject to his obligation to vacate the premises
anytime upon demand and obtained a TCT over the property. It repeatedly asked Riosa to vacate the premises but to
no avail and Riosas complaint was barred by prescription, laches, estoppel and indefeasibility of La Suertes title.
RTC ruled in favor of Riosa and found that there was really no sale that occurred. CA reversed he ruling and
held the validity of the sale. Thus, it was elevated to the SC. Riosa argued among others that there was no board
resolution authorizing Sia Ko Pio to purchase the property. La Suerte on the other hand averred that the absence of a
board resolution for the purchase of the property has no controlling consequence as La Suerte had ratified the act of
Sia Ko Pio.
Issue: Whether or not there is a valid sale when there was no board resolution to approve of the sale.
Held: NO. The Court notes that when Sia Ko Pios son, Juan was presented as an officer of La Suerte, he admitted that
he could not find in the records of the corporation any board resolution authorizing his father to purchase disputed
property.
SEC. 36. Corporate powers and capacity. Every corporation incorporated under this Code has the power
and capacity:
7. To purchase, receive, take or grant, hold, convey, sell, lease, pledge, mortgage and otherwise deal with
such real and personal property, including securities and bonds of other corporations, as the transaction of
a lawful business of the corporation may reasonably and necessarily require, subject to the limitations
prescribed by the law and the Constitution.
Under these provisions, the power to purchase real property is vested in the board of directors or trustees.
While a corporation may appoint agents to negotiate for the purchase of real property needed by the corporation, the
final say will have to be with the board, whose approval will finalize the transaction. A corporation can only exercise its
powers and transact its business through its board of directors and through its officers and agents when authorized by
a board resolution or its by-laws.
In the case at bench, Sia Ko Pio, although an officer of La Suerte, had no authority from its Board of Directors
to enter into a contract of sale of Riosas property. It is, thus, clear that the loan obtained by Riosa from Sia Ko Pio was
a personal loan from the latter, not a transaction between Riosa and La Suerte. There was no evidence to show that
Sia Ko Pio was clothed with authority to use his personal fund for the benefit of La Suerte. Evidently, La Suerte was
never in the picture.
WHEREFORE, the petition is GRANTED. The sale is null and void. This disposition is without prejudice to any
valid claim of the heirs of Sia Ko Pio against Riosa.

106
3B-Corporation Law

Topic:
Subtopic:
Provision:
Case Name:

Powers of Corporations
General Powers
Section 36, Corporation Code
Lanuza Jr. v BF Corporation

Facts:
- In 1993, BF Corporation filed a complaint for collection of money against Shangri-La and the members of its board of
directors: Alfredo C. Ramos, Rufo B. Colayco, Antonio O. Olbes, Gerardo Lanuza Jr., Maximo G. Licauco III and Benjamin
C. Ramos for the outstanding balance of the construction of Shangri-La mall and a multilevel parking structure along
EDSA.
- BF Corporation alleged that Shangri-Las directors were in bad faith in directing Shangri-Las affairs, therefore they
should be held jointly and severally liable with the company.
- Shangri-La and Alfredo C. Ramos, Rufo B. Colayco, Maximo G. Licauco III and Benjamin C. Ramos filed a motion to
suspend the proceeding because BF Corporation failed to submit the dispute to arbitration in accordance with their
contract. Which was later on denied by the RTC.
- Alfredo C. Ramos, Rufo B. Colayco, Antonio O. Olbes, Gerardo Lanuza Jr., Maximo G. Licauco III and Benjamin C.
Ramos filed an answer to BF Corporations complaint alleging that they had resigned as members of the Shangri-la
board of directors as of July 15, 1991.
- Thereafter, a controversy arose as to whether the board of directors impleaded in the complaint should be made
parties to the arbitration proceeding.

Issue: W/N the petitioners should be made parties to the arbitration proceedings?

Held: Yes, the petitioners should be made parties to the arbitration proceedings in order to determine if the distinction
between Shangri-La's personality and their personalities should be disregarded.
- A corporation is an artificial entity created by fiction of law. This means that while it is not a person, naturally, the law
gives it a distinct personality and treats it as such. A corporation, in the legal sense, is an individual with a personality
that is distinct and separate from other persons including its stockholders, officers, directors, representatives, and
other juridical entities.
- The law vests in corporations rights, powers, and attributes as if they were natural persons with physical existence
and capabilities to act on their own. For instance, they have the power to sue and enter into transactions or contracts.
- Because a corporation's existence is only by fiction of law, it can only exercise its rights and powers through its
directors, officers, or agents, who are all natural persons. A corporation cannot sue or enter into contracts without
them.
- A consequence of a corporation's separate personality is that consent by a corporation through its representatives is
not consent of the representative, personally. Its obligations, incurred through official acts of its representatives, are its
own. A stockholder, director, or representative does not become a party to a contract just because a corporation
executed a contract through that stockholder, director or representative.
- Hence, a corporation's representatives are generally not bound by the terms of the contract executed by the
corporation. They are not personally liable for obligations and liabilities incurred on or in behalf of the corporation.
- As a general rule, therefore, a corporation's representative who did not personally bind himself or
herself to an arbitration agreement cannot be forced to participate in arbitration proceedings made
pursuant to an agreement entered into by the corporation. He or she is generally not considered a party
to that agreement.
- However, there are instances when the distinction between personalities of directors, officers, and representatives,
and of the corporation, are disregarded. We call this piercing the veil of corporate fiction.

107
3B-Corporation Law

- When there are allegations of bad faith or malice against corporate directors or representatives, it becomes the duty
of courts or tribunals to determine if these persons and the corporation should be treated as one. Without a trial,
courts and tribunals have no basis for determining whether the veil of corporate fiction should be pierced. Courts or
tribunals do not have such prior knowledge. Thus, the courts or tribunals must first determine whether circumstances
exist to warrant the courts or tribunals to disregard the distinction between the corporation and the persons
representing it. The determination of these circumstances must be made by one tribunal or court in a proceeding
participated in by all parties involved, including current representatives of the corporation, and those persons whose
personalities are impliedly the same as the corporation. This is because when the court or tribunal finds that
circumstances exist warranting the piercing of the corporate veil, the corporate representatives are treated as the
corporation itself and should be held liable for corporate acts. The corporation's distinct personality is disregarded, and
the corporation is seen as a mere aggregation of persons undertaking a business under the collective name of the
corporation.
- Hence, when the directors, as in this case, are impleaded in a case against a corporation, alleging malice or bad faith
on their part in directing the affairs of the corporation, complainants are effectively alleging that the directors and the
corporation are not acting as separate entities. They are alleging that the acts or omissions by the corporation that
violated their rights are also the directors' acts or omissions. They are alleging that contracts executed by the
corporation are contracts executed by the directors. Complainants effectively pray that the corporate veil be pierced
because the cause of action between the corporation and the directors is the same.

108
3B-Corporation Law

Topic:
Subtopic:
Provision:
Case Name:

Powers of Corporations
Specific Powers: Powert to Deny Pre-emptive Right
Section 39, Corporation Code
Majority Stockholders of Ruby Ind. V Lim

FACTS:
Reeling from severe liquidity problems beginning in 1980, Ruby Industrial Corporation (Ruby), filed a petition for
suspension of payments with the Securities and Exchange Commission (SEC). In an order, SEC declared Ruby under
suspension of payments, enjoining the disposition of its properties pending hearing of the petition, except insofar as
necessary in its ordinary operations, and making payments outside of the necessary or legitimate expenses of its
business.
The SEC Hearing Panel created the management committee (Mancom) for Ruby tasked, among others, to take custody
and control over all existing assets and liabilities of Ruby, and determine the best way to salvage and protect the
interest of its investors and creditors.
Two rehabilitation plans were submitted to the SEC: the BENHAR/Ruby Rehabilitation Plan of the majority stockholders
led by Yu Kim Giang, and the Alternative Plan of the minority stockholders represented by Miguel Lim (Lim). Over 90%
of Rubys creditors objected to the Revised BENHAR/Ruby Plan and the creation of a new management committee.
Instead, they endorsed the minority stockholders Alternative Plan. Notwithstanding the objections, the SEC Hearing
Panel approved the Revised BENHAR/Ruby Plan and dissolved the existing Mancom. It also created a new Mancom and
appointed BENHAR as one of its members.
ISSUE: Whether the minoritys pre-emptive rights were violated
HELD:
Yes. Pre-emptive right under Sec. 39 of the Corporation Code refers to the right of a stockholder of a stock corporation
to subscribe to all issues or disposition of shares of any class, in proportion to their respective shareholdings. The right
may be restricted or denied under the articles of incorporation, and subject to certain exceptions and limitations. The
stockholder must be given a reasonable time within which to exercise their preemptive rights.
The validity of issuance of additional shares may be questioned if done in breach of trust by the controlling
stockholders. Thus, even if the pre-emptive right does not exist, an issue of shares may still be objectionable if the
directors acted in breach of trust and their primary purpose is to perpetuate or shift control of the corporation, or to
"freeze out" the minority interest.
The will of the majority shall govern in all matters within the limits of the act of incorporation and lawfully enacted bylaws not proscribed by law. It is, however, equally true that other stockholders are afforded the right to intervene
especially during critical periods in the life of a corporation like reorganization, or in this case, suspension of payments,
more so, when the majority seek to impose their will and through fraudulent means, attempt to siphon off Rubys
valuable assets to the great prejudice of Ruby itself, as well as the minority stockholders and the unsecured creditors.
Generally speaking, the voice of the majority of the stockholders is the law of the corporation, but there are exceptions
to this rule. There must necessarily be a limit upon the power of the majority. Without such a limit the will of the
majority will be absolute and irresistible and might easily degenerate into absolute tyranny.
Lamentably, the SEC refused to heed the plea of the minority stockholders for the SEC to order Ruby to commence
liquidation proceedings, which is allowed under Sec. 4-9 of the Rules on Corporate Recovery. Under the circumstances,
liquidation was the only hope of the minority stockholders for effecting an orderly and equitable settlement of Rubys
obligations, and compelling the majority stockholders to account for all funds, properties and documents in their
possession, and make full disclosure on the nullified credit assignments. Oblivious to these pending incidents so crucial
to the protection of the interest of the majority of creditors and minority shareholders, the SEC simply stated that in
the interim, Rubys corporate term was validly extended, as if such extension would provide the solution to Rubys
myriad
problems.
Extension of corporate term requires the vote of 2/3 of the outstanding capital stock in a stockholders meeting called
for the purpose. The actual percentage of shareholdings in RUBY as of September 3, 1996 -- when the majority
stockholders allegedly ratified the board resolution approving the extension of RUBY's corporate life to another 25
years was seriously disputed by the minority stockholders, and we find the evidence of compliance with the notice and
quorum requirements submitted by the majority stockholders insufficient and doubtful. Consequently, the SEC had no
basis for its ruling denying the motion of the minority stockholders to declare as without force and effect the extension
of Ruby's corporate existence.

109
3B-Corporation Law

Topic:
Subtopic:
Provision:
Case Name:

Powers of Corporations
Specific Powers: Power to Sell Al or Substantially all of the assets
Section 40, Corporation Code and Bulk Sales Law
Islamic Directorate of the Phils v CA

FACTS:
In 1971, the Islamic Directorate of the Philippines (IDP), was incorporated, the primary purpose of which is to establish
of a mosque, school, and other religious infrastructures in Quezon City. IDP purchased a lot in Quezon City. When
President Marcos declared martial law in 1972, most of the members of the 1971 Board of Trustees ("Tamano Group")
flew to the Middle East to escape political persecution. Thereafter, two contending groups claiming to be the IDP Board
of Trustees sprung: the Carpizo group and Abbas group.
In a suit between the two groups, SEC declared the election of both the Carpizo Group and the Abbas Group as IDP
board members to be null and void. SEC recommended that a new by-laws be approved and a new election be
conducted upon the approval of the by-laws. However, the SEC recommendation was not heeded.
In 1989, the Carpizo group passed a Board Resolution authorizing the sale of the land to Iglesia Ni Cristo (INC), and a
Deed of Sale was eventually executed. In 1991, the Tamano Group filed a petition before the SEC questioning the sale.
Meanwhile, INC filed a suit for specific performance before the RTC against the Carpizo group, to compel a certain
Leticia Ligon (who is apparently the mortgagee of the lot) to surrender the title. The Tamano group sought to
intervene, but the intervention was denied. In 1992, the Court subsequently ruled that the INC as the rightful owner of
the land, and ordered Ligon to surrender the titles for annotation.
ISSUE: Whether or not the sale between the Carpizo group and INC is null and void
HELD:
Yes.
Since the SEC has declared the Carpizo group as a void Board of Trustees, the sale it entered into with INC is void.
Without a valid consent of a contracting party, there can be no valid contract. The IDP, never gave its consent, through
a legitimate Board of Trustees. Therefore, this is a case not only of vitiated consent, but one where consent on the part
of one of the supposed contracting parties is totally wanting. Ineluctably, the subject sale is void and produces no
effect whatsoever.
Further, the Carpizo group failed to comply with Section 40 of the Corporation Code, which provides that: " ... a
corporation may, by a majority vote of its board of directors or trustees, sell, lease, exchange, mortgage, pledge or
otherwise dispose of all or substantially all of its property and assets... when authorized by the vote of the stockholders
representing at least two-thirds (2/3) of the outstanding capital stock; or in case of non-stock corporation, by the vote
of at least two-thirds (2/3) of the members, in a stockholders' or members' meeting duly called for the purpose...."
The subject lot constitutes the only property of IDP. Hence, its sale to a third-party is a sale or disposition of all the
corporate property and assets of IDP. For the sale to be valid, the majority vote of the legitimate Board of Trustees,
concurred in by the vote of at least 2/3 of the bona fide members of the corporation should have been obtained. These
twin requirements were not met in the case at bar.

Topic:
Powers of Corporations
Subtopic:
Specific Powers: Power to Sell Al or Substantially all of the assets
Provision:
Section 40, Corporation Code and Bulk Sales Law
Case Name:
Caltex Inc. v PNOC Shipping and Transport Corp
Name: Glenn Nio M. Sartillo
Topic: Specific Powers Power to Sell All or Substantially all of the assets
Name of Case: Caltex (Phil.), Inc. vs. PNOC Shipping and Transport Corp. (GR No. 150711, August 10, 2006)
Corporation Code Provision: Section 40
FACTS:
PNOC Shipping and Transport Corp (PSTC) and Luzon Stevedoring Corporation (LUSTEVECO) entered into an Agreement
of Assumption of Obligations (Agreement), which provides that PSTC shall assume all the obligations of LUSTEVECO

110
3B-Corporation Law

with respect to the claims enumerated in Annexes of the Agreement. The Agreement also provides that PSTC shall
control the conduct of any litigation pending or which may be filed with respect to the claims in the Annexes, among
others. Among the actions enumerated in the Annexes is Caltex (Phils.), Inc. v. Luzon Stevedoring Corporation pending
before the then Intermediate Appellate Court (IAC). In its Decision, the IAC ordered LUSTEVECO to pay Caltex.
However, the judgment was not satisfied because of the prior foreclosure of LUSTEVECOs properties. Caltex
subsequently learned of the Agreement between PSTC and LUSTEVECO. Caltex sent successive demands to PSTC.
Later, PSTC informed Caltex that it was not a party to the case and thus, PSTC would not pay LUSTEVECOs judgment
debt. PSTC advised Caltex to demand satisfaction of the judgment directly from LUSTEVECO.
Caltex filed a complaint for sum of money against PSTC. The trial court decided in favor of the Caltex, ordering PSTC to
pay the sums due the latter in the decision rendered by the Court of Appeals.
ISSUE: Whether or not PSTC is liable to Caltex
HELD:
Yes. Even without the Agreement, PSTC is still liable to Caltex.
While the disposition of all or substantially all of the assets of a corporation is allowed under Section 40 of The
Corporation Code of the Philippines, the transfer should not prejudice the creditors of the assignor. The only way the
transfer can proceed without prejudice to the creditors is to hold the assignee liable for the obligations of the assignor.
The acquisition by the assignee of all or substantially all of the assets of the assignor necessarily includes the
assumption of the assignors liabilities, unless the creditors who did not consent to the transfer choose to rescind the
transfer on the ground of fraud. To allow an assignor to transfer all its business, properties and assets without the
consent of its creditors and without requiring the assignee to assume the assignors obligations will defraud the
creditors. The assignment will place the assignors assets beyond the reach of its creditors.
Here, Caltex could not enforce the judgment debt against LUSTEVECO. The writ of execution could not be satisfied
because LUSTEVECOs remaining properties had been foreclosed by lienholders. In addition, all of LUSTEVECOs
business, properties and assets pertaining to its tanker and bulk business had been assigned to PSTC without the
knowledge of its creditors. Caltex now has no other means of enforcing the judgment debt except against PSTC.
If PSTC refuses to honor its written commitment to assume the obligations of LUSTEVECO, there will be fraud on the
creditors of LUSTEVECO. PSTC agreed to take over, and in fact took over, all the assets of LUSTEVECO upon its express
written commitment to pay all obligations of LUSTEVECO pertaining to those assets, including specifically the claim of
Caltex. LUSTEVECO no longer informed its creditors of the transfer of all of its assets presumably because PSTC
committed to pay all such creditors. Such transfer, leaving the claims of creditors unenforceable against the debtor, is
fraudulent and rescissible.

111
3B-Corporation Law

Topic:
Powers of Corporations
Subtopic:
Specific Powers: Power to Sell Al or Substantially all of the assets
Provision:
Section 40, Corporation Code and Bulk Sales Law
Case Name:
Metropolitan Bank and Trust Co. v Centro Development Corp
Topic: Powers of Corporations (Specific Powers: Powers to Sell All or Substantially all the assets)
Case: Metropolitan Bank Trust & Co vs Centro Development
Name: Carmela Abergos
FACTS:
On 20 March 1990, in a special meeting of the board of directors of respondent Centro Development Corporation
(Centro), its president Go Eng Uy was authorized to mortgage its properties and assets to secure the medium-term
loan of 84 million of Lucky Two Corporation and Lucky Two Repacking. This authorization was subsequently approved
on the same day by the stockholders.
Respondent Centro, represented by Go Eng Uy, executed a Mortgage Trust Indenture (MTI) with the Bank of the
Philippines Islands (BPI). Under the MTI, respondent Centro, together with its affiliates Lucky Two Corporation and
Lucky Two Repacking or Go Eng Uy, expressed its desire to obtain from time to time loans and other credit
accommodations from certain creditors for corporate and other business purposes. To secure these obligations from
different creditors, respondent Centro constituted a continuing mortgage on all or substantially all of its properties and
assets enumerated above unto and in favor of BPI, the trustee. On 31 March 1993, Centro and BPI amended the MTI to
allow an additional loan of 36 million and to include San Carlos Milling Company, Inc. (San Carlos) as a borrower in
addition to Centro, Lucky Two Corp. and Lucky Two Repacking. Then, on 28 July 1994, Centro and BPI again amended
the MTI for another loan of 24 million, bringing the total obligation to 144 million.
Respondent Centro, represented by Go Eng Uy, approached petitioner Metropolitan Bank and Trust Company
(Metrobank) sometime in 1994 and proposed that the latter assume the role of successor-trustee of the existing MTI.
Thereafter petitioner and respondent Centro executed the assailed MTI, amending the previous agreements by
appointing the former as the successor-trustee of BPI. It is worth noting that this MTI did not amend the amount of the
total obligations covered by the previous MTIs.
It was only sometime in 1998 that respondents herein, Chongking Kehyeng, Manuel Co Kehyeng and Quirino Kehyeng,
allegedly discovered that the properties of respondent Centro had been mortgaged, and that the MTI that had been
executed appointing petitioner as trustee. They alleged that they were not aware of any board or stockholders meeting
held on 12 August 1994, when petitioner was appointed as successor-trustee of BPI in the MTI.
Meanwhile, San Carlos obtained loans in the total principal amount of 812,793,513.23 from petitioner Metrobank. San
Carlos failed to pay these outstanding obligations despite demand. Thus, petitioner, as trustee of the MTI, initiated
foreclosure proceedings on the mortgaged properties.
Before the scheduled foreclosure, respondents herein filed a Complaint for the annulment of the 27 September 1994
MTI with a prayer for a temporary restraining order.
RTC dismissed the Complaint. It held that the evidence presented by respondents was insufficient to support their
claim that there were no meetings held authorizing the mortgage of Centros properties. It noted that the stocks of
respondents Kehyeng constituted only 30% of the outstanding capital stock, while the Go family owned the majority
70%, which represented more than the 2/3 vote required by Section 40 of the Corporation Code.
Centro and San Carlos filed a Complaint docketed as Civil Case No. 04-612 at Branch 56 of the RTC of Makati City
praying for the nullification of the foreclosure proceedings and prayed for the issuance of a TRO/injunction. While Civil
Case No. 04-612 was pending, the clerk of court and the ex-officio sheriff of the RTC of Makati City held an auction sale
of the disputed property, during which petitioner was adjudged as the highest bidder for 344,700,000. On June 2,
2004, a public auction sale was conducted and METROPOLITAN BANK & TRUST CO. submitted a bid for the sale to
him/it of the mortgaged property in the amount of 344,700,000 xxx, which was the highest bid hence declared as the
winning bidder and being the creditor he/it did not delivery or pay cash/monies to the Clerk of Court and Ex-Officio
Sheriff the bid price of 344,700,000 xxx and the selling price was credited as partial/full satisfaction of indebtedness
secured by the mortgage.
Because of this development, the Complaint in Civil Case No. 04-612 was amended, and Centro and San Carlos
prayed for the issuance of a writ of injunction to prevent the registration of the Certificate of Sale and the subsequent
transfer to petitioner of the title to the properties. However, Branch 56 of the RTC of Makati City subsequently denied
the application.

112
3B-Corporation Law

Respondent Centro thereafter filed before the CA a Petition for Certiorari . During this time, CA-G.R. CV No. 80778,
which involved the legality of the MTI, was still pending. CA promulgated the assailed Decision in CA-G.R. CV No.
80778. The appellate court first determined whether the requirements of Section 40 of the Corporation Code on the
sale of all or substantially all of the corporations property were complied with. Based on the 18 August 1994 Secretarys
Certificate, the CA found that only a quorum was present during the stockholders meeting. The appellate court thus
held that the 2/3 vote required by Section 40 was not met. It ruled that the minority stockholders were deprived of
their right to dissent from or to approve the proposed mortgage, considering that they had not been notified in writing
of the meeting in which the corporate action was to be discussed.
ISSUE:
Won the amendment of the MTI failed to meet the requirements of Section 40 of the Corporation Code on notice and
voting requirements.
HELD:
Reading carefully the Secretarys Certificate, it is clear that the main purpose of the directors Resolution was to
appoint petitioner as the new trustee of the previously executed and amended MTI. Going through the original and the
revised MTI, we find no substantial amendments to the provisions of the contract. We agree with petitioner that the act
of appointing a new trustee of the MTI was a regular business transaction. The appointment necessitated only a
decision of at least a majority of the directors present at the meeting in which there was a quorum, pursuant to Section
25 of the Corporation Code.
The second paragraph of the directors Resolution No. 005, s. 1994, which empowered Go Eng Uy "to sign the Real
Estate Mortgage and all documents/instruments with the said bank, for and in behalf of the Company which are
necessary and pertinent thereto," must be construed to mean that such power was limited by the conditions of the
existing mortgage, and not that a new mortgage was thereby constituted.
Moreover, it is worthy to note that respondents do not assail the previous MTI executed with BPI. They do not question
the validity of the mortgage constituted over all or substantially all of respondent Centros assets pursuant to the 21
March 1994 MTI in the amount of P 84 million. Nor do they question the additional loans increasing the value of the
mortgage to P 144 million; or the use of Centros properties as collateral for the loans of San Carlos, Lucky Two
Corporation, and Lucky Two Repacking.
Thus, Section 40 of the Corporation Code finds no application in the present case, as there was no new mortgage to
speak of under the assailed directors Resolution.
Nevertheless, while we uphold the validity of the stockholders Resolution appointing Metrobank as successor-trustee,
this is not to say that we uphold the validity of the extrajudicial foreclosure of the mortgage.
After a careful review of the records of this case, we find that petitioner failed to establish its right to be entitled to the
proceeds of the MTI. There is no evidence that petitioner, as creditor or as trustee, had a cause of action to move for
the extrajudicial foreclosure of the subject properties mortgaged under the MTI.

113
3B-Corporation Law

Topic:
Powers of Corporations
Subtopic:
Power to Invest in another purpose
Provision:
Section 40, Corporation Code and Bulk Sales Law
Case Name:
Republic v Sun Life
Topic: Powers of Corporations (Specific Powers: Power to invest in another purpose)
Case: Republic vs Sun life
Name: Carmela Abergos
FACTS:
On December 29, 1997, the [Court of Tax Appeals] (CTA) rendered its decision in Insular Life Assurance Co. Ltd. v.
[CIR], which held that mutual life insurance companies are purely cooperative companies and are exempt from the
payment of premium tax and DST. This pronouncement was later affirmed by this court in [CIR] v. Insular
Life Assurance Company, Ltd. Sun Life surmised that[,] being a mutual life insurance company, it was likewise exempt
from the payment of premium tax and DST. Hence, on August 20, 1999, Sun Life filed with the CIR
an administrativeclaim for tax credit of its alleged erroneously paid premium tax and DST for the aforestated tax
periods.
For failure of the CIR to act upon the administrative claim for tax credit and with the 2-year period to file a claim for tax
credit or refund dwindling away and about to expire, Sun Life filed with the CTA a petition for review. The CTA found in
favor of Sun Life.
Seeking reconsideration of the decision of the CTA, the CIR argued that Sun Life ought to have registered, foremost,
with the Cooperative Development Authority before it could enjoy theexemptions from premium tax and DST extended
to purely cooperative companies or associations under [S]ections 121 and 199 of the Tax Code. For its failure
to register, it could not avail of the exemptions prayed for. The CTA denied the CIRs motion for reconsideration.
ISSUE:
Whether or not respondent is exempted from payment of tax on life insurance premiums and documentary stamp tax
RULING:
YES. The Tax Code defines a cooperative as an associationconducted by the members thereof with the money
collected from among themselves and solely for their own protection and not for profit. Without a doubt, respondent
is a cooperative engaged in a mutual life insurance business.
First, it is managed by its members. Both the CA and the CTA found that the management and affairs of respondent
were conducted by its member-policyholders. SUNLIFE has been mutualized or converted from a stock life insurance
company to a nonstock mutual life insurance corporation pursuant to Section 266 of the Insurance Code of 1978. On
the basis of its bylaws, its ownership has been vested in its member-policyholders who are each entitled to one vote;
and who, in turn, elect from among themselves the members of its board of trustees.
Second, it is operated with money collected from its members. Since respondent is composed entirely of members who
are also its policyholders, all premiums collected obviously come only from them. The member-policyholders constitute
both insurer and insured who contribute, by a system of premiums or assessments, to the creation of a fund from
which all losses and liabilities are paid.
Third, it is licensed for the mutual protection of its members, not for the profit of anyone. A mutual life insurance
company is conducted for the benefit of its member-policyholders, who pay into its capital by way of premiums.
Under the Tax Code although respondent is a cooperative, registration with the Cooperative Development Authority
(CDA) is not necessary in order for it to be exempt from the payment of both percentage taxes on insurance premiums,
under Section 121; and documentary stamp taxes on policies of insurance or annuities it grants, under Section 199.

114
3B-Corporation Law

Topic:
Powers of Corporations
Subtopic:
Ultra Vires Acts
Provision:
Section 45, Corporation Coe
Case Name:
Zomer Development Corp v International Exchange Bank
Topic: Ultra Vires Acts
Case: ZOMER DEVELOPMENT CORP VS INTERNATIONAL EXCHANGE BANK
Name: Carmela Abergos
FACTS:
The Board of Directors of Zomer Development Company, Inc. (petitioner) approved a resolution authorizing it to apply
for and obtain a credit line with respondent International Exchange Bank (IEB) in the amount of P60,000,000 as well as
temporary excesses or permanent increases thereon as may be approved by IEB from time to time. The Board of
Directors also authorized petitioner to assign, pledge, or mortgage its properties as security for this credit line; and to
secure and guarantee the term loan and other credit facility of IDHI Prime Aggregates Corporation (Prime Aggregates)
with IEB.
Prime Aggregates obtained a term loan from IEB in the amount of P60,000,000. Petitioner, through its Treasurer
Amparo Zosa (Amparo) and its General Manager Manuel Zosa, Jr. (Zosa), executed a real estate mortgage covering
three parcels of land (the real estate mortgage) in favor of IEB to secure. Prime Aggregates subsequently obtained
several loans from IEB from September 1997 until September 1998.
Prime Aggregates failed to settle its outstanding obligation which stood at P90,267,854.96 and US$211,547.12 as of
September 15, 2000, drawing IEB to file a petition for extra-judicial foreclosure of mortgage before the RTC of Cebu
City.
Petitioner filed a complaint8 for Injunction with application for writ of preliminary injunction/temporary restraining order
alleging that the real estate mortgage was null and void because Amparo and Zosa were authorized to execute it to
secure only one obligation of Prime Aggregates denied. Petitioner filed an MR - denied.
Petitioner assailed the trial courts orders denying its prayer for the issuance of a writ of preliminary injunction before
the CA via certiorari alleging that the real estate mortgage it executed was null and void for being ultra vires as it was
not empowered to mortgage its properties as security for the payment of obligations of third parties; and that Amparo
and Zosa were authorized to mortgage its properties to secure only a P60,000,000 term loan and one credit facility of
Prime Aggregates.
CA - trial court committed no grave abuse of discretion in denying petitioners prayer for preliminary injunction. It
brushed aside petitioners arguments that the real estate mortgage was ultra vires and that Amparo and Zosa were
only authorized to mortgage petitioners properties to secure the P60,000,000 term loan and one credit facility of
Prime Aggregates. Hence, the present petition for review faulting the Court of Appeals
ISSUE:
Won the real estate mortgage was ultra vires
HELD:
The intention of the Members of the Board of Directors of the Petitioner, in approving the " Resolution," may be
ascertained xxx also from the contemporaneous and subsequent acts of the Petitioner, the Private Respondent and
Prime Aggregates. Given the factual milieu in the present recourse, as found and declared by the Respondent Court,
there can be no equivocation that, indeed the Petitioner conformed to and ratified, and hence, is bound by the
execution, by its Treasurer and General Manager, of the "Real Estate Mortgage" in favor of the Private respondent, with
its properties used as securities for the payment of the credit and loan availments of Prime Aggregates from the
Private Respondent on the basis of the "Resolution" approved by its Board of Directors. As our Supreme Court
declared, ratification and/or approval by the corporation of the acts of its agents/officers may be ascertained through x
x x the acquiescence in his acts of a particular nature, with actual or constructive thereof, whether within or beyond
the scope of his ordinary powers.
As it was, the Petitioner finally awoke from its slumber when the Private Respondent filed its "Petition" for the extrajudicial foreclosure of the "Real Estate Mortgage", with the Sheriff, and assailed the authority of its Board of Directors
to approve the said "Resolution" and of its Treasurer and General Manager to execute the deed and brand the said
"Resolution" and the said deed as "ultra vires" and hence, not binding on the Petitioner, and hurried off to the
Respondent Court and prayed for injunctive relief. Before then, the Petitioner maintained a stoic silence and adopted a
"hands off" stance. We find the Petitioners stance grossly inequitable. We must take heed and pay obeisance to the
equity rule that if one maintains silence when, in conscience he ought to speak, equity will debar him from speaking
when, in conscience, he ought to remain silent. He who remains silent when he ought to speak cannot be heard to

115
3B-Corporation Law

speak when he ought to be silent. More, the transactions between the Petitioner and the Private Respondent over its
properties are neither malum in se or malum prohibitum. Hence, the Petitioner cannot hide behind the cloak of
"ultra vires" for a defense.
The plea of "ultra vires" will not be allowed to prevail, whether interposed for or against a corporation, when it will not
advance justice but, on the contrary, will accomplish a legal wrong to the prejudice of another who acted in good
faith.

116
3B-Corporation Law

Topic:
Subtopic:
Provision:
Case Name:
MINUTES OF THE

Meetings
Minutes of Meetings
Section 49 to 52
Philippine National Bank v MErelo Aznar
MEETING

PHILIPPINE NATIONAL BANK v AZNAR


FACTS:

1958: Rural Insurance and Surety Company, Inc. (RISCO) ceased operation due to business reverses

In plaintiffs (Anzar et al.) desire to rehabilitate RISCO, they contributed a total amount of P212,720.00. This
was used to purchase 3 parcels of land in Cebu (2 in the Minicipality of Talisay and 1 in the District of Lahug)

After the purchase of the lots, titles were issued in the name of RISCO.

The amount contributed by plaintiffs constituted as liens and encumbrances on the properties as annotated in
the titles of said lots. Such annotation was made pursuant to the Minutes of the Special Meeting of the Board
of Directors of RISCO on March 14, 1961, and a part of it says:
And that the respective contributions above-mentioned (Aznar et al.) shall constitute as their lien or interest
on the property described above, if and when said property are titled in the name of RISCO, subject to
registration as their adverse claim in pursuance of the Provision of Land Registration Act, until such time their
respective contributions are refunded to them completely

Thereafter, various subsequent annotations were made on the same titles, including the Notice of Attachment
and Writ of Execution both dated August 3,1962 in favour of Philippine National Bank (PNB).

As a result, a Certificate of Sale was issued in favor of PNB, being the lone and highest bidder of the 3 parcels
of land. This prompted Aznar et al. to file the instant case seeking the quieting of their supposed title to the
subject properties.

Trial court ruled against PNB on the basis that there was an express trust created over the subject properties
whereby RISCO was the trustee and the stockholders, Aznar, et al., were the beneficiaries.

CA opined that the monetary contributions made by Aznar, et al. to RISCO can only be characterised as a load
secured by a lien on the subjected lots, rather than an expressed trust
ISSUE:
Whether there was a trust contract between RISCO and Aznar, et al.
HELD:
No.
At the outset, the Court agrees with the Court of Appeals that the agreement contained in the Minutes of the Special
Meeting of the RISCO Board of Directors held on March 14, 1961 was a loan by the therein named stockholders to
RISCO. Careful perusal of the Minutes relied upon by plaintiffs-appellees in their claim, showed that their contributions
shall constitute as lien or interest on the property. The term lien as used in the Minutes is defined as "a discharge on
property usually for the payment of some debt or obligation. Hence, from the use of the word "lien" in the Minutes, We
find that the money contributed by plaintiffs-appellees was in the nature of a loan, secured by their liens and interests
duly annotated on the titles. The annotation of their lien serves only as collateral and does not in any way vest
ownership of property to plaintiffs.
The Court was not persuaded by the contention of Aznar, et al., that the language of the subject Minutes created an
express trust.

117
3B-Corporation Law

Topic:
Stocks and Stockholders
Subtopic:
Trust Fund Doctrine
Provision:
Case Name:
Turner v Lorenzo
TRUST FUND DOCTRINE
TURNER v LORENZO SHIPPING
FACTS:

The petitioners (Philip and Elnora Turner) held 1,010,000 shares of stock of the respondent (Lorenzo Shipping
Corp.), a domestic corporation engaged primarily in cargo shipping activities. The respondent decided to
amend its articles of incorporation to remove the stockholders pre-emptive rights to newly issued shares of
stock. The petitioners voted against the amendment and demanded payment of their shares at the rate of
P2.276/share based on the book value of the shares, or a total of P2,298,760.00.

The respondent found the fair value of the shares demanded to be unacceptable. It insisted that the market
value on the date before the action to remove the pre-emptive right was taken should be the value, or
P0.41/share (P414,100.00) and that the payment could be made only if the respondent had unrestricted
retained earnings in its books to cover the value of the shares, which was not the case.

The disagreement on the valuation of the shares led the parties to constitute an appraisal committee pursuant
to Sec. 82 of the Corporation Code. The committee reported its valuation of P2.54/share, for an aggregate
value of P2,565,400.00

Subsequently, the petitioners demanded payment based on the valuation plus 2% month penalty from the
date of their original demand for payment, as well as the reimbursement of the amounts advanced as
professional fees to the appraisers.

Respondent refused the petitioners demand, explaining that pursuant to the Corporation Code, the dissenting
stockholders exercising their appraisal rights could be paid only when the corporation had unrestricted
retained earnings to cover the fair value of the shares, but that it had no retained earnings at the time of the
petitioners demand, as borne out by its Financial Statements for Fiscal Year 1999 showing a deficit of
P72,973,114.00.

Upon the respondents refusal to pay, the petitioners sued the respondent for collection and damages in the
RTC.

In a nutshell, RTC entertained the complaint and rendered a summary judgment and issued a writ of execution.

CA ruled otherwise holding that no payment shall be made to any dissenting stockholder unless the
corporation has unrestricted retained earnings in its books to cover such payment.
ISSUE:
Whether CA correctly qualified petitioners appraisal right under the Trust Fund Doctrine
HELD:
Yes.
In the instant case, it was established that there were no unrestricted retained earnings when the Turners filed their
complaint. In case the corporation has no available unrestricted retained earnings in its books, Section 83 of the
Corporation Code provides that if the dissenting stockholder is not paid the value of his shares within 30 days after the
award, his voting and dividend rights shall immediately be restored.
The trust fund doctrine backstops the requirement of unrestricted retained earnings to fund the payment of the shares
of stocks of the withdrawing stockholders. Under the doctrine, the capital stock, property, and other assets of a
corporation are regarded as equity in trust for the payment of corporate creditors, who are preferred in the distribution
of corporate assets. The creditors of a corporation have the right to assume that the board of directors will not use the
assets of the corporation to purchase its own stock for as long as the corporation has outstanding debts and liabilities.
There can be no distribution of assets among the stockholders without first paying corporate debts. Thus, any
disposition of corporate funds and assets to the prejudice of creditors is null and void.

Topic:
Stocks and Stockholders
Subtopic:
Trust Fund Doctrine
Provision:
Case Name:
Turner v Lorenzo
TRUST FUND DOCTRINE

118
3B-Corporation Law

LU v LU YM SR
FACTS:

The main purpose of the complaint filed before the RTC was the annulment of the issuance of the 600,000
LLDC shares of stocks because they had been allegedly issued for less than their par value. Thus, petitioners
sought the dissolution of the corporation and the appointment of receivers/management committee.

The value of the 600,000 shares of stock, which are the properties in litigation, should be the basis for the
computation of the filing fees.

RTC ruled in favor of petitioners by annulling the issuance of the shares of stock subscribed and paid by
respondent father and sons at less than par value, and ordering the dissolution and asset liquidation of LLDC.
The appeal of the trial court's Decision remains pending with the appellate court.
ISSUE:
Whether petitioners paid the correct docket fees for the complaint
HELD:
Yes. The nature of the principal action/remedy sought by petitioner is to declare null and void the issuance of 600,000
unsubscribed and unissued shares to respondents and not to recover properties of LLDC. Hence, the correct docket
fees were paid
It bears noting, however, that petitioners are not claiming to own the subject shares. They do not claim to be the
owners thereof entitled to be the transferees of the shares of stock. The mention of the real value of the shares of
stock, over which petitioners do not, it bears emphasis, interpose a claim of right to recovery, is merely narrative or
descriptive in order to emphasize the inequitable price at which the transfer was effected.
Under the trust fund doctrine, the capital stock, properties, and other assets of a corporation are regarded as held in
trust for the corporate creditors, who, being preferred in the distribution of the corporate assets, must first be paid
before any corporate assets may be distributed among the stockholders.
In the event of the dissolution of LLDC, therefore, petitioners would get only the value of their minority number of
shares, not the value of the 600,000 shares. Indeed, a basic concept in corporate law is that a shareholder's interest in
corporate property, if it exists at all, is indirect, contingent, remote, conjectural, consequential, and collateral. A share
of stock, although representing a proportionate or aliquot interest in the properties of the corporation, does not vest its
holder with any legal right or title to any of the properties, such holder's interest in the properties being equitable or
beneficial in nature. A shareholder is in no legal sense the owner of corporate properties, which are owned by the
corporation as a distinct legal person.

119
3B-Corporation Law

Topic:
Subtopic:
Provision:
Case Name:
Facts:

Issue:

Held:

Stocks and Stockholders


Trust Fund Doctrine
Donina Halley v Printwell

BMPI (Business Media Philippines Inc.) is a corporation under the control of its stockholders, including Donnina
Halley
In the course of its business, BMPI commissioned PRINTWELL to print Philippines, Inc. (a magazine published
and distributed by BMPI)
PRINTWELL extended 30-day credit accommodation in favor of BMPI and in a period of 9 mos. BMPI placed
several orders amounting to 316,000.
However, only 25,000 was paid hence a balance of 291,000
PRINTWELL sued BMPI for collection of the unpaid balance and later on impleaded BMPIs original stockholders
and incorporators to recover on their unpaid subscriptions.
It appears that BMPI has an authorized capital stock of 3M divided into 300,000 shares with P10 par value.
Only 75,000 shares worth P750,000 were originally subscribed of which P187,500 were paid up capital.
Halley subscribed to 35,000 shares worth P350,000 but only paid P87,500. Halley contends that: 1. They all
had already paid their subscriptions in full 2. BMPI had a separate and distinct personality 3. BOD and SH had
resolved to dissolve BMPI RTC and CA o Defendant merely used the corporate fiction as a cloak/cover to create
an injustice (against PRINTWELL) o Rejected allegations of full payment in view of irregularity in the issuance of
ORs (Payment made on a later date was covered by an OR with a lower serial number than payment made on
an earlier date.
WON a stockholder who was in active management of the business of the corporation and still has unpaid
subscriptions should be made liable for the debts of the corporation by piercing the veil of corporate fiction
YES!
Such stockholder should be made liable up to the extent of her unpaid subscription Ratio: It was found that at
the time the obligation was incurred, BMPI was under the control of its stockholders who know fully well that
the corporation was not in a position to pay its account (thinly capitalized). And, that the stockholders
personally benefited from the operations of the corporation even though they never paid their subscriptions
in full. The stockholders cannot now claim the doctrine of corporate fiction otherwise (to deny creditors to
collect from SH) it would create an injustice because creditors would be at a loss (limbo) against whom it would
assert the right to collect.
On piercing the veil: Although the corporation has a personality separate and distinct from its SH, such
personality is merely a legal fiction (for the convenience and to promote the ends of justice) which may be
disregarded by the courts if it is used as a cloak or cover for fraud, justification of a wrong, or an alter ego for
the sole benefit of the SH. As to the Trust Fund Doctrine:
The RTC and CA correctly applied the Trust Fund Doctrine Under which corporate debtors might look to the
unpaid subscriptions for the satisfaction of unpaid corporate debts Subscriptions to the capital of a
corporation constitutes a trust fund for the payment of the creditors (by mere analogy) In reality, corporation
is a simple debtor. Moreover, the corporation has no legal capacity to release an original subscriber to its
capital stock from the obligation of paying for his shares, in whole or in part, without valuable consideration, or
fraudulently, to the prejudice of the creditors.
The creditor is allowed to maintain an action upon any unpaid subscriptions and thereby steps into the shoes
of the corporation for the satisfaction of its debt. The trust fund doctrine is not limited to reaching the SHs
unpaid subscriptions. The scope of the doctrine when the corporation is insolvent encompasses not only the
capital stock but also other property and assets generally regarded in equity as a trust fund for the payment of
corporate debts.

120
3B-Corporation Law

Topic:
Subtopic:
Provision:
Case Name:
Facts:

Stocks and Stockholders


Subscription Contracts
Sections 60 and 61 of Corporation Code
Ong Yong et al v Tiu

In 1994, the construction of the Masagana Citimall in Pasay City was threatened with stoppage and
incompletion when its owner, the First Landlink Asia Development Corporation (FLADC), which was owned by
David S. Tiu, Cely Y. Tiu, Moly Yu Gow, Belen See Yu, D. Terence Y. Tiu, John Yu and Lourdes C. Tiu (the Tius),
encountered dire financial difficulties.
It was heavily indebted to the Philippine National Bank (PNB) for P190 million.
To stave off foreclosure of the mortgage on the two lots where the mall was being built, the Tius invited Ong
Yong, Juanita Tan Ong, Wilson T. Ong, Anna L. Ong, William T. Ong and Julia Ong Alonzo (the Ongs), to invest in
FLADC. Under the Pre-Subscription Agreement they entered into, the Ongs and the Tius agreed to maintain
equal shareholdings in FLADC: the Ongs were to subscribe to 1,000,000 shares at a par value of P100.00 each
while the Tius were to subscribe to an additional 549,800 shares at P100.00 each in addition to their already
existing subscription of 450,200 shares.
Furthermore, they agreed that the Tius were entitled to nominate the Vice President and the Treasurer plus 5
directors while the Ongs were entitled to nominate the President, the Secretary and 6 directors (including the
chairman) to the board of directors of FLADC.
Moreover, the Ongs were given the right to manage and operate the mall. Accordingly, the Ongs paid P100
million in cash for their subscription to 1,000,000 shares of stock while the Tius committed to contribute to
FLADC a four-storey building and two parcels of land respectively valued at P20 million (for 200,000 shares),
P30 million (for 300,000 shares) and P49.8 million (for 49,800 shares) to cover their additional 549,800 stock
subscription therein. The Ongs paid in another P70 million 3 to FLADC and P20 million to the Tius over and
above their P100 million investment, the total sum of which (P190 million) was used to settle the P190 million
mortgage indebtedness of FLADC to PNB.
The business harmony between the Ongs and the Tius in FLADC, however, was shortlived because the Tius, on
23 February 1996, rescinded the Pre-Subscription Agreement.
The Tius accused the Ongs of (1) refusing to credit to them the FLADC shares covering their real property
contributions; (2) preventing David S. Tiu and Cely Y. Tiu from assuming the positions of and performing their
duties as Vice-President and Treasurer, respectively, and (3) refusing to give them the office spaces agreed
upon. The controversy finally came to a head when the case was commenced by the Tius on 27 February 1996
at the Securities and Exchange Commission (SEC), seeking confirmation of their rescission of the PreSubscription Agreement. After hearing, the SEC, through then Hearing Officer Rolando G. Andaya, Jr., issued a
decision on 19 May 1997 confirming the rescission sought by the Tius.
On motion of both parties, the above decision was partially reconsidered but only insofar as the Ongs' P70
million was declared not as a premium on capital stock but an advance (loan) by the Ongs to FLADC and that
the imposition of interest on it was correct. Both parties appealed to the SEC en banc which rendered a
decision on 11 September 1998, affirming the 19 May 1997 decision of the Hearing Officer. The SEC en banc
confirmed the rescission of the PreSubscription Agreement but reverted to classifying the P70 million paid by
the Ongs as premium on capital and not as a loan or advance to FLADC, hence, not entitled to earn interest.
On appeal, the Court of Appeals (CA) rendered a decision on 5 October 1999, modifying the SEC order of 11
September 1998. Their motions for reconsideration having been denied, both parties filed separate petitions
for review before the Supreme Court. On 1 February 2002, the Supreme Court promulgated its Decision,
affirming the assailed decision of the Court of Appeals but with the modifications that the P20 million loan
extended by the Ongs to the Tius shall earn interest at 12% per annum to be computed from the time of
judicial demand which is from 23 April 1996; that the P70 million advanced by the Ongs to the FLADC shall
earn interest at 10% per annum to be computed from the date of the FLADC Board Resolution which is 19 June
1996; and that the Tius shall be credited with 49,800 shares in FLADC for their property contribution,
specifically, the 151 sq. m. parcel of land. The Court affirmed the fact that both the Ongs and the Tius violated
their respective obligations under the Pre-Subscription Agreement.
On 15 March 2002, the Tius filed before the Court a Motion for Issuance of a Writ of Execution. Aside from their
opposition to the Tius' Motion for Issuance of Writ of Execution, the Ongs filed their own "Motion for
Reconsideration; Alternatively, Motion for Modification (of the February 1, 2002 Decision)" on 15 March 2002.
Willie Ong filed a separate "Motion for Partial Reconsideration" dated 8 March 2002, pointing out that there
was no violation of the PreSubscription Agreement on the part of the Ongs, among others. On 29 January 2003,
the Special Second Division of this Court held oral arguments on the respective positions of the parties. On 27
February 2003, Dr. Willie Ong and the rest of the movants Ong filed their respective memoranda. On 28
February 2003, the Tius submitted their memorandum.

Issue

121
3B-Corporation Law


Held

Whether the pre-Subscription Agreement executed by the Ongs is actually a subscription contract.
FLADC was originally incorporated with an authorized capital stock of 500,000 shares with the Tius owning
450,200 shares representing the paid-up capital. When the Tius invited the Ongs to invest in FLADC as
stockholders, an increase of the authorized capital stock became necessary to give each group equal (50-50)
shareholdings as agreed upon in the Pre-Subscription Agreement.
The authorized capital stock was thus increased from 500,000 shares to 2,000,000 shares with a par value of
P100 each, with the Ongs subscribing to 1,000,000 shares and the Tius to 549,800 more shares in addition to
their 450,200 shares to complete 1,000,000 shares.
Thus, the subject matter of the contract was the 1,000,000 unissued shares of FLADC stock allocated to the
Ongs. Since these were unissued shares, the parties' Pre-Subscription Agreement was in fact a subscription
contract as defined under Section 60, Title VII of the Corporation Code.
A subscription contract necessarily involves the corporation as one of the contracting parties since the subject
matter of the transaction is property owned by the corporation its shares of stock. Thus, the subscription
contract (denominated by the parties as a Pre-Subscription Agreement) whereby the Ongs invested P100
million for 1,000,000 shares of stock was, from the viewpoint of the law, one between the Ongs and FLADC,
not between the Ongs and the Tius. Otherwise stated, the Tius did not contract in their personal capacities
with the Ongs since they were not selling any of their own shares to them.
It was FLADC that did. Considering therefore that the real contracting parties to the subscription agreement
were FLADC and the Ongs alone, a civil case for rescission on the ground of breach of contract filed by the Tius
in their personal capacities will not prosper. Assuming it had valid reasons to do so, only FLADC (and certainly
not the Tius) had the legal personality to file suit rescinding the subscription agreement with the Ongs
inasmuch as it was the real party in interest therein. Article 1311 of the Civil Code provides that "contracts
take effect only between the parties, their assigns and heirs. . ." Therefore, a party who has not taken part in
the transaction cannot sue or be sued for performance or for cancellation thereof, unless he shows that he has
a real interest affected thereby.

122
3B-Corporation Law

Topic:
Subtopic:
Provision:
Case Name:
Facts

Stocks and Stockholders


Trust Fund Doctrine
Yamamoto v Nishino Leather Industries

To disregard the separate juridical personality of a corporation, the wrongdoing or unjust act in contravention
of a plaintiffs legal rights must be clearly and convincingly established.
Also, without acceptance, a mere offer produces no obligation. Ryuichi Yamamoto and Ikuo Nishino agreed to
enter into a joint venture wherein Nishino would acquire such number of shares of stock equivalent to 70% of
the authorized capital stock of the corporation.
However, Nishino and his brother Yoshinobu Nishino acquired more than 70% of the authorized capital stock.
Negotiations subsequently ensued in light of a planned takeover by Nishino who would buy-out the shares of
stock of Yamamoto who was advised through a letter that he may take all the equipment/ machinery he had
contributed to the company (for his own use and sale) provided that the value of such machines is deducted
from the capital contributions which will be paid to him.
However, the letter requested that he give his comments on all the above, soonest. On the basis of the said
letter, Yamamoto attempted to recover the machineries but Nishino hindered him to do so, drawing him to file
a Writ of Replevin.
The Trial Court issued the writ.
However, on appeal, Nishino claimed that the properties being recovered were owned by the corporation and
the abovesaid letter was a mere proposal which was not yet authorized by the Board of Directors. Thus, the
Court of Appeals reversed the trial courts decision despite Yamamotos contention that the company is merely
an instrumentality of the Nishinos.

ISSUE:

Whether or not Yamamoto can recover the properties he contributed to the company in view of the Doctrine of
Piercing the Veil of Corporate Fiction and Doctrine of Promissory Estoppel.
HELD:

One of the elements determinative of the applicability of the doctrine of piercing the veil of corporate fiction is
that control must have been used by the defendant to commit fraud or wrong, to perpetuate the violation of a
statutory or other positive legal duty, or dishonest and unjust act in contravention of the plaintiffs legal rights.

To disregard the separate juridical personality of a corporation, the wrongdoing or unjust act in contravention
of a plaintiffs legal rights must be clearly and convincingly established; it cannot be presumed. Without a
demonstration that any of the evils sought to be prevented by the doctrine is present, it does not apply.
Estoppel may arise from the making of a promise. However, it bears noting that the letter was followed by a
request for Yamamoto to give his comments on all the above, soonest. What was thus proffered to Yamamoto
was not a promise, but a mere offer, subject to his acceptance. Without acceptance, a mere offer produces no
obligation. Thus, the machineries and equipment, which comprised Yamamotos investment, remained part of
the capital property of the corporation.

123
3B-Corporation Law

Topic:
Stocks and Stockholders
Subtopic:
Trust Fund Doctrine
Provision:
Case Name:
Ong Yong v Tiu
Ong Yong v. Tiu
Facts:
Tiu is the owner/developer of Masagana Citimall, under the First Landlink Asia Development Corporation (FLADC). Its
construction was threatened by financial constraints, thus Tiu invited Tiu to invest in said mall. A Pre-subscription
agreement was made to the effect that Ongs would subscribe to 1,000,000 shares while the Tius will subscribe to an
additional 549,800 shares in addition to their already existing subscription of 450,200 shares. Squabble later on
ensued which prompted Tiu to rescind the pre-subscription agreement.
Issue: Whether or not the rescission of the pre-subscription agreement was proper.
Held: No, it was not proper.
Ratio:
The Trust Fund Doctrine, first enunciated by this Court in the 1923 case of Philippine Trust Co. vs. Rivera, provides
that subscriptions to the capital stock of a corporation constitute a fund to which the creditors have a right to look for
the satisfaction of their claims. This doctrine is the underlying principle in the procedure for the distribution of capital
assets, embodied in the Corporation Code, which allows the distribution of corporate capital only in three instances: (1)
amendment of the Articles of Incorporation to reduce the authorized capital stock, (2) purchase of redeemable shares
by the corporation, regardless of the existence of unrestricted retained earnings, and (3) dissolution and eventual
liquidation of the corporation. Furthermore, the doctrine is articulated in Section 41 on the power of a corporation to
acquire its own shares and in Section 122 on the prohibition against the distribution of corporate assets and property
unless the stringent requirements therefor are complied with.
The distribution of corporate assets and property cannot be made to depend on the whims and caprices of the
stockholders, officers or directors of the corporation, or even, for that matter, on the earnest desire of the court a quo
"to prevent further squabbles and future litigations" unless the indispensable conditions and procedures for the
protection of corporate creditors are followed. Otherwise, the "corporate peace" laudably hoped for by the court will
remain nothing but a dream because this time, it will be the creditors' turn to engage in "squabbles and litigations"
should the court order an unlawful distribution in blatant disregard of the Trust Fund Doctrine.
In the instant case, the rescission of the Pre-Subscription Agreement will effectively result in the unauthorized
distribution of the capital assets and property of the corporation, thereby violating the Trust Fund Doctrine and the
Corporation Code, since rescission of a subscription agreement is not one of the instances when distribution of capital
assets and property of the corporation is allowed.
Contrary to the Tius' allegation, rescission will, in the final analysis, result in the premature liquidation of the
corporation without the benefit of prior dissolution in accordance with Sections 117, 118, 119 and 120 of the
Corporation Code. The Tius maintain that rescinding the subscription contract is not synonymous to corporate
liquidation because all rescission will entail would be the simple restoration of the status quo ante and a return to the
two groups of their cash and property contributions. We wish it were that simple. Very noticeable is the fact that the
Tius do not explain why rescission in the instant case will not effectively result in liquidation. The Tius merely refer in
cavalier fashion to the end-result of rescission (which incidentally is 100% favorable to them) but turn a blind eye to its
unfair, inequitable and disastrous effect on the corporation, its creditors and the Ongs.

124
3B-Corporation Law

Topic:
Stocks and Stockholders
Subtopic:
Certificate fo Stocks and Transfer of Shares
Provision:
Sections 63 and 64, Corporation Code
Case Name:
Puno v Puno Enterprises
RIGHTS TO INSPECT BOOKS AND RECORDS (Sec. 63 & 74, Corp. Code)
Puno vs Puno Enterprises
FACTS:

Carlos L. Puno, who died on June 25, 1963, was an incorporator of respondent Puno Enterprises, Inc.
On March 14, 2003, petitioner Joselito Musni Puno, claiming to be an heir of Carlos L. Puno, initiated a
complaint for specific performance against respondent. Petitioner averred that he is the son of the deceased
with the latters common-law wife, Amelia Puno. As surviving heir, he claimed entitlement to the rights and
privileges of his late father as stockholder of respondent. The complaint thus prayed that respondent allow
petitioner to inspect its corporate book, render an accounting of all the transactions it entered into from 1962,
and give petitioner all the profits, earnings, dividends, or income pertaining to the shares of Carlos L. Puno.
RTC ordered Puno Enterprises to let their records be inspected by plaintiff Joselito Puno.
CA dismissed.

Issue:
Whether or not Joselito Musni Puno as an heir is automatically entitled for the stocks upon the death of a shareholder.
Held:
NO.
The stockholders right of inspection of the corporations books and records is based upon his ownership of shares in
the corporation and the necessity for self-protection. After all, a shareholder has the right to be intelligently informed
about corporate affairs. Such right rests upon the stockholders underlying ownership of the corporations assets and
property
Upon the death of a shareholder, the heirs do not automatically become stockholders of the corporation and acquire
the rights and privileges of the deceased as shareholder of the corporation. The stocks must be distributed first to the
heirs in estate proceedings, and the transfer of the stocks must be recorded in the books of the corporation. Section 63
of the Corporation Code provides that no transfer shall be valid, except as between the parties, until the transfer is
recorded in the books of the corporation. During such interim period, the heirs stand as the equitable owners of the
stocks, the executor or administrator duly appointed by the court being vested with the legal title to the stock. Until a
settlement and division of the estate is effected, the stocks of the decedent are held by the administrator or executor.
Consequently, during such time, it is the administrator or executor who is entitled to exercise the rights of the
deceased as stockholder.
Thus, even if petitioner presents sufficient evidence in this case to establish that he is the son of Carlos L. Puno, he
would still not be allowed to inspect respondents books and be entitled to receive dividends from respondent, absent
any showing in its transfer book that some of the shares owned by Carlos L. Puno were transferred to him. This would
only be possible if petitioner has been recognized as an heir and has participated in the settlement of the estate of the
deceased.

125
3B-Corporation Law

Topic:
Stocks and Stockholders
Subtopic:
Certificate of Stocks and Transfer of Shares
Provision:
Sections 63 and 64, Corporation Code
Case Name:
Reyes v RTC of Makati
CERTIFICATE OF STOCKS AND TRANSFER OF SHARES (Sec. 63 & 64, Corp. Code)
Oscar Reyes vs Makati RTC, Zenith Insurance, Rodrigo Reyes
FACTS:

Rodrigo Reyes and Oscar Reyes are two of the four children of Pedro and Anastacia Reyes. They are all
stockholders of Zenith Insurance, a family corporation.

Pedro died in the 1970s and Anastacia died in 1993. Pedros estate, presumably including his shares in Zenith,
was partitioned, but Anastacias was not. In 1990, Oscar Reyes had 8,715,637 shares of stock, Rodrigo had
4,250 shares, and the deceased Anastacia still had 136,598 shares of stock.

Rodrigo and Zenith filed suit with the SEC against Oscar Reyes, for the accounting of funds and assets of
Zenith to determine the shares of stock of Pedro and Anastacia that were allegedly fraudulently and arbitrarily
appropriated by Oscar without the requisite partition or settlement of estate of Anastacia, to the detriment of
his siblings.

The case was transferred to the RTC acting as a Special Commercial Court (SCC) due to the passage of RA
8799 during the pendency of the case.
o Oscar filed a MTD, arguing that he acquired the shares by purchasing them with his own funds from
the unissued stocks of Zenith, and that the action is primarily for the settlement of the estate, and as
such should be with the probate court, not the RTC as a SCC.
o RTC denied the motion, holding that there are two causes of action; (1) a derivative suit for accounting
of the funds and assets of Zenith, and (2) determination of the shares of stock of Anastacio and Pedro
for distribution to the co-heirs which should be filed with the probate. RTC held that it will decide only
on the first cause of action.
o CA affirmed.
ISSUE:
Whether or not the suit is an intracorporate controversy properly within the jurisdiction of the RTC as a SCC.
HELD:
NO.
A review of relevant jurisprudence shows a development in the Courts approach in classifying what constitutes an
intra-corporate controversy. Initially, the main consideration in determining whether a dispute constitutes an intracorporate controversy was limited to a consideration of the intra-corporate relationship existing between or among the
parties.
The Court combined two tests, the relationship test (i.e. types of relationships embraced under Section 5(b) of PD 902A), and the nature of the controversy test (i.e. the controversy must not only be rooted in the existence of an intracorporate relationship, but must as well pertain to the enforcement of the parties correlative rights and obligations
under the Corporation Code and the internal and intra-corporate regulatory rules of the corporation) and thus declared
that jurisdiction should be determined by considering not only the status or relationship of the parties, but also the
nature of the question under controversy.
UNDER THE RELATIONSHIP TEST
While Rodrigo holds shares of stock in Zenith, he holds them in two capacities: in his own right with respect to the
4,250 shares registered in his name, and as one of the heirs of Anastacia Reyes with respect to the 136,598 shares
registered in her name. What is material in resolving the issues of this case under the allegations of the complaint is
Rodrigos interest as an heir since the subject matter of the present controversy centers on the shares of stocks
belonging to Anastacia, not on Rodrigos personally-owned shares nor on his personality as shareholder owning these
shares.
Upon Anastacias death, her children acquired legal title to her estate (which title includes her shareholdings in
Zenith), and they are, prior to the estates partition, deemed co-owners thereof. This status as co-owners, however,
does not immediately and necessarily make them stockholders of the corporation. Unless and until there is compliance
with Section 63 of the Corporation Code on the manner of transferring shares, the heirs do not become registered
stockholders of the corporation. Simply stated, the transfer of title by means of succession, though effective and valid
between the parties involved (i.e., between the decedents estate and her heirs), does not bind the corporation and

126
3B-Corporation Law

third parties. The transfer must be registered in the books of the corporation to make the transferee-heir a stockholder
entitled to recognition as such both by the corporation and by third parties.
Therfore, Rodrigo must first prove that there are shareholdings that will be left to him and his co-heirs, and this can be
determined only in a settlement of the decedents estate. No such proceeding has been commenced to date. Second,
he must register the transfer of the shares allotted to him to make it binding against the corporation. He cannot
demand that this be done unless and until he has established his specific allotment (and prima facie ownership) of the
shares. Without the settlement of Anastacias estate, there can be no definite partition and distribution of the estate to
the heirs. Without the partition and distribution, there can be no registration of the transfer. And without the
registration, he cannot be considered a stockholder who may invoke the existence of an intra-corporate relationship as
premise for an intra-corporate controversy within the jurisdiction of a special commercial court. Thus, he fails under
the relationship test.
UNDER THE NATURE OF CONTROVERSY TEST
Contrary to the findings of both the trial and appellate courts, there is only one cause of action alleged in the
complaint. The "derivative suit for accounting of the funds and assets of the corporation which are in the control,
custody, and/or possession of the respondent [herein petitioner Oscar]" does not constitute a separate cause of action
but is, as correctly claimed by Oscar, only an incident to the "action for determination of the shares of stock of
deceased spouses Pedro and Anastacia Reyes allegedly taken by respondent, its accounting and the corresponding
delivery of these shares to the parties brothers and sisters." There can be no mistake of the relationship between the
"accounting" mentioned in the complaint and the objective of partition and distribution.
That an accounting of the funds and assets of Zenith to determine the extent and value of Anastacias shareholdings
will be undertaken by a probate court and not by a special commercial court is completely consistent with the probate
courts limited jurisdiction. It has the power to enforce an accounting as a necessary means to its authority to
determine the properties included in the inventory of the estate to be administered, divided up, and distributed.
Beyond this, the determination of title or ownership over the subject shares (whether belonging to Anastacia or Oscar)
may be conclusively settled by the probate court as a question of collation or advancement. Thus, on this score, the
complaint must necessarily fail as well.

127
3B-Corporation Law

Topic:
Stocks and Stockholders
Subtopic:
Certificate of Stocks and Transfer of Shares
Provision:
Sections 63 and 64, Corporation Code
Case Name:
Ponce v Alsons Cement Corp
Certificate of Stock and Transfer of Shares
Ponce v. Alsons Cement Corp. (G.R. NO. 139802, December 10, 2002)

Vicente C. Ponce and Fausto Gaid, incorporator of Victory Cement Corporation, executed a Deed of
Undertaking and Indorsement whereby Gaid acknowledges that Ponce is the owner of the shares and he
was therefore assigning/endorsing it to Ponce
VCC was renamed Floro Cement Corporation and then to Alsons Cement Corporation
Up to the present, no certificates of stock corresponding to the 239,500 subscribed and fully paid shares of
Gaid were issued in the name of Fausto G. Gaid and/or the plaintiff.
Despite repeated demands, the ACC refused to issue the certificates of stocks
SEC Hearing Officer Enrique L. Flores, Jr. granted the motion to dismiss, upon appeal, the Commission En Banc
reversed the decision of the Hearing Officer
Ponce, filed a complaint with the SEC for mandamus but CA dismissed for failure to state a cause of action in
the absence of any allegation that the transfer of the shares was registered in the stock and transfer book

ISSUE: W/N the certificate of stocks of Gaid can be transferred to Ponce


Ruling: NO. The certificate of stocks cannot be transferred to Ponce.

SEC. 63. Certificate of stock and transfer of shares.The capital stock of stock corporations shall be divided into
shares for which certificates signed by the president or vice-president, countersigned by the secretary or
assistant secretary, and sealed with the seal of the corporation shall be issued in accordance with the by-laws.
Shares of stock so issued are personal property and may be transferred by delivery of the certificate or
certificates indorsed by the owner or his attorney-in-fact or other person legally authorized to make the
transfer. No transfer, however, shall be valid, except as between the parties, until the transfer is recorded in
the books of the corporation so as to show the names of the parties to the transaction, the date of the transfer,
the number of the certificate or certificates and the number of shares transferred. No shares of stock against
which the corporation holds any unpaid claim shall be transferable in the books of the corporation.
The stock and transfer book is the basis for ascertaining the persons entitled to the rights and subject to the
liabilities of a stockholder
Where a transferee is not yet recognized as a stockholder, the corporation is under no specific legal duty to
issue stock certificates in the transferees name.
In the current case, a mandamus should not issue to compel the secretary of a corporation to make a transfer
of the stock on the books of the company unless it affirmatively appears that he has failed or refused so to do,
upon the demand either of the person in whose name the stock is registered, or of some person holding a
power of attorney for that purpose from the registered owner of the stock.
Mere indorsee of a stock certificate, claiming to be the owner, will not necessarily be recognized as such by the
corporation and its officers, in the absence of express instructions of the registered owner to make such
transfer to the indorsee, or a power of attorney authorizing such transfer

Topic:
Stocks and Stockholders
Subtopic:
Certificate of Stocks and Transfer of Shares
Provision:
Sections 63 and 64, Corporation Code
Case Name:
Makati Sports Club v Cheng
Certificate of Stocks and Transfer of Shares

128
3B-Corporation Law

Makati Sports Club Inc v. Cecile Cheng (G.R. No. 178523, June 16, 2010)

Makati Sports Club Inc (MSCI) Board of Directors adopted a resolution authorizing the sale of 19 unissued
shares at a floor price of P400,000 and P450,000 per share for Class A and B, respectively.
Respondent Cheng was a Treasurer and Director of Makati Sports Club in 1995
Hodreal expressed his interest to buy a share, for this purpose he sent the letter requesting to be wait listed
McFoods acquired the shares of Makati Sports Club at P1,800,000 through Urban Bank. Stock certificate was
issued to McFoods.
McFoods advised its offer to resell , Hodreal paid McFoods P1,400,000 twice on different dates.
Cheng advised sale by McFoods to Hodreal of the share evidenced by a new certificate that was issued
Investigation showed that Cheng profited from the transaction because of her knowledge
MSCI sought judgment that would order respondents to pay the sum of P1,000,000.00, representing the
amount allegedly defrauded, together with interest and damages

ISSUE: W/N MSCI was defrauded by Cheng's collaboration with Mc Foods


Ruling: NO. Cheng did not defraud MSCI

No evidence on record that the Membership Committee acted on Hodreal's letter


SEC. 29. (a) The Membership Committee shall process applications for membership; ascertain that the
requirements for stock ownership, including citizenship, are complied with; submit to the Board its
recommended on applicants for inclusion in the Waiting List; take charge of auction sales of shares of stock;
and exercise such other powers and perform such other functions as may be authorized by the Board.
Membership Committee failed to question the alleged irregularities attending Mc Foods purchase
Price of P1,800,000.00 is P1,400,000.00 more than the floor price which was not detrimental
Upon payment and the execution of the Deed of Absolute Sale, it had the right to demand the delivery of the
stock certificate in its name. The right of a transferee to have stocks transferred to its name is an inherent
right flowing from its ownership of the stocks
Certificate of stock paper representative or tangible evidence of the stock itself and of the various interests
therein not a stock in the corporation but is merely evidence of the holders interest and status in the
corporation, his ownership of the share represented thereby
MSCI failed to repurchase Mc Foods Class "A" share within the 30 day pre-emptive period and there was no
proof that Cheng personally profited from the transaction

129
3B-Corporation Law

Topic:
Stocks and Stockholders
Subtopic:
Certificate of Stocks and Transfer of Shares
Provision:
Sections 63 and 64, Corporation Code
Case Name:
Republic v Sandiganbayan
Certificate of Stocks and Transfer of Shares
Republic v. Sandiganbayan (G.R. No. 166859, G.R. No. 169203, G.R. No. 180702, April 12, 2011)

The Republic commenced a Civil Case in the Sandiganbayan by complaint, impleading as defendants
respondent Eduardo M. Cojuangco, Jr. and 59 individual defendants.
Cojuangco allegedly purchased a block of 33,000,000 shares of SMC stock through the 14 holding companies
owned by the CIIF Oil Mills.
Petitioner contends that Cojuangco is the undisputed "coconut king" with unlimited powers to deal with the
coconut levy funds, who took undue advantage of his association, influence and connection, acting in unlawful
concert with Defendants Ferdinand E. Marcos, misused coconut levy funds to buy out majority of the
outstanding shares of stock of San Miguel Corporation.
Defendants Eduardo Cojuangco, Jr., and ACCRA law offices plotted, devised, schemed, conspired and
confederated with each other in setting up, through the use of coconut levy funds, the financial and corporate
framework and structures that led to the establishment of UCPB, UNICOM, COCOLIFE, COCOMARK. CIC, and
more than twenty other coconut levy-funded corporations, including the acquisition of San Miguel Corporation
shares and its institutionalization through presidential directives of the coconut monopoly.
Sandiganbayan dismissed the case for failure of plaintiff to prove by preponderance of evidence its causes of
action against defendants with respect to the twenty percent (20%) outstanding shares of stock of San Miguel
Corporation registered in defendants names
Petitioner appealed the case to the Supreme Court invoking that coconut levy funds are public funds. The SMC
shares, which were acquired by respondents Cojuangco, Jr. and the Cojuangco companies with the use of
coconut levy funds is in violation of respondent Cojuangco, Jr.s fiduciary obligation that are, necessarily, public
in character and should be reconveyed to the government.

ISSUE: W/N Respondent Cojuangco Jr. used the coconut levy funds to acquire SMC shares in violation of his fiduciary
obligation as a public officer?
Ruling: No, Respondent did not violate any fiduciary duties.

It does not suffice, as in this case, that the respondent is or was a government official or employee during the
administration of former Pres. Marcos. There must be a prima facie showing that the respondent unlawfully
accumulated wealth by virtue of his close association or relation with former Pres. Marcos and/or his wife.
It was Petitioners burden to establish by preponderance of evidence that respondents SMC shares had been
illegally acquired with coconut-levy funds was not discharged.
The conditions for the application of Articles 1455 and 1456 of the Civil Code (like the trustee sing trust funds
to purchase, or a person acquiring property through mistake or fraud), and Section 31 of the Corporation Code
(like a director or trustee willfully and knowingly voting for or assenting to patently unlawful acts of the
corporation, among others) require factual foundations to be first laid out in appropriate judicial proceedings.
Concluding that Cojuangco breached fiduciary duties as an officer and member of the Board of Directors of the
UCPB without competent evidence thereon would be unwarranted and unreasona le.Thus, the Sandiganbayan
could not fairly find that Cojuangco had committed breach of any fiduciary duties as an officer and member of
the Board of Directors of the UCPB.
The Amended Complaint contained no clear factual allegation on which to predicate the application of Articles
1455 and 1456 of the Civil Code, and Section 31 of the Corporation Code. Although the trust relationship
supposedly arose from Cojuangcos being an officer and member of the Board of Directors of the UCPB, the
link between this alleged fact and the borrowings or advances was not established.
There was no showing in the evidence, for fraud or breach of trust is never presumed, but must be alleged and
proved. The thrust of the Republic that the funds were borrowed or lent might even preclude any consequent
trust implication but is more inclined to be a contract of loan.
To say that a relationship is fiduciary when existing laws do not provide for such requires evidence that
confidence is reposed by one party in another who exercises dominion and influence.
Absent any special facts and circumstances proving a higher degree of responsibility, any dealings between a
lender and borrower are not fiduciary in nature.

130
3B-Corporation Law

The shares are declared to be exclusive property of Cojuangco.

131
3B-Corporation Law

Topic:
Subtopic:
Provision:
Case Name:

Stocks and Stockholders


Certificate of Stocks and Transfer of Shares
Sections 63 and 64, Corporation Code
Fontana Resort and Country Club v Spouses Tan
FONTANA RESORT AND COUNTRY CLUB, INC. AND RN DEVELOPMENT CORP.
- versus SPOUSES ROY S. TAN AND SUSAN C. TAN,
G.R. No. 154670, January 30, 2012

FACTS:

Respondent spouses Roy S. Tan and Susana C. Tan bought from petitioner RN Development Corporation (RNDC) two
class D shares of stock in petitioner Fontana Resort and Country Club, Inc. (FRCCI), worth P387,300.00, enticed by the
promises of petitioners sales agents that petitioner FRCCI would construct a park with first-class leisure facilities in
Clark Field, Pampanga, to be called Fontana Leisure Park (FLP); that FLP would be fully developed and operational by
the first quarter of 1998; and that FRCCI class D shareholders would be admitted to one membership in the country
club, which entitled them to use park facilities and stay at a two-bedroom villa for five (5) ordinary weekdays and two
(2) weekends every year for free.
Two years later, respondents filed before the SEC a Complaint for refund of the P387,300.00 they spent to purchase
FRCCI shares of stock from petitioners. Respondents alleged that they had been deceived into buying FRCCI shares
because of petitioners fraudulent misrepresentations. Construction of FLP turned out to be still unfinished and the
policies, rules, and regulations of the country club were obscure.
Sps Tan narrated that they were able to book and avail themselves of free accommodations at an FLP villa on
September 5, 1998, a Saturday. They requested that an FLP villa again be reserved for their free use on October 17,
1998, another Saturday, for the celebration of their daughters 18 th birthday, but were refused by petitioners.
Petitioners clarified that respondents were only entitled to free accommodations at FLP for one week annually
consisting of five (5) ordinary days, one (1) Saturday and one (1) Sunday[,] and that respondents had already
exhausted their free Saturday pass for the year. According to respondents, they were not informed of said rule
regarding their free accommodations at FLP, and had they known about it, they would not have availed themselves of
the free accommodations on September 5, 1998. In January 1999, respondents attempted once more to book and
reserve an FLP villa for their free use on April 1, 1999, a Thursday. Their reservation was confirmed by a certain Murphy
Magtoto. However, on March 3, 1999, another country club employee named Shaye called respondents to say that
their reservation for April 1, 1999 was cancelled because the FLP was already fully booked.
FONTANA countered that Sps Tan had been duly informed of the privileges given to them as shareholders of FRCCI
class D shares of stock since these were all explicitly provided in the promotional materials for the country club, the
Articles of Incorporation, and the By-Laws of FRCCI. It further denied that they unjustly cancelled Sps Tans reservation
for an FLP villa. Lastly, Fontana averred that when the spouses were first accommodated at FLP, only minor or finishing
construction works were left to be done and that facilities of the country club were already operational.

ISSUE:
Was the essence of the judgment of the SEC which ordered the return of the purchase price but not of the thing sold a
declaration of rescission or annulment of the contract of sale between RNDC and respondents?
HELD:
It does not matter that the Spouses Tan, in their Complaint, simply prayed for refund of the purchase price they had
paid for their FRCCI shares, without specifically mentioning the annulment or rescission of the sale of said shares. CA
treated respondents Complaint as one for annulment/rescission of contract and, accordingly, it did not simply order
petitioners to refund to respondents the purchase price of the FRCCI shares, but also directed respondents to comply
with their correlative obligation of surrendering their certificates of shares of stock to petitioners.

132
3B-Corporation Law

In this case, respondents have miserably failed to prove how petitioners employed fraud to induce respondents to buy
FRCCI shares. It can only be expected that petitioners presented the FLP and the country club in the most positive light
in order to attract investor-members. There is no showing that in their sales talk to respondents, petitioners actually
used insidious words or machinations, without which, respondents would not have bought the FRCCI
shares. Respondents appear to be literate and of above-average means, who may not be so easily deceived into
parting with a substantial amount of money. What is apparent to us is that respondents knowingly and willingly
consented to buying FRCCI shares, but were later on disappointed with the actual FLP facilities and club membership
benefits.

133
3B-Corporation Law

Topic:
Subtopic:
Provision:
Case Name:

Stocks and Stockholders


Certificate of Stocks and Transfer of Shares
Sections 63 and 64, Corporation Code
Forest Hills Golf and Country Club v Vertex Sales and Trading
FOREST HILLS GOLF & COUNTRY CLUB,
vs.
VERTEX SALES AND TRADING, INC.,
G.R. No. 202205, March 6, 2013

FACTS
Forest Hills is a domestic non-profit stock corporation that operates and maintains a golf and country club facility in
Antipolo City. Forest Hills was created as a result of a joint venture agreement between Kings Properties Corporation
(Kings) and Fil-Estate Golf and Development, Inc. (FEGDI). Accordingly, Kings and FEGDI owned the shares of stock of
Forest Hills, holding 40% and 60% of the shares, respectively.
In August 1997, FEGDI sold to RS Asuncion Construction Corporation (RSACC) one (1) Class "C" common share of
Forest Hills for P1.1 million. Prior to the full payment of the purchase price, RSACC transferred its interests over FEGDI's
Class "C" common share to respondent Vertex Sales and Trading, Inc. (Vertex). 4 RSACC advised FEGDI of the transfer
and FEGDI, in turn, requested Forest Hills to recognize Vertex as a shareholder. Forest Hills acceded to the request, and
Vertex was able to enjoy membership privileges in the golf and country club.
Despite the sale of FEGDI's Class "C" common share to Vertex, the share remained in the name of FEGDI, prompting
Vertex to demand for the issuance of a stock certificate in its name. As its demand went unheeded, Vertex filed a
complaint6 for rescission with damages against defendants Forest Hills, FEGDI, and Fil-Estate Land, Inc. (FELI) the
developer of the Forest Hills golf course. Vertex averred that the defendants defaulted in their obligation as sellers
when they failed and refused to issue the stock certificate covering the Class "C" common share. It prayed for the
rescission of the sale and the return of the sums it paid; it also claimed payment of actual damages for the defendants
unjustified refusal to issue the stock certificate.
Forest Hills denied transacting business with Vertex and claimed that it was not a party to the sale of the share; FELI
claimed the same defense. While admitting that no stock certificate was issued, FEGDI alleged that Vertex nonetheless
was recognized as a stockholder of Forest Hills and, as such, it exercised rights and privileges of one. FEGDI added that
during the pendency of Vertex's action for rescission, a stock certificate was issued in Vertex's name, but Vertex
refused to accept it.
RTC dismissed Vertex's complaint after finding that the failure to issue a stock certificate did not constitute a violation
of the essential terms of the contract of sale that would warrant its rescission. The RTC noted that the sale was already
consummated notwithstanding the non-issuance of the stock certificate. The issuance of a stock certificate is a
collateral matter in the consummated sale of the share; the stock certificate is not essential to the creation of the
relation of a shareholder. Hence, the RTC ruled that the non-issuance of the stock certificate is a mere casual breach
that would not entitle Vertex to rescind the sale. CA reversed the RTC. It declared that "in the sale of shares of stock,
physical delivery of a stock certificate is one of the essential requisites for the transfer of ownership of the stocks
purchased.
ISSUE:
Whether or not CA erred in declaring the rescission of the sale of one (1) Class "C" common share of Forest Hills to
Vertex and ordering the return by Forest Hills, FEGDI, and FELI to Vertex of the amount the latter paid by reason of the
sale.
HELD:

134
3B-Corporation Law

The Court found for Forest Hills. The question of rescission of the sale of the share is a settled matter that the
Court can no longer review in this petition. While Forest Hills questioned and presented its arguments against the CA
ruling rescinding the sale of the share in its petition, it is not the proper party to appeal this ruling.
As correctly pointed out by Forest Hills, it was not a party to the sale even though the subject of the sale was its share
of stock. The corporation whose shares of stock are the subject of a transfer transaction (through sale, assignment,
donation, or any other mode of conveyance) need not be a party to the transaction, as may be inferred from the terms
of Section 63 of the Corporation Code. However, to bind the corporation as well as third parties, it is necessary that the
transfer is recorded in the books of the corporation. In the present case, the parties to the sale of the share were FEGDI
as the seller and Vertex as the buyer (after it succeeded RSACC). As party to the sale, FEGDI is the one who may
appeal the ruling rescinding the sale. The remedy of appeal is available to a party who has "a present interest in the
subject matter of the litigation and is aggrieved or prejudiced by the judgment. A party, in turn, is deemed
aggrieved or prejudiced when his interest, recognized by law in the subject matter of the lawsuit, is
injuriously affected by the judgment, order or decree." The rescission of the sale does not in any way prejudice
Forest Hills in such a manner that its interest in the subject matter the share of stock is injuriously affected. Thus,
Forest Hills is in no position to appeal the ruling rescinding the sale of the share. Since FEGDI, as party to the sale, filed
no appeal against its rescission, we consider as final the CAs ruling on this matter.

135
3B-Corporation Law

Topic:
Subtopic:
Provision:
Case Name:

Stocks and Stockholders


Rights of Shareholders: Derivative Actions and other Actions of
Stockholders
Section 72, Corporation Code
Yu v Yugaykuan
ANTHONY YU et al.
vs.
JOSEPH YUKAYGUAN et al.
GR 177549, 18 June 2009

FACTS:
Petitioners and the respondents were all stockholders of Winchester Industrial Supply, Inc. On 15 October 2002,
respondents filed against petitioners a verified Complaint forAccounting, Inspection of Corporate Books and Damages
through Embezzlement and Falsification of Corporate Records and Accounts 1[6] before the RTC of Cebu. The said
Complaint was filed by respondents, in their own behalf and as a derivative suit on behalf of Winchester, Inc., and was
docketed as SRC Case No. 022-CEB. The factual background of the Complaint was stated in the attached Affidavit
executed by respondent Joseph.
According to respondents, Winchester, Inc. was established and incorporated on 12 September 1977, with petitioner
Anthony as one of the incorporators, holding 1,000 shares of stock worth P100,000.00. Petitioner Anthony paid for the
said shares of stock with respondent Josephs money, thus, making the former a mere trustee of the shares for the
latter.
The case at bar was initiated before the RTC by respondents as a derivative suit, on their own behalf and on behalf of
Winchester, Inc., primarily in order to compel petitioners to account for and reimburse to the said corporation the
corporate assets and funds which the latter allegedly misappropriated for their personal benefit.
ISSUE:
Whether or not the derivative suit is valid.
RULING:
YES.
The general rule is that where a corporation is an injured party, its power to sue is lodged with its board of directors or
trustees. Nonetheless, an individual stockholder is permitted to institute a derivative suit on behalf of the corporation
wherein he holds stocks in order to protect or vindicate corporate rights, whenever the officials of the corporation
refuse to sue, or are the ones to be sued, or hold the control of the corporation. In such actions, the suing stockholder
is regarded as a nominal party, with the corporation as the real party in interest. A derivative action is a suit by a
shareholder to enforce a corporate cause of action. The corporation is a necessary party to the suit. And the relief
which is granted is a judgment against a third person in favor of the corporation.
Glaringly, a derivative suit is fundamentally distinct and independent from liquidation proceedings. They are neither
part of each other nor the necessary consequence of the other. There is totally no justification for the Court of Appeals
to convert what was supposedly a derivative suit instituted by respondents, on their own behalf and on behalf of
Winchester, Inc. against petitioners, to a proceeding for the liquidation of Winchester, Inc.
While it may be true that the parties earlier reached an amicable settlement, in which they agreed to already
distribute the assets of Winchester, Inc., and in effect liquidate said corporation, it must be pointed out that
respondents themselves repudiated said amicable settlement before the RTC, even after the same had been partially
implemented; and moved that their case be set for pre-trial. Attempts to again amicably settle the dispute between
the parties before the Court of Appeals were unsuccessful.

136
3B-Corporation Law

Topic:
Subtopic:
Provision:
Case Name:

Stocks and Stockholders


Rights of Shareholders: Derivative Actions and other Actions of
Stockholders
Section 72, Corporation Code
Cua, Jr. v Ocampo

Facts: Petitioner is a minority stockholder of Philippine Racing Club Inc. (PRCI). PRCI wanted to developed its property
in Makati thus, it opted to acquire another domestic corporation, JTH Davies Holdings, Inc. (JTH). Resolution of the PRCI
Board of Directors on the property-for-shares exchange between PRCI and JTH was supposed to be presented for
approval by the stockholders, however, the petitioner filed before the RTC a Complaint, denominated as a Derivative
Suit with prayer for Issuance of TRO/Preliminary Injunction, against the rest of the directors of PRCI and/or JTH.
Petitioner complained that respondent Solomon, as PRCI President, with the acquiescence of the majority directors of
PRCI, maliciously refused and resisted the request of respondents Miguel, et al., for complete and adequate information
relative to the disputed Board Resolutions, brazenly and unlawfully violating the rights of the minority stockholders to
information and to inspect corporate books and records.
Issue: Whether or not the petitioner has cause of action.
Held: The petition is without merit.
Respondents Miguel, et al., allege another cause of action, other than the derivative suit -- the violation of their right to
information relative to the disputed Resolutions, i.e., the Resolutions dated 16 September 2006 and 11 May 2007 of the
PRCI Board of Directors.
Rule 7 of the IRPICC shall apply to disputes exclusively involving the rights of stockholders or members to inspect the
books and records and/or to be furnished with the financial statements of a corporation, under Sections 74 2and 753 of
the Corporation Code.
Rule 7, Section 2 of IRPICC enumerates the requirements particular to a complaint for inspection of corporate books
and records:
Sec. 2. Complaint. - In addition to the requirements in section 4, Rule 2 of these Rules, the
complaint must state the following:
(1) The case is for the enforcement of plaintiff's right of inspection of corporate orders or
records and/or to be furnished with financial statements under Sections 74 and 75 of the Corporation
Code of the Philippines;
(2) A demand for inspection and copying of books and records and/or to be
furnished with financial statements made by the plaintiff upon defendant;
(3) The refusal of defendant to grant the demands of the plaintiff and the reasons given for
such refusals, if any; and
(4) The reasons why the refusal of defendant to grant the demands of the plaintiff is unjustified
and illegal, stating the law and jurisprudence in support thereof. (Emphasis ours.)
As has already been previously established herein, the right to information, which includes the right to inspect
corporate books and records, is a right personal to each stockholder. After a closer reading of the Complaint in Civil
Case No. 07-610, the Court observes that only respondent Dulay actually made a demand for a copy of all the records,
documents, contracts, and agreements, emails, letters, correspondences, relative to the acquisition of JTH x x x. There
is no allegation that his co-respondents (who are his co-plaintiffs in Civil Case No. 07-610) made similar demands for
the inspection or copying of corporate books and records. Only respondent Dulay complied then with the requirement
under Rule 7, Section 2(2) of IRPICC.

2
3
137
3B-Corporation Law

Even so, respondent Dulays Complaint should be dismissed for lack of cause of action, for his demand for copies of
pertinent documents relative to the acquisition of JTH shares was not denied by any of the defendants named in the
Complaint in Civil Case No. 07-610, but by Atty. Jesulito A. Manalo (Manalo), the Corporate Secretary of PRCI, in a letter
dated 17 January 2006. Section 74 of the Corporation Code, the substantive law on which respondent Dulays Complaint
for inspection and copying of corporate books and records is based, states that:
Sec. 74. Books to be kept; stock transfer agent.
xxxx
Any officer or agent of the corporation who shall refuse to allow any director, trustees,
stockholder or member of the corporation to examine and copy excerpts from its records or minutes, in
accordance with the provisions of this Code, shall be liable to such director, trustee, stockholder or
member for damages, and in addition, shall be guilty of an offense which shall be punishable under
Section 144 of this Code: Provided, That if such refusal is pursuant to a resolution or order of the Board
of Directors or Trustees, the liability under this section for such action shall be imposed upon the
directors or trustees who voted for such refusal: x x x (Emphasis ours.)

Based on the foregoing, it is Corporate Secretary Manalo who should be held liable for the supposedly wrongful and
unreasonable denial of respondent Dulays demand for inspection and copying of corporate books and records; but, as
previously mentioned, Corporate Secretary Manalo is not among the defendants named in the Complaint in Civil Case
No. 07-610. There is also utter lack of any allegation in the Complaint that Corporate Secretary Manalo denied
respondent Dulays demand pursuant to a resolution or order of the PRCI Directors, so that the latter (who are actually
named defendants in the Complaint) could also be held liable for the denial.

138
3B-Corporation Law

Topic:
Subtopic:
Provision:
Case Name:

Stocks and Stockholders


Rights of Shareholders: Derivative Actions and other Actions of
Stockholders
Section 72, Corporation Code
Hi-Yield Realty, Inc. v CA

Facts:
Respondent, a stockholder of HTSI, filed a case in RTC Makati against petitioner Hi-Yield Realty for the annulment of
real estate mortgage and foreclosure sale of lands that belong to HTSI. Petitioner moved for the dismissal of the case
on ground of proper venue. Petitioner argues that the annulment of real estate mortgage and foreclosure sale is a real
action and thus it should be filed in Marikina and Quezon City where the lands are located. RTC and CA dismissed the
motion and appeal of the petitioner on ground that the action is primarily a derivative suit to redress the alleged
unauthorized acts of HTSI officers and major stockholders in connection with the lands.
Issue: Whether or not the action is a real action or a derivative suit.
Held: It is a derivative suit.
Ratio:
A derivative action is a suit by a shareholder to enforce a corporate cause of action. Under the Corporation Code,
where a corporation is an injured party, its power to sue is lodged with its board of directors or trustees. But an
individual stockholder may be permitted to institute a derivative suit on behalf of the corporation in order to protect or
vindicate corporate rights whenever the officials of the corporation refuse to sue, or are the ones to be sued, or hold
control of the corporation. In such actions, the corporation is the real party-in-interest while the suing stockholder, on
behalf of the coporation, is only a nominal party.
Derivative suits are governed by a special set of rules under A.M. No. 01-2-04 SC otherwise known as the Interim Rules
of Procedure Governing Intra-Corporate Controversies under RA No. 8799. Thus, the CA did not commit grave abuse of
discretion when it found that respondents correctly filed the derivative suite before the Makati RTC where HTSI had its
principal office.

139
3B-Corporation Law

Topic:
Subtopic:
Provision:
Case Name:

Stocks and Stockholders


Rights of Shareholders: Derivative Actions and other Actions of
Stockholders
Section 72, Corporation Code
Strategic Alliance Dev. Corp v Radstock Securities, LTD

Facts
Construction Development Corporation of the Philippines (CDCP) was incorporated in 1966. It was granted a franchise
to construct, operate and maintain toll facilities in the North and South Luzon Tollways and Metro Manila Expressway.
CDCP Mining Corporation (CDCP Mining), an affiliate of CDCP, obtained loans from Marubeni Corporation of
Japan(Marubeni). A CDCP official issued letters of guarantee for the loans although there was no CDCP Board
Resolution authorizing the issuance of such letters of guarantee. CDCP Mining secured the Marubeni loans when CDCP
and CDCP Mining were still privately owned and managed. In 1983, CDCPs name was changed to Philippine National
Construction Corporation (PNCC) in order to reflect that the Government already owned 90.3% of PNCC and only 9.70%
is under private ownership. Meanwhile, the Marubeni loans to CDCP Mining remained unpaid.
On 20 October 2000 and 22 November 2000, the PNCC Board of Directors (PNCC Board) passed Board Resolutions
admitting PNCCs liability to Marubeni. Previously, for two decades the PNCC Board consistently refused to admit any
liability for the Marubeni loans. In January 2001, Marubeni assigned its entire credit to Radstock Securities Limited
(Radstock), a foreign corporation. Radstock immediately sent a notice and demand letter to PNCC.PNCC and Radstock
entered into a Compromise Agreement. Under this agreement, PNCC shall pay Radstock the reduced amount of
P6,185,000,000.00 in full settlement of PNCCs guarantee of CDCP Minings debt allegedly totaling P17,040,843,968.00
(judgment debt as of 31 July 2006). To satisfy its reduced obligation, PNCC undertakes to (1) "assign to a third party
assignee to be designated by Radstock all its rights and interests" to the listed real properties of PNCC; (2) issue to
Radstock or its assignee common shares of the capital stock of PNCC issued at par value which shall comprise 20% of
the outstanding capital stock of PNCC; and (3) assign to Radstock or its assignee 50% of PNCCs 6% share, for the
next 27 years, in the gross toll revenues of the Manila North Tollways Corporation. Strategic Alliance Development
Corporation (STRADEC)moved for reconsideration. STRADEC alleged that it has a claim against PNCC as a bidder of the
National Governments shares, receivables, securities and interests in PNCC.

Sison, a stockholder and former PNCC President and Board Chairman, filed a Petition for Annulment of Judgment
Approving Compromise Agreement before the Court of Appeals. The case was docketed as CA-G.R. SP No. 97982.
Issue : Whether or not Sison has standing to file the petition

Held

Sison has legal standing to challenge the Compromise Agreement. Although there was no allegation that Sison filed
the case as a derivative suit in the name of PNCC, it could be fairly deduced that Sison was assailing the Compromise
Agreement as a stockholder of PNCC. In such a situation, a stockholder of PNCC can sue on behalf of PNCC to annul the
Compromise Agreement.
A derivative action is a suit by a stockholder to enforce a corporate cause of action. Under the Corporation Code,
where a corporation is an injured party, its power to sue is lodged with its board of directors or trustees. However, an
individual stockholder may file a derivative suit on behalf of the corporation to protect or vindicate corporate rights
whenever the officials of the corporation refuse to sue, or are the ones to be sued, or hold control of the corporation. 4
[27]
In such actions, the corporation is the real party-in-interest while the suing stockholder, on behalf of the corporation,
is only a nominal party.
In this case, the PNCC Board cannot conceivably be expected to attack the validity of the Compromise Agreement
since the PNCC Board itself approved the Compromise Agreement. In fact, the PNCC Board steadfastly defends the
Compromise Agreement for allegedly being advantageous to PNCC.

4
140
3B-Corporation Law

Topic:
Subtopic:
Provision:
Case Name:

Stocks and Stockholders


Rights of Shareholders: Derivative Actions and other Actions of
Stockholders
Section 72, Corporation Code
Majority Stockholders of Ruby Inc v Lim

Facts:
Ruby Industrial Corporation (RUBY) is a domestic corporation engaged in glass manufacturing. Reeling from severe
liquidity problems so they filed with SEC for petition for suspension of payments which the latter granted.
management committee (MANCOM) was created and (2) rehabilitation plans were submitted to the SEC: the
BENHAR/RUBY Rehabilitation Plan of the majority stockholders led by Yu Kim Giang, and the Alternative Plan of the
minority stockholders represented by Miguel Lim (Lim). The BENHARRUBY Plan shall allow Yu family (BENHAR) to
manage the company. This was opposed by 40% of the stockholders, including Lim, a minority shareholder and other
creditors who allegedly will be prejudiced by the plan. Pending Appeal, BENHAR without authority elected new
members of the Board and extended its corporate life. Lim together with the minority shareholders instituted a
derivative action where the real party in interest is the corporation itself.

Issue: WON Lim and the other minority stockholders have a right to sue as stockholders in behalf of the Corporation.
Held:

A derivative action is a suit by a shareholder to enforce a corporate cause of action. It is a remedy designed by equity
and has been the principal defense of the minority shareholders against abuses by the majority. For this purpose, it is
enough that a member or a minority of stockholders file a derivative suit for and in behalf of a corporation. An
individual stockholder is permitted to institute a derivative suit on behalf of the corporation wherein he holds stock in
order to protect or vindicate corporate rights, whenever officials of the corporation refuse to sue or are the ones to be
sued or hold the control of the corporation. In such actions, the suing stockholder is regarded as the nominal party,
with the corporation as the party in interest. There is no forum shopping in this case as each have distinct rights to
protect. Hence, yes, Lim et. Al. have the right to sue in behalf of the Corporation.

141
3B-Corporation Law

Topic:
Subtopic:
Provision:
Case Name:

Stocks and Stockholders


Rights of Shareholders: Derivative Actions and other Actions of
Stockholders
Section 72, Corporation Code
Lisam Enterprises v Banco de Oro Unibank

Facts:
Lisam Enterprises Inc. (LEI) acquired residential land in Legazpi City. Spouses Soriano in their personal capacity
obtained a loan with Banco de Oro and as president and treasurer, respectively LEI, but without authority and consent
of the board of said plaintiff and with the use of a falsified board resolution, executed a real estate mortgage on the
said property. Said spouses also by falsification managed to obtain a fake Deed of Assumption of Loans making LEI
liable for their personal loan. Plaintiffs commenced a derivative suit against defendants Lilian S. Soriano and the Estate
of Leandro A. Soriano, Jr., before the Securities and Exchange Commission and in regular court for the nullification of
the mortgage. PCIB raised that the parties have no locus standi. The court dismissed the case on this basis.
Issue : WON Petitioner Lolita Soriano (Corp. Secretary) has legal right to represent LEI.
Held:
In Hi-Yield Realty, Incorporated v. Court of Appeals, the Court enumerated the requisites for filing a derivative suit, as
follows:
a) the party bringing the suit should be a shareholder as of the time of the act or transaction
complained of, the number of his shares not being material;
b) he has tried to exhaust intra-corporate remedies, i.e., has made a demand on the board of directors
for the appropriate relief but the latter has failed or refused to heed his plea; and
c) the cause of action actually devolves on the corporation, the wrongdoing or harm having been, or
being caused to the corporation and not to the particular stockholder bringing the suit.
A reading of the amended complaint will reveal that all the foregoing requisites had been alleged
therein. Hence, the amended complaint remedied the defect in the original complaint and now sufficiently
states a cause of action. Hence, Lolita Soriano has legal standing to represent LEI in this Derivative Suit.

142
3B-Corporation Law

Topic:
Subtopic:
Provision:
Case Name:

Stocks and Stockholders


Rights of Shareholders: Derivative Actions and other Actions of
Stockholders
Section 72, Corporation Code
Legaspi Towers 300 Inc., v Amelia Muer

Facts:
Pursuant to the by-laws of Legaspi Towers 300, Inc., incumbent Board Members set the meeting and the election of the
new Board of the Condominium Corp. They found out that most of the proxy votes, at its face value, irregular, thus,
questionable; and for lack of time to authenticate the same, petitioners adjourned the meeting for lack of quorum.
Group of respondents challenged the adjournment of the meeting and proceeded with the election. Petitioner filed
complaint for nullification of the election results. The petitioners' motion to amend complaint to implead Legaspi
Towers 300, Inc. as plaintiff was denied on the ground that the vote is a personal right of a stockholder of a
corporation, such right can only be enforced through a direct action; hence, Legaspi Towers 300, Inc. cannot be
impleaded.
Issue: WON the corporation can be impleaded in this case.
Held:
Where a stockholder or member is denied the right of inspection, his suit would be individual because the wrong is
done to him personally and not to the other stockholders or the corporation. Where the wrong is done to a
group of stockholders, as where preferred stockholders' rights are violated, a class or representative suit will be
proper for the protection of all stockholders belonging to the same group. But where the acts complained
of constitute a wrong to the corporation itself, the cause of action belongs to the corporation and not to
the individual stockholder or member.
Because of the frequent occurrence of such a situation, the common law gradually recognized the right
of a stockholder to sue on behalf of a corporation in what eventually became known as a "derivative
suit." It has been proven to be an effective remedy of the minority against the abuses of management. Thus, an
individual stockholder is permitted to institute a derivative suit on behalf of the corporation wherein he
holds stock in order to protect or vindicate corporate rights, whenever officials of the corporation refuse
to sue or are the ones to be sued or hold the control of the corporation. In such actions, the suing
stockholder is regarded as thenominal party, with the corporation as the party-in- interest. Petitioners
complaint seek to nullify the said election, and to protect and enforce their individual right to vote, hence the
corporation should not be impleadead.

143
3B-Corporation Law

Topic:
Subtopic:
Provision:
Case Name:
FACTS:

Stocks and Stockholders


Rights of Shareholders: Derivative Actions and other Actions of
Stockholders
Section 72, Corporation Code
Ching v Subic Bay Golf and Country Club

On February 26, 2003, petitioners Nestor Ching and Andrew Wellington filed a Complaint with the RTC of Olongapo City
on behalf of the members of Subic Bay Golf and Country Club, Inc. (SBGCCI) against the said country club and its
Board of Directors and officers under the provisions of Presidential Decree No. 902-A in relation to Section 5.2 of the
Securities Regulation Code. The Subic Bay Golfers and Shareholders Incorporated (SBGSI), a corporation composed of
shareholders of the defendant corporation, was also named as plaintiff. The officers impleaded as defendants were the
following: (1) its President, Hu Ho Hsiu Lien alias Susan Hu; (2) its treasurer, Hu Tsung Chieh alias Jack Hu; (3)
corporate secretary Reynald Suarez; and (4) directors Hu Tsung Hui and Hu Tsung Tzu. The complaint alleged that the
defendant corporation sold shares to plaintiffs at US$22,000.00 per share, presenting to them the Articles of
Incorporation which contained the following provision:
No profit shall inure to the exclusive benefit of any of its shareholders, hence, no dividends shall be declared in
their favor. Shareholders shall be entitled only to a pro-rata share of the assets of the Club at the time of its dissolution
or liquidation
However, on June 27, 1996, an amendment to the Articles of Incorporation was approved by the Securities and
Exchange Commission (SEC), wherein the above provision was changed as follows:
No profit shall inure to the exclusive benefit of any of its shareholders, hence, no dividends shall be declared in
their favor. In accordance with the Lease and Development Agreement by and between Subic Bay Metropolitan
Authority and The Universal International Group of Taiwan, where the golf course and clubhouse component thereof
was assigned to the Club, the shareholders shall not have proprietary rights or interests over the properties of the
Club. x x x. (Emphasis supplied.)
Petitioners claimed in the Complaint that defendant corporation did not disclose to them the above
amendment which allegedly makes the shares non-proprietary, as it takes away the right of the shareholders to
participate in the pro-rata distribution of the assets of the corporation after its dissolution. According to petitioners,
this is in fraud of the stockholders who only discovered the amendment when they filed a case for injunction to
restrain the corporation from suspending their rights to use all the facilities of the club. Furthermore, petitioners
alleged that the Board of Directors and officers of the corporation did not call any stockholders meeting from the time
of the incorporation, in violation of Section 50 of the Corporation Code and the By-Laws of the corporation. Neither did
the defendant directors and officers furnish the stockholders with the financial statements of the corporation nor the
financial report of the operation of the corporation in violation of Section 75 of the Corporation Code. Petitioners also
claim that on August 15, 1997, SBGCCI presented to the SEC an amendment to the By-Laws of the corporation
suspending the voting rights of the shareholders except for the five founders shares. Said amendment was allegedly
passed without any stockholders meeting or notices to the stockholders in violation of Section 48 of the Corporation
Code.
ISSUE: Whether or not the complaint is a derivative suit?
HELD:
On the bases of these allegations of the petition, the Court finds that the case is a derivative suit. Being a
derivative suit in accordance with Rule 8 of the Interim Rules, the stockholders and members may bring an action in
the name of the corporation or association provided that he (the minority stockholder) exerted all reasonable efforts
and allege[d] the same with particularity in the complaint to exhaust of (sic) all remedies available under the articles
of incorporation, by-laws or rules governing the corporation or partnership to obtain the reliefs he desires. An
examination of the petition does not show any allegation that the petitioners applied for redress to the Board of
Directors of respondent corporation there being no demand, oral or written on the respondents to address their
complaints. Neither did the petitioners appl[y] for redress to the stockholders of the respondent corporation and
ma[k]e an effort to obtain action by the stockholders as a whole. Petitioners should have asked the Board of Directors
of the respondent corporation and/or its stockholders to hold a meeting for the taking up of the petitioners rights in
this petition

144
3B-Corporation Law

At the outset, it should be noted that the Complaint in question appears to have been filed only by the two
petitioners, namely Nestor Ching and Andrew Wellington, who each own one stock in the respondent corporation
SBGCCI. While the caption of the Complaint also names the "Subic Bay Golfers and Shareholders Inc. for and in behalf
of all its members," petitioners did not attach any authorization from said alleged corporation or its members to file
the Complaint. Thus, the Complaint is deemed filed only by petitioners and not by SBGSI.
On the issue of whether the Complaint is indeed a derivative suit, we are mindful of the doctrine that the
nature of an action, as well as which court or body has jurisdiction over it, is determined based on the allegations
contained in the complaint of the plaintiff, irrespective of whether or not the plaintiff is entitled to recover upon all or
some of the claims asserted therein.
We have also held that the body rather than the title of the complaint determines the nature of an action.
In Cua, Jr. v. Tan the Court previously elaborated on the distinctions among a derivative suit, an individual suit, and a
representative or class suit:
A derivative suit must be differentiated from individual and representative or class suits, thus:
"Suits by stockholders or members of a corporation based on wrongful or fraudulent acts of directors or other persons
may be classified into individual suits, class suits, and derivative suits. Where a stockholder or member is denied the
right of inspection, his suit would be individual because the wrong is done to him personally and not to the other
stockholders or the corporation. Where the wrong is done to a group of stockholders, as where preferred stockholders
rights are violated, a class or representative suit will be proper for the protection of all stockholders belonging to the
same group. But where the acts complained of constitute a wrong to the corporation itself, the cause of action belongs
to the corporation and not to the individual stockholder or member. Although in most every case of wrong to the
corporation, each stockholder is necessarily affected because the value of his interest therein would be impaired, this
fact of itself is not sufficient to give him an individual cause of action since the corporation is a person distinct and
separate from him, and can and should itself sue the wrongdoer. Otherwise, not only would the theory of separate
entity be violated, but there would be multiplicity of suits as well as a violation of the priority rights of creditors.
Furthermore, there is the difficulty of determining the amount of damages that should be paid to each individual
stockholder.
However, in cases of mismanagement where the wrongful acts are committed by the directors or trustees
themselves, a stockholder or member may find that he has no redress because the former are vested by law with the
right to decide whether or not the corporation should sue, and they will never be willing to sue themselves. The
corporation would thus be helpless to seek remedy. Because of the frequent occurrence of such a situation, the
common law gradually recognized the right of a stockholder to sue on behalf of a corporation in what eventually
became known as a "derivative suit." It has been proven to be an effective remedy of the minority against the abuses
of management. Thus, an individual stockholder is permitted to institute a derivative suit on behalf of the corporation
wherein he holds stock in order to protect or vindicate corporate rights, whenever officials of the corporation refuse to
sue or are the ones to be sued or hold the control of the corporation. In such actions, the suing stockholder is regarded
as the nominal party, with the corporation as the party in interest."
The reliefs sought in the Complaint, namely that of enjoining defendants from acting as officers and Board of
Directors of the corporation, the appointment of a receiver, and the prayer for damages in the amount of the decrease
in the value of the shares of stock, clearly show that the Complaint was filed to curb the alleged mismanagement of
SBGCCI. The causes of action pleaded by petitioners do not accrue to a single shareholder or a class of shareholders
but to the corporation itself.
However, as minority stockholders, petitioners do not have any statutory right to override the business
judgments of SBGCCIs officers and Board of Directors on the ground of the latters alleged lack of qualification to
manage a golf course. Contrary to the arguments of petitioners, Presidential Decree No. 902-A, which is entitled
REORGANIZATION OF THE SECURITIES AND EXCHANGE COMMISSION WITH ADDITIONAL POWERS AND PLACING THE
SAID AGENCY UNDER THE ADMINISTRATIVE SUPERVISION OF THE OFFICE OF THE PRESIDENT, does not grant minority
stockholders a cause of action against waste and diversion by the Board of Directors, but merely identifies the
jurisdiction of the SEC over actions already authorized by law or jurisprudence. It is settled that a stockholders right to
institute a derivative suit is not based on any express provision of the Corporation Code, or even the Securities

145
3B-Corporation Law

Regulation Code, but is impliedly recognized when the said laws make corporate directors or officers liable for
damages suffered by the corporation and its stockholders for violation of their fiduciary duties.
At this point, we should take note that while there were allegations in the Complaint of fraud in their
subscription agreements, such as the misrepresentation of the Articles of Incorporation, petitioners do not pray for the
rescission of their subscription or seek to avail of their appraisal rights. Instead, they ask that defendants be enjoined
from managing the corporation and to pay damages for their mismanagement. Petitioners only possible cause of
action as minority stockholders against the actions of the Board of Directors is the common law right to file a
derivative suit. The legal standing of minority stockholders to bring derivative suits is not a statutory right, there being
no provision in the Corporation Code or related statutes authorizing the same, but is instead a product of jurisprudence
based on equity. However, a derivative suit cannot prosper without first complying with the legal requisites for its
institution.
Section 1, Rule 8 of the Interim Rules of Procedure Governing Intra Corporate Controversies imposes the following
requirements for derivative suits:
(1) He was a stockholder or member at the time the acts or transactions subject of the action occurred and at
the time the action was filed;
(2) He exerted all reasonable efforts, and alleges the same with particularity in the complaint, to exhaust all
remedies available under the articles of incorporation, by-laws, laws or rules governing the corporation or
partnership to obtain the relief he desires;
(3) No appraisal rights are available for the act or acts complained of; and
(4) The suit is not a nuisance or harassment suit.
The RTC dismissed the Complaint for failure to comply with the second and fourth requisites above.
Upon a careful examination of the Complaint, this Court finds that the same should not have been dismissed on the
ground that it is a nuisance or harassment suit. Although the shareholdings of petitioners are indeed only two out of
the 409 alleged outstanding shares or 0.24%, the Court has held that it is enough that a member or a minority of
stockholders file a derivative suit for and in behalf of a corporation.
With regard, however, to the second requisite, we find that petitioners failed to state with particularity in the
Complaint that they had exerted all reasonable efforts to exhaust all remedies available under the articles of
incorporation, by-laws, and laws or rules governing the corporation to obtain the relief they desire. The Complaint
contained no allegation whatsoever of any effort to avail of intra-corporate remedies. Indeed, even if petitioners
thought it was futile to exhaust intra-corporate remedies, they should have stated the same in the Complaint and
specified the reasons for such opinion. Failure to do so allows the RTC to dismiss the Complaint, even motu proprio, in
accordance with the Interim Rules. The requirement of this allegation in the Complaint is not a useless formality which
may be disregarded at will.

146
3B-Corporation Law

Topic:
Subtopic:
Provision:
Case Name:

Stocks and Stockholders


Rights of Shareholders: Rights to Inspect Books and Records
Section 74, Corporation Code; Interim Rules for Intra-Corporate
Controversies
Cua Jr. v Ocampo Tan

FACTS:
Philippine Racing Club, Inc. (PRCI) is a corporation organized and established under Philippine Laws to carry on
a race horse business. PRCI holds a franchise to operate a horse track and manage betting stations.
In 1999, the Articles of Incorportaion of PRCI was amended to include a secondary purpose: To acquire real
properties and/or develop real properties. PRCI owns 2 properties: Makati and Cavite property. PRCI wanted to convert
the Makati Racetrack into an urban residential and commercial use. PRCI, then developed, the Cavite property into a
racetrack.
PRCI acquired a domestic corporation, JTH Davies Holdings, Inc. (JTH). PRCI wanted to manage and develop the
Makati property under this subsidiary company, JTH.
JTH was then owned by Jardine Matheson Europe B.V. (JME). To determine the value of JTH, PRCI engaged the services
of the accounting firm Sycip Gorres Velayo & Co. (SGV) to conduct a due diligence study.
Later on, in a meeting with SGV, it was determined that the Makati property can be transferred to JTH in
exchange for the unissued portion of the latters increased capital stock. The difference of P3,419,333,105.50 between
the total zonal value of the Makati property and the aggregate par value of the JTH shares to be issued in exchange for
the same, would be reflected as additional paid-in capital of PRCI in JTH. The matter of the proposed exchange was
taken up and approved by the PRCI Board of Directors in its meeting held on 11 May 2007, again with the lone dissent
of respondent Dulay.
The 11 May 2007 Resolution of the PRCI Board of Directors on the property-for-shares exchange between PRCI and JTH
was supposed to be presented for approval by the stockholders under the afore-quoted Items No. VII and No. VIII of the
Agenda.
However, on 10 July 2007, respondents Miguel, et al., as minority stockholders of PRCI, filed before the RTC a
Complaint, denominated as a Derivative Suit with prayer for Issuance of TRO/Preliminary Injunction, against the rest of
the directors of PRCI and/or JTH.
The Complaint was based on three causes of action: (1) the approval by the majority directors of PRCI of the
Board Resolutions dated 26 September 2006 and 11 May 2007 -- with undue haste and deliberate speed, despite the
absence of any disclosure and information -- was not only anomalous and fraudulent, but also extremely prejudicial
and inimical to interest of PRCI, committed in violation of their fiduciary duty as directors of the said corporation; (2)
respondent Solomon, as PRCI President, with the acquiescence of the majority directors of PRCI, maliciously refused
and resisted the request of respondents Miguel, et al., for complete and adequate information relative to the disputed
Board Resolutions, brazenly and unlawfully violating the rights of the minority stockholders to information and to
inspect corporate books and records; and (3) without being officially and formally nominated, the majority directors of
PRCI illegally and unlawfully constituted themselves as members of the Board of Directors and/or Executive Officers of
JTH, rendering all the actions they have taken as such null and void ab initio.

ISSUE: Whether or not there was a derivative suit

HELD:

147
3B-Corporation Law

The afore-quoted exposition is relevant considering the claim of respondents Miguel, et al., that its Complaint
in Civil Case No. 07-610 is not just a derivative suit, but also an intracorporate action arising from devices or schemes
employed by the PRCI Board of Directors amounting to fraud or misrepresentation. A thorough study of the said
Complaint, however, reveals that the distinction is deceptive. The supposed devices and schemes employed by the
PRCI Board of Directors amounting to fraud or misrepresentation are the very same bases for the derivative suit. They
are the very same acts of the PRCI Board of Directors that have supposedly caused injury to the corporation. From the
very beginning of their Complaint, respondents have alleged that they are filing the same as shareholders, for and in
behalf of the Corporation, in order to redress the wrongs committed against the Corporation and to protect or vindicate
corporate rights, and to prevent wastage and dissipation of corporate funds and assets and the further commission of
illegal acts by the Board of Directors. Although respondents Miguel, et al., also aver that they are seeking redress for
the injuries of the minority stockholders against the wrongdoings of the majority, the rest of the Complaint does not
bear this out, and is utterly lacking any allegation of injury personal to them or a certain class of stockholders to which
they belong.
It is important for the Court to mention that the 26 September 2006 Resolution of the PRCI Board of Directors not
only authorized the acquisition by PRCI of up to 100% of the common stock of JTH, but it also specifically appointed
petitioner Santiago Sr. to act as attorney-in-fact and proxy who could vote all the shares of PRCI in JTH, as well as
nominate, appoint, and vote into office directors and/or officers during regular and special stockholders meetings of
JTH. It was by this authority that PRCI directors were able to constitute the JTH Board of Directors. Thus, the protest of
respondents Miguel, et al., against the interlocking directors of PRCI and JTH is also rooted in the 26 September 2006
Resolution of the PRCI Board of Directors.
After a careful study of the allegations concerning this derivative suit, the Court rules that it is dismissible for
being moot and academic.
The derivative suit, with respect to the Resolution dated 11 May 2007 of the PRCI Board of Directors, is similarly
dismissible for lack of cause ofaction.
The basis of a stockholders suit is always one of equity. However, it cannot prosper without first complying
with the legal requisites for its institution.
Rule 8, Section 1 of the Interim Rules of Procedure for Intra-Corporate Controversies (IRPICC) lays down the
following requirements which a stockholder must comply with in filing a derivative suit:
Sec. 1. Derivative action. A stockholder or member may bring an action in the name of a
corporation or association, as the case may be, provided, that:
(1) He was a stockholder or member at the time the acts or transactions subject of the action
occurred and at the time the action was filed;
(2) He exerted all reasonable efforts, and alleges the same with particularity in the complaint, to
exhaust all remedies available under the articles of incorporation, by-laws, laws or rules governing the
corporation or partnership to obtain the relief he desires;
(3) No appraisal rights are available for the act or acts complained of; and
(4) The suit is not a nuisance or harassment suit. (Emphasis ours.)

148
3B-Corporation Law

Topic:
Subtopic:
Provision:
Case Name:

Stocks and Stockholders


Rights of Shareholders: Rights to Inspect Books and Records
Section 74, Corporation Code; Interim Rules for Intra-Corporate
Controversies
Ma. Belen Flordeliza C. Ang-Abaya et al v Eduardo Ang

FACTS:
Vibelle Manufacturing Corporation (VMC) and Genato Investments, Inc. (Genato) are family-owned corporations, where
petitioners Ma. Belen Flordeliza C. Ang-Abaya (Flordeliza), Francis Jason A. Ang (Jason), Vincent G. Genato (Vincent),
Hanna Zorayda A. Ang (Hanna) and private respondent Eduardo G. Ang (Eduardo) are shareholders, officers and
members of the board of directors.
VMC, Genato, and Oriana Manufacturing Corporation filed a case for damages with prayer for issuance of a TRO and/or
writ of preliminary injunction against herein respondent, together with Michael Edward Chi Ang, for allegedly conniving
to fraudulently wrest control/management of the corporations.
During the pendency of the case, Eduardo sought permission to inspect the corporate books of VMC and Genato on
account of petitioners alleged failure and/or refusal to update him on the financial and business activities of these
family corporations. Petitioners denied the request claiming that Eduardo would use the information obtained from said
inspection for purposes inimical to the corporations interests, considering that: "a) he is harassing and/or bullying the
Corporation[s] into writing off P165,071,586.55 worth of personal advances which he had unlawfully obtained in the
past; b) he is unjustly demanding that he be given the office currently occupied by Mr. Francis Jason Ang, the VicePresident for Finance and Corporate Secretary; c) he is usurping the rights belonging exclusively to the Corporation;
and d) he is coercing and/or trying to inveigle the Directors and/or Officers of the Corporation to give in to his baseless
demands involving specific corporate assets."
Eduardo filed an Affidavit-Complaint against petitioners Flordeliza and Jason, charging them with violation of Section
74, in relation to Section 144, of the Corporation Code of the Philippines. Ma. Belinda G. Sandejas, Vincent, and Hanna
were subsequently impleaded for likewise denying respondents request to inspect the corporate books.
TC granted the permanent injunction applied by the corporations. CA annulled permanent injunction. City Prosecutors
Office of Malabon City issued a Resolution recommending that petitioners be charged with two counts of violation of
Section 74 of the Corporation Code. DOJ denied Eduardos MR.
The appellate court ruled that the Secretary of Justice committed grave abuse of discretion amounting to lack or
excess of jurisdiction in reversing the Resolutions of the Malabon City Prosecutor and in finding that Eduardo did not
act in good faith when he demanded for the examination of VMC and Genatos corporate books. It further held that
Eduardo can demand said examination as a stockholder of both corporations; that Eduardo raised legitimate questions
that necessitated inspection of the corporate books and records; and that petitioners refusal to allow inspection
created probable cause to believe that they have committed a violation of Section 74 of the Corporation Code. CA
denied the Motions for Reconsideration filed by petitioners and the Secretary of Justice.
ISSUE: WON the Secretary of Justice committed grave abuse of discretion when it ruled that petitioners acted in good
faith when they denied private respondents demand for inspection for the corporate books.
HELD:
The stockholder's right of inspection of the corporation's books and records is based upon their ownership of
the assets and property of the corporation. It is, therefore, an incident of ownership of the corporate property, whether
this ownership or interest be termed an equitable ownership, a beneficial ownership, or a quasi-ownership. This right is
predicated upon the necessity of self-protection. It is generally held by majority of the courts that where the right is
granted by statute to the stockholder, it is given to him as such and must be exercised by him with respect to his
interest as a stockholder and for some purpose germane thereto or in the interest of the corporation. In other words,
the inspection has to be germane to the petitioner's interest as a stockholder, and has to be proper and lawful in
character and not inimical to the interest of the corporation.

149
3B-Corporation Law

Thus, contrary to Eduardos insistence, the stockholders right to inspect corporate books is not without
limitations. While the right of inspection was enlarged under the Corporation Code as opposed to the old Corporation
Law.
It is now expressly required as a condition for such examination that the one requesting it must not have been
guilty of using improperly any information secured through a prior examination, or that the person asking for such
examination must be acting in good faith and for a legitimate purpose in making his demand.
In a criminal complaint for violation of Section 74 of the Corporation Code, the defense of improper use or
motive is in the nature of a justifying circumstance that would exonerate those who raise and are able to prove the
same. Accordingly, where the corporation denies inspection on the ground of improper motive or purpose, the burden
of proof is taken from the shareholder and placed on the corporation.
Taken together, all these serve to justify petitioners allegation that Eduardo was not acting in good faith and
for a legitimate purpose in making his demand for inspection of the corporate books. Otherwise stated, there is lack of
probable cause to support the allegation that petitioners violated Section 74 of the Corporation Code in refusing
respondents request for examination of the corporation books.

150
3B-Corporation Law

Topic:
Subtopic:
Provision:

Stocks and Stockholders


Rights of Shareholders: Rights to Inspect Books and Records
Section 74, Corporation Code; Interim Rules for Intra-Corporate
Controversies
Case Name:
Dee Ping Wee v. Lee Hiong Wee
RIGHTS TO INSPECT BOOKS AND RECORDS (Sec. 74 & 75, Corp. Code, Rule 7, Interim Rule for Intra-Corporate
Controversies)
Dee Ping Wee, Araceli Wee, Marina Tan vs Lee Hiong Wee, Rosalind Wee
FACTS:

Dee Ping Wee, Lee Hiong Wee, and Marina Tan are siblings. Araceli is the spouse of Dee Ping Wee. Rosalind is
the spouse of Lee Hiong Wee.

Petitioners were the majority stockholders of:


(1)
Marcel Trading Corporation, a domestic corporation that is engaged in the business of manufacture
and sale of seaweeds.
(2)
Marine Resources Development Corporation, a domestic corporation that is primarily engaged in the
business of cultivating and selling marine products; and
(3)
First Marcel Properties, Inc., a real estate business.
Respondents were minority stockholders in the said corporations

Respondents sent a letter to petitioners, requesting to be allowed to see the financial statements for 2002 and
2003. Petitioners aver however that they shall only allow the same, PROVIDED that the respondents first repay
the proceeds of a bank loan which allegedly was procured by Marcel Trading, but was diverted by Lee Hiong to
fund the operations of Rico Philippines Industrial Corp (RPIC), and to give the financial statements of RPIC, as
well as for Lee Hiong to account for the export sales of RPIC which was diverted to lee Hiongs personal HK
account.

Respondents thus filed with the RTC 3 separate complaints for inspection of the corporate books.
o Petitioners filed Answers, alleging that the petitioners failed to state which particular records they
wanted to see, that they wanted to see the books to merely vex the petitioners, and to fish for
evidence to regain management control over Marcel Trading, Marine Resources Devt, and First Marcel
Properties.
o RTC granted the 3 claims, holding that there was no basis to deny them the right to see the books. All
3 suits were appealed, and was raffled to different divisions of the CA.
o CA dismissed two of the petitions for being the wrong remedy [certiorari was filed instead of appeal]. It
became final.
o CA granted the third complaint as regards Marine Resources Devt Corp. It became final.
o During the pendency of the appeals to the CA, RTC issued upon motion a Writ of Execution since the
Interim Rules provide that the decisions are immediately executory, except if restrained by the
appellate court.
o Petitioners sought to quash the writ of execution, by arguing that the final ruling of the CA on the
complaint against Marine Resources Devt, should be considered by the RTC as regards the one against
Marcel Trading since the three complaints were substantially the same.
o RTC denied. Hence this recourse
ISSUE:
Whether or not final decisions of the CA declaring as improper the intended inspection of corporate records of Marine
Resource Devt, and First Marcel Properties, are supervening events which would warrant the suspension of the
execution of the RTC decision as regards the inspection of the books of Marcel Trading
HELD:
NO.
Marine Resources Development and Marcel Trading are different corporations. Despite the fact that the parties to this
case are all stockholders in the said corporations and the respondents invoked the same provisions of law, the cases
filed before the RTC were entirely distinct from and independent of each other. The two corporations involved are
primarily engaged in different businesses and do not share exactly the same set of stockholders. The records of the
case are also silent with respect to the consolidation of the cases before the trial court. Thus, any ruling on the Marine
Resources Development Case would not materially alter the substance of the judgment in Marcel Trading Case, which
would render the execution of the latter case inequitable.
The burden of proof lies with the corporation who refuses to grant to the stockholder the right to inspect corporate
records. A stockholder has the statutory right of inspection per Sec. 74 of the Corp. Code, the only express limitation
being that the right of inspection should be exercised at reasonable hours on business days; 2) the person demanding

151
3B-Corporation Law

to examine and copy excerpts from the corporation's records and minutes has not improperly used any information
secured through any previous examination of the records of such corporation; and 3) the demand is made in good faith
or for a legitimate purpose. The latter two limitations, however, must be set up as a defense by the corporation if it is
to merit judicial cognizance.
While the decision dated March 11, 2005 of the Court of Appeals (Fourth Division) in CA-G.R. SP No. 85880 and the
Decision dated April 28, 2005 of the Court of Appeals (Eighth Division) in CA-G.R. SP No. 85879, which adopted the
ruling of the Fourth Division, had already become final and executory for failure of respondents to appeal therefrom,
the Court may no longer disturb the same in these proceedings. In any event, the applicability of the said decisions of
the Court of Appeals (Fourth and Eighth Divisions) is limited to the letter-demand for the inspection of corporate
records of Marine Resources Development Corporation and First Marine Properties, Inc. made by respondents.

152
3B-Corporation Law

Topic:
Subtopic:
Provision:

Stocks and Stockholders


Rights of Shareholders: Rights to Inspect Books and Records
Section 74, Corporation Code; Interim Rules for Intra-Corporate
Controversies
Case Name:
Sy Tiong Shion v Sy Chim
RIGHTS TO INSPECT BOOKS AND RECORDS (Sec. 74, Corp. Code, Sec. 144, Corp. Code)
Sy Tiong Shion, et al. vs Sy Chim, and Felicidad Sy
FACTS:

Sy Chim and Felicidad Sy are spouses who are directors and stockholders in Sy Siy Ho & Sons, Inc.

In 2003, the GIS was drawn up which showed allegedly showed a marked decrease in the shareholdings of the
spouses Sy from that in the 2002 GIS. The Sy spouses averred that they did not know how this happened as
they did not executed any conveyance of their shares.

They wrote a letter to petitioners Sy to inspect the books and records of the business. Petitioners Sy refused,
saying that a civil case for accounting and damages had already been filed against the respondent Sy spouses,
and that they cant do so until the Sy spouses have responded appropriately to the civil case.
o In the civil case, the petitioners Sy allege that the spouses Sy misappropriated certain amounts
from the sales of the corporation to the latters personal accounts, and that they suddenly moved
residences and were not heard from after the controversy was discovered. This is the rationale why
they filed the civil case, HOWEVER this was not alleged in their Answer to the criminal complaint.

Given the refusal, the Sy spouses filed a criminal complaint with the prosecutor. In the Answer to the criminal
complaint, petitioners Sy merely averred that there is a prejudicial question in the civil case, hence the
criminal complaint must be suspended.

Prosecutor suspended the criminal complaints. DOJ affirmed.

Upon R65 to the CA, CA ruled that the issue in the civil case can proceed separately from the complaint, since
a finding in the civil case that respondents mishandled or misappropriated the funds would not be
determinative of their guilt or innocence in the criminal complaint.
ISSUE:
Whether or not the civil case is a prejudicial question to the criminal complaint for violation of Sec. 74 in relation to
Sec. 144 of the Corporation Code.
HELD:
NO.
In order that probable cause to file a criminal case may be arrived at, or in order to engender the well-founded belief
that a crime has been committed, the elements of the crime charged should be present. This is based on the principle
that every crime is defined by its elements, without which there should beat the mostno criminal offense.
The requisites before the penal provision under Section 144 of the Corporation Code may be applied in a case of
violation of a stockholder or members right to inspect the corporate books/records as provided for under Section 74 of
the Corporation Code are:
1. A director, trustee, stockholder or member has made a prior demand in writing for a copy of excerpts from the
corporations records or minutes;
2. Any officer or agent of the concerned corporation shall refuse to allow the said director, trustee, stockholder or
member of the corporation to examine and copy said excerpts;
3. If such refusal is made pursuant to a resolution or order of the board of directors or trustees, the liability under
this section for such action shall be imposed upon the directors or trustees who voted for such refusal; and
4. Where the officer or agent of the corporation sets up the defense that the person demanding to examine and
copy excerpts from the corporations records and minutes has improperly used any information secured
through any prior examination of the records or minutes of such corporation or of any other corporation, or
was not acting in good faith or for a legitimate purpose in making his demand, the contrary must be shown or
proved
Thus, in a criminal complaint for violation of Section 74 of the Corporation Code, the defense of improper use or motive
is in the nature of a justifying circumstance that would exonerate those who raise and are able to prove the same.
Accordingly, where the corporation denies inspection on the ground of improper motive or purpose, the burden of
proof is taken from the shareholder and placed on the corporation. However, where no such improper motive or
purpose is alleged, and even though so alleged, it is not proved by the corporation, then there is no valid reason to
deny the requested inspection.

153
3B-Corporation Law

Even in their Joint Counter-Affidavit dated 23 September 2003, Sy Tiong Shiou, et al. did not make any allegation that
"the person demanding to examine and copy excerpts from the corporations records and minutes has improperly used
any information secured through any prior examination of the records or minutes of such corporation or of any other
corporation, or was not acting in good faith or for a legitimate purpose in making his demand." Instead, they merely
reiterated the pendency of the civil case. There being no allegation of improper motive, and it being undisputed that
Sy Tiong Shiou, et al. denied spouses Sys request for inspection, the Court rules and so holds that the DOJ erred in
dismissing the criminal charge for violation of Section 74 in relation to Section 144 of the Corporation Code

154
3B-Corporation Law

Topic:
Subtopic:
Provision:

Stocks and Stockholders


Rights of Shareholders: Rights to Inspect Books and Records
Section 74, Corporation Code; Interim Rules for Intra-Corporate
Controversies
Case Name:
Puno v Puno Enterprises
RIGHTS TO INSPECT BOOKS AND RECORDS (Sec. 63 & 74, Corp. Code)
Puno vs Puno Enterprises
FACTS:

Carlos L. Puno, who died on June 25, 1963, was an incorporator of respondent Puno Enterprises, Inc.
On March 14, 2003, petitioner Joselito Musni Puno, claiming to be an heir of Carlos L. Puno, initiated a
complaint for specific performance against respondent. Petitioner averred that he is the son of the deceased
with the latters common-law wife, Amelia Puno. As surviving heir, he claimed entitlement to the rights and
privileges of his late father as stockholder of respondent. The complaint thus prayed that respondent allow
petitioner to inspect its corporate book, render an accounting of all the transactions it entered into from 1962,
and give petitioner all the profits, earnings, dividends, or income pertaining to the shares of Carlos L. Puno.
RTC ordered Puno Enterprises to let their records be inspected by plaintiff Joselito Puno.
CA dismissed.

Issue:
Whether or not Joselito Musni Puno as an heir is automatically entitled for the stocks upon the death of a shareholder.
Held:
NO.
The stockholders right of inspection of the corporations books and records is based upon his ownership of shares in
the corporation and the necessity for self-protection. After all, a shareholder has the right to be intelligently informed
about corporate affairs. Such right rests upon the stockholders underlying ownership of the corporations assets and
property
Upon the death of a shareholder, the heirs do not automatically become stockholders of the corporation and acquire
the rights and privileges of the deceased as shareholder of the corporation. The stocks must be distributed first to the
heirs in estate proceedings, and the transfer of the stocks must be recorded in the books of the corporation. Section 63
of the Corporation Code provides that no transfer shall be valid, except as between the parties, until the transfer is
recorded in the books of the corporation. During such interim period, the heirs stand as the equitable owners of the
stocks, the executor or administrator duly appointed by the court being vested with the legal title to the stock. Until a
settlement and division of the estate is effected, the stocks of the decedent are held by the administrator or executor.
Consequently, during such time, it is the administrator or executor who is entitled to exercise the rights of the
deceased as stockholder.
Thus, even if petitioner presents sufficient evidence in this case to establish that he is the son of Carlos L. Puno, he
would still not be allowed to inspect respondents books and be entitled to receive dividends from respondent, absent
any showing in its transfer book that some of the shares owned by Carlos L. Puno were transferred to him. This would
only be possible if petitioner has been recognized as an heir and has participated in the settlement of the estate of the
deceased.

155
3B-Corporation Law

Topic:
Subtopic:
Provision:
Case Name:
Facts:
-

Stocks and Stockholders


Rights of Shareholders: Rights to Inspect Books and Records
Section 74, Corporation Code; Interim Rules for Intra-Corporate
Controversies
Loreli Lim Po v DOJ

Herein private respondent, Jasper T. Tan (Tan), is a stockholder of Coastal Highpoint Ventures, Inc. (CHVI), a real
estate development company. Antonio Ng Chiu1 (Chiu) is its President. Tan claimed that Loreli Lim Po (Po) is
Chius personal accountant. Po asserted otherwise and instead alleged that she is merely a consultant for CHVI.
Tan states that pertinent information of CHVI was withheld from him such as Financial Statements. Consequently,
he filed a complaint against Chiu and Po for violation of the Corporate Code.
Chiu and Po argues that Tan could have acquired said documents without problem himself and that his
inspectors were not restricted from conducting an audit of the corporations account.
Tan argues otherwise

Issue:
- Whether or not there is a violation of Section 74 of the Corporation Code
Held:
-

YES
In the case at bar, we find no grave abuse of discretion on the part of the CA when it rendered its Decision28
dated January 11, 2011.
There is ample evidence on record to support the said decision. To name one, accountants Creest O. Morales and
Jay Arr T. Hernandez, who were part of the Inspection Team sent by Tan to CHVI, executed a Joint Affidavit29
stating that the documents made available to them for inspection were limited. Further, they claimed that on the
day of the inspection, they brought a portable photocopying machine to CHVIs premises but they were not
allowed to use the same. The offense punishable under Section 74, in relation to Section 144 of the Corporation
Code, for which Chiu was indicted, requires the unjustified disallowance or refusal by a suspect, of a
stockholders written request to examine or copy excerpts of a corporations books or minutes. The absence of
any ascribed ill motives on the part of the aforementioned accountants to make statements adverse or
unfavorable to Chiu lends credibility to their declarations.
Besides, as we ruled in Metrobank,30 in a preliminary investigation, the prosecutor is bound to determine merely
the existence of probable cause that a crime has been committed and that the accused has committed the
same. The rules do not require that a prosecutor has moral certainty of the guilt of a person for the latter to be
indicted for an offense after the conduct of a preliminary investigation. Further, we have repeatedly ruled that
the determination of probable cause, for purposes of preliminary investigation, is an executive function. Such
determination should be free from the courts interference save only in exceptional cases where the DOJ gravely
abuses its discretion in the issuance of its orders or resolutions.

156
3B-Corporation Law

Topic:
Subtopic:
Provision:
Case Name:

Stocks and Stockholders


Rights of Shareholders: Rights to Inspect Books and Records
Section 74, Corporation Code; Interim Rules for Intra-Corporate
Controversies
Yuico v Quaimbao

Facts: Respondents filed with the RTC a complaint against STRADEC. The complaint prays that: (1) the election of
Board of Directors be nullified on the ground of improper venue, pursuant to Section 51 of the Corporation Code; (2) all
ensuing transactions conducted by the elected directors be likewise nullified; and (3) a special stockholders meeting
be

held

anew.

Petitioner asserts that it is the SEC which has jurisdiction over the case. Furthermore, the action has already prescribed
since the case was filed beyond 15 day period from the day of the election. The respondents, in their comment,
counter that the appellate court correctly ruled that the power to hear and decide controversies involving intracorporate disputes, as well as to act on matters incidental and necessary thereto, have been transferred from the SEC
to

the

RTCs

designated

as

Special

Commercial

Courts.

Issues:
(1) Whether only the SEC, not the RTC, has jurisdiction to order the holding of a special stockholders meeting involving
an
(2)

intra-corporate
Whether

the

action

contrive.
has

already

prescribed.

Held:
(1) No. Clearly, the RTC has the power to hear and decide the intra-corporate controversy of the parties herein.
Concomitant to said power is the authority to issue orders necessary or incidental to the carrying out of the powers
expressly granted to it. Thus, the RTC may, in appropriate cases, order the holding of a special meeting of stockholders
or

members

of

corporation.

(2) Yes. Under Section 3, Rule 6 of the Interim Rules of Procedure Governing Intra-Corporate Controversies under R.A.
No. 8799, an election contest must be "filed within 15 days from the date of the election." It was only on August 16,
2004 that respondents instituted an action questioning the validity of the March 1, 2004 stockholders election, clearly
beyond the 15-day prescriptive period.

157
3B-Corporation Law

Topic:
Subtopic:
Provision:
Case Name:

Stocks and Stockholders


Rights of Shareholders: Appraisal Right
Section 74, Corporation Code; Interim Rules for Intra-Corporate
Controversies
Turner v Lorenzo Shipping Corp

Facts:
This case concerns the right of dissenting stockholders to demand payment of the value of their shareholdings.
The disagreement on the valuation of the shares led the parties to constitute an appraisal committee pursuant to
Section 82 of the Corporation Code, each of them nominating a representative, who together then nominated the third
member who would be chairman of the appraisal committee. Thus, the appraisal committee came to be made up of
Reynaldo Yatco, the petitioners nominee; Atty. Antonio Acyatan, the respondents nominee; and Leo Anoche of the
Asian Appraisal Company, Inc., the third member/chairman.
On October 27, 2000, the appraisal committee reported its valuation of P2.54/share, for an aggregate value of
P2,565,400.00 for the petitioners.2
Subsequently, the petitioners demanded payment based on the valuation of the appraisal committee, plus 2%/month
penalty from the date of their original demand for payment, as well as the reimbursement of the amounts advanced as
professional fees to the appraisers.3
In its letter to the petitioners dated January 2, 2001,4 the respondent refused the petitioners demand, explaining that
pursuant to the Corporation Code, the dissenting stockholders exercising their appraisal rights could be paid only when
the corporation had unrestricted retained earnings to cover the fair value of the shares, but that it had no retained
earnings at the time of the petitioners demand, as borne out by its Financial Statements for Fiscal Year 1999 showing
a deficit of P72,973,114.00 as of December 31, 1999.
Upon the respondents refusal to pay, the petitioners sued the respondent for collection and damages in the RTC in
Makati City on January 22, 2001. The case, docketed as Civil Case No. 01-086, was initially assigned to Branch 132.5
It was granted for partial performance. Petitioner sues to assail such decision
Issue:
WoN appraisal right was exercised properly
Held:
The Corporation Code defines how the right of appraisal is exercised, as well as the implications of the right of
appraisal, as follows:
1)

The appraisal right is exercised by any stockholder who has voted against the proposed corporate action by

making a written demand on the corporation within 30 days after the date on which the vote was taken for the
payment of the fair value of his shares. The failure to make the demand within the period is deemed a waiver of the
appraisal right.
2)

If the withdrawing stockholder and the corporation cannot agree on the fair value of the shares within a period of

60 days from the date the stockholders approved the corporate action, the fair value shall be determined and
appraised by three disinterested persons, one of whom shall be named by the stockholder, another by the corporation,
and the third by the two thus chosen. The findings and award of the majority of the appraisers shall be final, and the
corporation shall pay their award within 30 days after the award is made. Upon payment by the corporation of the
agreed or awarded price, the stockholder shall forthwith transfer his or her shares to the corporation.
3)

All rights accruing to the withdrawing stockholders shares, including voting and dividend rights, shall be

suspended from the time of demand for the payment of the fair value of the shares until either the abandonment of

158
3B-Corporation Law

the corporate action involved or the purchase of the shares by the corporation, except the right of such stockholder to
receive payment of the fair value of the shares.
4)

Within 10 days after demanding payment for his or her shares, a dissenting stockholder shall submit to the

corporation the certificates of stock representing his shares for notation thereon that such shares are dissenting
shares. A failure to do so shall, at the option of the corporation, terminate his rights under this Title X of the
Corporation Code. If shares represented by the certificates bearing such notation are transferred, and the certificates
are consequently canceled, the rights of the transferor as a dissenting stockholder under this Title shall cease and the
transferee shall have all the rights of a regular stockholder; and all dividend distributions that would have accrued on
such shares shall be paid to the transferee.
5)

If the proposed corporate action is implemented or effected, the corporation shall pay to such stockholder, upon

the surrender of the certificates of stock representing his shares, the fair value thereof as of the day prior to the date
on which the vote was taken, excluding any appreciation or depreciation in anticipation of such corporate action.
Notwithstanding the foregoing, no payment shall be made to any dissenting stockholder unless the corporation has
unrestricted retained earnings in its books to cover the payment. In case the corporation has no available unrestricted
retained earnings in its books, Section 83 of the Corporation Code provides that if the dissenting stockholder is not
paid the value of his shares within 30 days after the award, his voting and dividend rights shall immediately be
restored.
The trust fund doctrine backstops the requirement of unrestricted retained earnings to fund the payment of the shares
of stocks of the withdrawing stockholders. Under the doctrine, the capital stock, property, and other assets of a
corporation are regarded as equity in trust for the payment of corporate creditors, who are preferred in the distribution
of corporate assets. The creditors of a corporation have the right to assume that the board of directors will not use the
assets of the corporation to purchase its own stock for as long as the corporation has outstanding debts and liabilities.
There can be no distribution of assets among the stockholders without first paying corporate debts. Thus, any
disposition of corporate funds and assets to the prejudice of creditors is null and void.

159
3B-Corporation Law

Topic:
Subtopic:
Provision:
Case Name:

Merger and Consolidation


Section 76 to 80 Corporation Code
Bank of the Philippine Islands v BPI Employees Union

Facts:

The BSP approved the Articles of Merger executed on January 20, 2000 by and between BPI, and FEBTC. This Article
and Plan of Merger was approved by the SEC on April 7, 2000.Pursuant to the Article and Plan of Merger, all the assets
and liabilities of FEBTC were transferred to and absorbed by BPI as the surviving corporation. FEBTC employees,
including those in its different branches across the country, were hired by petitioner as its own employees, with their
status and tenure recognized and salaries and benefits maintained. Respondent BPI Employees Union-Davao ChapterFederation of Unions in BPI Unibank is the exclusive bargaining agent of BPIs rank and file employees in Davao City.
The former FEBTC rank-and-file employees in Davao City did not belong to any labor union at the time of the merger.

Prior to the effectivity of the merger, respondent union invited said FEBTC employees to a meeting regarding the Union
Shop Clause of the existing CBA between petitioner BPI and respondent union. The parties both advert to certain
provisions of the existing CBA. After the meeting called by the union, some of the former FEBTC employees joined the
union, while others refused. Later, however, some of those who initially joined retracted their membership. Respondent
union then sent notices to the former FEBTC employees who refused to join, as well as those who retracted their
membership and called them to a hearing regarding the matter. When these former FEBTC employees refused to
attend the hearing, the president of the Union requested BPI to implement the Union Shop Clause of the CBA and to
terminate their employment.

After two months of management inaction on the request, respondent informed petitioner of its decision to refer the
issue of the implementation of the Union Shop Clause of the CBA to the Grievance Committee. However, the issue
remained unresolved at this level and so it was subsequently submitted for voluntary arbitration by the parties.
Voluntary Arbitrator ruled in favor of petitioner BPI. Respondent Union filed a motion for reconsideration, but the
voluntary arbitrator denied the same. It appealed to the CA and the CA reversed and set aside the decision of the
voluntary arbitrator. Hence, this petition.
Issue: WON a corporation invoke its merger with another corporation as a valid ground to exempt its absorbed
employees from the coverage of a union shop clause contained in its existing CBA with its own certified labor union?
HELD:
the Articles of Merger and Plan of Merger dated April 7, 2000 did not contain any specific stipulation with respect to the
employment contracts of existing personnel of the non-surviving entity which is FEBTC. Unlike the Voluntary Arbitrator,
this Court cannot uphold the reasoning that the general stipulation regarding transfer of FEBTC assets and liabilities to
BPI as set forth in the Articles of Merger necessarily includes the transfer of all FEBTC employees into the employ of BPI
and neither BPI nor the FEBTC employees allegedly could do anything about it.
Even if it is so, it does not follow that the absorbed employees should not be subject to the terms and conditions of
employment obtaining in the surviving corporation.
The rule is that unless expressly assumed, labor contracts such as employment contracts and collective bargaining
agreements are not enforceable against a transferee of an enterprise, labor contracts being in personam, thus binding
only between the parties. A labor contract merely creates an action in personam and does not create any real right
which should be respected by third parties. This conclusion draws its force from the right of an employer to select his
employees and to decide when to engage them as protected under our Constitution, and the same can only be
restricted by law through the exercise of the police power.

160
3B-Corporation Law

Topic:
Subtopic:
Provision:
Case Name:

Merger and Consolidation


Section 76 to 80 Corporation Code
Mindanao Savings and Loan Association v Wilkom

Facts:
First Iligan Savings and Loan Association, Inc. (FISLAI) and Davao Savings and Loan Association, Inc. (DSLAI) are
entities duly registered with the SEC primarily engaged in the business of granting loans and receiving deposits from
the general public, and treated as banks.
In 1985, FISLAI and DSLAI entered into a merger, with DSLAI as the surviving corporation but their articles of merger
were not registered with the SEC due to incomplete documentation. DSLAI changed its corporate name to MSLAI by
way of an amendment to its Articles of Incorporation which was approved by the SEC.
In 1986, the Board of Directors of FISLAI passed and approved Board Resolution assigning its assets in favor of DSLAI
which in turn assumed the formers liabilities. The business of MSLAI, however, failed. Hence, the Monetary Board of
the Central Bank of the Philippines ordered its liquidation with PDIC as its liquidator.
Prior to the closure of MSLAI, Uy filed with the RTC of Iligan City, an action for collection of sum of money against
FISLAI. The RTC issued a summary decision in favor of Uy, directing FISLAI to pay. As a consequence, 6 parcels of land
owned by FISLAI were levied and sold to Willkom.
In 1995, MSLAI, represented by PDIC, filed before the RTC a complaint for the annulment of the Sheriffs Sale alleging
that the sale on execution of the subject properties was conducted without notice to it and PDIC. Respondents, in its
answer, averred that MSLAI had no cause of action because MSLAI is a separate and distinct entity from FISLAI on the
ground that the unofficial merger between FISLAI and DSLAI (now MSLAI) did not take effect considering that the
merging companies did not comply with the formalities and procedure for merger or consolidation as prescribed by the
Corporation Code of the Philippines. RTC dismissed the case for lack of jurisdiction.
CA affirmed but ruled that there was no merger between FISLAI and MSLAI (formerly DSLAI) for their failure to follow
the procedure laid down by the Corporation Code for a valid merger or consolidation.
Issue: Was the merger between FISLAI and DSLAI (now MSLAI) valid and effective?
Held: NO.
In merger, one of the corporations survives while the rest are dissolved and all their rights, properties, and liabilities
are acquired by the surviving corporation. Although there is a dissolution of the absorbed or merged corporations,
there is no winding up of their affairs or liquidation of their assets because the surviving corporation automatically
acquires all their rights, privileges, and powers, as well as their liabilities.
The merger, however, does not become effective upon the mere agreement of the constituent corporations. Since a
merger or consolidation involves fundamental changes in the corporation, as well as in the rights of stockholders and
creditors, there must be an express provision of law authorizing them. The steps necessary to accomplish a merger or
consolidation, as provided for in Sections 76,[24] 77,[25] 78,[26] and 79[27] of the Corporation Code, are:
The board of each corporation draws up a plan of merger or consolidation. Such plan must include any amendment, if
necessary, to the articles of incorporation of the surviving corporation, or in case of consolidation, all the statements
required in the articles of incorporation of a corporation;
(2) Submission of plan to stockholders or members of each corporation for approval. A meeting must be called and at
least two (2) weeks notice must be sent to all stockholders or members, personally or by registered mail. A summary

161
3B-Corporation Law

of the plan must be attached to the notice. Vote of two-thirds of the members or of stockholders representing twothirds of the outstanding capital stock will be needed. Appraisal rights, when proper, must be respected;
(3) Execution of the formal agreement, referred to as the articles of merger or consolidation, by the corporate officers
of each constituent corporation. These take the place of the articles of incorporation of the consolidated corporation, or
amend the articles of incorporation of the surviving corporation;
(4) Submission of said articles of merger or consolidation to the SEC for approval;
(5) If necessary, the SEC shall set a hearing, notifying all corporations concerned at least two weeks before;
(6) Issuance of certificate of merger or consolidation.
Clearly, the merger shall only be effective upon the issuance of a certificate of merger by the SEC, subject to its prior
determination that the merger is not inconsistent with the Corporation Code or existing laws. In this case, it is
undisputed that the articles of merger between FISLAI and DSLAI were not registered with the SEC due to incomplete
documentation. Consequently, the SEC did not issue the required certificate of merger. Even if it is true that the
Monetary Board of the Central Bank of the Philippines recognized such merger, the fact remains that no certificate was
issued by the SEC. Such merger is still incomplete without the certification. The issuance of the certificate of merger is
crucial because not only does it bear out SECs approval but it also marks the moment when the consequences of a
merger take place. By operation of law, upon the effectivity of the merger, the absorbed corporation ceases to exist
but its rights and properties, as well as liabilities, shall be taken and deemed transferred to and vested in the surviving
corporation. There being no merger between FISLAI and DSLAI (now MSLAI), for third parties such as respondents, the
two corporations shall not be considered as one but two separate corporations. Being separate entities, the property of
one cannot be considered the property of the other.
Thus, in the instant case, as far as third parties are concerned, the assets of FISLAI remain as its assets and cannot be
considered as belonging to DSLAI and MSLAI, notwithstanding the Deed of Assignment wherein FISLAI assigned its
assets and properties to DSLAI, and the latter assumed all the liabilities of the former. As provided in Article 1625 of
the Civil Code, an assignment of credit, right or action shall produce no effect as against third persons, unless it
appears in a public instrument, or the instrument is recorded in the Registry of Property in case the assignment
involves real property. The certificates of title of the subject properties were clean and contained no annotation of the
fact of assignment. Respondents cannot, therefore, be faulted for enforcing their claim against FISLAI on the properties
registered under its name. Accordingly, MSLAI, as the successor-in-interest of DSLAI, has no legal standing to annul the
execution sale over the properties of FISLAI. With more reason can it not cause the cancellation of the title to the
subject properties of Willkom and Go.

162
3B-Corporation Law

Topic:
Subtopic:
Provision:
Case Name:

Merger and Consolidation


Section 76 to 80 Corporation Code
Bank of the Philippine Islands v Lee

Facts:
On April 26, 1988, respondent Carlito Lee (Lee) filed a complaint for sum of money with damages and
application for the issuance of a writ of attachment against Trendline and Buelva before the RTC seeking to recover his
total investment in the amount of P5.8 million as he was enticed to invest his money with Trendline upon Buelvas
misrepresentation that she was its duly licensed investment consultant or commodity saleswoman. His investments,
however, were lost without any explanation from the defendants.
RTC issued a writ of preliminary attachment whereby the Check-O-Matic Savings Accounts of Trendline with
Citytrust Banking Corporation, Ayala Branch, in the total amount of P700,962.10 were garnished. RTC found
defendants jointly and severally liable to Lee for the full amount of his investment plus legal interest, attorneys fees
and costs of suit. The defendants appealed the RTC decision to the CA.
On October 4, 1996, Citytrust and BPI merged, with the latter as the surviving corporation. The Articles of
Merger provide, among others, that "all liabilities and obligations of Citytrust shall be transferred to and become the
liabilities and obligations of BPI in the same manner as if the BPI had itself incurred such liabilities or obligations."
After the RTC judgment had become final and executory, Lee filed a Motion for Execution and was granted. He
sought the release of the garnished deposits of Trendline. When the writ was implemented, however, BPI Manager
Samuel Mendoza, Jr. denied having possession, control and custody of any deposits or properties belonging to
defendants, prompting Lee to seek the production of their records of accounts with BPI. However, on the manifestation
of BPI that it cannot locate the defendants' bank records with Citytrust, the RTC denied the motion.
CA annulled the questioned orders, finding grave abuse of discretion on the part of the RTC in denying Lees
motion to enforce the garnishment against Trendlines attached bank deposits with Citytrust, which have been
transferred to BPI by virtue of their merger. It found BPI liable to deliver to the RTC the garnished bank deposit of
Trendline in the amount of P700,962.10, which Citytrust withheld pursuant to the RTC's previously-issued writ of
attachment.
Issues:
1. Whether or not BPI is a party of the case by virtue of the merger.
2. Whether or not the loss of bank records of a garnished deposit is not a ground for the dissolution of
garnishment.
Held:
1. YES. Section 5, Rule 65 of the Revised Rules of Court requires that persons interested in sustaining the
proceedings in court must be impleaded as private respondents. Upon the merger of Citytrust and BPI, with the latter
as the surviving corporation, and with all the liabilities and obligations of Citytrust transferred to BPI as if it had
incurred the same, BPI undoubtedly became a party interested in sustaining the proceedings, as it stands to be
prejudiced by the outcome of the case. Citytrust, therefore, upon service of the notice of garnishment and its
acknowledgment that it was in possession of defendants' deposit accounts in its letter-reply dated June 28, 1988,
became a "virtual party" to or a "forced intervenor" in the civil case. As such, it became bound by the orders and
processes issued by the trial court despite not having been properly impleaded therein. Consequently, by virtue of its
merger with BPI on October 4, 1996, BPI, as the surviving corporation, effectively became the garnishee, thus the
"virtual party" to the civil case.
2. NO. By virtue of the writ of garnishment, the deposits of the defendants with Citytrust were placed in
custodia legis of the court. From that time onwards, their deposits were under the sole control of the RTC and Citytrust
holds them subject to its orders until such time that the attachment or garnishment is discharged, or the judgment in
favor of Lee is satisfied or the credit or deposit is delivered to the proper officer of the court. Thus, Citytrust, and
thereafter BPI, which automatically assumed the formers liabilities and obligations upon the approval of their Articles
of Merger, is obliged to keep the deposit intact and to deliver the same to the proper officer upon order of the court.
BPI cannot avoid the obligation attached to the writ of garnishment by claiming that the fund was not
transferred to it, in light of the Articles of Merger which provides that "all liabilities and obligations of Citytrust shall be
transferred to and become the liabilities and obligations of BPI in the same manner as if the BPI had itself incurred
such liabilities or obligations, and in order that the rights and interest of creditors of Citytrust or liens upon the
property of Citytrust shall not be impaired by merger.

163
3B-Corporation Law

Topic:
Subtopic:
Provision:
Case Name:

Merger and Consolidation


Section 76 to 80 Corporation Code
Bank of Commerce v Radio Philippines Network, Inc

FACTS:
The Traders Royal Bank (TRB) proposed to sell to Bank of Commerce (Bancommerce) its banking business consisting of
specified assets and liabilities. Bancommerce agreed subject to prior Bangko Sentral ng Pilipinas' (BSP's) approval of
their Purchase and Assumption (P & A) Agreement. On November 8, 2001 the BSP approved that agreement subject to
the condition that Bancommerce and TRB would set up an escrow fund with another bank to cover TRB liabilities for
contingent claims.
Bancommerce entered into a P & A Agreement with TRB and acquired its specified assets and liabilities, excluding
liabilities arising from judicial actions which were to be covered by the BSP-mandated escrow of P50 million. To comply
with the BSP mandate, TRB placed P50 million in escrow with Metropolitan Bank and Trust Co. (Metrobank).
Acting in G.R. 138510, Traders Royal Bank v. Radio Philippines Network (RPN), Inc., this Court ordered TRB to pay
respondents RPN, Intercontinental Broadcasting Corporation, and Banahaw Broadcasting Corporation (collectively,
RPN, et al.) actual damages. Based on this decision, RPN, et al. filed a motion for execution against TRB before the
Regional Trial Court (RTC) of Quezon City. But rather than pursue a levy in execution of the corresponding amounts on
escrow with Metrobank, RPN, et al. filed a Supplemental Motion for Execution where they described TRB as "now Bank
of Commerce" based on the assumption that TRB had been merged into Bancommerce. Bancommerce denied that
there was a merger between TRB and Bancommerce.
ISSUE: Whether or not TRB had been merged into Bancommerce as to make the latter liable for TRBs judgment debts
HELD:
No.
Merger is the absorption of one or more corporations by another existing corporation, which retains its identity and
takes over the rights, privileges, franchises, properties, claims, liabilities and obligations of the absorbed
corporation(s). The absorbing corporation continues its existence while the life or lives of the other corporation(s) is or
are terminated. The Corporation Code requires the following steps for merger or consolidation:
(1) The board of each corporation draws up a plan of merger or consolidation. Such plan must include any
amendment, if necessary, to the articles of incorporation of the surviving corporation, or in case of
consolidation, all the statements required in the articles of incorporation of a corporation.
(2) Submission of plan to stockholders or members of each corporation for approval. A meeting must be called
and at least two (2) weeks notice must be sent to all stockholders or members, personally or by registered
mail. A summary of the plan must be attached to the notice. Vote of two-thirds of the members or of
stockholders representing two thirds of the outstanding capital stock will be needed. Appraisal rights,
when proper, must be respected.
(3) Execution of the formal agreement, referred to as the articles of merger o[r] consolidation, by the corporate
officers of each constituent corporation. These take the place of the articles of incorporation of the
consolidated corporation, or amend the articles of incorporation of the surviving corporation.
(4) Submission of said articles of merger or consolidation to the SEC for approval.
(5) If necessary, the SEC shall set a hearing, notifying all corporations concerned at least two weeks before.
(6) Issuance of certificate of merger or consolidation.
Indubitably, it is clear that no merger took place between Bancommerce and TRB as the requirements and procedures
for a merger were absent. A merger does not become effective upon the mere agreement of the constituent
corporations. Section 79 of the Corporation Code further provides that the merger shall be effective only upon the
issuance by the Securities and Exchange Commission (SEC) of a certificate of merger.
Here, Bancommerce and TRB remained separate corporations with distinct corporate personalities. What happened is
that TRB sold and Bancommerce purchased identified recorded assets of TRB in consideration of Bancommerces
assumption of identified recorded liabilities of TRB including booked contingent accounts. There is no law that prohibits
this kind of transaction especially when it is done openly and with appropriate government approval. In strict sense, no
merger or consolidation took place as the records do not show any plan or articles of merger or consolidation. More
importantly, the SEC did not issue any certificate of merger or consolidation.

164
3B-Corporation Law

Topic:
Subtopic:
Provision:
Case Name:

Non-Stock Corporations
Sections 87 to 95, Corporation Code; Sec Memorandum Circular 8
Dennis R. Manzanal v Ramon K. Ilusorio

FACTS:
A penthouse unit at the Baguio Country Club Corporation (BCCC) building was assigned to Ilusorio with the conformity
of BCCC. For a period of five years since the assignment, Ilusorio enjoyed the use of the unit and the clubs facilities.
But that when conflict within the family arose in 1998 and escalated to great proportions, he was barred from using the
unit and was almost expelled as member of the club. This spawned his filing of multiple suits against BCCC before the
courts and SEC.
Ilusorio requested from BCCC his current statement of account. Replying, BCCC charged him the amount
of P102,076.74, which he paid under protest. He, however, requested a breakdown of the amount, which BCCC, thru
Manzanal, complied with, via letter to which was attached respondents Statement of Account itemizing the amount
which in fact totalled P2,928,223.26.
Respondent averred that, inter alia, he should not be charged for the use of the unit as he, as owner, is entitled to its
use and enjoyment. And he cast doubt on billing him for charges dating back to 1995.
The trial court dismissed the complaint. On appeal, CA reversed the RTC Makati and ordered the reinstatement of
respondents complaint, ratiocinating that if the allegations in the complaint that Ilusorio is a member of BCCC and an
owner of one of the units of the Clubs House Building, the defendants-appellees had been unreasonably charging him.
ISSUE: Whether or not BCCC is correct in charging Ilusorio in the amount of P2,928,223.26
HELD:
The petition is meritorious.
The Court finds from the tenor of the demand letters that it did not deviate from the standard practice of pursuing the
satisfaction of a club members obligations. Respondent did not indicate in his complaint how tenuous petitioners
claim for unpaid charges is. In his reply to petitioners final letter of demand, he in fact did not contradict petitioners
statement that his work partners and employees used his unit, thereby admitting that he welched on his undertaking
in the contract that only family members are allowed free usage.
As an exclusive organization which primarily derives life from membership fees and charges, BCCC is expected to
enforce claims from members in default of their contractual obligations.

165
3B-Corporation Law

Topic:
Non-Stock Corporations
Subtopic:
Provision:
Sections 87 to 95, Corporation Code; Sec Memorandum Circular 8
Case Name:
Valley Golf and Country Club Inc. v Caram
Name: Glenn Nio M. Sartillo
Topic: Non-Stock Corporations
Name of Case: Valley Golf & Country Club, Inc. vs. Caram (GR No. 158805, April 16, 2009)
Corporation Code Provision: Sections 76 to 80
FACTS:
Cong. Fermin Z. Caram, Jr., respondents husband, subscribed and paid in full 1 Golf Share of Valley Golf and was
subsequently issued with a stock certificate which indicated a par value of P9,000.00. Valley Golf alleged that Caram
stopped paying his monthly dues and that it has sent 5 letters to Caram concerning his delinquent account.
The Golf Share was subsequently sold at public auction. Caram thereafter died and his wife initiated
intestate
proceedings before the RTC. Unaware of the pending controversy over the Golf Share, the Caram family and the RTC
included the Golf Share as part of Carams estate. Upon partition, the Golf Share was adjudicated to the wife. It was
only through a letter that the heirs of Caram learned of the sale of the Golf Share following their inquiry with Valley
Golf about the Golf Share.
ISSUE: Whether or not non-stock corporation can seize and dispose of the membership share of a fully-paid member
on account of its unpaid debts to the corporation when it is authorized to do so under the corporate by-laws but not by
the Articles of Incorporation
HELD:
The SEC hearing officer noted that under Section 67, paragraph 2 of the Corporation Code, a share stock could only be
deemed delinquent and sold in an extrajudicial sale at public auction only upon the failure of the stockholder to pay
the unpaid subscription or balance for the share. The section could not have applied in Carams case since he had fully
paid for the Golf Share and he had been assessed not for the share itself but for his delinquent club dues. Proceeding
from the foregoing premises, the SEC hearing officer concluded that the auction sale had no basis in law and was thus
a nullity.
There is a specific provision under Title XI on Non-Stock Corporations of the Corporation Code dealing with the
termination of membership in a non-stock corporation such Valley Golf. Section 91 of the Corporation Code provides:
SEC. 91. Termination of membership.Membership shall be terminated in the manner and for the
causes provided in the articles of incorporation or the by-laws. Termination of membership shall have the
effect of extinguishing all rights of a member in the corporation or in its property, unless otherwise
provided in the articles of incorporation or the by-laws.
On the basis of Section 91, the Supreme Court ruled that the right of a non-stock corporation such as Valley Golf to
expel a member through the forfeiture of the Golf Share may be established in the by-laws alone, as is the situation in
this case.
However, the Supreme Court proceed to declare the sale as invalid. The Supreme Court found that Valley Golf acted in
bad faith when it sent the final notice to Caram under the pretense they believed him to be still alive, when in fact they
had very well known that he had already died.

166
3B-Corporation Law

Topic:
Non-Stock Corporations
Subtopic:
Provision:
Sections 87 to 95, Corporation Code; Sec Memorandum Circular 8
Case Name:
Calatagan Golf and Country Club, Inc. v Sixto Clemente, Jr.
Topic: Non- Stock Corporations
Case: Calatagan Golf and Country Club vs Clemente
Name: Carmela Abergos
FACTS:
Clemente applied to purchase one share of stock of Calatagan. The provision on monthly dues is incorporated in
Calatagans Articles of Incorporation and By-Laws. It is also reproduced at the back of each certificate of stock. When
Clemente became a member, the monthly charge stood at P400.00. He paid P3,000.00 for his monthly dues on 21
March 1991 and another P5,400.00 on 9 December 1991. Then he ceased paying the dues. Ten (10) months later,
Calatagan declared Clemente delinquent for having failed to pay his monthly dues. On 1 December 1992, Calatagans
board of directors adopted a resolution authorizing the foreclosure of shares of delinquent members, including
Clementes; and the public auction of these shares.
On 7 December 1992, Calatagan sent a third and final letter to Clemente, this time signed by its Corporate Secretary,
Atty. Benjamin Tanedo, Jr., warning that unless Clemente settles his outstanding dues, his share would be included
among the delinquent shares to be sold at public auction. Again, this letter was sent to Clementes mailing address
that had already been closed. Clemente learned of the sale of his share only in November of 1997.
He filed a claim with the Securities and Exchange Commission (SEC) seeking the restoration of his shareholding in
Calatagan with damages. SEC rendered a decision dismissing Clementes complaint. The SEC further held that
Calatagan had complied with all the requirements for a valid sale of the subject share.
Clemente filed a petition for review with the CA. CA promulgated a decision reversing the SEC. Hence, the present
appeal.
ISSUE:
Won Clemente is entitled to the restoration of his shareholding in Calatagan and award for damages
RULING:
Yes. It is plain that Calatagan had endeavored to install a clear and comprehensive procedure to govern the payment
of monthly dues, the declaration of a member as delinquent, and the constitution of a lien on the shares and its
eventual public sale to answer for the members debts. Under Section 91 of the Corporation Code, membership in a
non-stock corporation "shall be terminated in the manner and for the causes provided in the articles of incorporation or
the by-laws." The By-law provisions are elaborate in explaining the manner and the causes for the termination of
membership in Calatagan, through the execution on the lien of the share. The Court is satisfied that the By-Laws, as
written, affords due protection to the member by assuring that the member should be notified by the Secretary of the
looming execution sale that would terminate membership in the club. In addition, the By-Laws guarantees that after
the execution sale, the proceeds of the sale would be returned to the former member after deducting the outstanding
obligations. If followed to the letter, the termination of membership under this procedure outlined in the By-Laws would
accord with substantial justice.
Calatagan argues that it exercised due diligence before the foreclosure sale and "sent several notices to Clementes
specified mailing address. We do not agree; we cannot label as due diligence Calatagans act of sending the December
7, 1992 letter to Clementes mailing address knowing fully well that the P.O. Box had been closed. Due diligence or
good faith imposes upon the Corporate Secretary the chief repository of all corporate records the obligation to
check Clementes other address which, under the By-Laws, have to be kept on file and are in fact on file.
The by-law provisions was clearly conceived to afford due notice to the delinquent member of the impending sale, and
not just to provide an intricate faade that would facilitate Calatagans sale of the share. But then, the bad faith on
Calatagans part is palpable. As found by the Court of Appeals, Calatagan very well knew that Clementes postal box to
which it sent its previous letters had already been closed, yet it persisted in sending that final letter to the same postal
box. It is also noteworthy that Clemente in his membership application had provided his residential address along with

167
3B-Corporation Law

his residence and office telephone numbers. Nothing in Section 32 of Calatagans By-Laws requires that the final notice
prior to the sale be made solely through the members mailing address.
The utter bad faith exhibited by Calatagan brings into operation Articles 19, 20 and 21 of the Civil Code, under the
Chapter on Human Relations. These provisions, which the Court of Appeals did apply, enunciate a general obligation
under law for every person to act fairly and in good faith towards one another. A non-stock corporation like Calatagan
is not exempt from that obligation in its treatment of its members. The obligation of a corporation to treat every
person honestly and in good faith extends even to its shareholders or members, even if the latter find themselves
contractually bound to perform certain obligations to the corporation. A certificate of stock cannot be a charter of
dehumanization.
And therefore, the award of actual damages is of course warranted since Clemente has sustained pecuniary injury by
reason of Calatagans wrongful violation of its own By-Laws. It would not be feasible to deliver Clementes original
Certificate of Stock because it had already been cancelled and a new one issued in its place in the name of the
purchases at the auction who was not impleaded in this case. However, the Court of Appeals instead directed that
Calatagan to issue to Clemente a new certificate of stock. That sufficiently redresses the actual damages sustained by
Clemente. After all, the certificate of stock is simply the evidence of the share.

168
3B-Corporation Law

Topic:
Subtopic:
Provision:
Case Name:

Non-Stock Corporations
Sections 87 to 95, Corporation Code; Sec Memorandum Circular 8
Tan v Sycip

Topic: Non- Stock Corporations


Case: Tan vs Sycip
Name: Carmela Abergos
FACTS:
Grace Christian High School (GCHS) is a nonstock, non-profit educational corporation w/ 15 regular members, who also
constitute the board of trustees. During the annual members meeting on April 6, 1998, only 11 living membertrustees, as 4 had already died. 7 attended the meeting through their respective proxies.
The meeting was convened and chaired by Atty. Sabino Padilla Jr. over the objection of Atty. Antonio C. Pacis, who
argued that there was no quorum. While petitioners Ernesto Tanchi, Edwin Ngo, Virginia Khoo, and Judith Tan were
voted to replace the 4 deceased member-trustees.
SEC ruled that the meeting was void due to lack of quorum. CA dismissed.
ISSUE:
Won dead members should still be counted in the quorum
HELD: NO.
Having determined that the quorum in a members meeting is to be reckoned as the actual number of members of the
corporation, the next question to resolve is what happens in the event of the death of one of them.
In stock corporations, shareholders may generally transfer their shares. Thus, on the death of a shareholder, the
executor or administrator duly appointed by the Court is vested with the legal title to the stock and entitled to vote
it. Until a settlement and division of the estate is effected, the stocks of the decedent are held by the administrator or
executor.
On the other hand, membership in and all rights arising from a nonstock corporation are personal and nontransferable, unless the articles of incorporation or the bylaws of the corporation provide otherwise. In other words, the
determination of whether or not dead members are entitled to exercise their voting rights (through their executor or
administrator), depends on those articles of incorporation or bylaws.
Under the By-Laws of GCHS, membership in the corporation shall, among others, be terminated by the death of the
member. Section 91 of the Corporation Code further provides that termination extinguishes all the rights of a member
of the corporation, unless otherwise provided in the articles of incorporation or the bylaws.
Applying Section 91 to the present case, we hold that dead members who are dropped from the membership roster in
the manner and for the cause provided for in the By-Laws of GCHS are not to be counted in determining the requisite
vote in corporate matters or the requisite quorum for the annual members meeting. With 11 remaining members, the
quorum in the present case should be 6. Therefore, there being a quorum, the annual members meeting, conducted
with six, members present, was valid

169
3B-Corporation Law

Topic:
Non-Stock Corporations
Subtopic:
Provision:
Sections 87 to 95, Corporation Code; Sec Memorandum Circular 8
Case Name:
Primo Co., Sr. v Philippine Canine Club, Inc.
Topic: Non- Stock Corporations
Case:
PRIMO CO, SR., EDGARDO CRUZ, FE LANNY L. ALEGADO, JESTER B. ONGCHUAN, JOSEPH
ONGCHUAN and LUCIANNE CHAM vs. PHILIPPINE CANINE CLUB INC.
Name: Carmela Abergos
FACTS:
The respondent, Philippine Canine Club, Inc. (PCCI), is a non-stock, non-profit organization established in 1963 for the
principal purpose of promoting the breeding of purebred dogs. The petitioners, Primo Co, Sr.(Co), Edgardo Cruz (Cruz),
Fe Lanny L. Alegado (Alegado), Jester B. Ongchuan (Jester), Joseph Ongchuan (Joseph), and Lucianne Cham (Cham)
were members of PCCI.
Sometime in 2008, the Asian Kennel Club Union of the Philippines, Inc. (AKCUPI) was established as a corporate entity,
and made known its intention to hold and to sponsor dog shows and events similar to those being held and conducted
by other kennel clubs in the Philippines, including the PCCI.
Believing that there was no conflict in the goals and the objectives of PCCI and AKCUPI, and that there was no
prohibition on members of PCCI whether express or implied from joining and affiliating themselves with other kennel
clubs, the petitioners registered their dogs with AKCUPI.
PCCI amended its By-laws, allegedly without the participation of its non-voting members, including the petitioners. SEC
issued a Certification approving PCCIs Amended By-laws. Shortly thereafter, PCCI sent Co, Cruz and Jester identical
letters dated December 15, 2008, informing them of their expulsion from the organization due to their alleged
conduct prejudicial to the best interest of PCCI. PCCIs General Manager personally advised Alegado about her
expulsion, albeit, she has never actually received any formal communication to that effect. As regards Joseph and
Cham, PCCI allegedly threatened them with the same sanctions imposed on the other petitioners.
The petitioners filed a case for Annulment of the Amended By-laws, Injunction and Damages with application for the
issuance of a TRO. They claim that the adoption of the questioned Amended By-Laws, particularly Article VI on
suspension, expulsion and termination of membership, without the participation of PCCIs non-voting members,
constitutes a violation of Section 6 of the Corporation Code.
RTC found that although the petitioners are non-voting members of PCCI, they are still entitled to vote on the
amendments of the by-laws under Section 6 of the Corporation Code. Since the May 17, 2008 Amended By-laws was
voted upon only by its voting members and without the participation of its nonvoting members, including the
petitioners, the RTC held that the Amended By-laws is questionable. The RTC thus declared that the petitioners are
entitled to the injunctive writ they prayed for.
The CA reversed the RTC Order. The petitioners filed MR denied. Hence, this petition.
ISSUE:
Can the court enjoin the enforcement of the PCCI Amended By-laws, which has already been in effect, and the
enforcement of the penalty of expulsion against the petitioners, which has already been implemented?
RULING:
It is a universal principle of law that an injunction will not issue to restrain the performance of an act already done. A
writ of injunction becomes moot and academic after the act sought to be enjoined has already been consummated. In
the present case, the act sought to be restrained by the petitioners has already been partly accomplished. The actual
suspension and expulsion of Co, Cruz, Alegado and Jester from PCCI rendered their prayer for injunctive relief moot.
Evidently, it is no longer possible to grant the relief they were seeking that is, to stop PCCI from implementing their
suspension and expulsion as the same has already been consummated. The status quo can no longer be restored.
Furthermore, Dayrit v. Delos Santos, the case cited by the petitioners is not squarely applicable to the present case. In
Dayrit, the party praying for an injunction alleged not only acts that were already committed or consummated, but
also those acts that the defendant could still continue to execute unless restrained. Moreover, the acts sought to be
restrained in that case (i.e., making excavations, opening a ditch, and construction of a dam) are capable of being
continued or repeated. In other words, the defendant's questioned acts, even if partly or initially executed, are capable
of continuation, as these acts consist of several stages that are not consummated by a mere single act. In the present
case, the suspension and expulsion of petitioners Co, Cruz, Alegado and Jester are finished completed acts and which
can only be restored depending on the final outcome of the case on the merits. This is different from the acts enjoined
in Dayrit which consisted of the making of excavations, opening a ditch, and construction of a dam, which were all

170
3B-Corporation Law

continuing. Hence, we cannot apply the ruling that "not only the commission or execution of such acts, but also their
continuation can be prevented or prohibited by an injunction."
Thus, we hold that the trial court's issuance of the writ of preliminary injunction, insofar as petitioners Co, Cruz,
Alegado and Jester, is improper, as the same may no longer be availed of.

171
3B-Corporation Law

Topic:
Subtopic:
Provision:
Case Name:

Special Corporations
Educational Corporations
Sections 106 to 108, Corporation Code
Barayuga v Adventist University of the Philippines

EDUCATIONAL CORPORATIONS
BARAYUGA v ADVENTIST UNIVERSITY OF THE PHILIPPINES (AUP)
FACTS:

The North Philippine Union Mission elected the members of the Board of Trustees of respondent AUP, including
the chairman and the secretary. Dayson was elected chairman while petitioner Barayuga was chosen
secretary. In January 2001, the Board of Trustees appointed Barayuga as president of AUP.

In January 2003, a special meeting was held wherein the members of the Board of Trustees, by secret ballot,
voted to remove him as President based on the findings of the audit, to appoint an interim committee
consisting of 3 members to assume the powers and functions of president, and to recommend him to the
NPUM for consideration as Associate Director of Secondary Education.

Barayuga asked for reconsideration; however, it was denied by the Board because his reasons were not
meritorious. He brought a suit for injunction and damages in the RTC praying for the issuance of a TRO against
AUP and its Board of Trustees, which was granted by the RTC.

The respondents filed a petition for certiorari in the CA, which rendered its decision nullifying the RTCs writ of
preliminary injunction.
ISSUE:
Whether petitioners term as President was five years
HELD:
No.
Section 108 of the Corporation, paragraph 2 provides that although the term set for the members of the Board of
Trustees is at five years, the duration is subject to qualification as to what is otherwise provided in the articles of
incorporation or by-laws of the educational corporation. That contrary provision controls on the term of office.
Under the amended By-Laws of AUP, the term of office of the members of the Board of Trustees was only of two years;
and the officers, who included the President, were to be elected from among the members of the Board during their
organizational meeting, which was held during the election of the Board of Trustees every two years, which means the
officers, including the President, were to exercise the powers vested by the amended By-Laws of AUP for a term of only
2years, not five.
By the time of his removal for cause as President, he was alreadyo ccupying the office in a hold-over capacity, and
could be removed at any time, without cause, upon the election or appointment of his successor. Thus, his removal as
President of AUP, being made in accordance with the AUP Amended By-Laws, was valid.

172
3B-Corporation Law

Topic:
Special Corporations
Subtopic:
Religious Corporations
Provision:
Sections 106 to 108, Corporation Code
Case Name:
Iglesia Evangelica Metodista Enlas Islas Filipinas, Inc. v Lazaro
RELIGIOUS CORPORATIONS
IGLESIA EVANGELICA METODISTA ENLAS ISLAS FILIPINAS INC v LAZARO
FACTS:

In 1909, Bishop Nicolas Zamora established the petitioner Iglesia Evangelica Metodista En Las Islas Filipinas,
Inc. (IEMELIF) as a corporation sole with Bishop Zamora acting as its "General Superintendent."

Although the IEMELIF remained a corporation sole on paper (with all corporate powers theoretically lodged in
the hands of one member, the General Superintendent), it had always acted like a corporation aggregate.

During its 1973 General Conference, the general membership voted to put things right by changing IEMELIF's
organizational structure from a corporation sole to a corporation aggregate. The Securities and Exchange
Commission (SEC) approved the vote. For some reasons, however, the corporate papers of the IEMELIF
remained unaltered as a corporation sole.

Petitioners claim that a complete shift from IEMELIF's status as a corporation sole to a corporation aggregate
required, not just an amendment of the IEMELIF's articles of incorporation, but a complete dissolution of the
existing corporation sole followed by a re-incorporation.

RTC dismissed the action for Declaration of Nullity of Amended Articles of Incorporation which CA affirmed after
an appeal was filed by the petitioners.
ISSUE:
Whether CA erred in affirming the RTC ruling that a corporation sole may be converted into a corporation aggregate by
mere amendment of its articles of incorporation
HELD:
Yes.
The Corporation Code provides no specific mechanism for amending the articles of incorporation of a corporation sole.
However, Section 109 of the Corporation Code allows the application to religious corporations of the general provisions
governing non-stock corporations.
For non-stock corporations, the power to amend its articles of incorporation lies in its members. The code requires
two-thirds of their votes for the approval of such an amendment. So how will this requirement apply to a corporation
sole that has technically but one member (the head of the religious organization) who holds in his hands its broad
corporate powers over the properties, rights, and interests of his religious organization?
Although a non-stock corporation has a personality that is distinct from those of its members who established it, its
articles of incorporation cannot be amended solely through the action of its board of trustees. The amendment needs
the concurrence of at least two-thirds of its membership. If such approval mechanism is made to operate in a
corporation sole, its one member in whom all the powers of the corporation technically belongs, needs to get the
concurrence of two-thirds of its membership. The one member, here the General Superintendent, is but a trustee,
according to Section 110 of the Corporation Code, of its membership. He can, with membership concurrence, increase
the technical number of the members of the corporation from sole or one to the greater number authorized by its
amended articles.

173
3B-Corporation Law

Topic:
Subtopic:
Provision:
Case Name:

Special Corporations
Religious Corporations
Sections 106 to 108, Corporation Code
United Church of Christ in the Philippines v Bradford United Church of
Christ, Inc.
RELIGIOUS CORPORATIONS
UNITED CHURCH OF CHRIST IN THE PHILIPPINES, INC. (UCCP) v BRADFORD UNITED CHURCH OF CHRIST,
INC. (BUCCI)
FACTS:

UCCP has three (3) governing bodies namely: the General Assembly, the Conference and the Local Church,
each having distinct and separate duties and powers. As a UCCP local church located in Cebu, BUCCI belonged
to the Cebu Conference, Inc. (CCI) with whom it enjoyed peaceful co-existence until late 1989 when BUCCI
started construction of a fence that encroached upon the right-of-way allocated by UCCP for CCI and Visayas
jurisdiction.

In 1992, BUCCI disaffiliated from UCCP. The effectivity of the disaffiliation was made to retroact to when BUCCI
severed its ties from CCI.

Consequently, BUCCI filed its Amended Articles of Incorporation and By-Laws which provided for and effected
its disaffiliation from UCCP. SEC approved the same on 2 July 1993.

Thereafter, UCCP filed before SEC a complaint/protest initially for the disallowance of the continued use of
BUCCI as corporate name. It was alleged in the amended complaint/protest, however, that the separate
incorporation and registration of BUCCI is not allowed under the UCCP Constitution and By-laws; and that
BUCCI and the respondents must be enjoined from using the name BUCCI, both in its Amended AOI and its
dealings with the public, and from using its properties.

SEC en banc dismissed UCCP's petition (which was later affirmed by CA) to declare as null and void the
amendments made to the Articles of Incorporation of BUCCI.
ISSUE:
Whether the disaffiliation of respondents purely an ecclesiastical affair, hence, outside the jurisdiction of civil courts
HELD:
No. The Court believes that the matter at hand is not purely an ecclesiastical affair.
An ecclesiastical affair is one that concerns doctrine, creed or form of worship of the church, or the adoption and
enforcement within a religious association of needful laws and regulations for the government of the membership, and
the power of excluding from such associations those deemed unworthy of membership. Based on this definition, an
ecclesiastical affair involves the relationship between the church and its members and relate to matters of faith,
religious doctrines, worship and governance of the congregation. To be concrete, examples of this so-called
ecclesiastical affairs to which the State cannot meddle are proceedings for excommunication, ordinations of religious
ministers, administration of sacraments and other activities attached with religious significance.
The SC upheld BUCCI's decision as it concerns its legal right as a religious corporation to disaffiliate from another
religious corporation via legitimate means a secular matter well within the civil courts' purview.

174
3B-Corporation Law

Topic:
Subtopic:
Provision:
Case Name:

Dissolution and Liquidation


Sections 117 to 122, Corporation Code
Metropolitan Bank and Trust Co. v The Board of Trustees of Riverside
Mills Corp, Provident Retirement Fund

FACTS:

RMC established a Provident and Retirement Plan for its regular employees. Under the Plan, RMC and its
employees shall each contribute 2% of the employees current basic monthly salary, with RMCs contribution
to increase by 1% every five (5) years up to a maximum of 5%.

The contributions shall form part of the provident fund (the Fund) which shall be held, invested and distributed
by the Commercial Bank and Trust Company. On October 15, 1979, the Board of Trustees of RMCPRF (the
Board) entered into an Investment Management Agreement with Philbank (petitioner Metropolitan Bank and
Trust Company).

Pursuant to the Agreement, petitioner shall act as an agent of the Board and shall hold, manage, invest and
reinvest the Fund in Trust Account No. 1797 in its behalf.

The Agreement shall be in force for one (1) year and shall be deemed automatically renewed unless sooner
terminated either by petitioner bank or by the Board. In 1984, RMC ceased business operations.

Nonetheless, petitioner continued to render investment services to respondent Board. In a letter dated
September 27, 1995, petitioner informed respondent Board that Philbanks Board of Directors had decided to
apply the remaining trust assets held by it in the name of RMCPRF against part of the outstanding obligations
of RMC.

Subsequently, respondent RMC Unpaid Employees Association, Inc. (Association), representing the terminated
employees of RMC, learned of Trust Account No. 1797. Through counsel, they demanded payment of their
share in a letter dated February 4, 1997.

When such demand went unheeded, the Association, along with the individual members of RMCPRF, filed a
complaint for accounting against the Board and its officers.
ISSUE:

Whether or not the functions of the Board of Trustees ceased upon with RMCs closure.
RULING:

NO.

Under Section 122 of the Corporation Code, a dissolved corporation shall nevertheless continue as a body
corporate for three (3) years for the purpose of prosecuting and defending suits by or against it and enabling it
to settle and close its affairs, to dispose and convey its property and to distribute its assets, but not for the
purpose of continuing the business for which it was established. Within those three (3) years, the corporation
may appoint a trustee or receiver who shall carry out the said purposes beyond the three (3)-year winding-up
period.

Thus, a trustee of a dissolved corporation may commence a suit which can proceed to final judgment even
beyond the three (3)-year period of liquidation. In the same manner, during and beyond the three (3)-year
winding-up period of RMC, the Board of Trustees of RMCPRF may do no more than settle and close the affairs of
the Fund. The Board retains its authority to act on behalf of its members, albeit, in a limited capacity. It may
commence suits on behalf of its members but not continue managing the Fund for purposes of maximizing
profits.

Here, the Boards act of issuing the Resolution authorizing petitioner to release the Fund to its beneficiaries is
still part of the liquidation process, which is, satisfaction of the liabilities of the Plan, and does not amount to
doing business. Hence, it was properly within the Boards power to promulgate.

175
3B-Corporation Law

Topic:
Subtopic:
Provision:
Case Name:
Facts:

Dissolution and Liquidation


Sections 117 to 122, Corporation Code
Majority Stockholder s of Ruby Inc v Lim

Ruby Industrial Corporation (RUBY) is a domestic corporation engaged in glass manufacturing. Reeling from
severe liquidity problems beginning in 1980, RUBY filed on December 13, 1983 a petition for suspension of
payments with the SEC which was granted.
On August 10, 1984, the SEC Hearing Panel created the management committee (MANCOM) for RUBY,
composed of representatives from Rubys creditors. One of the many task of MANCOM is study, review and
evaluate the proposed rehabilitation plan for RUBY.
Subsequently, two (2) rehabilitation plans were the BENHAR/RUBYsubmitted to the SEC Rehabilitation Plan of
the majority stockholders led by Yu Kim Giang, and the Alternative Plan of the minority stockholders
represented by Miguel Lim (Lim). But the implementation of both majority plans has been enjoined by the SEC
and CA. Later, the SC issued a final injunction on the implementation.
Sept 18, 1991: Notwithstanding the injunction order, SEC issued an Order approving the Revised BENHAR/RUBY
Plan and creating a new management committee to oversee its implementation. It also dissolves the MANCOM.
The Revised BENHAR/RUBY Plan had proposed the calling for subscription of unissued shares through a Board
Resolution from the P11.814 million of theP23.7 million ACS in order to allow the long overdue program of the
REHAB Program.
Oct 2, 1991: To implement the Revised plan, RUBYs board of directors held a special meeting and took up the
capital infusion of P11.814 Million representing the unissued and unsubscribed portion of the present ACS of
P23.7 Million.
The Board resolved that: The corporation be authorized to issue out of the unissued portion of the authorized
capital stocks of the corporation in the form of common stocks 11.8134.00 [Million] to be subscribed and paid
in full by the present stockholders in proportion to their present stockholding in the corporation on staggered
basis and that should any of the stockholders fail to exercise their rights to buy the number of shares they
are qualified to buy by making the first installment payment of 25% on or before October 13, 1991, then the
other stockholders may buy the same and that only when none of the present stockholders are interested in
the shares may there be a resort to selling them by public auction.
The minority directors claimed they were not notified of said board meeting.
Sept 1, 1996: Lim receive a Notice of Stockholders Meeting scheduled on September 3, 1996. The matters
that will be taken up in said meeting include the extension of RUBYs corporate term for another twenty-five
(25) years and election of Directors.
Sept 3, 1996: Lim together with other minority stockholders, appeared in order to put on record their
objections on the validity of holding thereof and the matters to be taken therein. Specifically, they questioned
the percentage of stockholders present in the meeting which the majority claimed stood at 74.75%(from
59.829%) of the outstanding capital stock of RUBY. Lim argued that the majority stockholders claimed to have
increased their shares to 74.75% by subscribing to the unissued shares of the authorized capital stock (ACS).
Lim pointed out that such move of the majority was in implementation of the BENHAR/RUBY Plan which calls
for capital infusion of P11.814 Million representing the unissued and unsubscribed portion of the present ACS
of P23.7 Million.
Jan 20, 1998: the SC affirmed CA decision setting aside the SEC orders approving the Revised BENHAR/RUBY
Plan because it not only recognized the void deeds of assignments entered into with some of RUBYs creditors
in violation of the CAs decision in CA-G.R. SP No. 18310, but also maintained a financing scheme which will
just make the rehabilitation plan more costly and create a worse situation for RUBY.
Mar 17, 2000, Lim filed a Motion informing the SEC of acts being performed by BENHAR and RUBY. Allegedly,
the implementation of the new percentage stockholdings of the majority stockholders and the calling of
stockholders meeting and the subsequent resolution approving the extension of corporate life of RUBY for
another twenty-five (25) years, were all done in violation of the decisions of the CA and this Court, and without
compliance with the legal requirements under the Corporation Code.
There being no valid extension of corporate term, RUBYs corporate life had legally ceased. Consequently, Lim
moved that the SEC: (1) declare as null and void the infusion of additional capital made by the majority
stockholders and restore the capital structure of RUBY to its original structure prior to the time injunction was
issued; and (2) declare as null and void the resolution of the majority stockholders extending the corporate life
of RUBY for another twenty-five (25) years.
Sept 18, 2002, the SEC overruled the objections raised by the minority stockholders regarding the
questionable issuance of shares of stock by the majority stockholders and extension of RUBYs corporate term
because the filing of the amendment of articles of incorporation by RUBY in 1996 complied with all the legal
requisites and hence the presumption of regularity in the act of a government entity stands. It pointed out that
Lim raised the issue only in the year 2000. Moreover, the SEC found that notwithstanding his allegations of

176
3B-Corporation Law

Issue:

fraud, Lim never proved the illegality of the additional infusion of the capitalization by RUBY so as to warrant a
finding that there was indeed an unlawful act.
Before the CA, Lim demonstrated the following evidence to rebut the presumption of regularity:
(1) it was the board of directors and not the stockholders which conducted the meeting without the approval of
the MANCOM;
(2) there was no written waivers of the minority stockholders pre-emptive rights and thus it was irregular to
merely notify them of the board of directors meeting and ask them to exercise their option;
(3) there was an existing permanent injunction against any additional capital infusion on the BENHAR/RUBY
Plan, while the CA and this Court both rejected the Revised BENHAR/RUBY Plan;
(4) there was no General Information Sheet reports made to the SEC on the alleged capital infusion, as per
certification by the SEC. CA Decision (which is cited by SC in its decision): - SEC erred in not finding that the
October 2, 1991 meeting held by RUBYs board of directors was illegal because the MANCOM was neither
involved nor consulted in the resolution approving the issuance of additional shares of RUBY.
The CA further noted that the October 2, 1991 board meeting was conducted on the basis of the September
18, 1991 order of the SEC Hearing Panel approving the Revised BENHAR/RUBY Plan, which plan was set by CA
and SC.
The CA pointed out that records confirmed the proposed infusion of additional capital for RUBYs rehabilitation,
approved during said meeting, as implementing the Revised BENHAR/RUBY Plan. Necessarily then, such capital
infusion is covered by the final injunction against the implementation of the revised plan.
The CA likewise faulted the SEC in relying on the presumption of regularity on the matter of the extension of
RUBYs corporate term through the filing of amended articles of incorporation. SEC should have invalidated the
resolution extending the corporate term of RUBY for another twenty-five (25) years.
With the expiration of the RUBYs corporate term, the CA ruled that it was error for the SEC in not commencing
liquidation proceedings.
WON the additional capital infusion is valid?

RULING

No because the issuance of additional shares was done in breach of trust by the controlling stockholders.

Here, the majority sought to impose their will and, through fraudulent means, attempt to siphon off Rubys
valuable assets to the great prejudice of Ruby itself, as well as the minority stockholders and the unsecured
creditors.]

Ratio: A stock corporation is expressly granted the power to issue or sell stocks. The power to issue shares of
stock in a corporation is lodged in the board of directors and no stockholders meeting is required to consider it
because additional issuances of shares of stock do not need approval of the stockholders.

What is only required is the board resolution approving the additional issuance of shares. The corporation shall
also file the necessary application with the SEC to exempt these from the registration requirements under the
Revised Securities Act (now the Securities Regulation Code). But CA found, which the Court affirmed, that: the
foregoing payment schedules as embodied in the said Revised plan which gives Benhar undue advantage over
the other creditors goes against the very essence of rehabilitation, which requires that no creditor should be
preferred over the other.

One of the salient features of the Revised Benhar/Ruby Plan is to Call on unissued shares forP11.814 M and if
minority will take up their pre-emptive rights and dilute minority shareholdings. With the nullification of the
Revised BENHAR/RUBY Plan by both CA and SC on Jan 20, 1998, the legitimate concerns of the minority
stockholders and MANCOM who objected to the capital infusion which resulted in the dilution of their
shareholdings, the expiration of RUBYs corporate term and the pending incidents on the void deeds of
assignment of credit all these should have been duly considered and acted upon by the SEC when the case
was remanded to it for further proceedings. With the final rejection of the courts of the Revised BENHAR/RUBY
Plan, it was grave error for the SEC not to act decisively on the motions filed by the minority stockholders who
have maintained that the issuance of additional shares did not help improve the situation of RUBY except to
stifle the opposition coming from the MANCOM and minority stockholders by diluting the latters shareholdings.

Worse, the SEC ignored the evidence adduced by the minority stockholders indicating that the correct amount
of subscription of additional shares was not paid by the majority stockholders and that SEC official records still
reflect the 60%-40% percentage of ownership of RUBY.

The SEC remained indifferent to the reliefs sought by the minority stockholders, saying that the issue of the
validity of the additional capital infusion was belatedly raised. Even assuming the October 2, 1991 board
meeting indeed took place, the SEC did nothing to ascertain whether indeed, as the minority claimed: (1) the
minority stockholders were not given notice as required and reasonable time to exercise their preemptive
rights; and (2) the capital infusion was not for the purpose of rehabilitation but a mere ploy to divest the
minority stockholders of their 40.172% shareholding and reduce it to a mere 25.25%. Pre-emptive right under

177
3B-Corporation Law

Sec. 39 of the Corporation Code refers to the right of a stockholder of a stock corporation to subscribe to all
issues or disposition of shares of any class, in proportion to their respective shareholdings.
The right may be restricted or denied under the articles of incorporation, and subject to certain exceptions and
limitations. The stockholder must be given a reasonable time within which to exercise their preemptive rights.
Upon the expiration of said period, any stockholder who has not exercised such right will be deemed to have
waived it. The validity of issuance of additional shares may be questioned if done in breach of trust by the
controlling stockholders.
Thus, even if the preemptive right does not exist, either because the issue comes within the exceptions in
Section 39 or because it is denied or limited in the articles of incorporation, an issue of shares may still be
objectionable if the directors acted in breach of trust and their primary purpose is to perpetuate or shift control
of the corporation, or to freeze out the minority interest. In this case, the following relevant observations
should have signaled greater circumspection on the part of the SEC -- upon the third and last remand to it
pursuant to our January 20, 1998 decision -- to demand transparency and accountability from the majority
stockholders, in view of the illegal assignments and objectionable features of the Revised BENHAR/RUBY Plan,
as found by the CA and as affirmed by this Court: There can be no gainsaying the well-established rule in
corporate practice and procedure that the will of the majority shall govern in all matters within the limits of the
act of incorporation and lawfully enacted by-laws not proscribed by law.
It is, however, equally true that other stockholders are afforded the right to intervene especially during critical
periods in the life of a corporation like reorganization, or in this case, suspension of payments, more so, when
the majority seek to impose their will and through fraudulent means, attempt to siphon off Rubys valuable
assets to the great prejudice of Ruby itself, as well as the minority stockholders and the unsecured creditors.
Certainly, the minority stockholders and the unsecured creditors are given some measure of protection by the
law from the abuses and impositions of the majority, more so in this case, considering the give-away signs of
private respondents perfidy strewn all over the factual landscape.
Indeed, equity cannot deprive the minority of a remedy against the abuses of the majority, and the present
action has been instituted precisely for the purpose of protecting the true and legitimate interests of Ruby
against the Majority Stockholders. On this score, the Supreme Court, has ruled that: Generally speaking, the
voice of the majority of the stockholders is the law of the corporation, but there are exceptions to this rule.
There must necessarily be a limit upon the power of the majority. Without such a limit the will of the majority
will be absolute and irresistible and might easily degenerate into absolute tyranny. x x x[67] (Additional
emphasis supplied.) Lamentably, the SEC refused to heed the plea of the minority stockholders and MANCOM
for the SEC to order RUBY to commence liquidation proceedings, which is allowed under Sec. 4-9 of the Rules
on Corporate Recovery.
Under the circumstances, liquidation was the only hope of the minority stockholders for effecting an orderly
and equitable settlement of RUBYs obligations, and compelling the majority stockholders to account for all
funds, properties and documents in their possession, and make full disclosure on the nullified credit
assignments. In fine, no error was committed by the CA when it set aside the September 18, 2002 Order of the
SEC and declared the nullity of the acts of majority stockholders in implementing capital infusion through
issuance of additional shares in October 1991 and the board resolution approving the extension of RUBYs
corporate term for another 25 years.

178
3B-Corporation Law

Topic:
Subtopic:
Provision:
Case Name:

Dissolution and Liquidation


Sections 117 to 122, Corporation Code
Premiere Development Bank v Flores

FACTS:

Panacor Marketing Corporation (Panacor for brevity), a newly-formed corporation, acquired an exclusive
distributorship of products manufactured by Colgate Palmolive Philippines, Inc. (Colgate for short). To meet the
capital requirements of the exclusive distributorship, which required an initial inventory level of P7.5 million,
Panacor applied for a loan of P4.1 million with Premiere Development Bank. After an extensive study of
Panacors creditworthiness, Premiere Bank rejected the loan application and suggested that its affiliate
company, Arizona Transport Corporation (Arizona for short), should instead apply for the loan on condition that
the proceeds thereof shall be made available to Panacor.

Since the P2.7 million released by Premiere Bank fell short of the P4.1 million credit line which was previously
approved, Panacor negotiated for a take-out loan with IBA-Finance Corporation (hereinafter referred to as IBAFinance) in the sum of P10 million, P7.5 million of which will be released outright in order to take-out the loan
from Premiere Bank and the balance of P2.5 million (to complete the needed capital of P4.1 million with
Colgate) to be released after the cancellation by Premiere of the collateral mortgage on the property covered
by TCT No. T-3475.

Premiere Bank still refused to release the requested mortgage documents specifically, the owners duplicate
copy of TCT No. T-3475 despite IBA-Finance payment to Premiere Bank the amount of P6,235,754.79,
representing the full outstanding loan account of Arizona.

Panacor failed to generate the required capital to meet its distribution and sales targets. On December 7,
1995, Colgate informed Panacor of its decision to terminate their distribution agreement.

Panacor and Arizona filed a complaint for specific performance and damages against Premiere Bank before the
Regional Trial Court of Pasig City, docketed as Civil Case No. 65577.

On June 11, 1996, IBA-Finance filed a complaint-in-intervention praying that judgment be rendered ordering
Premiere Bank to pay damages in its favor.

RTC Ruled in favor of Panacor and IBA


ISSUE:

Whether the Premiere can collect its deficiency claims from the involuntarily dissolved corporations (ARIZONA
TRANSPORT CORPORATION and PANACOR MARKETING CORPORATION)?
HELD:

No.
The fear of Premiere Development Bank that they would have difficulty collecting its alleged loan deficiencies
from respondent corporations since they were already involuntarily dissolved due to their failure to file
reportorial requirements with the Securities and Exchange Commission is neither here nor there. In any event,
the law specifically allows a trustee to manage the affairs of the corporation in liquidation, and the dissolution
of the corporation would not serve as an effective bar to the enforcement of rights for or against it.

As early as 1939,23 this Court held that, although the time during which the corporation, through its own
officers, may conduct the liquidation of its assets and sue and be sued as a corporation is limited to three
years from the time the period of dissolution commences, there is no time limit within which the trustees must
complete a liquidation placed in their hands. What is provided in Section 12224 of the Corporation Code is that
the conveyance to the trustees must be made within the three-year period. But it may be found impossible to
complete the work of liquidation within the three-year period or to reduce disputed claims to judgment. The
trustees to whom the corporate assets have been conveyed pursuant to the authority of Section 122 may sue
and be sued as such in all matters connected with the liquidation.

Furthermore, Section 145 of the Corporation Code clearly provides that "no right or remedy in favor of or
against any corporation, its stockholders, members, directors, trustees, or officers, nor any liability incurred by
any such corporation, stockholders, members, directors, trustees, or officers, shall be removed or impaired
either by the subsequent dissolution of said corporation." Even if no trustee is appointed or designated during
the three-year period of the liquidation of the corporation, the Court has held that the board of directors may
be permitted to complete the corporate liquidation by continuing as "trustees" by legal implication.25
Therefore, no injustice would arise even if the Court does not stay the execution of G.R. 159352.

179
3B-Corporation Law

Topic:
Dissolution and Liquidation
Subtopic:
Provision:
Sections 117 to 122, Corporation Code
Case Name:
Alabang Development Corp. v Alabang Hills Village Association, Inc.
Facts:
1. Alabang Development Corporation,developer of Alabang Hills Village filed a complaint for Injunction and Damages
against Alabang Hills Village Association Inc., and its president, Rafael for allegedly starting the construction of a multipurpose hall and a swimming pool on one of the parcels of land still owned by ADC, without the latters consent and
approval, and despite demand, failed to desist from constructing thereof.
2. In its answer with counter-claim, AHVAI denied ADCs allegations and made the following claims:
a. That ADC has no legal capacity to sue because its corporate existence was already dissolved by the Securities and
Exchange Corporation on May 26, 2003.
b. That ADC has no cause of action as it was merely holding the property in trust for AHVAI as beneficial owner
thereof.
c. That the lot is part of the open space required by law to be provided in the subdivision.
3.
a.
by
b.

The RTC dismissed ADCs complaint holding that:


It has no personality to sue and that subject area is a reserved area for the benefit of the homeowners as required
law.
HLURB has exclusive jurisdiction over the dispute between ADC and AHVAI.

4. ADC filed a Notice of Appeal to elevate the case to the CA, which also denied its appeal, holding that it had no
capacity to sue as it was already defunct.
Held:
The Supreme Court:
Anent the first assigned error, the Court does not agree that the CA erred in relying on the case of Columbia Pictures,
Inc. v. Court of Appeals.
The CA cited the case for the purpose of restating and distinguishing the jurisprudential definition of the terms
lack of capacity to sue and lack of personality to sue; and of applying these definitions to the present case. Thus,
the fact that, unlike in the instant case, the corporations involved in the Columbia case were foreign corporations is of
no moment. The definition of the term lack of capacity to sue enunciated in the said case still applies to the case at
bar. Indeed, as held by this Court and as correctly cited by the CA in the case of Columbia: [l]ack of legal capacity to
sue means that the plaintiff is not in the exercise of his civil rights, or does not have the necessary qualification to
appear in the case, or does not have the character or representation he claims[;] lack of capacity to sue refers to a
plaintiffs general disability to sue, such as on account of minority, insanity, incompetence, lack of juridical personality
or any other general disqualifications of a party. In the instant case, petitioner lacks capacity to sue because it no
longer possesses juridical personality by reason of its dissolution and lapse of the three-year grace period provided
under Section 122 of the Corporation Code, as will be discussed below.
With respect to the second assigned error, Section 122 of the Corporation Code provides as follows:
SEC. 122. Corporate liquidation. Every corporation whose charter expires by its own limitation or is annulled by
forfeiture or otherwise, or whose corporate existence for other purposes is terminated in any other manner, shall
nevertheless be continued as a body corporate for three (3) years after the time when it would have been so dissolved,
for the purpose of prosecuting and defending suits by or against it and enabling it to settle and close its affairs, to
dispose of and convey its property and to distribute its assets, but not for the purpose of continuing the business for
which it was established.
At any time during said three (3) years, said corporation is authorized and empowered to convey all of its
property to trustees for the benefit of stockholders, members, creditors, and other persons in interest. From and after
any such conveyance by the corporation of its property in trust for the benefit of its stockholders, members, creditors
and others in interest, all interest which the corporation had in the property terminates, the legal interest vests in the
trustees, and the beneficial interest in the stockholders, members, creditors or other persons in interest.
Upon winding up of the corporate affairs, any asset distributable to any creditor or stockholder or member who
is unknown or cannot be found shall be escheated to the city or municipality where such assets are located.
Except by decrease of capital stock and as otherwise allowed by this Code, no corporation shall distribute any
of its assets or property except upon lawful dissolution and after payment of all its debts and liabilities.
This Court has held that:

180
3B-Corporation Law

It is to be noted that the time during which the corporation, through its own officers, may conduct the liquidation of its
assets and sue and be sued as a corporation is limited to three years from the time the period of dissolution
commences; but there is no time limit within which the trustees must complete a liquidation placed in their hands. It is
provided only (Corp. Law, Sec. 78 [now Sec. 122]) that the conveyance to the trustees must be made within the threeyear period. It may be found impossible to complete the work of liquidation within the three-year period or to reduce
disputed claims to judgment. The authorities are to the effect that suits by or against a corporation abate when it
ceased to be an entity capable of suing or being sued (7 R.C.L., Corps., par. 750); but trustees to whom the corporate
assets have been conveyed pursuant to the authority of Sec. 78 [now Sec. 122] may sue and be sued as such in all
matters connected with the liquidation
In the absence of trustees, this Court ruled, thus:
Still in the absence of a board of directors or trustees, those having any pecuniary interest in the assets, including
not only the shareholders but likewise the creditors of the corporation, acting for and in its behalf, might make proper
representations with the Securities and Exchange Commission, which has primary and sufficiently broad jurisdiction in
matters of this nature, for working out a final settlement of the corporate concerns.
In the instant case, there is no dispute that petitioners corporate registration was revoked on May 26, 2003.
Based on the above-quoted provision of law, it had three years, or until May 26, 2006, to prosecute or defend any suit
by or against it. The subject complaint, however, was filed only on October 19, 2006, more than three years after such
revocation.
It is likewise not disputed that the subject complaint was filed by petitioner corporation and not by its directors
or trustees. In fact, it is even averred, albeit wrongly, in the first paragraph of the Complaint that [p]laintiff is a duly
organized and existing corporation under the laws of the Philippines, with capacity to sue and be sued. x x x
Petitioner, nonetheless, insists that a corporation may still sue, even after it has been dissolved and the threeyear liquidation period provided under Section 122 of the Corporation Code has passed. Petitioner cites the cases
of Gelano v. Court of Appeals, Knecht v. United Cigarette Corporation, and Pepsi-Cola Products Philippines, Inc. v. Court
of Appeals, as authority to support its position. The Court, however, agrees with the CA that in the abovecited cases,
the corporations involved filed their respective complaints while they were still in existence. In other words, they
already had pending actions at the time that their corporate existence was terminated.
The import of this Courts ruling in the cases cited by petitioner is that the trustee of a corporation may
continue to prosecute a case commenced by the corporation within three years from its dissolution until rendition of
the final judgment, even if such judgment is rendered beyond the three-year period allowed by Section 122 of the
Corporation Code. However, there is nothing in the said cases which allows an already defunct corporation to initiate a
suit after the lapse of the said three-year period. On the contrary, the factual circumstances in the abovecited cases
would show that the corporations involved therein did not initiate any complaint after the lapse of the three-year
period. In fact, as stated above, the actions were already pending at the time that they lost their corporate existence.
In the present case, petitioner filed its complaint not only after its corporate existence was terminated but also
beyond the three-year period allowed by Section 122 of theCorporation Code. Thus, it is clear that at the time of the
filing of the subject complaint petitioner lacks the capacity to sue as a corporation. To allow petitioner to initiate the
subject complaint and pursue it until final judgment, on the ground that such complaint was filed for the sole purpose
of liquidating its assets, would be to circumvent the provisions of Section 122 of the Corporation Code.
As to the last issue raised, the basic and pivotal issue in the instant case is petitioners capacity to sue as a
corporation and it has already been settled that petitioner indeed lacks such capacity. Thus, this Court finds no cogent
reason to depart from the ruling of the CA finding it unnecessary to delve on the other issues raised by petitioner.

WHEREFORE, the subject judgment of the lower court ordering the register of deeds of Metro Manila, Makati Branch IV
to reconstitute from Decree No. 15170 and the plan and technical descriptions submitted, the alleged certificate of
title, original and owner's duplicate copy, in the name of Manuela Aquial is hereby annulled and set aside, and the
petition for reconstitution is ordered dismissed.

The temporary restraining order of June 27, 1980 issued against respondents is hereby made and declared permanent.
With costs jointly and severally against private respondents.

181
3B-Corporation Law

The Division Clerk of Court is hereby directed to furnish the Honorable Minister of Justice a copy of the decision at bar
(as well as a copy, for ready reference, of the decision of January 27, 1981 in the related Bernal case, G.R. No. L-45168,
previously ordered furnished to him) for the institution of appropriate criminal proceedings against private respondents
and all others who have assisted or conspired with them as may be warranted by the evidence of record.

SO ORDERED.

182
3B-Corporation Law

Topic:
Subtopic:
Provision:
Case Name:

Dissolution and Liquidation


Sections 117 to 122, Corporation Code
Vitaliano Aguirre II v FQB+7

FACTS:

Vitaliano filed a Complaint for intra-corporate dispute, injunction, inspection of corporate books and records,
and damages, against respondents Nathaniel, Priscila and Antonio for the usurpation of the management
powers and prerogatives of the "real" Board of Directors.
The application was granted when the respondents failed to attend the hearing. The respondents filed a
Petition for Certiorari and Prohibition before the CA seeking the annulment of all the proceedings
The CA postulated that Section 122 of the Corporation Code allows a dissolved corporation to continue as a
body corporate for the limited purpose of liquidating the corporate assets and distributing them to its creditors,
stockholders, and others in interest. It does not allow the dissolved corporation to continue its business.
That being the state of the law, the CA determined that Vitalianos Complaint, being geared towards the
continuation of FQB+7, Inc.s business, should be dismissed because the corporation has lost its juridical
personality.
Moreover, the CA held that the trial court does not have jurisdiction to entertain an intra-corporate dispute
when the corporation is already dissolved.

ISSUE:

Whether the RTC has jurisdiction over an intra-corporate dispute involving a dissolved corporation.
HELD:

Intra-corporate disputes remain even when the corporation is dissolved. Jurisdiction over the subject matter is
conferred by law. R.A. No. 8799 conferred jurisdiction over intra-corporate controversies on courts of general
jurisdiction or RTCs, to be designated by the Supreme Court. Thus, as long as the nature of the controversy is
intra-corporate, the designated RTCs have the authority to exercise jurisdiction over such cases.

183
3B-Corporation Law

Topic:
Subtopic:
Provision:
Case Name:

Dissolution and Liquidation


Sections 117 to 122, Corporation Code
Viguilla et al v Philipine College of Criminology

FACTS:

Petitioners are janitorial employees of MBMSI service agency and work in the maintenance department of
petitioner PCCR.

In 2008, the Certificate of Incorporation of MBMSI was revoked and PCCR terminated its contract with MBMSI
thus causing petitioners to be dismissed from service under MBMSI.

Petitioners filed complaints for illegal dismissal. They allege that their real employer was not MBMSI but PCCR,
contending that:
o PCCR had direct control and supervision over their work;
o PCCR had direct control over MBMSIs operations (because Atty. Seril was Senior VP for Admin of PCCR,
and President and General Managemer of MBMI);
o there was no contractual relationship between PCCR and MBMSI; and
o PCCR selected and hired its employees.

PCCR denied such contentions and insist that MBMSI is the direct employer of petitioners. PCCR submitted
documents to prove that they were employees of MBMSI. This included notarized quitclaims that relieve
MBMSI of liability for any and all claims after receipt of separation pay.

LA and NLRC both found that PCCR is the real principal employer and ordered backwages, reinstatement and
payment of other benefits. MBMSI and Atty. Seril are deemed as a labor only contractor and thus are solidarily
liable with PCCR and Bautista for the valid claims of petitioners. However, due to the valid quitclaim executed
to MBMSI, the liability has been extinguished.

CA affirms, hence this case.


ISSUE:
Whether the dissolved corporation can enter into an agreement such as releases, waivers and quitclaims beyond the
3-year winding up period under Section 122 of the Corporation Code
HELD:
Yes.
The executed releases, waivers and quitclaims are valid and binding notwithstanding the revocation of MBMSIs
Certificate of Incorporation. The revocation does not result in the termination of its liabilities. Section 122 of the
Corporation Code provides for a three-year winding up period for a corporation whose charter is annulled by forfeiture
or otherwise to continue as a body corporate for the purpose, among others, of settling and closing its affairs.
Even if said documents were executed in 2009, six (6) years after MBMSIs dissolution in 2003, the same are still valid
and binding upon the parties and the dissolution will not terminate the liabilities incurred by the dissolved corporation
pursuant to Sections 122 and 14528 of the Corporation Code. In the case of Premiere Development Bank v. Flores, the
Court held that a corporation is allowed to settle and close its affairs even after the winding up period of three (3)
years, to wit:
Although the time during which the corporation, through its own officers, may conduct the liquidation of its assets and
sue and be sued as a corporation is limited to three years from the time the period of dissolution commences, there is
no time limit within which the trustees must complete a liquidation placed in their hands. What is provided in Section
122 of the Corporation Code is that the conveyance to the trustees must be made within the three-year period. But it
may be found impossible to complete the work of liquidation within the three-year period or to reduce disputed claims
to judgment. The trustees to whom the corporate assets have been conveyed pursuant to the authority of Section 122
may sue and be sued as such in all matters connected with the liquidation.

184
3B-Corporation Law

Topic:
Foreign Corporations
Subtopic:
Provision:
Sections 123 to 136, Corporation Code
Case Name:
Tuna Processing Inc v Philippine Kingford Inc
Foreign Corporations
Tuna Processing, Inc. vs. Philippine Kingford, Inc. (G.R. No. 185582, February 29, 2012)

Kanemitsu Yamaoka, co-patentee of a US Patent, Philippine Letters Patent, and an Indonesian Patent, entered
into a Memorandum of Agreement with five Philippine tuna processors including Respondent Philippine
Kingford, Inc.. The MOA provides for the enforcing of the abovementioned patents, granting licenses under the
same, and collecting royalties, and for the establishment of herein Petitioner Tuna Processors, Inc..
Due to a series of events, the tuna processors, including Respondent KINGFORD, withdrew from Petitioner TPI
and correspondingly reneged on their obligations.
Petitioner TPI submitted the dispute for arbitration before the International Centre for Dispute Resolution in the
State of California, United States and won the case against Respondent KINGFORD.
To enforce the award, Petitioner TPI filed a Petition for Confirmation, Recognition, and Enforcement of Foreign
Arbitral Award before the RTC of Makati City.
Respondent KINGFORD filed a Motion to Dismiss, which the RTC denied for lack of merit.
Respondent KINGFORD then sought for the inhibition of the RTC judge, Judge Alameda, and moved for the
reconsideration of the order denying the Motion.
Judge Alameda inhibited himself notwithstanding the unfounded allegations and unsubstantiated assertions in
the motion. The case was re-raffled to Judge Ruiz who granted Respondent KINGFORDSs Motion for
Reconsideration and dismissed the Petition on the ground that Petitioner TPI lacked legal capacity to sue in the
Philippines.
Petitioner TPI is a corporation established in the State of California and not licensed to do business in the
Philippines.

ISSUE: W/N a foreign corporation not licensed to do business in the Philippines, but which collects royalties from
entities in the Philippines, sue here to enforce a foreign arbitral award?
RULING: Yes Petitioner can sue to enforce a foreign arbitral award in the Philippines.

Petitioner TPI, although not licensed to do business in the Philippines, may seek recognition and enforcement
of the foreign arbitral award in accordance with the provisions of the Alternative Dispute Resolution Act of
2004.
A foreign corporations capacity to sue in the Philippines is not material insofar as the recognition and
enforcement of a foreign arbitral award is concerned.
Rule 13.1 of the Special Rules provides that [a]ny party to a foreign arbitration may petition the court to
recognize and enforce a foreign arbitral award. The contents of such petition are enumerated in Rule 13.5.
Capacity to sue is not included. Oppositely, in the rule on local arbitral awards or arbitrations in instances
where the place of arbitration is in the Philippines, it is specifically required that a petition to determine any
question concerning the existence, validity and enforceability of such arbitration agreement available to the
parties before the commencement of arbitration and/or a petition for judicial relief from the ruling of the
arbitral tribunal on a preliminary question upholding or declining its jurisdiction after arbitration has already
commenced should state [t]he facts showing that the persons named as petitioner or respondent have legal
capacity to sue or be sued.
it is in the best interest of justice that in the enforcement of a foreign arbitral award, the Court deny
availment by the losing party of the rule that bars foreign corporations not licensed to do business in the
Philippines from maintaining a suit in Philippine courts. When a party enters into a contract containing a
foreign arbitration clause and, as in this case, in fact submits itself to arbitration, it becomes bound by the
contract, by the arbitration and by the result of arbitration, conceding thereby the capacity of the other
party to enter into the contract, participate in the arbitration and cause the implementation of the result.
Arbitration, as an alternative mode of settlement, is gaining adherents in legal and judicial circles here and
abroad. If its tested mechanism can simply be ignored by an aggrieved party, one who, it must be stressed,
voluntarily and actively participated in the arbitration proceedings from the very beginning, it will destroy the
very essence of mutuality inherent in consensual contracts.

185
3B-Corporation Law

On the matter of capacity to sue, a foreign arbitral award should be respected not because it is favored over
domestic laws and procedures, but because Republic Act No. 9285 has certainly erased any conflict of law
question.
Even assuming, only for the sake of argument, that the RTC correctly observed that the Model Law, not the
New York Convention, governs the subject arbitral award, Petitioner TPI may still seek recognition and
enforcement of the award in Philippine court, since the Model Law prescribes substantially identical exclusive
grounds for refusing recognition or enforcement.

186
3B-Corporation Law

Topic:
Foreign Corporations
Subtopic:
Provision:
Sections 123 to 136, Corporation Code
Case Name:
Philippine Deposit Insurance Corp v Citibank
Foreign Corporations
Philippine Deposit Insurance Corporation VS Citibank (G.R. No. 170290, April 11, 2012)

Philippine Deposit Insurance Corporation (PDIC), a government instrumentality created by virtue of Republic
Act No. 3591, conducted an examination of the books of account of Citibank.
Citibank is a banking corporation while respondent Bank of America is a national banking association, both of
which is duly organized and existing under the laws of the USA and duly licensed to do business in the
Philippines.
PDIC discovered that Citibank received from its head office and other foreign branches a total of
P11,923,163,908.00 in dollars, covered by Certificates of Dollar Time Deposit that were interest-bearing.
These funds, lodged in the books of Citibank under the account Their Account-Head Office/Branches-Foreign
Currency, were not reported to PDIC as deposit liabilities that were subject to assessment for insurance.
PDIC assessed Citibank for deficiency in the sum of P1,595,081.96.
PDIC examined the books of accounts of BA which revealed that from BA received from its head office and its
other foreign branches a total of P629,311,869.10 in dollars, covered by Certificates of Dollar Time Deposit
that were interest-bearing with corresponding maturity dates and lodged in their books under the account
Due to Head Office/Branches. BA also excluded these from its deposit liabilities.
PDIC wrote seeking the remittance of P109,264.83 representing deficiency premium assessments.
Citibank and BA sought a declaratory judgment that the money placements they received from their head
office and other foreign branches were not deposits and did not give rise to insurable deposit liabilities under
Sections 3 and 4 of the PDIC Charter and the deficiency assessments were improper and erroneous.
RTC favored respondent, contending that there was no depositor-depository relationship between the
respondents and their head office or other branches. As a result, such deposits were not included as third-party
deposits that must be insured. Rather, they were considered inter-branch deposits which were excluded from
the assessment base, in accordance with the practice of the United States Federal Deposit Insurance
Corporation(FDIC) after which PDIC was patterned.
CA affirmed the decision of the RTC.

ISSUE: W/N the funds placed in the Philippine branch by the head office and foreign branches of Citibank and BA are
insurable deposits under the PDIC Charter and, as such, are subject to assessment for insurance premiums.

RULING: No, Respondents are not subject to assessment for insurance premiums

A foreign corporation can establish its presence in the Philippines by choosing to incorporate its own subsidiary
as a domestic corporation. Such subsidiary would have its own separate and independent legal personality to
conduct business in the country. In the alternative, it may create a branch in the Philippines, which would not
be a legally independent unit, and simply obtain a license to do business in the Philippines.
In the case of Citibank and BA, they both did not incorporate a separate domestic corporation to represent its
business interests in the Philippines. Their Philippine branches are merely branches without a separate legal
personality from their parent company. Being the same entity, the funds placed by the respondents in their
respective branches in the Philippines should not be treated as deposits made by third parties subject to
deposit insurance under the PDIC Charter.
In the leading case of Sokoloff vs.The National City Bank of New York, the Supreme Court of New York held:
Where a bank maintains branches, each branch becomes a separate business entity with separate books of
account. When considered with relation to the parent bank they are not independent agencies; they are, what
their name imports, merely branches, and are subject to the supervision and control of the parent bank
In United States v. BCCI Holdings Luxembourg, the US CA, emphasized that while individual bank branches
may be treated as independent of one another, each branch, unless separately incorporated, must be viewed
as a part of the parent bank rather than as an independent entity. Philippine banking laws also state that the
head office of a foreign bank and its branches are considered as one legal entity. Section 75 of The General

187
3B-Corporation Law

Banking Law and Section 5 of R.A. No. 7221 (An Act Liberalizing the Entry of Foreign Banks) both require the
head office of a foreign bank to guarantee the prompt payment of all the liabilities of its Philippine branch PDIC
must be reminded of the purpose for its creation as the Corporation which is to protect the depositing public
in the event of a bank closure. Suppose the Philippine branch of Citibank suddenly closes for some reason,
Citibank N.A. would then be required to answer for the deposit liabilities of Citibank Philippines. If the Court
were to adopt the posture of PDIC that the head office and the branch are two separate entities and that the
funds placed by the head office and its foreign branches with the Philippine branch are considered deposits, it
would result to the incongruous situation where Citibank, as the head office, would be placed in the ridiculous
position of having to reimburse itself, as depositor, for the losses it may incur occasioned by the closure of
Citibank Philippines. Surely our law makers could not have envisioned such a preposterous circumstance when
they created PDIC

188
3B-Corporation Law

Topic:
Subtopic:
Provision:
Case Name:

Foreign Corporations
Sections 123 to 136, Corporation Code
Steelcase Inc. v Design International Selections Inc.

Facts:
Petitioner Steelcase, Inc. is a foreign corporation existing under the laws of Michigan, USA and is engaged in the
manufacture of office furniture with dealers worldwide. Design InternationalSelections, Inc. (DISI) is a corporation
existing under Philippine Laws and engaged in the furniture business, including the distribution of furniture. Steelcase
and DISI orally entered into a dealership agreement whereby Steelcase granted DISI the right to market, sell,
distribute, install and service its products to end-user customers within the Philippines. The business relationship
continued smoothly until it was terminated after the agreement was breached in 1999. Steelcase filed a complaint for
sum of money against DISI alleging that DISI had an unpaid account of $600,000. It also prayed that DISI be ordered to
pay actual or compensatory damages, exemplary damages, attorneys fees and costs of suit. Meanwhile, DISI alleged
that the complaint failed to state a cause of action and that the complaint should be dismissed because of Steelcases
lack of legal capacity to sue in Philippine courts due to that fact that it doesnt have a license to operate in the country.
The RTC dismissed Steelcases complaint. It has likewise concluded that Steelcase wasdoing business in the
Philippines as contemplated by RA 7042 (The Foreign Investments Act of 1991) and since it did not have the license to
do business in the country, it was barred from seeking redress from Philippine courts until it obtained the requisite
license to do so. The CA affirmed the ruling of the RTC. Steelcase contends that DISI is an independent distributor of
Steelcase products and not an agent or conduit of Steelcase.
Moreover, DISI is acting as Steelcases appointed local distributor, and is transacting business in its own name and for
its own account.
Issue:
Whether or not Steelcase had been doing business in the Philippines without a license
Held:
The phrase doing business is clearly defined in Section 3(d) of RA 7042 (Foreign Investments Act of 1991) which
states that the phrase doing business shall include soliciting orders, service contracts, opening offices, whether
called liaison offices or branches; appointing representatives or distributors domiciled in the Philippines totaling 180
days or more; participating in the management, supervision or control of any domestic business, firm, entity or
corporation in the Philippines; and any other act or acts that imply a continuity of commercial dealings or
arrangements, and contemplate to that extent the performance of acts or works, or the exercise of some of the
functions normally incident to, and in the progressive prosecution of, commercial gain or of the purpose and object of
the business organization. The second sentence of Section 3(d) states that the phrase doing business shall not be
deemed to include mere investment as a shareholder by a foreign entity in domestic corporations duly registered to do
business nor appointing a representative or distributor domiciled in the Philippines which transacts business in its
own name and for its own account.
On such account, the appointment of a distributor in the Philippines is not sufficient to constitute doing business
unless it is under the full control of the foreign corporation. Steelcase, therefore, is foreign corporation not doing
business in the Philippines by its act of appointing a distributor falls under one of the exceptions under RA 7042.

189
3B-Corporation Law

Topic:
Subtopic:
Provision:
Case Name:

Foreign Corporations
Sections 123 to 136, Corporation Code
Cargill Inc. v Intra Strata Assurance Corp.
CARGILL, INC.
vs.
INTRA STRATA ASSURANCE CORPORATION
G.R. No. 168266
March 15, 2010

FACTS:
Cargill (foreign) is a corporation organized and existing under the laws of theState of Delaware. Cargill executed a
contract with Northern Mindanao Corporation (NMC)(domestic), whereby NMC agreed to sell to petitioner 20,000 to
24,000 metrictons of molasses to be delivered from Jan 1 to 30 1990 for $44 per metric ton. The contract provided that
CARGILL was to open a Letter of Credit with theBPI. NMC was permitted to draw up 500,000 representing the minimum
priceof the contract. The contract was amended 3 times (in relation to the amount and the price).But the third
amendment required NMC to put up a performance bond whichwas intended to guarantee NMCs performance to
deliver the molasses duringthe prescribed shipment periods.
In compliance, INTRA STRATA issued a performance bond to guaranteeNMCs delivery. NMC was only able to deliver
219551 metric tons out of the agreed 10,500.Thus CARGILL sent demand letters to INTRA claiming payment under
theperformance and surety bonds. When INTRA failed to pay, CARGILL filed acomplaint.
CARGILL NMC and INTRA entered into a compromise agreement approvedby the court, such provided that NMC would
pay CARGILL 3 million uponsigning and would deliver to CARGILL 6,991 metric tons of molasses. ButNMC still failed to
comply.
ISSUE:
Whether or not petitioner is doing or transacting business in the Philippines in contemplation of the law and
established jurisprudence.
RULING:
NO.
The determination of whether a foreign corporation is doing business in the Philippines must be based on the facts of
each case. In the case at bar, the transactions entered into by the respondent with the petitioners are not a series of
commercial dealings which signify an intent on the part of the respondent to do business in the Philippines but
constitute an isolated one which does not fall under the category of "doing business." The records show that the only
reason why the respondent entered into the second and third transactions with the petitioners was because it wanted
to recover the loss it sustained from the failure of the petitioners to deliver the crude coconut oil under the first
transaction and in order to give the latter a chance to make good on their obligation.
In the present case, petitioner is a foreign company merely importing molasses from a Philipine exporter. A foreign
company that merely imports goods from a Philippine exporter, without opening an office or appointing an agent in the
Philippines, is not doing business in the Philippines.

190
3B-Corporation Law

Topic:
Subtopic:
Provision:
Case Name:

Foreign Corporations
Sections 123 to 136, Corporation Code
Global Business Holdings Inc.

Facts:

Surecomp Software is a foreign corp organized under the laws of the Netherlands. In 1999, it entered into a software
license agreement to let Asian Bank Corp (ABC) use Surecomps IMEX Software System for 20 yrs. Pursuant to the
agreement, Surecomp installed the system and ABC also undertook to pay professional services and annual
maintenance fees for 5 yrs, and committed to purchase some products at discounted prices. ABC also requested
Surecomp to purchase a certain software with a promise to reimburse. However, Global failed to reimburse despite
Surecomps delivery of the product. Sometime in 2000, ABC merged with Global Business. When Global took over
operations, it found the IMEX system unworkable and informed Surecomp that it was going to discontinue with the
software agreement and that it was going to stop payments.
Surecomp filed a complaint for breach of contract with damages in RTC-Makati for Globals failure to pay its
obligations in the agreement despite demands. Surecomp demanded payment of actual damages and an additional
amount for Globals unilateral pretermination of the agreement, and damages.

Instead of filing an answer, Global filed an MTD based on two grounds:

1.

license; and

2.

That Surecomp had no capacity to sue because it was doing business in the Philippines without a

That the claim on which the action was founded was unenforceable under the Intellectual Property
Code of the Philippines. Being a technology transfer arrangement, Surecomp failed to comply with
Sec 87 & 88 of the Intellectual Property Code of the Philippines.

RTC ruled that: On Ground 1: Global is estopped from denying Surecomps capacity to sue. Globals argument that it
was not the one who actually contracted with Surecomp is of no moment. It does not relieve Global of its contractual
obligation. On Gorund 2: This will require a hearing before the MTD can be resolved. Surecomp moved for an outright
denial of the MTD. RTC denied MTD. RTC says it sees no reason to belabor the issue on Surecomps capacity to sue
since there is a prima facie showing that Global entered into a contract with Surecomp and having done so, willingly, it
cannot now be made to raise the issue of capacity to sue (Merrill Lynch Futures, Inc. v. CA).

As to unenforceability of the contract, it is an executed, rather than an executor contract. The statute of frauds finds
no application here. Global filed a petition for certiorari with prayer for the issuance of a TRO and/or writ of
preliminary injunction under Rule 65 before the CA, saying that the RTC abused its discretion and acted in excess of
its jurisdiction. CA denied the petition. MR denied.
Issue:
Whether Global is estopped from questioning Surecomps capacity to sue.
HELD:
YES. The determination of a corporations capacity is a factual question that requires the elicitation of a preponderant
set of facts. As a rule, unlicensed foreign non-resident corporations doing business in the Philippines cannot file suits in
the Philippines. This is mandated under Section 133 of the Corporation Code. A corporation has a legal status only
within the state or territory in which it was organized. For this reason, a corporation organized in another country has
no personality to file suits in the Philippines. In order to subject a foreign corporation doing business in the country to
the jurisdiction of our courts, it must acquire a license from the Securities and Exchange Commission and appoint an
agent for service of process. Without such license, it cannot institute a suit in the Philippines.
The exception to this rule is the doctrine of estoppel. Global is estopped from challenging Surecomps capacity to sue.A
foreign corporation doing business in the Philippines without license may sue in Philippine courts a Filipino citizen or a
Philippine entity that had contracted with and benefited from it. A party is estopped from challenging the personality of
a corporation after having acknowledged the same by entering into a contract with it. The principle is applied to

191
3B-Corporation Law

prevent a person contracting with a foreign corporation from later taking advantage of its noncompliance with the
statutes, chiefly in cases where such person has received the benefits of the contract.
Due to Globals merger with ABC and because it is the surviving corporation, it is as if it was the one which entered into
contract with Surecomp. In the merger of two existing corporations, one of the corporations survives and continues the
business, while the other is dissolved, and all its rights, properties, and liabilities are acquired by the surviving
corporation.This is particularly true in this case. Based on the findings of fact of the RTC, as affirmed by the CA, under
the terms of the merger or consolidation, Global assumed all the liabilities and obligations of ABC as if it had incurred
such liabilities or obligations itself. In the same way, Global also has the right to exercise all defenses, rights,
privileges, and counter-claims of every kind and nature which ABC may have or invoke under the law. These findings of
fact were never contested by Global in any of its pleadings filed before the Court.

192
3B-Corporation Law

Topic:
Subtopic:
Provision:
Case Name:

Foreign Corporations
Sections 123 to 136, Corporation Code
Van Zuiden Bros LTD v GTVL Industries Inc.

FACTS:
Petitioner Zuiden, is a corporation, incorporated under the laws of Hong Kong. Zuiden is not engaged in business in the
Philippines, but is suing before the Philippine Courts, for the reasons hereinafter stated. It is engaged in the
importation and exportation of several products, including lace products. On several occasions, GTVL purchased lace
products from petitioner.
The procedure for these purchases, as per the instructions of GTVL, was that Zuiden delivers the products purchased
by GTVL, to a certain Hong Kong corporation, known as Kenzar Ltd. and the products are then considered as sold, upon
receipt by Kenzar of the goods purchased by GTVL. Kenzar had the obligation to deliver the products to the Philippines
and/or to follow whatever instructions GTVL had on the matter.
Insofar as Zuiden is concerned, upon delivery of the goods to KENZAR in Hong Kong, the transaction is concluded; and
GTVL became obligated to pay the agreed purchase price. However, commencing October 31, 1994 up to the present,
GTVL has failed and refused to pay the agreed purchase price for several deliveries ordered by it and delivered by
Zuiden.
ISSUE:
Whether or not petitioner, an unlicensed foreign corporation, has legal capacity to sue before Philippine courts.
RULING:
YES.
An unlicensed foreign corporation not doing business in the Philippines can sue before Philippine courts. In the present
case, the series of transactions between petitioner and respondent cannot be classified as "doing business" in the
Philippines under Section 3(d) of RA 7042.
An essential condition to be considered as "doing business" in the Philippines is the actual performance of specific
commercial acts within the territory of the Philippines for the plain reason that the Philippines has no jurisdiction over
commercial acts performed in foreign territories.
Here, there is no showing that petitioner performed within the Philippine territory the specific acts of doing business
mentioned in Section 3(d) of RA 7042. Petitioner did not also open an office here in the Philippines, appoint a
representative or distributor, or manage, supervise or control a local business. While petitioner and respondent
entered into a series of transactions implying a continuity of commercial dealings, the perfection and consummation of
these transactions were done outside the Philippines.
Further, the series of transactions between petitioner and respondent transpired and were consummated in Hong
Kong. There was no single activity which petitioner performed here in the Philippines pursuant to its purpose and
object as a business organization. Moreover, petitioners desire to do business within the Philippines is not discernible
from the allegations of the complaint or from its attachments. Therefore, there is no basis for ruling that petitioner is
doing business in the Philippines.
Considering that petitioner is not doing business in the Philippines, it does not need a license in order to initiate and
maintain a collection suit against respondent for the unpaid balance of respondents purchases.

193
3B-Corporation Law

Topic:
Subtopic:
Provision:
Case Name:

Foreign Corporations

Sections 123 to 136, Corporation Code


Agilent Technologies Singapore v Integrated Silicon Technology
Philippines Corp
AGILENT TECHNOLOGIES SINGAPORE (PTE) LTD., vs. INTEGRATED SILICON TECHNOLOGY PHILIPPINES CORP et al

FACTS: Petitioner Agilent is a foreign corporation, which, by its own admission, is not licensed to do business in the
Philippines. Respondent Integrated Silicon is a private domestic corporation, 100% foreign owned, which is engaged in
the business of manufacturing and assembling electronics components.
The juridical relation among the various parties in this case can be traced to a 5-year Value Added Assembly Services
Agreement (VAASA), between Integrated Silicon and HP-Singapore. Under the terms of the VAASA, Integrated Silicon
was to locally manufacture and assemble fiber optics for export to HP-Singapore. HP-Singapore, for its part, was to
consign raw materials to Integrated Silicon. The VAASA had a five-year term with a provision for annual renewal by
mutual written consent. Later, with the consent of Integrated Silicon, HP-Singapore assigned all its rights and
obligations in the VAASA to Agilent.
Later, Integrated Silicon filed a complaint for Specific Performance and Damages against Agilent and its officers. It
alleged that Agilent breached the parties oral agreement to extend the VAASA. Agilent filed a separate complaint
against Integrated Silicon for Specific Performance, Recovery of Possession, and Sum of Money with Replevin,
Preliminary Mandatory Injunction, and Damages.

Respondents filed a MTD in the 2nd case, on the grounds of lack of Agilents legal capacity to sue; litis
pendentia; forum shopping; and failure to state a cause of action.
ISSUE: WON an unlicensed foreign corporation not doing business in the Philippines lacks the legal capacity to file suit.
HELD: The petition is GRANTED. The Decision of the CA which dismissed the 2nd case is REVERSED and SET ASIDE.
The Order denying the MTD is REINSTATED. Agilents application for a Writ of Replevin is GRANTED.
Ratio:
A foreign corporation without a license is not ipso facto incapacitated from bringing an action in Philippine courts. A
license is necessary only if a foreign corporation is transacting or doing business in the country. The Corporation
Code provides:

Sec. 133. Doing business without a license. No foreign corporation transacting business in the Philippines without a
license, or its successors or assigns, shall be permitted to maintain or intervene in any action, suit or proceeding in any
court or administrative agency of the Philippines; but such corporation may be sued or proceeded against before
Philippine courts or administrative tribunals on any valid cause of action recognized under Philippine laws.
The aforementioned provision prevents an unlicensed foreign corporation doing business in the Philippines from
accessing our courts.

The principles regarding the right of a foreign corporation to bring suit in Philippine courts may thus be condensed in
four statements:
if a foreign corporation does business in the Philippines without a license, it cannot sue before the Philippine courts;
if a foreign corporation is not doing business in the Philippines, it needs no license to sue before Philippine courts on an
isolated transaction or on a cause of action entirely independent of any business transaction;

194
3B-Corporation Law

if a foreign corporation does business in the Philippines without a license, a Philippine citizen or entity which has
contracted with said corporation may be estopped from challenging the foreign corporations corporate personality in
a suit brought before Philippine courts; and
if a foreign corporation does business in the Philippines with the required license, it can sue before Philippine courts on
any transaction.
**
The challenge to Agilents legal capacity to file suit hinges on whether or not it is doing business in the Philippines.
However, there is no definitive rule on what constitutes doing, engaging in, or transacting business in the
Philippines. The Corporation Code itself is silent as to what acts constitute doing or transacting business in the
Philippines.
In the Mentholatum case this Court discoursed on the two general tests to determine whether or not a foreign
corporation can be considered as doing business in the Philippines. The first of these is the substance test, thus:
The true test [for doing business], however, seems to be whether the foreign corporation is continuing the body of the
business or enterprise for which it was organized or whether it has substantially retired from it and turned it over to
another.
The second test is the continuity test, expressed thus:
The term [doing business] implies a continuity of commercial dealings and arrangements, and contemplates, to that
extent, the performance of acts or works or the exercise of some of the functions normally incident to, and in the
progressive prosecution of, the purpose and object of its organization.]
**
The Foreign Investments Act of 1991 (the FIA; Republic Act No. 7042, as amended), defines doing business as
follows:
Sec. 3, par. (d). The phrase doing business shall include soliciting orders, service contracts, opening offices, whether
called liaison offices or branches; appointing representatives or distributors domiciled in the Philippines or who in
any calendar year stay in the country for a period or periods totaling one hundred eighty (180) days or more;
participating in the management, supervision or control of any domestic business, firm, entity, or corporation in the
Philippines; and any other act or acts that imply a continuity of commercial dealings or arrangements, and
contemplate to that extent the performance of acts or works, or the exercise of some of the functions normally
incident to, and in the progressive prosecution of, commercial gain or of the purpose and object of the business
organization.
An analysis of the relevant case law, in conjunction with Sec 1 of the IRR of the FIA (as amended by RA 8179), would
demonstrate that the acts enumerated in the VAASA do not constitute doing business in the Philippines. The said
provision provides that the following shall not be deemed doing business:
(1) Mere investment as a shareholder by a foreign entity in domestic corporations duly registered to do business,
and/or the exercise of rights as such investor;
(2) Having a nominee director or officer to represent its interest in such corporation;
(3) Appointing a representative or distributor domiciled in the Philippines which transacts business in the
representatives or distributors own name and account;
(4) The publication of a general advertisement through any print or broadcast media;
(5) Maintaining a stock of goods in the Philippines solely for the purpose of having the same processed by another
entity in the Philippines;

195
3B-Corporation Law

(6) Consignment by a foreign entity of equipment with a local company to be used in the processing of products for
export;
(7) Collecting information in the Philippines; and
(8) Performing services auxiliary to an existing isolated contract of sale which are not on a continuing basis, such as
installing in the Philippines machinery it has manufactured or exported to the Philippines, servicing the same, training
domestic workers to operate it, and similar incidental services.
By and large, to constitute doing business, the activity to be undertaken in the Philippines is one that is for profitmaking.
By the clear terms of the VAASA, Agilents activities in the Philippines were confined to (1) maintaining a stock of
goods in the Philippines solely for the purpose of having the same processed by Integrated Silicon; and (2)
consignment of equipment with Integrated Silicon to be used in the processing of products for export. As such, we hold
that, based on the evidence presented thus far, Agilent cannot be deemed to be doing business in the Philippines.
Respondents contention that Agilent lacks the legal capacity to file suit is therefore devoid of merit. As a foreign
corporation not doing business in the Philippines, it needed no license before it can sue before our courts.

196
3B-Corporation Law

Topic:
Subtopic:
Provision:
Case Name:

Penal Provisions
Sections 144, Corporation Code
Loreli Lim Po v DOJ

Facts: Tan is a stockholder of CHVI, a real estate development company. Chiu is the president and Loreli Lim Po is the
accountant. Tan requested a copy of the companys financial statement and be allowed to inspect the corporate
books. His request, however, was denied despite repeated demands. Tan thus filed a case against Chiu and Po.
Issue: Whether or not Tan has a cause of action.
Held: Yes, Tan has a right to inspect the companys financial statements and other business documents
Ratio:
There is ample evidence on record to support the said decision. To name one, accountants Creest O. Morales and Jay
Arr T. Hernandez, who were part of the Inspection Team sent by Tan to CHVI, executed a Joint AffidaviT stating that the
documents made available to them for inspection were limited. Further, they claimed that on the day of the inspection,
they brought a portable photocopying machine to CHVIs premises but they were not allowed to use the same. The
offense punishable under Section 74, in relation to Section 144 of the Corporation Code, for which Chiu was indicted,
requires the unjustified disallowance or refusal by a suspect, of a
stockholders written request to examine or copy excerpts of a corporations books or minutes. The absence of any
ascribed ill motives on the part of the aforementioned accountants to make statements adverse or unfavorable to Chiu
lends credibility to their declarations.

197
3B-Corporation Law

Topic:
Subtopic:
Provision:
Case Name:

Penal Provisions
Sections 144, Corporation Code
Yuico v Quiambao

Facts:
- On Mar. 1 2004, Aderito Z. Yujuico was elected and replaced Quiambao as the president and chairman of Strategic
Alliance Development Corporation, a domestic corporation operating as a business development and investment
company.
- Yujuico then appointed Sumbilla as treasurer and Blando, who replaced Pilapil, as corporate secretary.
- Yujuico demanded to Quaimbao for the turnover of the corporate records of the company which were in possession of
Casanova, the accountant of STRADEC, who was in possession of the said records on behalf of Quiambao because the
latter needed the same as part of his defense in a pending case in court.
- Quiambao and Casanova then caused the removal of the corporate records of STRADEC from the companys office.
- Blando likewise demanded Pilapil for the turnover of the stock and transfer book of STRADEC, to which he refused.
Pilapil then proposed that such records be deposited in a safety deposit box in Equitable-PCI Bank. Blando accepted
the proposal with the agreement that it was only to be opened in the presence of both Quiambao and Blando.
- Quiambao and Blando withdrew the safety deposit box and brought it to the offices of STRADCOM. Quiambao asked
Blando to proceed to the STRADCOM offices and upon arriving thereat, Quiambao pressured Blando to make certain
entries in the stock and transfer books. After making such entries, Blando demanded that he be given the possession
of the stock and transfer book, Quiambao refused.
- Even after several demands of Blando for the turnover of the stock and transfer book of STRADEC, Quiambao
repeatedly refused to turn over such.
- Thus, on August 12, 2005, Yujuico filed a criminal case against Quiambao, Pilapil and Casanova for the violation of
Sec. 74 in relation to Sec. 144 of the Corporation Code.
- The RTC ruled that the act of refusing to allow inspection of the stock and transfer book, even though it may be a
violation of Section 74(4), is not punishable as an offense under the Corporation Code because under Sec. 74, the
application of Section 144 is expressly mentioned only in relation to the act of "refusing to allow any director, trustees,
stockholder or member of the corporation to examine and copy excerpts from the corporation's records or minutes"
that excludes its stock and transfer book.

Issue: W/N the RTCs interpretation of Sec. 74 in relation to Sec. 144 of the Corporation Code is correct

Held: No, RTCs interpretation of Sec. 74 in relation to Sec. 144 of the Corporation Code.
- While Section 74 of the Corporation Code expressly mentions the application of Section 144 only in relation to the act
of "refusing to allow any director, trustees, stockholder or member of the corporation to examine and copy excerpts
from the corporation's records or minutes," it same does not mean that Sec. 144 no longer applies to any other
possible violations of Sec. 74.
- It must be emphasized that Section 144 already purports to penalize "violations" of "any provision" of the Corporation
Code "not otherwise specifically penalized therein." Hence, we find inconsequential the fact that that Section 74
expressly mentions the application of Section 144 only to a specific act, but not with respect to the other possible
violations of the former section.
- Indeed, we find no cogent reason why Section 144 of the Corporation Code cannot be made to apply to violations of
the right of a stockholder to inspect the stock and transfer book of a corporation under Section 74(4) given the already
unequivocal intent of the legislature to penalize violations of a parallel right, i.e., the right of a stockholder or member
to examine the other records and minutes of a corporation under Section 74(2). Certainly, all the rights guaranteed to
corporators under Section 74 of the Corporation Code are mandatory for the corporation to respect. All such rights are

198
3B-Corporation Law

just the same underpinned by the same policy consideration of keeping public confidence in the corporate vehicle thru
an assurance of transparency in the corporation's operations.
- Verily, we find inaccurate the pronouncement of the RTC that the act of refusing to allow inspection of the stock and
transfer book is not a punishable offense under the Corporation Code. Such refusal, when done in violation of Section
74(4) of the Corporation Code, properly falls within the purview of Section 144 of the same code and thus may be
penalized as an offense.

199
3B-Corporation Law

Topic:
Subtopic:
Provision:
Case Name:

Securities and Exchange Commission


Securities Regulations Code, Kinds of Securities
Securities and Regulations Code, PD-902-A
Power Homes Unlimited Corp v SEC

Facts:
Power Homes Inc, a domestic corporation is duly registered with public respondent SEC and its primary purpose is to
develop real estates. Respondent Noel Manero requested public respondent SEC to investigate petitioners business.
Upon investigation, SEC found that petitioner is engaged in the sale or offer for sale or distribution of investment
contracts, which are considered securities under Sec. 3.1 (b) of Republic Act (R.A.) No. 8799 (The Securities Regulation
Code), but failed to register them in violation of Sec. 8 of the Code. SEC issued a Cease and Desist Order (CDO) hence
this suit.
Issue: whether petitioners business constitutes an investment contract which should be registered with public
respondent SEC before its sale or offer for sale or distribution to the public.
Held:
An investment contract is defined in the Amended Implementing Rules and Regulations of R.A. No. 8799 as a "contract,
transaction or scheme (collectively contract) whereby a person invests his money in a common enterprise and is led
to expect profits primarily from the efforts of others." Our R.A. No. 8799 appears to follow this flexible concept for it
defines an investment contract as a contract, transaction or scheme (collectively "contract") whereby a person invests
his money in a common enterprise and is led to expect profits not solely but primarily from the efforts of
others. Thus, to be a security subject to regulation by the SEC, an investment contract in our jurisdiction must be
proved to be: (1) an investment of money, (2) in a common enterprise, (3) with expectation of profits,
(4) primarily from efforts of others.
Hence, petitioner was engaged in the sale or distribution of an investment contract. In the SEC v. Turner case, the
Court ruled that It is apparent from the record that what is sold is not of the usual "business motivation" type of
courses. Rather, the purchaser is really buying the possibility of deriving money from the sale of the plans
by Dare to individuals whom the purchaser has brought to Dare. The promotional aspects of the plan, such as
seminars, films, and records, are aimed at interesting others in the Plans. Their value for any other purpose is, to put it
mildly, minimal. Once an individual has purchased a Plan, he turns his efforts toward bringing others into
the organization, for which he will receive a part of what they pay. His task is to bring prospective purchasers
to "Adventure Meetings." The business scheme of petitioner in the case at bar thus, its business constitutes an
investment contract.

200
3B-Corporation Law

Topic:
Subtopic:
Provision:
Case Name:

Securities
Securities
Securities
Securities

and Exchange Commission


Regulations Code, Kinds of Securities
and Regulations Code, PD-902-A
and Exchange Commission v Prosperity Com Inc.

Facts:
Prosperity.Com, Inc. (PCI) sold computer software and hosted websites without providing internet service. To make a
profit, PCI devised a scheme in which, for the an amount, a buyer could acquire from it an internet website of a 15Mega Byte (MB) capacity and

by referring to PCI his own down-line buyers, a first-time buyer could earn

commissions.PCI patterned its scheme from that of Golconda Ventures, Inc. (GVI) which SEC issued a CDO before. SEC
alleged that PCIs scheme constitutes an Investment contract and, following the Securities Regulations Code, it should
have first registered such contract or securities with the SEC

Issue:. WON PCIs scheme constitutes and Investment Contract.


Held:
The United States Supreme Court held in Securities and Exchange Commission v. W.J. Howey Co.[ that, for an
investment contract to exist, the following elements, referred to as the Howey test must concur: (1) a contract,
transaction, or scheme; (2) an investment of money; (3) investment is made in a common enterprise; (4) expectation
of profits; and (5) profits arising primarily from the efforts of others. Thus, to sustain the SEC position in this case, PCIs
scheme or contract with its buyers must have all these elements.
Here, PCIs clients do not make such investments. They buy a product of some value to them: an Internet website of a
15-MB capacity. The client can use this website to enable people to have internet access to what he has to offer to
them, say, some skin cream. The buyers of the website do not invest money in PCI that it could use for running some
business that would generate profits for the investors. The last requisite in the Howey test is lacking in the marketing
scheme that PCI has adopted. Thus, its scheme does not constitute an Investment Contract.

201
3B-Corporation Law

Topic:
Subtopic:
Provision:
Case Name:

Securities
Securities
Securities
Securities

and Exchange Commission


Regulations Code, Kinds of Securities
and Regulations Code, PD-902-A
and Exchange Commission v Santos

FACTS:
Sometime in 2007, yet another investment scam was exposed with the disappearance of its primary
perpetrator, Michael H.K. Liew (Liew), a self- styled financial guru and Chairman of the Board of Directors of
Performance Investment Products Corporation (PIPC-BVI), a foreign corporation registered in the British Virgin Islands.
To do business in the Philippines, PIPC-BVI incorporated herein as Philippine International Planning Center
Corporation (PIPC Corporation).
Because the head of PIPC Corporation had gone missing and with it the monies and investment of a significant
number of investors, the SEC was flooded with complaints from thirty-one (31) individuals against PIPC Corporation, its
directors, officers, employees, agents and brokers for alleged violation of certain provisions of the Securities
Regulation Code, including Section 28 thereof. Santos was charged in the complaints in her capacity as investment
consultant of PIPC Corporation, who supposedly induced private complainants Luisa Mercedes P. Lorenzo (Lorenzo) and
Ricky Albino P. Sy (Sy), to invest their monies in PIPC Corporation.
Soon thereafter, the SEC, through its Compliance and Endorsement Division, filed a complaint-affidavit for
violation of Sections 8,4 265 and 286 of the Securities Regulation Code before the Department of Justice which was
docketed as I.S. No. 2007-1054. Among the respondents in the complaint-affidavit were the principal officers of PIPC:
Liew, Chairman and President; Cristina Gonzalez-Tuason, Director and General Manager; Ma. Cristina Bautista-Jurado,
Director; and herein respondent Santos.
On the whole, Lorenzo and Sy charge Santos in her capacity as investment consultant of PIPC Corporation who
actively engaged in the solicitation and recruitment of investors. Private complainants maintain that Santos, apart
from being PIPC Corporations employee, acted as PIPC Corporations agent and made representations regarding its
investment products and that of the supposed global corporation PIPC-BVI. Facilitating Lorenzos and Sys investment
with PIPC Corporation, Santos represented to the two that investing with PIPC Corporation, an affiliate of PIPC-BVI,
would be safe and full-proof.
Santos defense consisted in: (1) denying participation in the conspiracy and fraud perpetrated against the
investor-complainants of PIPC Corporation, specifically Sy and Lorenzo; (2) claiming that she was initially and merely
an employee of, and subsequently an independent information provider for, PIPC Corporation; (3) PIPC Corporation
being a separate entity from PIPC-BVI of which Santos has never been a part of in any capacity; (4) her not having
received any money from Sy and Lorenzo, the two having, in actuality, directly invested their money in PIPC-BVI; (5)
Santos having dealt only with Sy and the latter, in fact, deposited money directly into PIPC-BVIs account; and (6) on
the whole, PIPC-BVI as the other party in the investment contracts signed by Sy and Lorenzo, thus the only corporation
liable to Sy and Lorenzo and the other complainants.
On 18 April 2008, the DOJ, in I.S. No. 2007-1054, issued a Resolution signed by a panel of three (3)
prosecutors, with recommendation for approval of the Assistant Chief State Prosecutor, and ultimately approved by
Chief State Prosecutor Jovencito R. Zuo, indicting: (a) Liew and Gonzalez-Tuason for violation of Sections 8 and 26 of
the Securities Regulation Code; and (b) herein respondent Santos, along with Cristina Gonzalez-Tuason and 12 others
for violation of Section 28 of the Securities Regulation Code. The same Resolution likewise dismissed the complaint
against 8 of the respondents therein for insufficiency of evidence. In the 18 April 2008 Resolution, the DOJ discussed at
length the liability of PIPC Corporation and its officers, employees, agents and all those acting on PIPC Corporations
behalf.
In sum, the DOJ panel based its finding of probable cause on the collective acts of the majority of the
respondents therein, including herein respondent Santos, which consisted in their acting as employees-agent and/or
investor-agents of PIPC Corporation and/or PIPC-BVI. Specifically alluding to Santos as Investment Consultant of PIPC
Corporation, the DOJ found probable cause to indict her for violation of Section 28 of the Securities Regulation Code for
engaging in the business of selling or offering for sale securities, on behalf of PIPC Corporation and/or PIPC-BVI (which
were found to be an issuer13 of securities without the necessary registration from the SEC) without Santos being
registered as a broker, dealer, salesman or an associated person.
On separate motions for reconsideration of the respondents therein, including herein respondent Santos, the
DOJ panel issued a Resolution dated 2 September 2008 modifying its previous ruling and excluding respondent Victor
Jose Vergel de Dios from prosecution for violation of Section 28 of the Securities Regulation Code.
Respondent Santos filed a petition for review before the Office of the Secretary of the DOJ assailing the
Resolutions dated 18 April 2008 and 2 September 2008 and claiming that she was a mere clerical
employee/information provider who never solicited nor recruited investors, in particular complainants Sy and Lorenzo,

202
3B-Corporation Law

for PIPC Corporation or PIPC- BVI. Santos also claimed dearth of evidence indicating she was a salesman/agent or an
associated person of a broker or dealer, as defined under the Securities Regulation Code.
The SEC filed its Comment opposing Santos petition for review. Thereafter, the Office of the Secretary of the
DOJ, through its then Undersecretary Ricardo R. Blancaflor, issued a Resolution dated 1 October 2009 which, as
previously adverted to, excluded respondent Santos from prosecution for violation of Section 28 of the Securities
Regulation Code.
ISSUE:
Whether or not respondent Santos acted as agent of PIPC Corp. or had enticed Luisa Mercedes P. Lorenzo or
Ricky Albino P. Sy to buy PIPC Corp. or PIPC-BVIs investment products.
HELD: NO.
This appeal by certiorari raising the sole error of Santos exclusion from the Information for violation of Section
28 of the Securities Regulation Code.
Generally, at the preliminary investigation proper, the investigating prosecutor, and ultimately, the Secretary of the
DOJ, is afforded wide latitude of discretion in the exercise of its power to determine probable cause to warrant criminal
prosecution. The determination of probable cause is an executive function where the prosecutor determines merely
that a crime has been committed and that the accused has committed the same.17 The rules do not require that a
prosecutor has moral certainty of the guilt of a person simply for preliminary investigation purposes.
However, the authority of the prosecutor and the DOJ is not absolute; it cannot be exercised arbitrarily or
capriciously. Where the findings of the investigating prosecutor or the Secretary of the DOJ as to the existence of
probable cause are equivalent to a gross misapprehension of facts, certiorari will lie to correct these errors.18
While it is our policy not to interfere in the conduct of preliminary investigations, we have, on more than one
occasion, adhered to some exceptions to the general rule.
In excluding Santos from the prosecution of the supposed violation of Section 28 of the Securities Regulation
Code, the Secretary of the DOJ, as affirmed by the appellate court, debunked the DOJ panels finding that Santos was
prima facie liable for either: (1) selling securities in the Philippines as a broker or dealer, or (2) acting as a salesman, or
an associated person of any broker or dealer on behalf of PIPC Corporation and/or PIPC-BVI without being registered as
such with the SEC.
To get to that conclusion, the Secretary of the DOJ and the appellate court ruled that no evidence was adduced
showing Santos actual participation in the final sale by PIPC Corporation and/or PIPC-BVI of unregistered securities
since the very affidavits of complainants Lorenzo and Sy proved that Santos had never signed, neither was she
mentioned in, any of the investment documents between Lorenzo and Sy, on one hand, and PIPC Corporation and/or
PIPC-BVI, on the other hand.
The conclusions made by the Secretary of the DOJ and the appellate court are a myopic view of the investment
solicitations made by Santos on behalf of PIPC Corporation and/or PIPC-BVI while she was not licensed as a broker or
dealer, or registered as a salesman, or an associated person of a broker or dealer.
We sustain the DOJ panels findings which were not overruled by the Secretary of the DOJ and the appellate
court, that PIPC Corporation and/or PIPC-BVI was: (1) an issuer of securities without the necessary registration or
license from the SEC, and (2) engaged in the business of buying and selling securities.
To determine whether the DOJ Secretarys Resolution was tainted with grave abuse of discretion, we pass upon
the elements for violation of Section 28 of the Securities Regulation Code: (a) engaging in the business of buying or
selling securities in the Philippines as a broker or dealer; or (b) acting as a salesman; or (c) acting as an associated
person of any broker or dealer, unless registered as such with the SEC.
Tying it all in, there is no quarrel that Santos was in the employ of PIPC Corporation and/or PIPC-BVI, a
corporation which sold or offered for sale unregistered securities in the Philippines. To escape probable culpability,
Santos claims that she was a mere clerical employee of PIPC Corporation and/or PIPC-BVI and was never an agent or
salesman who actually solicited the sale of or sold unregistered securities issued by PIPC Corporation and/or PIPC-BVI.
Solicitation is the act of seeking or asking for business or information; it is not a commitment to an
agreement.20
Santos, by the very nature of her function as what she now unaffectedly calls an information provider, brought
about the sale of securities made by PIPC Corporation and/or PIPC-BVI to certain individuals, specifically private
complainants Sy and Lorenzo by providing information on the investment products of PIPC Corporation and/or PIPC- BVI
with the end in view of PIPC Corporation closing a sale.
While Santos was not a signatory to the contracts on Sys or Lorenzos investments, Santos procured the sale
of these unregistered securities to the two (2) complainants by providing information on the investment products being
offered for sale by PIPC Corporation and/or PIPC-BVI and convincing them to invest therein.

203
3B-Corporation Law

No matter Santos strenuous objections, it is apparent that she connected the probable investors, Sy and
Lorenzo, to PIPC Corporation and/or PIPC-BVI, acting as an ostensible agent of the latter on the viability of PIPC
Corporation as an investment company. At each point of Sys and Lorenzos investment, Santos participation thereon,
even if not shown strictly on paper, was prima facie established.
In all of the documents presented by Santos, she never alleged or pointed out that she did not receive extra
consideration for her simply providing information to Sy and Lorenzo about PIPC Corporation and/or PIPC-BVI. Santos
only claims that the monies invested by Sy and Lorenzo did not pass through her hands. In short, Santos did not
present in evidence her salaries as a supposed mere clerical employee or information provider of PIPC-BVI. Such
presentation would have foreclosed all questions on her status within PIPC Corporation and/or PIPC-BVI at the lowest
rung of the ladder who only provided information and who did not use her discretion in any capacity.
We cannot overemphasize that the very information provided by Santos locked the deal on unregistered
securities with Sy and Lorenzo.
In fact, Sy alleged in his affidavit, which allegation was not refuted by Santos, that he was introduced to Santos while
he performed routine transactions at his bank.
What is palpable from the foregoing is that Sy and Lorenzo did not go directly to Liew or any of PIPC
Corporations and/or PIPC-BVIs principal officers before making their investment or renewing their prior investment.
However, undeniably, Santos actively recruited and referred possible investors to PIPC Corporation and/or PIPC-BVI and
acted as the go-between on behalf of PIPC Corporation and/or PIPC-BVI.
The DOJs and Court of Appeals reasoning that Santos did not sign the investment contracts of Sy and Lorenzo
is specious. The contracts merely document the act performed by Santos.
Individual complainants and the SEC have categorically alleged that Liew and PIPC Corporation and/or PIPC-BVI
is not a legitimate investment company but a company which perpetrated a scam on 31 individuals where the
president, a foreign national, Liew, ran away with their money. Liews absconding with the monies of 31 individuals and
that PIPC Corporation and/or PIPC-BVI were not licensed by the SEC to sell securities are uncontroverted facts.
The transaction initiated by Santos with Sy and Lorenzo, respectively, is an investment contract or
participation in a profit sharing agreement that falls within the definition of the law. When the investor is relatively
uninformed and turns over his money to others, essentially depending upon their representations and their honesty
and skill in managing it, the transaction generally is considered to be an investment contract. The touchstone is the
presence of an investment in a common venture premised on a reasonable expectation of profits to be derived from
the entrepreneurial or managerial efforts of others.
At bottom, the exculpation of Santos cannot be preliminarily established simply by asserting that she did not
sign the investment contracts, as the facts alleged in this case constitute fraud perpetrated on the public. Especially so
because the absence of Santos signature in the contract is, likewise, indicative of a scheme to circumvent and evade
liability should the pyramid fall apart.
Lastly, we clarify that we are only dealing herein with the preliminary investigation aspect of this case. We do not
adjudge respondents guilt or the lack thereof. Santos' defense of being a mere employee or simply an information
provider is best raised and threshed out during trial of the case.

204
3B-Corporation Law

Topic:
Subtopic:
Provision:
Case Name:
FACTS:

Securities and Exchange Commission


Insider Trading, Short Swing Transactions, and Manipulation of
Security Prices and other Fraudulent acts
Securities and Regulations Code, PD-902-A
Abacus Securities Corp v Ampil

Petitioner corporation was engaged in business as a broker and dealer of securities of listed companies at the
Philippine Stock Exchange Center. On April 8, 1997, respondent opened a cash account with petitioner for his
transactions in securities and, on April 10, started trading on that account. As a result of his trading activities, he
accumulated an outstanding obligation in favor of the corporation in the principal sum of P6,617,036.22 as of April 30,
1997. Respondent failed to settle his account upon the lapse of the required period and the extension given by
petitioner, prompting it to sell his securities on May 6, 1997, to offset his unsettled obligations. After the sale of his
securities and the application of the proceeds against his account, his remaining accountabilities to petitioner totalled
P3,364,313.56. This obligation he failed to settle despite its demands. The trial court and the Court of Appeals (CA)
both held that the parties were in pari delicto and, hence, without recourse against each other. The lower courts said
that petitioner had violated Sections 23 and 25 of the Revised Securities Act (RSA) and Rule 25-1 of the Rules
Implementing the Act (RSA Rules). The violation was committed when it failed 1) to require respondent to pay for his
stock purchases within three (T+3) or four days (T+4) from trading; and 2) to request from the appropriate authority
an extension of time for the payment of his cash purchases. The trial court noted that despite his nonpayment within
the required period, petitioner did not cancel his purchases. Neither did it require him to deposit cash payments before
it executed buy and/or sell orders subsequent to the first unsettled transaction.

ISSUES:
Two issues were raised by the parties:
(1) whether the pari delicto rule was applicable to the present case; and
(2) whether the trial court had jurisdiction over the case.

HELD:
First Issue: Pari Delicto
Pari Delicto Sections 23 and 25 and Rule 25-1, otherwise known as the mandatory close-out rule,[1] clearly vested
an obligation, not just a right, in petitioner. That obligation was to cancel or otherwise liquidate a customers order, if
payment was not received within three days from the date of purchase. Subsequent to an unpaid order, the broker
should require its customer to deposit funds in the account sufficient to cover each purchase, prior to the execution of
the transaction. These duties were imposed upon the broker to ensure faithful compliance with the margin
requirements of the law, which forbade the broker from extending undue credit to a cash customer.
Respondent Liable for the First Respondent Liable for the First But Not for the Subsequent Trade But Not for the
Subsequent Trade Nonetheless, these margin requirements were applicable only to transactions entered into by the
parties subsequent to the initial trades of April 10 and 11, 1997. Thus, petitioner could still collect from respondent to
the extent of the difference between his outstanding obligation as of April 11, 1997, less the proceeds from the
mandatory sellout of the shares pursuant to the RSA Rules. Its right to collect was justified under the general law on
obligations and contracts.[2] Petitioner could not be denied the right to collect, as the initial transactions had been
entered into pursuant to the instructions of respondent. His obligation for stock transactions made and entered into on
April 10 and 11, 1997, remained outstanding. Those transactions were valid, and the obligations he incurred in regard
to his stock purchases on those dates subsisted. At the time, there was yet no violation of the RSA. Petitioner
committed a fault only when it failed 1) to liquidate the transactions on April 14 and 15, 1997, or the fourth day
following the stock purchases; and 2) to complete its liquidation no later than ten days after, by applying the proceeds
as payment for his outstanding obligation.[3]
Elucidating further, since the buyer was not able to pay for the transactions that had taken place on April 10 and 11 -that is, at T+4 -- the broker was duty-bound to advance the payment to the settlement banks, without prejudice to its
right to collect from the client later on.[4] It should be clear that Congress had imposed the margin requirements to

205
3B-Corporation Law

protect the general economy, not to give the customer a free ride at the expense of the broker.[5] Not to require
respondent to pay for his April 10 and 11 trades would put a premium on his circumvention of the laws and would
enable him to enrich himself unjustly at the expense of petitioner. By failing to ensure his payment of his first purchase
transactions within the period prescribed by law, thereby allowing him to make subsequent purchases, petitioner
effectively converted his cash account into a credit account. The extension or maintenance of credits on nonmargin
transactions, however, were specifically prohibited under Section 23(b). Thus, petitioner was remiss in its duty and
could not be said to have come to court with clean hands, insofar as it intended to collect on transactions
subsequent to the initial trades of April 10 and 11, 1997. Respondent Equally Guilty Respondent Equally Guilty for
Subsequent Trades for Subsequent Trades On the other hand, respondent was found to be equally guilty of entering
into transactions in violation of the RSA and RSA Rules. The Court was not prepared to accept his self-serving
assertions of being an innocent victim in all the transactions. Obviously, he knowingly speculated on the market by
taking advantage of the nocash-out arrangement extended to him by petitioner. It was respondents privilege to
gamble or speculate, as he apparently did by asking for extensions of time and refraining from giving orders to his
broker to sell, in the hope that the prices would rise. Sustaining his argument would have amounted to relieving him of
the risks of his own speculation and saddling petitioner with the consequences after the result turned out to be
unfavorable.[6] His conduct as an investor was precisely the sort deplored by the law. Thus, with respect to his
counterclaim for damages for having been allegedly induced by petitioner to generate additional purchases despite his
outstanding obligations, the Court held that he deserved no legal or equitable relief. In the final analysis, both parties
had acted in violation of the law and did not come to court with clean hands as regards the transactions subsequent to
the initial one made on April 10 and 11, 1997. In this case, the pari delicto rule applied only to transactions entered
into after those initial trades. Pursuant to RSA Rule 25-1, petitioner should have liquidated the transactions (sold the
stocks) on the fourth day after (at T+4) and completed its liquidation not later than ten days following the last day for
the customer to pay (effectively at T+14). Respondents outstanding obligation, therefore, was to be determined on
the basis of the closing prices -- at T+14 -- of the stocks purchased.
Second Issue: Jurisdiction
Jurisdiction The instant controversy related to acts committed by the parties in the course of their business
relationship. An ordinary civil case seeking to enforce rights arising from the Agreement (AOF) between the parties, the
suit was intended to enable petitioner to collect on the alleged outstanding debt incurred by respondent for his stock
purchases. To be sure, the RSA and its Rules were to be read into the Agreement that the parties had entered into.
Thus, to determine whether they had fulfilled their obligations under this Agreement, the Court passed upon their
compliance with the RSA and its Rules. In no way did it thereby deprive the Securities and Exchange Commission (SEC)
of the authority to determine willful violations of the RSA and impose appropriate sanctions, as provided under
Sections 45 and 46 of the Act. Thus, the Court upheld the SEC in its Opinion, as follows: As to the issue of jurisdiction,
it is settled that a party cannot invoke the jurisdiction of a court to secure affirmative relief against his opponent and
after obtaining or failing to obtain such relief, repudiate or question that same jurisdiction. Indeed, after voluntarily
submitting a cause and encountering an adverse decision on the merits, it is too late for petitioner to question the
jurisdictional power of the court. It is not right for a party who has affirmed and invoked the jurisdiction of a court in a
particular matter to secure an affirmative relief, to afterwards deny that same jurisdiction to escape a penalty.

206
3B-Corporation Law

Topic:
Subtopic:
Provision:
Case Name:

Securities and Exchange Commission


Insider Trading, Short Swing Transactions, and Manipulation of
Security Prices and other Fraudulent acts
Securities and Regulations Code, PD-902-A
SEC v Interport Resources Corp

FACTS:
The Board of Directors of IRC approved a Memorandum of Agreement with GHB (Ganda Holdings Berhad).
Under said memorandum of agreement, IRC acquired 100% of the entire capital stock of GEHI (Ganda Energy Holdings
Inc.) which would own and operate a 102 megawatt gas turbine power generating barge. In exchange, IRC will issue to
GHB 55% of the expanded capital stock of IRC. On the side, IRC would acquire 67% of the entire capital of PRCI
(Philippine Racing Club).
It is alleged herein that a press release announcing the approval of the agreement was sent to the Philippine
Stock Exchange through facsimile and the SEC, but the facsimile machine of the SEC could not receive it. However, the
SEC received reports that the IRC failed to make timely public disclosures of its negotiations with GHB and that some
of its directors, heavily traded IRC shares utilizing this material insider information. For this reason, the SEC required
the directors to appear before the SEC to explain the alleged failure to disclose material information as required by the
Rules on Disclosure of Material Facts. Unsatisfied with the explanation, the SEC issued an order finding that the IRC
violated the Rules in connection with the then Old Securities Act when it failed to make timely disclosures of its
negotiations with GHB. In addition, the SEC found that the directors of IRC entered into transactions involving IRC
shares in violation of the Revised Securities Act.
Respondents, however, questioned the authority of the SEC to investigate on said matter since according to PD
902-A, jurisdiction upon the matter was conferred upon the PED (Prosecution and Enforcement Department) of the SEC
however, this issue is already moot since pending the disposition of the case, the Securities Regulation Code was
passed thereby effectively repealing PD 902-A and abolishing the PED. They also contended that their right to due
process was violated when the SEC required them to appear before the SEC to show cause why sanctions should not
be imposed upon them since such requirement shifted the burden of proof to respondents.
Issue:
1.) Whether or not the SEC has authority to file suit against respondents for violations of the RSA.
2.) Whether or not their right to due process was violated when the SEC denied the parties of their right to cross
examination.
Held:
The Revised Securities Act does not require the enactment of implementing rules to make it binding and
effective. The provisions of the RSA are sufficiently clear and complete by themselves. The requirements are
specifically set out and the acts which are enjoined are determinable. To tule that absence of implementing rules can
render ineffective an act of Congress would empower administrative bodies to defeat the legislative will by delaying
the implementing rules. Where the statute contains sufficient standards and an unmistakable intent (as in this case,
the RSA) there should be no impediment as to its implementation.
The court does not discern any vagueness or ambiguity in the RSA such that the acts proscribed and/or
required would not be understood by a person of ordinary intelligence. The provision explains in simple terms that the
insider's misuse of nonpublic and undisclosed information is the gravamen of illegal conduct and that the intent of the
law is the protection of investors against fraud committed when an insider, using secret information, takes advantage
of an uninformed investor. Insiders are obligatd to disclose material information to the other party or abstain from
trading the shares of his corporation. This duty to disclose or abstain is based n 2 factors: 1) the existence of a
relationship giving access, directly or indirectly to information intended to be available only for a corporate purpose
and not for the personal benefit of anyone and 2) the inherent unfairness involved when a party takes advantage of
such information knowing it is unavailable to those with whom he is dealing.
This obligation to disclose is imposed upon "insiders" which are particularly officers, directors or controlling
stockholders but that definition has already been expanded and not includes those persons whose relationship of
former relationship to the issuer or the security that is not generally available and the one who learns such a fact from
an insider knowing that the person from whom he learns such fact is an insider. In some case, however, there may be
valid corporate reasons for the nondisclosure of material information but it should not be used for non-corporate
purposes.
Respondent contends that the terms "material fact", "reasonable person", "nature and reliability" and "generally
available" are vaguely used in the RSA because under the provision of the said law what is required to be disclosed is a
fact of special significance, meaning:

207
3B-Corporation Law

1.
2.

a material fact which would be likely to affect the market price of a security or;
one which a reasonable person would consider especially important in determining his course of action with
regard to the shares of stock.

But the court dismissed said contention and stated that material fact is already defined and explained as one
which induces or tends to induce or otherwise affect the sale or purchase of securities. On the other hand, "reasonable
person" has already been used many times in jurisprudence and in law since it is a standard on which most of legal
doctrines stand (even the doctrine on negligence uses such standard) and it has been held to mean " a man who relies
on the calculus of common sense of which all reasonable men have in abundance"
As to "nature and reliability" the proper adjudicative body would be able to determine if facts of a certain nature
and reliability can influence a reasonable person's decision to retain, buy or sell securities and thereafter explain and
justify its factual findings in its decision since the same must be viewed in connection with the particular
circumstances of a case.
As to "generally available", the court held also that such is a matter which may be adjudged given the particular
circumstances of the case. The standards of which cannot remain at a standstill.
There is no violation of due process in this case since the proceedings before the PED are summary in nature. The
hearing officer may require the parties to submit their respective verified position papers together will all supporting
documents and affidavits of witnesses. A formal hearing is not mandatory and it is within the discretion of the hearing
officer to determine whether or not there is a need for a formal hearing. Moreover, the law creating the PED empowers
it to investigate violations of the rules and regulations and to file and prosecute such cases. It does not have an
adjudicatory powers. Thus, the PED need not comply with the provisions of the Administrative Code on adjudication.
The SEC retained jurisdiction to investigate violations of the RSA, reenacted in the Securities Regulations Code
despite the abolition of the PED. In this case, the SEC already commenced investigating the respondents for violations
of the RSA but during the pendency of the case the Securities and Regulations Code was passed thereby repealing the
RSA. However, the repeal cannot deprive the SEC of its jurisdiction to continue investigating the case.
Investigations by the SEC is a requisite before a criminal case may be referred to the DOJ since the SEC is an
administrative agency with the special competence to do so. According to the doctrine of primary jurisdiction, the
courts will not determine a controversy involving a question within the jurisdiction of an administrative tribunal where
the question demands the exercise of sound administrative discretion requiring the specialized knowledge and
expertise of said administrative tribunal to determine technical and intricate matters of fact.

208
3B-Corporation Law

Topic:
Subtopic:

Securities and Exchange Commission


Insider Trading, Short Swing Transactions, and Manipulation of
Security Prices and other Fraudulent acts
Provision:
Securities and Regulations Code, PD-902-A
Case Name:
La Bugal BLaan Tribal Corp v Ramos
INSIDER TRADING, SHORT SWING TRANSACTIONS AND MANIPULATION OF SECURITY PRICES AND OTHER FRAUDULENT
ACTS
La Bugal-BLaan Tribal Association vs DENR Secretary Ramos
FACTS:

On July 25, 1987, then President Corazon C. Aquino issued Executive Order (E.O.) No. 279 authorizing the DENR
Secretary to accept, consider and evaluate proposals from foreign-owned corporations or foreign investors for
contracts or agreements involving either technical or financial assistance for large-scale exploration,
development, and utilization of minerals, which, upon appropriate recommendation of the Secretary, the
President may execute with the foreign proponent.

On March 3, 1995, then President Fidel V. Ramos approved R.A. No. 7942 to "govern the exploration,
development, utilization and processing of all mineral resources." R.A. No. 7942 defines the modes of mineral
agreements for mining operations, outlines the procedure for their filing and approval, assignment/transfer and
withdrawal, and fixes their terms. Similar provisions govern financial or technical assistance agreements.

On April 9, 1995, 30 days following its publication, R.A. No. 7942 took effect. Shortly before the effectivity of
R.A. No. 7942, however, or on March 30, 1995, the President entered into an FTAA with WMCP covering 99,387
hectares of land in South Cotabato, Sultan Kudarat, Davao del Sur and North Cotabato.

On August 15, 1995, then DENR Secretary Victor O. Ramos issued DENR Administrative Order (DAO) No. 95-23,
s. 1995, otherwise known as the Implementing Rules and Regulations of R.A. No. 7942. This was later repealed
by DAO No. 96-40, s. 1996 which was adopted on December 20, 1996.

On January 10, 1997, counsels for petitioners sent a letter to the DENR Secretary demanding that the DENR
stop the implementation of R.A. No. 7942 and DAO No. 96-40, giving the DENR fifteen days from receipt to act
thereon.

Petitioners filed suit to permanently enjoin respondents from acting on any FTAA agreement, declare the RA
7942 and its IRR as unconstitutional and null and void, and cancel the FTAA with Western Mining. SC en banc
ruled in favor of petitioners.

Hence this MR. During the pendency of the suit, the WMCP shares were bought by Sagittarius Mining, a
Philippine corporation.
Issue:
Whether or not Sec. 7.9 of the FTAA to WMCP is valid.
Held:
NO.
While Section 7.7 gives the government a 60 percent share in the net mining revenues of WMCP from the
commencement of commercial production, Section 7.9 deprives the government of part or all of the said 60 percent.
Under the latter provision, should WMCP's foreign shareholders -- who originally owned 100 percent of the equity -- sell
60 percent or more of its outstanding capital stock to a Filipino citizen or corporation, the State loses its right to
receive its 60 percent share in net mining revenues under Section 7.7.
Evidently, what Section 7.7 grants to the State is taken away in the next breath by Section 7.9 without any offsetting
compensation to the State. Thus, in reality, the State has no vested right to receive any income from the FTAA for the
exploitation of its mineral resources. Furthermore, it appears that what is given to the State in Section 7.7 is by mere
tolerance of WMCP's foreign stockholders, who can at any time cut off the government's entire 60 percent share. They
can do so by simply selling 60 percent of WMCP's outstanding capital stock to a Philippine citizen or corporation, as
what happened in this case, when they sold the shares to Sagittarius Mining. Moreover, the proceeds of such sale will
of course accrue to the foreign stockholders of WMCP, not to the State.
The sale of 60 percent of WMCP's outstanding equity to a corporation that is 60 percent Filipino-owned and 40 percent
foreign-owned will still trigger the operation of Section 7.9. Effectively, the State will lose its right to receive all 60
percent of the net mining revenues of WMCP; and foreign stockholders will own beneficially up to 64 percent of WMCP,
consisting of the remaining 40 percent foreign equity therein, plus the 24 percent pro-rata share in the buyercorporation.
It is possible that the inclusion of the offending provision was initially prompted by the desire to provide some form of
incentive for the principal foreign stockholder in WMCP to eventually reduce its equity position and ultimately divest in
favor of Filipino citizens and corporations. However, as finally structured, Section 7.9 has the deleterious effect of

209
3B-Corporation Law

depriving government of the entire 60 percent share in WMCP's net mining revenues, without any form of
compensation whatsoever. Such an outcome is completely unacceptable. By their mere divestment of up to 60 percent
equity in WMCP in favor of Filipino citizens and/or corporations, the local and foreign stockholders get a windfall. Their
share in the net mining revenues of WMCP is automatically increased, without their having to pay the government
anything for it. In short, the provision in question is without a doubt grossly disadvantageous to the government,
detrimental to the interests of the Filipino people, and violative of public policy.

210
3B-Corporation Law

Topic:
Securities and Exchange Commission
Subtopic:
Tender Offer, Margin Trading, and Other Regulated Activities
Provision:
Securities and Regulations Code, PD-902-A
Case Name:
CEMCO Holdings Inc v National Life
TENDER OFFER, MARGIN TRADING & OTHER REGULATED ACTIVITIES (Sec. 19, RA 8799)
CEMCO Holdings vs National Life
FACTS:

Union Cement Corporation (UCC), a publicly-listed company, has two principal stockholders UCHC, a nonlisted company, with shares amounting to 60.51%, and petitioner Cemco with 17.03%. Majority of UCHCs
stocks were owned by BCI with 21.31% and ACC with 29.69%. Cemco, on the other hand, owned 9% of UCHC
stocks.

In a disclosure letter dated 5 July 2004, BCI informed the PSE that it and its subsidiary ACC had passed
resolutions to sell to Cemco BCIs stocks in UCHC equivalent to 21.31% and ACCs stocks in UCHC equivalent to
29.69%.

In the PSE Circular for Brokers, it was stated that as a result of petitioner Cemcos acquisition of BCI and ACCs
shares in UCHC, petitioners total beneficial ownership, direct and indirect, in UCC has increased by 36% and
amounted to at least 53% of the shares of UCC. PSE inquired with the SEC on whether the Tender Offer Rule in
Rule 19 of the IRR of Securities Regulation Code is applicable or not. SEC answered in the negative.

National Life, a minority stockholder of UCC, sent a letter to Cemco demanding the mandatory Tender Offer
Rule. Cemco denied.

National Life filed a complaint with the SEC to declare the purchase of Cemco of the ACC and BCI shares to be
void.
o SEC ruled for National Life, and ordered Cemco to make a tender offer to National Life.
o CA affirmed.
ISSUE:
WON the Tender Offer Rule applies to the indirect acquisition of shares in a listed company, in this case, the indirect
acquisition by Cemco of 36% of UCC, a publicly-listed company, through its purchase of the shares in UCHC, a nonlisted company .
HELD:
YES.
Tender offer is a publicly announced intention by a person acting alone or in concert with other persons to acquire
equity securities of a public company. A public company is defined as a corporation which is listed on an exchange, or
a corporation with assets exceeding P50,000,000.00 and with 200 or more stockholders, at least 200 of them holding
not less than 100 shares of such company. Stated differently, a tender offer is an offer by the acquiring person to
stockholders of a public company for them to tender their shares therein on the terms specified in the offer. Tender
offer is in place to protect minority shareholders against any scheme that dilutes the share value of their investments.
It gives the minority shareholders the chance to exit the company under reasonable terms, giving them the
opportunity to sell their shares at the same price as those of the majority shareholders.
Under existing SEC Rules, the 15% and 30% threshold acquisition of shares under the Section 19 of RA 8799 was
increased to thirty-five percent (35%). It is further provided therein that mandatory tender offer is still applicable even
if the acquisition is less than 35% when the purchase would result in ownership of over 51% of the total outstanding
equity securities of the public company.
The SEC and the Court of Appeals ruled that the indirect acquisition by petitioner of 36% of UCC shares through the
acquisition of the non-listed UCHC shares is covered by the mandatory tender offer rule. The SEC and the Court of
Appeals accurately pointed out that the coverage of the mandatory tender offer rule covers not only direct acquisition
but also indirect acquisition or "any type of acquisition."
The legislative intent of Section 19 of the Code is to regulate activities relating to acquisition of control of the listed
company and for the purpose of protecting the minority stockholders of a listed corporation. Whatever may be the
method by which control of a public company is obtained, either through the direct purchase of its stocks or through
an indirect means, mandatory tender offer applies.

211
3B-Corporation Law

Topic:
Securities and Exchange Commission
Subtopic:
Tender Offer, Margin Trading, and Other Regulated Activities
Provision:
Securities and Regulations Code, PD-902-A
Case Name:
Abacus Securities Corp v Ampil
TENDER OFFER, MARGIN TRADING & OTHER REGULATED ACTIVITIES (Section 23 & 25, RA 8799)
Abacus Securities vs Ampil
FACTS:

Petitioner corporation was engaged in business as a broker and dealer of securities of


listed
companies at the Philippine Stock Exchange Center.

On April 8, 1997, respondent opened a cash account with petitioner for his transactions in securities
and, on April 10, started trading on that account. As a result of his trading activities, he accumulated an
outstanding obligation in favor of the corporation in the principal sum of P6,617,036.22 as of April 30, 1997.

Respondent failed to settle his account upon the lapse of the required period and the extension
given by petitioner, prompting it to sell his securities on May 6, 1997, to offset his unsettled obligations.
After the sale of his securities and the application of the proceeds against his account, his remaining
accountabilities to petitioner totalled P3,364,313.56. This obligation he failed to settle despite its demands.

The trial court and the CA both held that the parties were in pari delicto and, hence, without recourse
against each other. The lower courts said that petitioner had violated Sections 23 and 25 of the
Revised Securities Act (RSA) and Rule 25-1 of the Rules Implementing the Act (RSA Rules).
The
violation was committed when it failed 1) to require respondent to pay for his stock purchases within
three (T+3) or four days (T+4) from trading; and 2) to request from the appropriate authority an
extension of time for the payment of his cash purchases. The trial court noted that despite his
nonpayment within the required period, petitioner did not cancel his purchases. Neither did it require
him to deposit cash payments before it executed buy and/or sell orders subsequent to the first
unsettled transaction.
ISSUE:
1) Whether the pari delicto rule is applicable in the present case, and
2) Whether the trial court had jurisdiction over Abacus alleged violation of the Revised Securities Act.
HELD:
YES.
Sections 23 and 25 and Rule 25-1, otherwise known as the mandatory close-out rule, [1] clearly vested an obligation,
not just a right, in petitioner. That obligation was to cancel or otherwise liquidate a customers order, if payment was
not received within three days from the date of purchase. Subsequent to an unpaid order, the broker should require its
customer to deposit funds in the account sufficient to cover each purchase, prior to the execution of the transaction.
These duties were imposed upon the broker to ensure faithful compliance with the margin requirements of the law,
which forbade the broker from extending undue credit to a cash customer.
It will be noted that trading on credit (or margin trading) allows investors to buy more securities than their cash
position would normally allow. Investors pay only a portion of the purchase price of the securities; their broker
advances for them the balance of the purchase price and keeps the securities as collateral for the advance or loan.
Brokers take these securities/stocks to their bank and borrow the balance on it, since they have to pay in full for the
traded stock. Hence, increasing margins i.e., decreasing the amounts which brokers may lend for the speculative
purchase and carrying of stocks is the most direct and effective method of discouraging an abnormal attraction of
funds into the stock market and achieving a more balanced use of such resources.

212
3B-Corporation Law

You might also like